You are on page 1of 304

CHILD

NEUROLOGY
CASE BASED

Muhammad Saeed
MB, FCPS (Pak), MRCP, FRCP (Glasgow),
MRCPCH, FRCPCH (London), FRCP (Ireland)
Consultant Pediatric Neurologist
Armed Forces Hospital,
Saudi Arabia

Paramount Books (Pvt.) Ltd.


Karachi | Lahore | Islamabad | Sukkur | Faisalabad | Peshawar | Abbottabad
Paramount Books (Pvt.) Ltd.

Child Neurology Case Based


by
Muhammad Saeed

All rights reserved. No part of this publication may be reproduced, stored in a retrieval system,
or transmitted in any form or by any means, electronic, mechanical, photocopying, recording or
otherwise, without the prior permission of the Copyright Holders.

This book is sold subject to the condition that it shall not, by way of trade or otherwise, be lent, resold,
hired out or otherwise circulated without the publisher’s prior consent in any form of binding or cover
other than that in which it is published and without a similar condition including this condition being
imposed on the subsequent purchaser.

Medical knowledge is constantly changing. As new information becomes available, changes in


treatment, procedures, equipment and the use of drugs become necessary. The editors, contributors
and the publishers have, as far as it is possible, taken care to ensure that the information given
in this text is accurate and up-to-date. However, readers are strongly advised to confirm that the
information, especially with regard to drug usage, complies with the latest legislation and standards
of practice. Neither the publisher nor the authors assume any responsibility for any loss or injury
and/or damage to person or property arising out of or related to any use of the material contained
in this handbook.

Copyright © 2019
All Rights Reserved

First Edition ......................................


......................................2019
2019

Paramount Books (Pvt.) Ltd.


152/O, Block-2, P.E.C.H.S., Karachi-75400.
Tel: +92-21-34310030, info@paramountbooks.com.pk
www.paramountbooks.com.pk

ISBN: 978-969-637-538-8
Printed in Pakistan
CONTENTS
Preface v
Acknowledgement vi
Case 01 Mitochondrial Encephalomyopathy, Lactic Acidosis 1
Case 02 Guillain-Barré Syndrome 6
Case 03 Cephalic Tetanus 13
Case 04 Early Infantile Epileptic Encephalopathy 18
Case 05 Infantile Krabbe Disease (Globoid Cell Leukodystrophy) 23
Case 06 Pyridoxal 5'-Phosphate-Responsive Epilepsy 29
Case 07 Acute Cerebellar Ataxia 34
Case 08 Juvenile Myasthenia Gravis 40
Case 09 Spasmus Nutans 46
Case 10 Dopa-Responsive Dystonia 51
Case 11 Cyanotic Breath-Holding Spells 57
Case 12 Juvenile Spinal Muscular Atrophy 62
Case 13 Left Carotid Artery Dissection 69
Case 14 Autoimmune Encephalitis 75
Case 15 Posterior Reversible Encephalopathy Syndrome 81
Case 16 Systemic Primary Carnitine Deficiency 86
Case 17 Duchenne Muscular Dystrophy 93
Case 18 Congenital Disorder of Glycosylation 101
Case 19 Benign Childhood Epilepsy with Centrotemporal Spikes 106
Case 20 Type IV Hereditary Sensory Autonomic Neuropathy 111
Case 21 Acute Transverse Myelitis 117
Case 22 Ataxia Telangiectasia 125
Case 23 Anti-N-Methyl-D-Aspartates Receptor Encephalitis 134
Case 24 X-Linked Myotubular Myopathy 140
Case 25 Childhood Absence Epilepsy 146
Case 26 Cerebral Sinus Venous Thrombosis 150
Case 27 Doose Syndrome 157
Case 28 Acute Disseminated Encephalomyelitis 162
Case 29 Juvenile Myoclonic Epilepsy 168
Case 30 Landau-Kleffner Syndrome 172
Case 31 Wilson’s Disease 177
Case 32 Refractory Convulsive Status Epilepticus 184
Case 33 Lennox–Gastaut Syndrome 188
Case 34 Subacute Sclerosing Panencephalitis 194
Case 35 Post-infectious (Idiopathic) Opsoclonus Myoclonus Syndrome 199
Case 36 Autism 204
Case 37 Rett Syndrome 212
Case 38 Adrenoleukodystrophy 217
Case 39 Tuberous Sclerosis Complex 223
Case 40 Hemiplegic Migraine 229
Case 41 Sydenham Chorea 238
Case 42 Hyperekplexia 244
Case 43 Leigh Syndrome 249
Case 44 Tuberculous Meningitis 254
Case 45 Myotonia Congenita 263
Case 46 Biotin-Thiamine-Responsive Basal Ganglia Disease 268
Case 47 Panayiotopoulos Syndrome 275
Case 48 Infantile Spasms 280
Case 49 Nonketotic Hyperglycinaemia 285
Case 50 Pompe Disease 292
Comments of Well Known Neurologists 298
PREFACE
A considerable proportion of the patients in pediatric medical department
has pediatric neurological issues 20%-30% are accompanied by neurologi-
cal issues. This shows that neurological diseases account for a large volume
of pediatric patients. I realize that there is no current book in pediatric
neurology which is case based, that might serve as a study aid for the post-
graduate examination.
In Child Neurology (Case Based), both common and rare pediatric
neurological disorders are covered, but majority of the cases are based
on actual presentations. The cases are presented randomly and not in se-
quence of diseases, in order to stimulate the experience for theoretical and
clinical examination. This book is intended to help doctors gain a founda-
tion on knowledge in pediatric neurology and develop a practical approach
to clinical decision making.

Dr. Muhammad Saeed


MBBS, FCPS, MRCPCH, FRPCPCH (UK),
MRCPS, FRCPS (Glasgow), FRCPI (Ireland)
Consultant Pediatric Neurologist
Armed Forces Hospital,
Saudi Arabia
ACKNOWLEDGEMENT
I must thank all my patients since without them this book would not have
been possible. Special thanks are due to my family, my wife Dr. Saima
Saeed, my son Muhammad Haider Saeed, and my daughter Mahnoor
Saeed for their forbearance. This is a befitting opportunity for me to thank
my friend Dr. Tahir Nadeem for his immeasurable time for proofreading.
Case 01
A 9 year old boy was admitted with the complaints of fever, headache,
vomiting, convulsions and left sided weakness of the body for two
days. Seizures were generalised tonic-clonic in nature that lasted for fifteen
minutes. He had a history of repeated episodes of weakness of one side of
his body associated with seizures for the last four years. Parents noticed that
the boy was reasonably well up to 4 years of age, but since then, he was less
interested to engage in any physical activity and became easily fatigued. His
developmental mile stone was age appropriate up to 4 years of age, since then
motor and language delays were evident. He complained of poor vision and
hearing impairment. His school performance was below average. The boy had
a normal antenatal and perinatal history. There is no history of consanguinity
of marriage in his family. All of his family members including maternal uncle
or cousin are in good health.
On physical examination, the boy was apathetic, least interested in his sur-
roundings, grossly emaciated, stunted, and his vital signs were within normal
limits. Neurological examination revealed slurred speech, poor vision and
hearing impairment. Cranial nerves, except for the 2nd and the 3rd were intact.
Muscle bulk, tone and strength were symmetrically reduced. Deep Tendon
Reflexes (DRT) were decreased, bilateral plantar reflexes were flexor; sensory
function was intact.
On fundoscopic examination, signs of optic atrophy were found in both
eyes. Audiometry revealed a bilateral, moderate, mixed type of hearing loss.
Laboratory evaluation revealed positive findings including, blood lactate 58.84
mg/dl (normal range 3-7 mg/dl), very high SGPT 1091 U/L (normal range
up to 40 U/L) Ultrasonogram of the abdomen showed cholelithiasis and
pancreatic calculi. An Electroencephalogram (EEG) showed focal epileptiform
discharge. Computed Tomography (CT scan) of the brain showed a hypodense
signal in the right temporoparietal region suggestive of infarction involving an
area of the middle cerebral artery and also bilateral basal ganglia calcification.

Answer the following questions:


• What is the most likely diagnosis?
• Summarise the case.
• Discuss the differential diagnosis.
• Discuss the diagnostic work-up.
• What is the appropriate management of the patient?

1
Child Neurology

DIAGNOSIS
Mitochondrial Encephalomyopathy, Lactic Acidosis, and Stroke like episodes
(MELAS)

SUMMARY
A previously healthy boy who presented with the complains of fever, head-
ache, vomiting, convulsions and left sided weakness of the body for two days.
He had a history of repeated episodes of weakness of one side of his body asso-
ciated with seizures for the last four years. On examination, was apathetic, least
interested in his surroundings, grossly emaciated, stunted and CNS examina-
tion revealed slurred speech, poor vision and hearing impairment.

DIFFERENTIAL DIAGNOSIS
MELAS is a complicated, multisystem disease. Since the symptoms, signs, and
acute clinical presentations are so varied and the disease is relatively rare, the
disease is misdiagnosed frequently.
• Mitochondrial encephalomyopathy, lactic acidosis, and stroke like episodes
(MELAS). Patients with the syndrome of mitochondrial encephalomyopa-
thy, lactic acidosis, and stroke like episodes (MELAS) have stroke like events
that are acute in onset, often transient, and occasionally associated with a
febrile illness. The vascular territories of focal brain lesions and the prior
medical history of these patients differ substantially from those of typical
patients with stroke. Although MELAS remains a largely untreatable con-
dition, prevention and management of medical complications may prolong
survival, adding to the benefit of establishing a diagnosis. If the history and
physical findings are suggestive of MELAS, lactic acid and pyruvate may
be used as screening tests, after first eliminating the more common causes
of lactic acidosis, such as tissue hypoxic-ischaemic injury, hyperglycaemia,
and hypoglycaemia. The more uncommon amino acid and fatty acid meta-
bolic disorders also need to be considered. Arterial lactate and pyruvate are
high and Cerebrospinal Fluid (CSF) lactate also may be high. Lactate and
pyruvate may increase substantially with exercise. Lactate/pyruvate ratio
may be increased. The increased lactate-to-pyruvate ratio is observed in the
face of a normal O2 saturation, as opposed to tissue-injury lactic acidosis in
which the increased ratio is associated with decreased O2 saturation. MRI
studies show hyperintense T2 lesions predominantly in the grey and sub-
cortical white matter in the temporal, parietal, and occipital lobes. Lesions
spare the deep white matter and cross vascular boundaries. Basal ganglia
calcifications and atrophy also are reported. Generalised cerebral atrophy
is frequent. Serial MRI studies often demonstrate lesion resolution, dif-
ferentiating these lesions from typical ischaemic strokes.
• By using MR spectroscopy, several groups have shown that lactic acid lev-
els in the brain parenchyma and ventricles may be increased during the
acute phase of the disease and in chronic lesions. Findings of non-invasive

2
Case 01

and cerebral angiographic studies are generally normal or show focal cap-
illary blush or early venous filling in affected cortical regions
• Single-Photon Emission Computed Tomography (SPECT) and Positron
emission Tomography (PET) studies have been reported variably to show
normal or increased (and occasionally diminished) cerebral blood flow to
regions structurally abnormal on CT scan and MRI. Metabolic PET stud-
ies demonstrate focally deranged metabolic states
• Electroencephalography is often performed when seizures are a concern.
This is especially necessary in MELAS, since patients occasionally have
intractable status epilepticus as a terminal condition
• Electrocardiography may reveal pre-excitation or incomplete heart block.
Echocardiography may demonstrate cardiomyopathy
• Muscle biopsy demonstrates ragged red fibres on modified Gomori
trichrome stain in at least 85% of cases. Ragged red fibres, common to
MELAS, Myoclonic Epilepsy with Ragged Red Fibres (MERRF), Kearns-
Sayer, and overlap syndromes, reflect proliferation of abnormal mito-
chondria under the sarcolemma. Negative muscle biopsy findings do not
preclude consideration of this syndrome. Pathologic examination of brain
tissues demonstrates multiple cortical and subjacent white matter ischae-
mic regions, spongiform degeneration of the cortex, and calcium deposi-
tion in the capillary walls of the globus pallidus. Endothelial and smooth
muscle cells of pial arterioles and small arteries exhibit increased numbers
of structurally abnormal mitochondria, and capillary lumens are narrowed
due to endothelial hypertrophy
• Mitochondrial DNA analysis. Mitochondrial DNA analysis is now availa-
ble commercially from several sources to identify the mutations responsi-
ble for this disease. At least 30 different point mutations have been associ-
ated with MELAS. The most common defect, present in 80% of patients,
is a point mutation at nucleotide position 3243 in the transfer RNA for
leucine. A number of other point mutations and double point mutations
have also been associated with MELAS
• A multidisciplinary team approach is needed in the diagnosis and further
care of patients with MELAS. Genetic testing and counseling should be
offered to family members. The differential diagnosis of the syndrome
of mitochondrial encephalomyopathy, lactic acidosis, and stroke like epi-
sodes (MELAS) includes the following:
• Basilar Artery Thrombosis
• ADEM (Acute Disseminated Encephalomyelitis)
• Cerebral Venous Thrombosis
• Childhood Migraine Variants
• Inherited Metabolic Disorders: Organic acidemias
• Migraine Headache
• Moyamoya Disease

3
Child Neurology

• Multiple Sclerosis
• Posterior Cerebral Artery Stroke

MANAGEMENT
A number of agents have been tried experimentally in patients with MELAS.
Trials of coenzyme Q10, idebenone, dichloroacetate, cytochrome C,
L-carnitine, L-arginine, and various B vitamins all have been reported in small
groups of patients as short-and long-term treatments.
• Coenzyme Q10 levels of coenzyme Q10 are not reduced in patients suf-
fering from MELAS; its therapeutic benefit is presumed to be due to the
increase in production of ATP at the inner mitochondrial membrane.
Some success with use of coenzyme Q10 at a dose of 4 mg/kg/day has
been reported. Idebenone (an analogue of coenzyme Q10) has been used
in a few patients.
• Both intravenous formulations and oral formulations of dichloroacetate
have been used in the acute treatment of stroke like episodes, as well as
in long-term prophylaxis of stroke in patients with MELAS. The possible
therapeutic effect of dichloroacetate was thought to be mediated via re-
duction in lactate levels in blood and the brain.
• Repeated intravenous injections of cardiochrome have been reported to
produce similar biochemical and clinical improvement, in terms of de-
creased number of stroke like episodes. Cardiochrome, a combination of
cytochrome C and vitamins B-1 and B-2, may increase the effectiveness of
the electron transport chain of the inner mitochondrial membrane.
• Valproic acid should not be used as an anti-epileptic. Its detrimental effect
on mitochondria has been reported to exacerbate seizures and to precipi-
tate stroke like episodes in patients with MELAS.

SUPPORTIVE CARE
The general supportive care measures used in acute stroke syndromes also
should be followed. Death in patients with MELAS is usually the result of
cardiac failure, pulmonary embolus, or renal failure. Status epilepticus can oc-
casionally be fatal; seizures should be treated aggressively.

CONSULTATIONS
• A neurologist should be consulted for diagnosis and care of a patient with
stroke like episodes, seizures, and encephalopathy.
• Advice on the proper handling and processing of a muscle biopsy should
be sought from a pathologist or neuromuscular disease specialist.
• If psychosis is present, the patient may benefit from psychiatric consultation.
• Post stroke rehabilitation needs are best addressed through consultations
with a physical therapist, occupational therapist, speech-language patholo-
gist, and rehabilitation specialist (neurologist or psychiatrist).

4
Case 01

• Transfer to a tertiary care centre with multispecialty facilities is appropriate


for patients suspected of having MELAS, particularly if muscle biopsy and
MRI facilities are not readily available for diagnosis.
• Immediate life-threatening issues, such as acidosis, seizures, pulmonary
embolus, and cardiac arrhythmias, may need to be addressed locally prior
to transfer.

REFERENCES
1. Meseguer S, Martínez-Zamora A, García-Arumí E, Andreu AL, Armengod ME.
The ROS-sensitive microRNA-9/9* controls the expression of mitochondrial
tRNA-modifying enzymes and is involved in the molecular mechanism of ME-
LAS syndrome. Hum Mol Genet. 2014 Aug 22.
2. Testai FD, Gorelick PB. Inherited metabolic disorders and stroke part 1: Fabry dis-
ease and mitochondrial myopathy, encephalopathy, lactic acidosis, and strokelike
episodes. Arch Neurol. 2010 Jan. 67(1):19-24.
3. Scarpelli M, Zappini F, Filosto M, Russignan A, Tonin P, Tomelleri G. Mitochon-
drial Sensorineural Hearing Loss: A Retrospective Study and a Description of
Int.. 2012. 2012:287432.
Cochlear Implantation in a MELAS Patient. Genet Res Int
4. Singmaneesakulchai S, Limotai N, Jagota P, Bhidayasiri R. Expanding spectrum of
abnormal movements in MELAS syndrome (mitochondrial encephalomyopathy,
lactic acidosis, and stroke-like episodes). Mov Disord.
Disord. 2012 Oct. 27(12):1495-7.
5. Fayssoil A. Heart diseases in mitochondrial encephalomyopathy, lactic acidosis,
and stroke syndrome. Congest Heart Fail. 2009 Nov-Dec. 15(6):284-7.
6. Ciafaloni E, Ricci E, Shanske S, et al.
al. MELAS: clinical features, biochemistry, and
molecular genetics. Ann Neurol. 1992 Apr. 31(4):391-8.
7. Kaufmann P, Engelstad K, Wei Y, et al.. Dichloroacetate causes toxic neuropathy in
MELAS: a randomized, controlled clinical trial. Neurology
Neurology. 2006 Feb 14. 66(3):324-
30.
8. Koga Y, Akita Y, Nishioka J, et al.. L-arginine improves the symptoms of strokelike
episodes in MELAS. Neurology.
Neurology. 2005 Feb 22. 64(4):710-2.
9. Matsumoto J, Saver JL, Brennan KC, Ringman JM. Mitochondrial encephalo-
myopathy with lactic acidosis and stroke (MELAS). Rev Neurol Dis. 2005 Winter.
2(1):30-4.
10. Scaglia F, Northrop JL. The mitochondrial myopathy encephalopathy, lactic acido-
sis with stroke-like episodes (MELAS) syndrome: a review of treatment options.
CNS Drugs.
Drugs 2006. 20(6):443-64.
11. Tay SH, Nordli DR Jr, Bonilla E, Null E, Monaco S, Hirano M. Aortic rupture in
mitochondrial encephalopathy, lactic acidosis, and stroke-like episodes. Arch Neu-
rol. 2006 Feb. 63(2):281-3.
12. Thambisetty M, Newman NJ, Glass JD, Frankel MR. A practical approach to the
diagnosis and management of MELAS: case report and review. Neurologist. 2002
Sep. 8(5):302-12.

5
02 Case
A 4-year-old healthy boy presented with a 3-day history of sore throat,
non-productive cough, decreased activity and worsening diffuse pain
in the anterior part of the thighs, buttocks and calf muscles. Four days prior,
his complaints started as pain in the legs and now the pain is more diffuse
throughout the body. Now he refuses to walk even though he is able to. Next,
it prevented him from walking up stairs. In the Emergency Room, he was
afebrile and appearing non-toxic, and general paediatric examination was
remarkable only for hoarseness and mild tachycardia. Generalised weakness
and diffuse pain were noted but no other specified finding.
On physical examination and hospital course: Meningism was suspected
and Cerebrospinal Fluid (CSF) examination revealed two WBC/mm3, 298
RBC/mm3 , total protein of 117 mg/dL (range 15-45) and a glucose level of
73 mg/dL (range 60-80). Extensive laboratory studies including an infectious
work-up were negative. During hospital stay on day 3 to 4, progressive
weakness developed with worsening head control and reduced truncal
strength, accompanied by weakness of the arms and legs. His limb pain was
minimally responsive to non-steroidal anti-inflammatory drugs. He was
intubated for 3 days because of progressive respiratory failure.
A neurology consultation was obtained, revealing an alert patient with
symmetrically flaccid limbs, grade two power in all muscle groups. Deep
Tendon Reflexes were absent, and there was no Babinski sign. In the Intensive
Care Unit (ICU), he developed mild dysautonomia including intermittent
tachycardia, bradycardia.

Answer the following questions:


• Give the most likely diagnosis.
• Briefly summarise the case.
• Provide a differential diagnosis.
• Discuss an appropriate diagnostic work-up.
• Discuss the management of the patient.

6
Case 02

DIAGNOSIS
Guillain-Barré Syndrome (GBS)

SUMMARY
A previously healthy boy presented with a 3-day history of sore throat, non-
productive cough, decreased activity and worsening diffuse pain in the legs.
He developed progressive weakness with worsening head control and re-
duced truncal strength, accompanied by weakness of the arms and the legs.
Examination revealed an alert patient with symmetrically flaccid limbs. Deep
tendon reflexes were absent, and there was no Babinski sign.

DIFFERENTIAL DIAGNOSIS
Guillain-Barré Syndrome (GBS)
Guillain-Barré Syndrome (GBS) can be described as a collection of clinical
syndromes that manifests as an acute inflammatory polyradiculoneuropathy
with resultant weakness and diminished reflexes.
Although the classic description of GBS is that of a demyelinating neuropathy
with ascending weakness, many clinical variants have been well documented
in medical literature. The typical patient with GBS, which in most cases will be
Acute Inflammatory Demyelinating Polyradiculoneuropathy (AIDP), presents
2-4 weeks following a relatively benign respiratory or gastrointestinal illness
with complaints of finger dysesthesias and proximal muscle weakness of the
lower extremities. The weakness may progress over hours to days to involve
the arms, truncal muscles, cranial nerves, and muscles of respiration. The clas-
sic clinical picture of weakness is ascending and symmetrical in nature. The
lower limbs are usually involved before the upper limbs. Proximal muscles
may be involved earlier than the more distal ones. Trunk, bulbar, and respira-
tory muscles can be affected as well. Weakness develops acutely and progresses
over days to weeks. Severity may range from mild weakness to complete tetra-
plegia with ventilatory failure.
Most patients complain of paraesthesia, numbness, or similar sensory changes.
Paraesthesia mostly begins in the toes and fingertips, progressing upward, but
generally not extending beyond the wrists or ankles. Pain associated with GBS
is most severe in the shoulder girdle, back, buttocks, and thighs and may occur
with even the slightest movements. The pain is often described as aching or
throbbing in nature. Patients may be unable to stand or walk despite reason-
able strength, especially when ophthalmoparesis or impaired proprioception is
present. Respiratory muscle weakness with shortness of breath may be present.
Common complaints associated with cranial nerve involvement in GBS in-
clude the following: Facial droop (may mimic Bell palsy), Diplopia, Dysarthria,
Dysphagia, Ophthalmoplegia, and Pupillary disturbances.

7
Child Neurology

Autonomic changes in GBS can include the following:


• Tachycardia
• Bradycardia
• Facial flushing
• Paroxysmal hypertension
• Orthostatic hypotension
• Anhidrosis and/or diaphoresis
• Urinary retention
Typical respiratory complaints in GBS include the following:
• Dyspnoea on exertion
• Shortness of breath
• Difficulty swallowing
• Slurred speech
Ventilatory failure with required respiratory support occurs in up to one-third
of patients at some time during the course of their disease. Sensory symptoms
are usually mild. In most cases, objective findings of sensory loss tend to be
minimal and variable. Dysesthetic symptoms are observed in approximately
50% of patients during the course of their illness. Dysesthesia frequently is de-
scribed as burning, tingling, or shocklike sensations and are often more preva-
lent in the lower extremities than in the upper extremities. Dysesthesia may
persist indefinitely in 5-10% of patients.
• Polymyositis. Presence of pain and muscle tenderness usually in the
shoulder and upper arm, involvement of flexor neck muscle dispropor-
tionate to limb weakness, absence of sensory symptoms, preservation of
reflexes, absence of dysautonomia.
• Transverse verse myelitis. Spinal cord disorders including transverse my-
elitis present with asymmetrical motor or sensory loss usually involving
the lower extremities, early bowel or bladder dysfunction with persistent
incontinence, and segmental radicular pain. Physical examination dem-
onstrates upper motor neuron signs (hyperreflexia, positive Babinski’s
response) and a sensory level.
• Myasthenia gravis. Early involvement of muscle groups including
the extra-ocular, levator, pharyngeal jaw, neck, and respiratory muscles.
Sometimes presents without limb weakness. Excessive fatigability and a
variation of symptoms and signs throughout the day is common. Reflexes
are preserved, and sensory features, dysautonomia, and bladder dysfunc-
tion are absent.
• Lambert Eaton Myasthenic Syndrome (LEMS). Can be difficult to
differentiate because of similar clinical characteristics. However, some
characteristics are more typical for LEMS. These include slower develop-
ment of clinical symptoms, dry mouth, lack of objective sensory loss, rare
involvement of the respiratory muscle group, and potentiation of reflexes
after exercise or contraction.

8
Case 02

• Botulism. History of ingesting food tainted with botulinum toxin.


Descending paralysis begins in the bulbar muscles then the limbs, face,
neck, and respiratory muscles. Respiratory muscles are involved with mild
limb weakness, and reflexes are usually preserved. Ptosis, dilated non-
reactive pupils are present. Dilated non-reactive pupils are uncommon in
GBS, but more common in botulism. Constipation is also a characteristic
feature of botulism.
• Acute poliomyelitis a disease of the anterior horn motor neurons of the
spinal cord and brain stem caused by the poliovirus. Flaccid asymmetric
weakness and muscle atrophy are the hallmarks of its clinical manifesta-
tions, due to loss of motor neurons and denervation of their associated
skeletal muscles. Usually asymmetric proximal weakness is present with
more involvement of the lumbar than the cervical segments and more
spinal cord than brainstem segments. The trunk muscles are affected least
and sensations should be within normal limits objectively. Deep tendon
reflexes are diminished or absent. Atrophy of muscle may be detected 3
weeks after onset of paralysis, which becomes maximal at 12-15 weeks and
remains permanent.
• Vasculitic neuropathy. Common features include painful asymmetrical
presentation of muscle weakness, uncommon involvement of cranial nerves,
respiratory paralysis, and sphincter dysfunction. Usually patients complain of
fever, fatigue, weakness, and arthralgia.
The following spinal cord lesions may be considered in the differential
diagnosis:
• Epidural abscess
• Tumours
• Enteroviral infections of the anterior horn cells
• Vascular malformations
• Cord infarctions
• Cord compression
• Trauma
Peripheral neuropathies from the following may produce a GBS-like picture:
• Vincristine Toxicity
• Glue sniffing
• Heavy metals
• Organophosphate pesticides
• Diphtheria
• Lyme disease
• Inborn errors of metabolism
• Leigh disease
• Porphyria

9
Child Neurology

DIAGNOSTIC WORK-UP
• Guillain-Barré Syndrome (GBS) is generally diagnosed on clinical
grounds.
• Electromyography (EMG) and Nerve Conduction Studies (NCS) can
be very helpful in the diagnosis. Abnormalities in NCS that are consist-
ent with demyelination are sensitive and represent specific findings for
classic GBS. Delayed distal latencies, slowed nerve conduction velocities,
temporal dispersion of waveforms, conduction block, prolonged or absent
F waves, and prolonged or absent H-reflexes are all the findings that sup-
port demyelination. Needle EMG may be normal in acute nerve lesions,
and it may take 3-4 weeks for fibrillation to develop. In the acute phase,
the only needle EMG abnormality may be abnormal motor recruitment,
with decreased recruitment and rapid firing motor units in weak muscles.
Unfortunately, electrodiagnostic studies can be completely normal in acute
GBS and a normal study does not rule GBS.
• Lumbar puncturencture for Cerebrospinal Fluid (CSF) studies is recommend-
ed. During the acute phase of GBS, characteristic findings on CSF analy-
sis include albuminocytologic dissociation, which is an elevation in CSF
protein (>0.55 g/L) without an elevation in white blood cells. The in-
crease in CSF protein is thought to reflect the widespread inflammation
of the nerve roots.
• Imaging studies, such as Magnetic Resonance Imaging (MRI) and
Computed Tomography (CT) scanning of the spine, may be more helpful
in excluding other diagnoses, such as mechanical causes of myelopathy,
than in assisting in the diagnosis of GBS.
• However, MRI findings in Guillain–Barré syndrome are surface thickening
and contrast enhancement on the conus medullaris and on the nerve roots of
the cauda equine. The most common site of enhancement in Guillain–Barré
syndrome is considered to be the anterior nerve roots, although enhancement
of the posterior nerve roots is also seen. Although cranial nerve roots can also
be affected but the facial nerve (CN VII) is the most commonly affected.
• Serum autoantibodies are not measured routinely in the work-up of GBS,
but results may be helpful in patients with a questionable diagnosis or a
variant of GBS. Anti-bodies to glycolipids are observed in the sera of 60-
70% of patients with GBS during the acute phase, with gangliosides being
the major target antigens.
Specific anti-bodies found in association with GBS include the following:
• Anti-bodies to GM1: Frequently found in the sera of patients with the
Acute Motor Axonal Neuropathy (AMAN) or Acute Inflammatory
Demyelinating Polyradiculoneuropathy (AIDP) variants of GBS. Elevated
titers are closely associated antecedent C jejuni infections.
• Anti-GQ1b anti-bodies: Found in patients with GBS with ophthalmople-
gia, including patients with the Miller-Fisher variant. Other anti-bodies
to different major and minor gangliosides also have been found in GBS
patients.

10
Case 02

MANAGEMENT OF GBS
• Patients who are diagnosed with GBS should be admitted to a hospital for
close monitoring until it has been determined that the course of the dis-
ease has reached a plateau or undergone reversal usually 2 weeks or more.
• Although the weakness may initially be mild and non-disabling, symp-
toms can progress rapidly over just a few days. Continued progression
may result in a neuromuscular emergency with profound paralysis, res-
piratory insufficiency, and/or autonomic dysfunction with cardiovascular
complications.
• Approximately one-third of patients require admission to an ICU, pri-
marily because of respiratory failure. Patients with persistent functional
impairments may need to be transferred to an inpatient rehabilitation unit
• Early recognition and treatment of GBS also may be important in the
long-term prognosis.
• Immunomodulatory treatment has been used to hasten recovery.
Intravenous Immunoglobulin (IVIG) 2 Gm/kg/over 2-5 days and plasma
exchange have proved equally effective.
• Patients should be monitored closely for changes in blood pressure, heart
rate, and arrhythmias. Treatment is rarely needed for tachycardia. Atropine
is recommended for symptomatic bradycardia.
• Because of the lability of dysautonomia, hypertension is best treated with
short-acting agents, such as a short-acting beta blocker or nitroprusside.
Hypotension from dysautonomia usually responds to intravenous fluids
and supine positioning. Temporary pacing may be required for patients
with second-and third-degreee heart block.
• Intubation should be performed on patients who develop any degree of
respiratory failure. Clinical indicators for intubation in the ED include the
following: Hypoxia, rapidly declining respiratory function, poor or weak
cough, suspected aspiration and typically, intubation is indicated when the
Forced Vital Capacity (FVC) is less than 15 ml/kg.
Regular spirometry, respiratory rate monitoring and blood gas analysis pe-
riodically are good tool for assessment of the need for intubation.
• Prevention of pressure sores and contractures entails careful positioning,
frequent postural changes, and daily Range-of-Motion (ROM) exercises.
• Although bowel and bladder dysfunction is generally transitory, manage-
ment of these functions is needed to prevent other complications.
• Patients can take months and even years to recover. Most people with
Guillain-Barré syndrome experience that recovery begins, usually lasting
six to 12 months, though for some people it could take as long as three
years. Children, generally recover more completely than adults.
• Consequently, physical and occupational therapy should be instituted
early in the disease course to promote mobility and strength building. In
addition, as pain may be a significant symptom, involvement of a pain-
management team may be helpful.

11
Child Neurology

REFERENCES
1. Lee JH, Sung IY, Rew IS. Clinical presentation and prognosis of childhood Guil-
lain-Barré syndrome. J Paediatr Child Health. 2008 Jul-Aug. 44(7-8):449-54.
2. Souayah N, Nasar A, Suri MF, Qureshi AI. Guillain-Barré syndrome after vaccina-
tion in United States: data from the Centres for Disease Control and Prevention/
Food and Drug Administration Vaccine Adverse Event Reporting System (1990-
2005). J Clin Neuromuscul Dis. 2009 Sep. 11(1):1-6.
3. Nachamkin I, Arzarte Barbosa P, Ung H, Lobato C, Gonzalez Rivera A, Rodriguez
P, et al.. Patterns of Guillain-Barre syndrome in children: results from a Mexican
population. Neurology. 2007 Oct 23. 69(17):1665-71.
4. Kalra V, Chaudhry R, Dua T, Dhawan B, Sahu JK, Mridula B. Association of
Campylobacter jejuni infection with childhood Guillain-Barré syndrome: a case-
control study. J Child Neurol.. 2009 Jun. 24(6):664-8.
5. Korinthenberg R, Schessl J, Kirschner J. Clinical presentation and course of child-
hood Guillain-Barré syndrome: a prospective multicentre study. Neuropediatrics.
2007 Feb. 38(1):10-7.
6. Shafqat S, Khealani BA, Awan F, Abedin SE. Guillain-Barré syndrome in Pa-
kistan: similarity of demyelinating and axonal variants. Eur J Neurol.. 2006 Jun.
13(6):662-5.
7. Kalra V, Sankhyan N, Sharma S, Gulati S, Choudhry R, Dhawan B. Outcome in
childhood Guillain-Barré syndrome Indian J Pediatr
Pediatr.. 2009 Aug. 76(8):795-9.
8. Roodbol J, de Wit MC, Walgaard C, de Hoog M, Catsman-Berrevoets CE, Jacobs
BC. Recognizing Guillain-Barre syndrome in preschool children. Neurology
Neurology. 2011
Mar 1. 76(9):807-10.
9. Gorson KC, Ropper AH, Muriello MA, Blair R. Prospective evaluation of MRI
lumbosacral nerve root enhancement in acute Guillain-Barré syndrome. Neurology.
1996 Sep. 47(3):813-7.
10. Nishimoto Y, Susuki K, Yuki N. Serologic marker of acute motor axonal neuropa-
thy in childhood. Pediatr Neurol.
Neurol. 2008 Jul. 39(1):67-70.
11. Schessl J, Koga M, Funakoshi K, Kirschner J, Muellges W, Weishaupt A, et al. Pro-
spective study on anti-ganglioside anti-bodies in childhood Guillain-Barré syn-
drome. Arch Dis Child
Child.. 2007 Jan. 92(1):48-52.
12. Roodbol J, de Wit MC, Aarsen FK, Catsman-Berrevoets CE, Jacobs BC. Long-
term outcome of Guillain-Barré syndrome in children. J Peripher Nerv Syst. 2014
Jun. 19(2):121-6.
13. Hughes RA, Wijdicks EF, Barohn R, Benson E, Cornblath DR, Hahn AF, et al.
Practice parameter: immunotherapy for Guillain-Barré syndrome: report of the
Quality Standards Subcommittee of the American Academy of Neurology. Neurol-
ogy. 2003 Sep 23. 61(6):736-40.
14. Korinthenberg R, Schessl J, Kirschner J, Mönting JS. Intravenously administered
immunoglobulin in the treatment of childhood Guillain-Barré syndrome: a rand-
omized trial. Pediatrics. 2005 Jul. 116(1):8-14.
15. Baranwal AK, Ravi RN, Singh R. Exchange transfusion: a low-cost alterna-
tive for severe childhood Guillain-Barré syndrome. J Child Neurol. 2006 Nov.
21(11):960-5.

12
Case 03
A 3-year-old unimmunised girl was admitted with complaints of a ten-
day history of purulent left ear discharge, inability to open the mouth,
and neck stiffness of five days duration associated with cough and fever of two
days prior to presentation. No previous history of supportive otitis media. She
never received a primary immunisation vaccine due to parental negligence.
Physical examination revealed an acutely ill-looking febrile girl with
resistance in opening the mouth, neck stiffness, and frequent contact spasms
of the facial muscles. There were no obvious spasms in other parts of the
body. She was conscious, with left VII cranial nerve palsy, of lower motor
neuron lesion. Left ear examination revealed purulent discharge associated
with poor visualisation of the tympanic membrane. She was dysphonic with
wide spread coarse crepitation on chest auscultation. Her gag was present.
The tongue was midline. She had normal bulk, tone and strength 5/5 of all
her muscle groups. Deep tendon reflexes were 2+ throughout with bilateral
plantar flexor responses.

Answer the following questions:


• Give the most likely diagnosis.
• Localise the examination findings.
• Provide the differential diagnosis.
• Discuss an appropriate diagnostic work-up.
• Discuss the management of the patient.

13
Child Neurology

DIAGNOSIS
Cephalic Tetanus

SUMMARY
A 3-year-old girl admitted with Suppurative Otitis Media (SOM) and an
inability to open the mouth, and neck stiffness of five days. She never re-
ceived primary immunisation with a tetanus vaccine. The biochemistry and
microscopic profile of the cerebrospinal fluid was not suggestive of bacterial
meningitis.

Localisation in Tetanus
Muscle rigidity and spasms ensue, often manifesting as trismus/lockjaw, dys-
phagia, opisthotonus, or rigidity and spasms of the respiratory, laryngeal, and
abdominal muscles. Thus, a constellation of the typical symptoms points to a
disorder in this patient, release of neurotransmitters, and blocking inhibitor
impulses. This leads to unopposed muscle contraction and spasm so it seems
more likely cephalic tetanus.

DIFFERENTIAL DIAGNOSIS
The incubation period ranges from 3 to 21 days, usually about 10 days. In gen-
eral, the further the injury site is from the central nervous system, the longer
the incubation period. A shorter incubation period is associated with more se-
vere disease, complications, and a higher chance of death. In neonatal tetanus,
symptoms usually appear from 4 to 14 days after birth, averaging about 7 days.
Tetanus may be categorised into the following 4 clinical types:
• Generalised tetanus
• Localised tetanus
• Cephalic tetanus
• Neonatal tetanus
Approximately 50-75% of patients with generalised tetanus present with tris-
mus (“lockjaw”), which is the inability to open the mouth secondary to mas-
seter muscle spasm. Nuchal rigidity and dysphagia are also early complaints
that cause risus sardonicus, the scornful smile of tetanus, resulting from facial
muscle involvement.
Cephalic tetanus is a rare form of localised tetanus defined as trismus plus
paralysis of one or more cranial nerves and accounts for 1-3 percent of the
total number of reported cases of tetanus. Cephalic tetanus occasionally fol-
lows SOM, as with our patient; or craniofacial injuries. About two-thirds of
cephalic tetanus cases progress to generalised tetanus with bad prognosis. In
those who do not progress to generalised tetanus, the prognosis is good. The
role of SOM as a portal of entry in post-neonatal tetanus has been elucidated
through many studies.

14
Case 03

• Strychnine poisoning is the only condition that truly mimics tetanus.


However, a number of conditions (e.g., dental or other local infections,
hysteria, neoplasms, and encephalitis) may cause trismus, and these must
be differentiated from the conditions from tetanus. The following condi-
tions listed do not cause manifestations of tetanus other than trismus:
• Dental infections
• Neoplasms
• Malignant hyperthermia
• Stimulant use
• Hepatic encephalopathy
• Intracranial haemorrhage
• Dystonic drug reactions (e.g., phenothiazines, metoclopramide)
• Seizure disorder (partial or generalised)
• Serotonin syndrome

DIAGNOSTIC WORK-UP
• Tetanus diagnosis is strictly clinical; there are no confirmatory laboratory
tests.
• The WHO definition of tetanus requires at least one of the following
signs: trismus (inability to open the mouth) or risus sardonicus (sustained
spasm of the facial muscles); or painful muscular contractions. Although
this definition requires a history of injury or a wound, tetanus may also
occur in patients who are unable to recall a specific wound or injury.

MANAGEMENT
Treatment consists of nursing care, aural toileting, sedation, muscle relaxation,
and anti-biotics, with good outcome.
• General measures: If possible a separate ward/location should be desig-
nated for tetanus patients. Patients should be placed in a quiet shaded area
and protected from tactile and auditory stimulation as much as possible.
All wounds should be cleaned and debrided as indicated.
• Immunotherapy: If available, administer human TIG 500 units by in-
tramuscular injection or intravenously (depending on the available prepa-
ration) as soon as possible; in addition, administer age-appropriate TT-
containing vaccine, 0.5 cc by intramuscular injection at a separate site.
[Tetanus disease does not induce immunity; patients without a history of
primary TT vaccination should receive a second dose 1-2 months after the
first dose and a third dose 6-12 months later].
• Anti-biotic treatment: Metronidazole is preferred (30 mg/kg/day PO/
IV q6h First Dose: 7.5 mg/kg PO/IV x 1 (Max: 4 g/day). Penicillin G
(100,000-200,000 IU/kg/day intravenously, given in 2-4 divided doses).

15
Child Neurology

Tetracyclines, macrolides, clindamycin, cephalosporins and chloram-


phenicol are also effective.
• Muscle spasm control: Benzodiazepines are preferred. In children, start
with doses of 0.1-0.2 mg/kg every 2-6 hours, titrating upward as needed).
Oral preparations could be used but must be accompanied by careful mon-
itoring to avoid respiratory depression or arrest. Magnesium sulphate can
be used alone or in combination with benzodiazepines to control spasms
and autonomic dysfunction: 5 gm (or 75 mg/kg) intravenous loading dose,
then 2-3 grams per hour until spasm control is achieved. To avoid over-
dose, monitor patellar reflex as areflexia (absence of patellar reflex) oc-
curs at the upper end of the therapeutic range (4 mmol/L). If areflexia
develops, dose should be decreased. Other agents used for spasm control
include baclofen, dantrolene (1-2 mg/kg intravenous or by mouth every 4
hours), barbiturates, preferably short-acting (100-150 mg every 1-4 hours
in adults; 6-10 mg/kg in children; by any route), and chlorpromazine (50-
150 mg by intramuscular injection every 4-8 hours in adults; 4-12 mg
every by intramuscular injection every 4-8 hours in children).
• Autonomic dysfunction control: Magnesium sulphate as above; or mor-
phine. Note: β-blockers
-blockers such as propranolol were used in the past but can
cause hypotension and sudden death; only esmolol is currently recommended.
• Airway/respiratory control: Drugs used to control spasm and provide
sedation can result in respiratory depression. If mechanical ventilation is
available, this is less of a problem; if not, patients must be carefully moni-
tored and medication doses adjusted to provide maximal spasm and auto-
nomic dysfunction control while avoiding respiratory failure. If spasm, in-
cluding laryngeal spasm, is impeding or threatening adequate ventilation,
mechanical ventilation is recommended when possible. Early tracheos-
tomy is preferred as endotracheal tubes can provoke spasm and exacerbate
airway compromise.
• Adequate fluids and nutrition should be provided, as tetanus spasms
result in high metabolic demands and a catabolic state. Nutritional sup-
port will enhance chances of survival. Prior to the availability of a vaccine
and mechanical ventilation (during the 1920s-30s), careful monitoring
and nursing care improved survival. If patients can be supported through
one to two weeks of spasm and other complications, the chances of com-
plete recovery greatly increase, particularly in non-elderly and previously
healthy patients.

How is Tetanus Prevented?


The Centre for Disease Control and Prevention (CDC) recommends that
children need 5 DTaP shots. A DTaP shot is a combination vaccine that pro-
tects against 3 diseases: diphtheria, tetanus, and pertussis.
• The first 3 shots are given at 2, 4, and 6 months of age.
• Between 15 and 18 months of age, the fourth shot is given.

16
Case 03

• A fifth is given when a child enters school at 4 to 6 years of age.


• At regular checkups for 11-or 12-year-olds, a preteen should get a dose of
Tdap. The Tdap booster also contains tetanus, diphtheria, and pertussis.
• If an adult did not get a Tdap as a preteen or teen, then he or she should
get a dose of Tdap instead of the Td booster. Adults should then get a Td
booster every 10 years, but it can be given before the 10-year mark.

REFERENCES
1. Centres for Disease Control and Prevention (CDC). Tetanus surveillance United
States, 2001-2008. MMWR Morb Mortal Wkly Rep 2011; 60:365.
2. Afshar M, Raju M, Ansell D, Bleck TP. Narrative review: tetanus-a health threat
after natural disasters in developing countries. Ann Intern Med 2011; 154:329.
3. American Academy of Pediatrics. Tetanus (lockjaw). In: Red Book: 2009 Report of
the Committee on Infectious Diseases, 28th, American Academy of Pediatrics, Elk
Grove Village, IL 2009. p.655.
4. Thwaites CL, Yen LM, Loan HT, et al.. Magnesium sulphate for treatment of severe
tetanus: a randomised controlled trial. Lancet 2006; 368:1436.
5. Ceneviva GD, Thomas NJ, Kees-Folts D. Magnesium sulfate for control of mus-
cle rigidity and spasms and avoidance of mechanical ventilation in pediatric teta-
nus. Pediatr Crit Care Med 2003; 4:480.
6. Tiwari TSP. Manual for the Surveillance of Vaccine-Preventable Diseases. Chapter
16: Tetanus. Centres for Disease Control and Prevention. Available at http://www.
cdc.gov/vaccines/pubs/surv-manual/chpt16-tetanus.html. April 1, 2014; Accessed:
June 16, 2016.
7. Ceneviva GD, Thomas NJ, Kees-Folts D. Magnesium sulfate for control of mus-
cle rigidity and spasms and avoidance of mechanical ventilation in pediatric teta-
nus. Pediatr Crit Care Med. 2003 Oct. 4(4):480-4.
8. Thwaites CL, Yen LM, Loan HT, Thuy TT, Thwaites GE, Stepniewska K, et al.
Magnesium sulphate for treatment of severe tetanus: a randomised controlled trial.
Lancet. 2006 Oct 21. 368(9545):1436-43.

17
04 Case
A twenty-day-old baby girl was admitted with refractory seizures since
the second day of life. Following an uncomplicated pregnancy, she
was born at term by emergency caesarean section due to failure to progress.
Her birth weight was 3220g and Apgar scores were normal and there was no
family history of neonatal seizures. She was having frequent, brief, extensor
tonic spasms in the sleep state initially that occurred many times (15-35/day)
increasing in frequency in spite of multiple anti-convulsants.
On physical examination. She was a well-developed, well-nourished girl
and her general physical exam was unremarkable. During the examination
it was noted that she was having frequent, brief, extensor tonic spasms in
the awake and sleep states. She appeared to be awake but she did not fix and
follow. She was brachycephalic, diffusely hypotonic, and hyporeflexic, but no
other abnormalities were noted.
Her EEG was grossly abnormal with bursts of moderate to high amplitude
irregular spike waves followed by prolonged segments of background
attenuation giving rise to typical burst suppression pattern. The inter-burst
intervals ranged between 6-15 seconds. A video/EEG evaluation showed
Suppression-Burst (SB) in the awake and sleep states. The extensor spasms
were associated with attenuation of the SB pattern.
MRI scan was normal. She had a modest improvement with a combination
of topiramate and phenobarbital and was able to be discharged home.
Adrenocorticotrophic hormone also tried but no benefit. At five months of
age there was a change in the character of her seizures from tonic extensor
spasms to flexor spasms. The spasms still occurred in clusters but now were
noted to be upon awakening in the morning and after a nap, and no longer
in sleep. In spite of multiple medication changes, she continued to have
occasional infantile spasms until 14 months of age when, again, the character
of the seizures gradually changed. The flexor spasms diminished and finally
stopped, but were replaced by multiple-seizure types including tonic seizures,
atypical absence spells, and generalised tonic-clonic seizures. A repeat EEG
showed 2 Hz spike and wave discharges on a slow background.

Answer the following questions:


• What is the most likely diagnosis?
• Provide appropriate differential diagnosis.
• Discuss diagnostic work-up.
• Describe the management strategy.

18
Case 04

DIAGNOSIS
Early Infantile Epileptic Encephalopathy (EIEE), Ohtahara syndrome

SUMMARY
A new born admitted with refractory convulsions since the second day of life.
She was having frequent, brief, extensor tonic spasms initially but later on, at
five months of age there was a change in the character of her seizures from
tonic extensor spasms to flexor spasms. She was brachycephalic, diffusely hy-
potonic, and hyporeflexic, but no other abnormalities were noted. A video/
EEG evaluation showed Suppression-Burst (SB) in the awake and sleep states.
In spite of multiple medications and even ACTH failed to control the seizures,
she continued to have occasional infantile spasms until 14 months of age.

DIFFERENTIAL DIAGNOSIS
Onset of EIEE occurs within the first 3 months of life but some present within
the first few weeks after birth. Neonates have poor suckling reflexes, hypo-
tonia and manifest with generalised and symmetrical tonic spasms that can
appear in clusters or singly and can last for up to 10 seconds. The pattern of
these spasms remains unchanged during wakefulness and sleep and they can
occur hundreds of times per day. Other seizure types, including generalised
tonic-clonic seizures, focal motor seizures and hemiconvulsions, are seen in
1/3 of cases. Those who live past the age of 2 years manifest with severe psy-
chomotor deficits. In some, EIEE can transition into West syndrome (between
2-6 months of age) and later into Lennox-Gastaut syndrome (see these terms).
Certain genetic variants manifest with additional signs such as dyskinetic
movements and an atypical Rett-syndrome phenotype. Death is often due to
pneumonia or other complications of a complex disability. EIEE may be the
result of different etiologies. Many cases have been associated with structural
brain abnormalities. Some cases are due to metabolic disorders (cytochrome
C oxidase deficiency, carnitine palmitoyl transferase II deficiency) or brain
malformations (such as porencephaly, hemimegalencephaly) that may or may
not be genetic in origin. Genetic variants of EIEE have been associated with
mutations in certain genes such as ARX (Xp22.13), CDKL5 (Xp22), SL25A22
(11p15.5) and STXBP1 (9q34.1), among others. The genetic abnormalities are
thought to lead to EIEE as they are related to neuronal dysfunction or brain
dysgenesis. A careful history and physical/neurologic examination is essential
to identify common causes of early onset seizures. Once it is clear that there is
no ready explanation for seizures, electroencephalography is the next step in
the evaluation, and is very helpful in distinguishing between the many seizure
disorders that can present at an early age. Differential diagnoses include other
epileptic encephalopathies such as early myoclonic encephalopathy, West syn-
drome and other early onset epileptic encephalopathies.
• Infants who have one of the benign epilepsy syndromes will generally
have a normal EEG background rhythm, and may or may not have epi-
leptiform discharges.

19
Child Neurology

• Ohtahara Syndrome (EIEE) and EME are both associated with an SB pat-
tern. Infantile spasm is relatively easily distinguished electrographically
from the other two encephalopathic syndromes by the presence of hyp-
sarrhythmia, modified hypsarrhythmia, or multifocal independent spike
wave discharges. Thus, in the right clinical setting, an SB EEG would be
indicative of either EIEE or EME. It should be noted, however, that SB is
not specific to epileptic encephalopathy.
• Neonatal Hypoxic Ischaemic Encephalopathy (HIE) is a common cause
of SB, and SB can occur in many other disorders. Although the character
of the SB is subtly different in EIEE compared with EME, it is very dif-
ficult to differentiate between the two on the basis of the EEG. Thus, the
semiology of the seizures serves as an important point of differentiation
because Ohtahara patients have very frequent tonic spasms that occur in
the awake and sleep states, and EME patients have fragmentary myoclonic
seizures
• An MRI scan should be performed in all cases of epileptic encephalopathy.
Approximately 75% of EIEE patients will be found to have a structural
lesion on MRI scan. Most lesions will involve some type of cortical dys-
plasia such as lissencephaly, hemimegalencephaly, Aicardi syndrome, or
focal cortical dysplasia.
• On the other hand, patients with EME are unlikely to have a structural
lesion on MRI. They are, however, likely to have a metabolic disorder
such as nonketotic hyperglycaemia, propionic acidemia, methylmalonic
acidemia, molybdenum cofactor deficiency, mitochondrial dysfunction,
or Menkes disease.
• An evaluation for metabolic disease should be performed in patients who
have epileptic encephalopathy, whether or not it appears clinically that
EME or EIEE is the most likely diagnosis.

DIAGNOSTIC WORK-UP
Diagnosis is based on clinical and electroencephalographic findings. The char-
acteristic Electroencephalogram (EEG) displays a suppression burst pattern
(which appears with the onset of spasms) that is comprised of bursts of high
amplitude spikes and polyspikes that alternate with periods of low voltage ba-
sic rhythm (suppression). This EEG pattern is continuous and remains un-
changed during both waking and sleeping states. Structural abnormalities can
often be seen on cranial MRI.

MANAGEMENT
There is no cure for EIEE and patients require constant supervision and care.
Anti-epileptic drugs such as benzodiazepines, valproate, levetiracetam, zon-
isamide and phenobarbital have shown limited success in controlling seizures
as has pyridoxine. A ketogenic diet has been reported to show some success
in seizure control. In those with associated metabolic disorders, once these

20
Case 04

conditions have been treated there can be an improvement in the course of


EIEE. Similarly, EIEE patients with certain structural abnormalities have ben-
efited from surgical intervention, if unilateral.
Prognosis is poor with death usually occurring in infancy (50% before the
age of 2). Survivors have severe psychomotor impairments with continuing
seizures.
• Seizures associated with EIEE are generally found to be medically intrac-
table. Anti-epileptic medications are routinely used, but seizures with OS
are usually difficult to control. Medications that are often tried include
clobazam, clonazepam, vigabatrin, topiramate, zonisamide, Phenobarbital,
valproate, or felbamate
• Steroid therapy with ACTH or prednisone has been helpful in some
children
• It is reasonable to attempt treatment with any of the available medications
because there is no single medication that has been shown to be more ef-
fective, and the cases are sufficiently rare that a definitive study is unlikely
• However, patients who have focal cortical dysplasia or hemimegalenceph-
aly have been reported to improve after surgical resection; therefore, a
surgical evaluation is indicated if there is any suggestion of a localised area
of cortical abnormality. It has been seen many patients’ seizures tend to
improve as they get older.
• There is significant morbidity and mortality associated with Ohtahara
syndrome. Many patients will not survive the first two years of life and,
essentially, all of the survivors will be significantly mentally retarded.
Ohtahara has reported that approximately 75% of patients with Ohtahara
syndrome progress to West syndrome during the first year of life. Many
patients with West syndrome evolve to Lennox–Gastaut syndrome later
in childhood. As noted, seizures will often become easier to control after
a few years
• Devices or dietary therapy: A vagus nerve stimulator or the ketogenic diet
may also be considered when medicines do not work well
• Autosomal recessive inheritance has been reported, but most cases of
EIEE are sporadic (de novo mutations). Genetic counseling is therefore
very valuable to inform parents that their risk of having further children
with EIEE is low.

REFERENCES
1. Proposal for revised classification of epilepsies and epileptic syndromes. Commis-
sion on Classification and Terminology of the International League Against Epi-
lepsy. Epilepsia 1989; 30:389.
2. Berg AT, Berkovic SF, Brodie MJ, et al. Revised terminology and concepts for or-
ganization of seizures and epilepsies: report of the ILAE Commission on Classifi-
cation and Terminology, 2005-2009. Epilepsia 2010; 51:676.

21
Child Neurology

3. Grinton BE, Heron SE, Pelekanos JT, et al. Familial neonatal seizures in 36 fami-
lies: Clinical and genetic features correlate with outcome. Epilepsia 2015; 56:1071.
4. Ottman R, Hirose S, Jain S, et al. Genetic testing in the epilepsies--report of the
ILAE Genetics Commission. Epilepsia 2010; 51:655.
5. Beal JC, Cherian K, Moshe SL. Early-onset epileptic encephalopathies: Ohtahara
syndrome and early myoclonic encephalopathy. Pediatr Neurol 2012; 47:317.
6. Bahi-Buisson N, Girard B, Gautier A, et al.. Epileptic encephalopathy in a girl with
an interstitial deletion of Xp22 comprising promoter and exon 1 of the CDKL5
gene. Am J Med Genet B Neuropsychiatr Genet 2010; 153B:202.
7. Kato M, Saitoh S, Kamei A, et al.. A longer polyalanine expansion mutation in the
ARX gene causes early infantile epileptic encephalopathy with suppression-burst
pattern (Ohtahara syndrome). Am J Hum Genet 2007; 81:361.
8. Saitsu H, Kato M, Okada I, et al.. STXBP1 mutations in early infantile epileptic
encephalopathy with suppression-burst pattern. Epilepsia 2010; 51:2397.
9. Aicardi J, Ohtahara S. Severe neonatal epilepsies with suppression-burst pattern.
In: Roger J, Thomas P, Bureau M, Hirsch D, Dravet C, et al. al. Undefined. Syndromes
in Infancy, Childhood and Adolescence. 4th Edition. John Libbey Eurotext; 2005:Chap-
ter 3.
10. Ohtahara S, Yamatogi Y. Ohtahara syndrome: with special reference to its devel-
opmental aspects for differentiating from early myoclonic encephalopathy. Epilepsy
Res. 2006 Aug. 70 Suppl 1:S58-67.

22
Case 05
A 10-month-old boy is a product of non-consanguineous parents born at
term after an uneventful pregnancy. His birth and family history were
unremarkable. Birth weight was 3.4 kg, length 50 cm, and head circumference
34 cm. At 2 months of age, he presented with feeding problems and irritability.
At 6 months of age he demonstrated neurodevelopmental regression,
abnormal movements, myoclonic jerks, and episodes of hyperextension of the
lower limbs.
On physical examination he was noted to have generalised hypertonia
with an opisthotonic posture, hyperreflexia, clenched fist and plantar extensor
response. He was very irritable and prone to cry when awake. He had markedly
reduced spontaneous movements and could not localise sound. He continued
to deteriorate and died at 17 months of age following a respiratory arrest. His
head circumference was on the ninth centile.
A sleep electroencephalogram showed a diffusely slow and disorganised
background with superimposed isolated spikes in the left temporal region.
Routine blood and urine examinations were unremarkable. Urine organic
acids were normal. Cerebrospinal Fluid (CSF) examination showed a normal
glucose level, a cell count of 26, and a protein of 2.90 g/L (normal range 0.15-
0.45 g/L).

Answer the following questions:


• Give the most likely diagnosis.
• Discuss the differential diagnosis.
• Discuss an appropriate diagnostic work-up.
• Discuss the management of this patient.

23
Child Neurology

DIAGNOSIS
Infantile Krabbe disease (Globoid cell Leukodystrophy)

SUMMARY
An infant presented with feeding problems and irritability, with regression of
developmental milestones. He also has seizure and episodes of hyperextension
of the lower limbs. On physical examination he showed generalised hypertonia
with an opisthotonic posture, irritability, hyperreflexia, clenched fist and plan-
tar extensor response. A review of the MRI images (symmetrical signals abnor-
malities in the periventricular regions of the posterior cerebral hemisphere)
suggested the diagnosis of Krabbe disease. This was confirmed by the dem-
onstration of markedly reduced fibroblast activity of β-galactosylceramidase
(0.17nmoL/mg/h, normal range 1.50-8.00nmoL/mg/h). Molecular analysis of
the GALC gene revealed compound heterozygosity for a G537R mutation (in-
herited from the mother) and the recurrent deletion of exons 11-17, inherited
from the father.

DIFFERENTIAL DIAGNOSIS
Krabbe disease (globoid cell leukodystrophy) is an autosomal recessive disorder
caused by the deficiency of Galactocerebrosidase (GALC). GALC is respon-
sible for the liposomal hydrolysis of galactolipids formed during white matter
myelination. The pathologic changes in the peripheral and central nervous
system (globoid cell formation and decreased myelin) appear to result from
the toxic nature of psychosine and the accumulation of galactocerebroside (ga-
lactosylceramide) that cannot be degraded because of the GALC deficiency.
Most patients with Krabbe disease present with symptoms within the first six
months of life; approximately 10 percent present later in life, including adult-
hood. Peripheral motor sensory neuropathy occurs in all patients, but the early
onset forms are dominated by symptoms related to CNS dysfunction.
Infantile onset. Infantile onset disease is associated with developmental de-
lay, limb stiffness, hypotonia, absent reflexes, optic atrophy, and microcephaly
with extreme irritability. Seizures eventually appear, and tonic extensor spasms
associated with stimulation with light, sound, or touch occur. These children
regress rapidly to a decerebrate condition, with most dying before reaching
two years of age.
Juvenile onset. Patients with juvenile onset disease typically present with
weakness, loss of skills, and vision loss. Late infantile and juvenile patients
regress at an unpredictable rate, but all become severely incapacitated and die
two to seven years after diagnosis.
Adult onset. Adult onset disease may be manifested initially by loss of manual
dexterity, burning paraesthesias in the extremities, weakness, or predominantly
peripheral motor sensory neuropathy with loss of distal sensation and muscle
atrophy with scoliosis. Some adolescents and adults have symptoms confined to

24
Case 05

weakness without intellectual deterioration, whereas others become bedridden


and continue to deteriorate mentally and physically.
Symptoms of the following disorders can be similar to those of Krabbe
leukodystrophy.

GM2 Gangliosidoses
The classic infantile forms
orms of the 3 subgroups of GM2 gangliosidoses have
their onset in infancy and lead to death by age 2-4 years. Clinical symptoms
of progressive neurodegeneration and developmental delay are caused by ac-
cumulation of GM2 ganglioside in lysosomes. Children with type I gangliosi-
doses do not appear to be affected at birth. Loss of milestones occurs in infancy.
Neurologic symptoms as well as muscular weakness, which leads to paralysis,
start by age 3-5 months. Most patients never walk and exhibit increasing apa-
thy and inattention. Hyperacusis (i.e. persistent extension response to sound,
startle reaction) can aid in early diagnosis. After age 8-10 months, deteriora-
tion is rapid as neurodegeneration progresses. Macrocephaly due to reactive
cerebral gliosis typically begins at age 18 months. Eventually, patients have
difficulty swallowing, uncontrolled seizures, spasticity, blindness, and demen-
tia. Electroretinographic and EEG findings are normal, but visual-evoked re-
sponses are abnormal.

Gaucher Disease
Gaucher disease has traditionally been divided into the following three clinical
subtypes, delineated by the absence or presence of neurologic involvement
and its progression:
• Type 1: Non neuronopathic form
• Type 2: Acute neuronopathic form
• Type 3: Chronic neuronopathic form
Patients with type 2 disease may present at birth or during infancy with in-
creased tone, seizures, strabismus, and organomegaly. Failure to thrive, swal-
lowing abnormalities, oculomotor apraxia, hepatosplenomegaly, and stridor
due to laryngospasm are typical in infants with type 2 disease.
Organomegaly, oculomotor apraxia, and stridor differentiate it from Karabbe
disease.

Metachromatic Leukodystrophy
Metachromatic Leukodystrophy (MLD) Arylsulphatase deficiency is part of
a larger group of lysosomal storage diseases, some of which are progressive,
inherited, and neurodegenerative disorders (metachromatic leukodystrophy
included). Four types of metachromatic leukodystrophy occur with varying
ages of onset and courses (i.e. late infantile, early juvenile, late juvenile, adult).
Patients with the late infantile form are usually aged 4 years or younger and
typically present initially with gait disturbances, loss of motor developmental

25
Child Neurology

milestones, optic atrophy, and diminished deep tendon reflexes. In addition,


progressive loss of both motor and cognitive functions is fairly rapid, and death
results within approximately 5 years after the onset of clinical symptoms.
Clinically may mimic but radiologically different, in Krabbe hyperintensities.

Sphingomyelinase Deficiency
Niemann-Pick Disease (NPD) types A and B result from a deficiency of
acid sphingomyelinase and lysosomal accumulation of sphingomyelin. NPD
type A is characterised by early-onset, progressive neurodegenerative course;
systemic disease manifestations, including massive hepatosplenomegaly, in-
terstitial lung disease, and cherry-red macula; and death in early childhood.
Niemann-Pick Disease (NPD) type A disease is fatal in early childhood. The
clinical presentation and course of NPD type A is relatively uniform and char-
acterised by normal appearance at birth, followed by progressive hepatosple-
nomegaly from age 3 months and severe, neurodegenerative course leading to
death by age 3 years.
Massive hepatosplenomegaly and a cherry red spot separate this entity from
Krabbe disease.

Canavan Disease
Canavan is characterised by evidence of developmental delays by age three to
five months with severe hypotonia and failure to achieve independent sitting,
ambulation, or speech. Hypotonia evolves into spasticity and assistance with
feeding becomes necessary. Life expectancy is usually into the second decade.
Most individuals with Canavan disease have macrocephaly, which is a variable
finding in individuals with Krabbe disease. MRI shows prominent involve-
ment of subcortical white matter. The finding of elevated N-acetylaspartic acid
concentration in urine confirms the diagnosis of Canavan disease.
Pelizaeus-Merzbacher Disease (PMD) PMD typically manifests in infancy
or early childhood with nystagmus, hypotonia, and cognitive impairment, and
progresses to severe spasticity and ataxia. Life span is shortened. Molecular ge-
netic testing of PLP1 is diagnostic. There is complete loss of white matter myeli-
nation at an early age with capsular and subcortical white matter extensively lost.
Alexander disease is a disorder of cortical white matter. Two forms are com-
mon, infantile (80% of affected individuals) and juvenile (~14%), although
neonatal and adult forms are also recognised. The infantile form presents in
the first two years of life typically with megalencephaly, seizures, progressive
psychomotor retardation with loss of developmental milestones, and quadri-
paresis. Affected individuals survive a few weeks to several years.

DIAGNOSTIC WORK-UP
• Individuals with the infantile form of Krabbe disease can present with
any or all of the features; Irritability, muscle hypertonicity, progressive

26
Case 05

neurologic deterioration, peripheral neuropathy, evidence of white mat-


ter disease on neuroimaging, and elevation of Cerebrospinal Fluid (CSF)
protein concentration.
• While most individuals have the infantile form, older individuals ranging
in age from six months to the seventh decade have also been diagnosed
with galactocerebrosidase deficiency. Galactocerebrosidase (GALC) en-
zyme activity: Symptomatic individuals. All individuals with Krabbe dis-
ease have very low GALC enzyme activity (0%-5% of normal activity)
in leukocytes isolated from whole heparinised blood and cultured skin
fibroblasts.
• Neuroimaging. Progressive, diffuse, and symmetric cerebral atrophy is
observed by neuroimaging. In general, MRI detects demyelination in the
brain stem and cerebellum more clearly than CT at the early stage of the
disease; MRI revealed symmetric hyperdensity involving the cerebellum,
thalami, caudate, corona radiata, and brain stem. Individuals with Krabbe
disease who have severe demyelination show high-intensity lesions on
T2-weighted images with a loss of diffusional anisotropy and relatively
high signal on diffusion-weighted images.
• Newborn screening. With improvements in treatment options for pre-
symptomatic individuals, efforts to develop newborn screening methods are
underway. A method using dried blood spots and tandem mass spectrom-
etry to measure GALC enzyme activity. In August 2006, New York State
instituted newborn screening for Krabbe disease; to date over 1,000,000
newborns have been screened. Other high-risk individuals were identified
by confirming enzymatic studies and by molecular genetic testing
testing.
• Molecular genetic testing gene. GALC is the gene most commonly
known to be associated with Krabbe disease.

MANAGEMENT
• There is no specific treatment for Krabbe disease. However, the irritability
encountered in the infantile form of Krabbe disease has been successfully
controlled with low dose of morphine
• Some people have had a bone marrow transplant in the early stages of
the disease, but this treatment has risks. Following the emergence of
Haematopoietic Stem Cell Transplantation (HSCT) as a potential treat-
ment for Krabbe disease. Numerous studies in human and animal models
have shown varying degrees of benefit with HSCT, with greatest benefit
occurring in patients who are asymptomatic or mildly symptomatic and
when transplanted within the 1st month of life. HCST should be con-
sidered in individuals with late-onset or slowly progressive Krabbe dis-
ease and in individuals with infantile-onset disease, in the early neonatal
asymptomatic period. Short-term benefits with HSCT are reported in
medical literature, including a suggestion of delayed progression and im-
proved survival; however, transplantation mortality rates are 15%.

27
Child Neurology

Prevention
Genetic counseling is recommended for persons with a family history of
Krabbe disease who are considering having children. A blood test can be done
to see if you carry the gene for Krabbe disease. Prenatal tests (amniocentesis
or chorionic villus sampling) can be done to screen a developing baby for this
condition.

REFERENCES
1. Duffner PK, Caggana M, Orsini JJ, et al.. Newborn screening for Krabbe disease:
the New York State model. Pediatr Neurol 2009; 40:245.
2. Wenger DA, Rafi MA, Luzi P. Molecular genetics of Krabbe disease (globoid cell
leukodystrophy): diagnostic and clinical implications. Hum Mutat 1997; 10:268.
3. Duffner PK, Barczykowski A, Jalal K, et al.. Early infantile Krabbe disease: results of
the World-Wide Krabbe Registry. Pediatr Neurol 2011; 45:141.
4. Duffner PK, Jalal K, Carter RL. The Hunter’s Hope Krabbe family database. Pedi-
atr Neurol 2009; 40:13.
5. Abdelhalim AN, Alberico RA, Barczykowski AL, Duffner PK. Patterns of magnetic
resonance imaging abnormalities in symptomatic patients with Krabbe disease cor-
respond to phenotype. Pediatr Neurol 2014; 50:127.
6. Li Y, Brockmann K, Turecek F, et al.. Tandem mass spectrometry for the direct as-
say of enzymes in dried blood spots: application to newborn screening for Krabbe
disease. Clin Chem 2004; 50:638.
7. Krivit W, Shapiro EG, Peters C, et al.. Hematopoietic stem-cell transplantation in
globoid-cell leukodystrophy. N Engl J Med 1998; 338:1119.
8. Escolar ML, Poe MD, Provenzale JM, et al.. Transplantation of umbilical-cord
blood in babies with infantile Krabbe’s disease. N Engl J Med 2005; 352:2069.
9. Siddiqi ZA, Sanders DB, Massey JM. Peripheral neuropathy in Krabbe disease: ef-
fect of hematopoietic stem cell transplantation. Neurology 2006; 67:268.

28
Case 06
A newborn premature of 32 weeks’ gestation, female, suffered repeated
convulsions, myoclonus, rotatory eye movements, and sudden clonic
contractions since 5 hours of age. The pregnancy was complicated, with intra-
uterine growth retardation and oligohydramnios. Her birth weight was 2300
g, and Apgar scores at 1 and 5 min were 9 and 9, respectively. She exhibited
symptoms of myoclonic seizures 5 hours after birth.
On physical examination, the patient was non-dysmorphic, the skin was pink
and well perfused. There were moderate subcostal retractions, shallow respi-
rations, and bronchial breath sounds that were slightly diminished in inten-
sity; the remainder of the examination was normal for the gestational age. The
patient was intubated (developed RDS) and assisted ventilation was started.
A full-septic work-up and metabolic investigations were performed with un-
remarkable results. The patient was treated with phenobarbital, which was
slowly titrated to maximum dose, at the age of 10 days; the patient underwent
several episodes of myoclonic seizures. A Computed Tomography (CT) scan
showed cerebral oedema. The status epilepticus was refractory to multiple
anti-epileptic medications, including phenytoin, phenobarbital, levetiracetam,
and Clonazepam. A trial of intravenous pyridoxine did not stop the clinical
seizure.
Cerebral function monitoring during high-dose midazolam infusion showed
a burst-suppression pattern. The patient brain showed normal results when
subjected to magnetic resonance imaging and magnetic resonance spec-
trometry; in addition, her urine organic acid screening was unremarkable.
Discontinuation of the midazolam treatment resulted in a gradual return of
continuous EEG pattern and the patient regaining consciousness. The patient
was then discharged at the age of 21 days. Two days after the hospital dis-
charge, the patient experienced multiple clusters of multifocal and generalised
myoclonic seizures, intermixed with eye deviation and inconsolable crying,
which developed into myoclonic status epilepticus.

Answer the following questions:


• Give the most likely diagnosis and discuss the differential diagnosis.
• Discuss the appropriate diagnostic work-up.
• Discuss the management of this patient.

29
Child Neurology

DIAGNOSIS
Pyridoxal 5'-phosphate-responsive epilepsy
(pyridox(am)ine-5-prime-phosphate oxidase deficiency)

SUMMARY
A premature new born experienced multiple clusters of multifocal, general-
ised and myoclonic seizures, intermixed with eye deviation and inconsolable
crying, which developed into myoclonic status epilepticus. The status epilep-
ticus was refractory to multiple anti-epileptic medications. A trial of intra-
venous pyridoxine did not stop the clinical seizure and 10 mg/kg Pyridoxal
5-Phosphate (PLP) trial resulted in an increased suppression ratio of the
Electroencephalogram (EEG). Because of her dramatic response to PLP, PLP-
responsive epilepsy was suspected. The maintenance of a PLP dose of 30 mg
helped control the seizures.

DIFFERENTIAL DIAGNOSIS
Pyridoxal 5’-phosphate-dependent epilepsy is a condition that involves sei-
zures beginning soon after birth or, in some cases, before birth. The seizures
typically are myoclonic, abnormal eye movements, and other types of convul-
sions. Most babies with this condition are born prematurely and may have a
temporary, potentially toxic, increase in lactic acid in the blood (lactic acidosis).
Additionally, some infants have foetal distress.
Anti-convulsant drugs, which are usually given to control seizures, are inef-
fective in babies with pyridoxal 5'-phosphate-dependent epilepsy. Instead, in-
dividuals with this type of epilepsy are medically treated with large daily doses
of pyridoxal 5’-phosphate (active form of vitamin B6). If left untreated, people
with this condition can develop severe brain dysfunction (encephalopathy),
which can lead to death. Even though seizures can be controlled with pyri-
doxal 5’-phosphate, neurological problems such as developmental delay and
learning disorders may still occur. Pyridoxal-5-Phosphate, or P5P as it is com-
monly known, is the active form of vitamin B6.
Neonatal seizures are common clinical problems confronting pediatricians,
neonatologists and paediatric neurologists. Seizures presenting in a new-born
represent a symptom of underlying central nervous system dysfunction that
may result from a variety of causes including developmental anomalies of
brain structure, infectious or inflammatory conditions, cerebrovascular dis-
ease, trauma, or metabolic disturbance. Certain rare inborn errors of metabo-
lism that are clinically responsive to specific vitamin therapy may present as a
neonatal encephalopathy with anti-convulsant-resistant seizures.
• Pyridoxine-dependent seizures. Pyridoxine-Dependent Seizures
(PDS) is the best understood of the neonatal vitamin-responsive epileptic
encephalopathies. Classically, patients with PDS present with neonatal sei-
zures that are intractable to treatment with conventional anti-convulsants

30
Case 06

and only come under control once pharmacologic doses of pyridoxine are
administered and then continued on a regular basis. Most recently muta-
tions in the ALDH7A1 gene were discovered which are responsible for
the biochemical abnormalities underlying PDS. The disorder may pre-
sent within hours of birth as an epileptic encephalopathy that may mimic
hypoxic-ischaemic encephalopathy. In addition, some mothers may report
having experienced unusual foetal movements that likely represent intrau-
terine foetal seizures. PDS may present with seizures at a later time during
the first several weeks of life. Even more unusual are examples of patients
with PDS that develop clinical features after 2 months of age, and these are
considered to be late-onset cases. Universally, all patients with PDS have
clinical seizures, which either recur serially or evolve into status epilepti-
cus despite treatment with large doses of one or more conventional anti-
convulsants. The typical dose of PLP is 30-50 mg/kg/day in 3-4 divided
doses as an enteral preparation. Early diagnosis and treatment is the most
important predictor of outcome. Untreated cases have high mortality, and
survivors are left with poor neurocognitive outcome.
• Folinic acid-responsive seizures. Infants with folinic acid depend-
ent epilepsy all present with seizures and encephalopathy within the first
5 days of life, and the seizures are resistant to anti-convulsant therapy.
Folinic acid responsive epilepsies are also commonly known as cerebral
folate deficiency syndrome. Although cerebral folate deficiency is associ-
ated with diverse neurological conditions, only a few conditions are re-
sponsive to treatment with folinic acid. Neuroimaging shows brain atro-
phy and variable white matter abnormalities. Some patients respond to
pyridoxine alone, or have an initial response to it, only to become resistant
later on and responding to folinic acid. Mutations in the FOLR1 gene
coding for Folate Receptor alpha (FR (FRα) predominantly acting cerebral
folate transport leads to low CSF 5-MTHF levels. The choroid plexus is
rich in FRα,, and is the main site of folate delivery to the brain.
• Glycine encephalopathy. Glycine encephalopathy, which is also known
as nonketotic hyperglycinaemia or NKH. Glycine encephalopathy is
caused by the shortage of an enzyme that normally breaks down glycine in
the body. A lack of this enzyme allows excess glycine to build up in tissues
and organs, particularly the brain, leading to serious medical problems.
The most common form of glycine encephalopathy, called the classical
type, appears shortly after birth. Affected infants experience a progressive
lack of energy (lethargy), feeding difficulties, hypotonia, seizures, and life-
threatening problems with breathing.

DIAGNOSTIC WORK-UP
• Seizures ceased with the administration of PLP, having been resist-
ant to treatment with pyridoxine, suggesting a defect of pyridox(am)ine
5-prime-phosphate oxidase.

31
Child Neurology

• Diagnosis may be made in individuals experiencing status epilepticus or


repetitive clinical seizures that are not controlled with anti-epileptic drugs.
• The diagnosis of pyridoxal 5-phosphate-dependent epilepsy is typically
made on clinical grounds and may be confirmed through biochemical
and/or molecular genetic testing. Pyridoxine-5-P oxidase catalyses the
terminal step in vitamin B6 metabolism, the biosynthesis of pyridoxal-5’-
phosphate, the biologically active form of vitamin B6 which acts as an es-
sential cofactor.
• CSF concentration of the Dopamine metabolitie, homovanillic acid, and
of the serotonin metabolite, 5-Hydroxyindoleacetic acid, are low, whereas
the L-dopa metabolite 3-O-methyldopa is increased.
• The urinary excretion of another L-dopa metabolite, vanillyl-lactic acid, is
increased. These changes indicated reduced activity of the PLP-dependent
enzyme, aromatic L-amino acid decarboxylase.
• Excessive vanillyl-lactic acid excretion in urine organic acid profile (nor-
mal concentration is very low) is a biochemical hallmark of PNPO defi-
ciency that may be detected with initial metabolic screening.
• Mutations of the PNPO gene (chromosome 17q21.2) have been de-
scribed, and result in varying degree of reduced pyridox(am)ine phos-
phate oxidase activity.

MANAGEMENT
• Pyridoxamine 5-phosphate oxidase deficiency is an extremely rare autoso-
mal recessive disorder that deprives the brain from the only active cofac-
tor of the vitamin B6 precursors pyridoxine and pyridoxamine, namely
pyridoxal 5’-phosphate.
• In the case of PNPO deficiency, seizures do not respond to treatment with
anti-convulsants or pyridoxine but resolve with the administration of PLP.
• At the time of designation, no satisfactory methods of treatment were
authorised in the EU for pyridoxamine 5’-phosphate oxidase deficiency.
Products containing PLP (the active form of vitamin B6, Capsule-50 mg)
were given to patients, and it is very effective when given via a nasogastric
tube ( sick neonate) or orally, following recovery from seizures.
• In a PNPO-deficient patient, nasogastric administration of 50 mg of PLP
led to cessation of seizures within an hour, but this was associated with
profound hypotonia and unresponsiveness, and also some hypotension.
• A trail of treatment with PLP should be undertaken only in a setting where
full resuscitation and intensive care facilities are available.
• If left untreated, people with this condition can develop severe brain dys-
function (encephalopathy), which can lead to death. Even though seizures
can be controlled with pyridoxal 5’-phosphate, neurological problems
such as developmental delay and learning disorders may still occur.

32
Case 06

• Early recognition and treatment of cofactor intractable neonatal seizures


should include empiric treatment with pyridoxine as first line with add on
pyridoxal 5 phosphate for incomplete or absent seizure control.

REFERENCES
1. Bagci S, Zschocke J, Hoffmann GF, Bast T, Klepper J, Müller A, Heep A, Bart-
mann P, Franz AR. Pyridoxal phosphate-dependent neonatal epileptic encepha-
lopathy. Arch Dis Child Fetal Neonatal Ed. 2008 Mar;93(2):F151-2. doi: 10.1136/
adc.2006.115162.
2. Hoffmann GF, Schmitt B, Windfuhr M, Wagner N, Strehl H, Bagci S, Franz AR,
Mills PB, Clayton PT, Baumgartner MR, Steinmann B, Bast T, Wolf NI, Zschocke
J. Pyridoxal 5’-phosphate may be curative in early-onset epileptic encephalopathy.
J Inherit Metab Dis. 2007 Feb;30(1):96-9. Epub 2006 Dec 23.
3. Khayat M, Korman SH, Frankel P, Weintraub Z, Hershckowitz S, Sheffer VF, Ben
Elisha M, Wevers RA, Falik-Zaccai TC. PNPO deficiency: an under diagnosed
inborn error of pyridoxine metabolism. Mol Genet Metab. 2008 Aug;94(4):431-4.
doi: 10.1016/j.ymgme.2008.04.008. Epub 2008 May 15.
4. Mills PB, Surtees RA, Champion MP, Beesley CE, Dalton N, Scambler PJ, Heales
SJ, Briddon A, Scheimberg I, Hoffmann GF, Zschocke J, Clayton PT. Neona-
tal epileptic encephalopathy caused by mutations in the PNPO gene encoding
pyridox(am)ine 5’-phosphate oxidase. Hum Mol Genet. 2005 Apr 15;14(8):1077-
86. Epub 2005 Mar 16.
5. Guerin A, Aziz AS, Mutch C, Lewis J, Go CY, and Mercimek-Mahmutoglu S.
Pyridox(am)ine-5-Phosphate Oxidase Deficiency Treatable Cause of Neonatal
Epileptic Encephalopathy With Burst Suppression: Case Report and Review of the
Literature. J Child Neurol. August 2015; 30(9):1218-25.
6. Veeravigrom M, Damrongphol P, Ittiwut R, Ittiwut C, Suphapeetiporn K, and
Shotelersuk V. Pyridoxal 5’-phosphate-responsive epilepsy with novel mutations
in the PNPO gene: a case report. Genet Mol Res. October 30 2015; 14(4):14310-5
7. Kniffen CL. Pyridoxal 5-Prime-Phosphate Oxidase Deficiency. Online Mendelian
Inheritance in Man. March 1 2016;

33
07 Case
A 6-year-old girl was referred to the paediatric neurology clinic with an
acute complaint of instability of gait and frequent falls while walking,
for the last 12 hours. She had complained of fever, and flu like illness 12
days before. Her General Practitioner treated her with a five-day course of
Augmentin. Her vaccinations schedule was up-to-date. She denied ingestion
of unknown/illicit substances and there was no history of head trauma. Her
past medical history included an isolated reflex anoxic syncopal event aged 22
months.
On physical examination, the vital signs including pulse rate and blood
pressure were normal. Glasgow coma scale was 15/15. Her gait examination
showed a broad-based gait and she was markedly ataxic, unable to walk or sit
unsupported, and was noted to have titubation, and dysmetria.
She had bilateral cerebellar signs in the form of hypotonia of all the limbs,
dysarthria, nystagmus, bilateral dysdiadochokinesia, and bilateral finger-nose
and heel-knee incoordination. Tone, power and the deep tendon reflexes were
normal and plantars were flexor. Cranial nerves were intact, and there were
no signs of meningeal irritation. The rest of the physical and neurological
examination was unremarkable.
Initial investigations were done to determine a possible infective cause. The
LP was normal, and Varicella serology was negative. An MRI scan of her brain
showed no intracranial lesions, structural changes or features of infection.

Answer the following questions:


• Give the most likely diagnosis.
• Provide a differential diagnosis.
• Localise the examination findings.
• Discuss the appropriate diagnostic work-up.
• Discuss the management of the patient.

34
Case 07

DIAGNOSIS
Acute cerebellar ataxia

SUMMARY
This girl presented with acute complaint of instability of gait and frequent
falls while walking, for the last 12 hours and had upper respiratory infection
12 days before. She had bilateral cerebellar signs in the form of hypotonia of
all the limbs, dysarthria, nystagmus, bilateral dysdiadochokinesia, and bilateral
finger-nose and heel-knee incoordination. Tone, power and the deep tendon
reflexes were normal and plantars were flexor. Cranial nerves were intact, and
there were no signs of meningeal irritation.

Where is the Localisation of the Lesion?


Neurological findings of this patient localise to the cerebellum diffusely as she
has truncal and appendicular ataxia. This shows pancerebellar syndrome and
there was no other focal feature.

DIFFERENTIAL DIAGNOSIS
Acute cerebellar ataxia is a syndrome which has been observed following many
infectious diseases, including varicella, Epstein-Barr virus, roseola (human
herpesvirus 6), enterovirus, rubeola, parvovirus, and mycoplasma infection.
The syndrome most often is seen in young children with the majority of cases
occurring between two and five years of age. The pathogenesis, clinical pres-
entation, evaluation, and prognosis of acute cerebellar ataxia will be reviewed
here. Acute cerebellar ataxia usually occurs in children under six years of age.
The true incidence of this disorder is not known. Acute cerebellar ataxia is
characterised by the acute onset of ataxia four days to three weeks after the
inciting illness. The symptoms are typically maximal at onset. Gait disturbance
is the primary symptom, but the cerebellar dysfunction may be limited to fine
motor control problems or tremor. Associated symptoms may include nystag-
mus (roughly one-half of cases reported), slurred or garbled speech, vomiting,
dysarthria, or, in older children, headache. Fever, meningismus, and seizures
are absent.
The differential for acute ataxia in children is broad; however, the diagnostic
approach is designed to identify one of the following alternate causes for the
clinical presentation.

Toxins Ingestion
The incidence of accidental drug ingestion is greatest between ages 1 and 4
years. This is the most common cause of acute ataxia. Excessive use of anti-
histamines in the treatment of an infant or young child with allergy or an upper
respiratory tract infection may cause ataxia. This is especially true in children
with otitis media, who may have underlying unsteadiness because of middle ear

35
Child Neurology

infection. An overdose of most psychoactive drugs causes ataxia, disturbances


in personality or sensorium, and sometimes seizures. Toxic doses of anti-epi-
leptic drugs, especially phenytoin, may cause marked nystagmus and ataxia

CNS Infections
Central Nervous System infections such as bacterial meningitis, encephalitis,
and cerebellitis also commonly present with acute cerebellar abnormalities.
Ataxia may be the initial feature of viral encephalitis affecting primarily the
structures of the posterior fossa.

Cerebellar Vascular (Stroke Haemorrhage/Thrombosis)


Cerebellar strokes are very unusual in childhood. Clinical presentation can be
very wide in spectrum. In young adults less than 40-years-old the most com-
mon mechanism of cerebellar infarctions is likely to be arterial occlusion as a
result of intracranial vertebral artery dissection (40%), mainly with PICA in-
volvement. Cardiac diseases constitute an important etiological differential in
childhood strokes. It is recommended to screen for those disorders in all child-
hood strokes. A non-conclusive list might include gait ataxia, intentional trem-
ors, vertigo, dizziness, nausea, vomiting, visual changes, nystagmus, headache,
depressed sensorium, slurred speech, dysphagia, hearing impairment and is-
sues with pain and temperature perception. Spontaneous cerebellar haemor-
rhage in children, in the absence of a coagulopathy, is due to arteriovenous
malformation, even though less than 10% of intracranial arteriovenous mal-
formations in children are in the cerebellum.

Acute Inflammatory Demyelinating Polyneuropathy (AIDP)


Ataxia, ophthalmoplegia, and areflexia characterise the Miller Fisher syn-
drome. A similar disorder with ataxia and areflexia but without ophthalmo-
plegia is acute ataxic neuropathy. Some believe that Miller Fisher syndrome
is a variant of Guillain-Barré syndrome; others believe that it is a form of
brainstem encephalitis. Decreased peripheral sensory input probably causes
areflexia, and more prominent, limb than trunk ataxia. The clinical distinction
between Miller Fisher syndrome and brainstem encephalitis can be difficult.
Disturbances of sensorium, multiple cranial nerve palsies, an abnormal EEG,
or prolongation of the interpeak latencies of the brainstem auditory evoked
response should suggest brainstem encephalitis. The cerebrospinal fluid pro-
file in Miller Fisher syndrome parallels that of the Guillain-Barré Syndrome.

Acute Disseminated Encephalomyelitis (ADEM)


Acute Disseminated Encephalomyelitis (ADEM) is an immune-mediated in-
flammatory demyelinating condition that predominately affects the white mat-
ter of the brain and spinal cord. The common presenting symptoms are fever,
vomiting, headache, gait disturbance and generalised seizures. Neurological
manifestations included altered sensorium, multiple cranial nerve involvement,

36
Case 07

quadriplegia and paraplegia, dystonia and choreiform movements, nystagmus,


bladder involvement (both incontinence and retention), speech defect and
double vision. The diagnosis of ADEM is made based on the clinical presenta-
tion and suggestive MRI findings. Magnetic resonance imaging was the neu-
roimaging study of choice for establishing the diagnosis. Prognosis for survival
and outcome is excellent.

Tumours
Approximately 85% of primary brain tumours in children 2 to 12 years of age
are located in the posterior fossa. The four major tumours of the posterior
fossa are cerebellar astrocytoma, brainstem glioma, ependymoma, and primi-
tive neuroectodermal tumour (medulloblastoma). Gait disturbances occur
with equal frequency whether supratentorial tumours are in the midline or
the hemispheres, whereas cerebellar signs are more common with midline tu-
mours. Ataxia is present in three fourths, dysmetria in one-half, and nystagmus
in only one-fourth. Ataxia varies in severity from a wide-based, lurching gait to
a subtle alteration of gait observed only with tandem walking or quick turning.
Its cause is partly the cerebellar location of the tumour and partly hydrocepha-
lus. When the tumour is in the cerebellar hemisphere, ipsilateral or bilateral
dysmetria may be present. MRI is the best imaging study for diagnosis.

Trauma
Mild head injuries are common in children and are an almost daily occurrence
in toddlers. Ataxia may follow even mild head injuries. In most cases, ataxia is
part of the so-called post-concussion syndrome, in which imaging studies do
not show any structural derangement of the nervous system. In others, a cer-
ebellar contusion or posterior fossa haematoma may be present (see Chapter 2).
Ataxia may also occur after cervical injuries, especially during sports. Trauma to
the vertebrobasilar artery is causal.

Metabolic
Hartnup and Maple syrup urine disease are rare disorder; both transmitted
by autosomal recessive inheritance. Many patients have episodes of limb atax-
ia, sometimes associated with nystagmus. Affected individuals are normal at
birth. Between 5 months and 2 years, minor infections, surgery, or a diet rich
in protein provokes episodes of ataxia, irritability, and progressive lethargy. The
length of an attack varies; most children recover spontaneously, but some die
of severe metabolic acidosis.

Conversion Disorder
Hysterical gait disturbances are relatively common in children, especially girls
between 10 and 15 years of age. Hysteria is involuntary and usually provides
a secondary gain. In contrast, malingering is a voluntary act. Hysterical gait
disturbances are often extreme. The child appears to sit without difficulty, but

37
Child Neurology

when brought to standing, immediately begins to sway from the waist. Stance
is not wide to improve stability. Instead, the child lurches, staggers, and oth-
erwise travels across the room from object to object. The lurching maneuvers
are often complex and require extraordinary balance. Strength, tone, sensation,
and tendon reflexes are normal. The diagnosis of hysterical gait disturbances
is by observation; laboratory tests are not usually required to exclude other
possibilities.

DIAGNOSTIC APPROACH
• The mostt common alternate diagnosis involves a toxic ingestion; there-
fore, a thorough history should be performed to determine access to vari-
ous pharmaceutical and chemical agents and a routine toxicology screen
should be performed.
• Routine laboratory studies should identify an occult metabolic process
and may suggest infection.
• Neuroimaging study is mandatory to exclude hydrocephalus or a struc-
tural cerebellar lesion, such as a haematoma, tumour, cerebellitis, or ab-
scess. The white matter lesions that characterise ADEM and more severe
cases of ACA are best visualised on MRI.
• Lumbar puncture should be performed, especially in the presence of fe-
ver, altered mental status, or abnormally elevated WBCs. The challenge
lies in differentiating between meningitis, meningoencephalitis, and ACA/
ADEM, all of which can have WBCs in the CSF.
• The availability of rapid (PCR for EBV, varicella, campylobacter, myco-
plasma or other organism) assays for the presence of viral and bacterial
assays may facilitate this decision.

MANAGEMENT
• There is no specific
pecific therapy at the present time for patients with ACA.
Over 90 percent of patients with ACA will recover completely from their
illness, with a low incidence of residual ataxia or cognitive deficits.
• Worse outcomes have been reported with older patients and in association
with EBV or HSV infections. Once the other conditions have been ruled
out, management
mana is primarily symptomatic.
• Affected children may often benefit from physical and occupational ther-
apy during the recovery phase of their illness.
• For refractory cases, corticosteroids as well as intravenous immunoglobu-
lin have been tried but reports of efficacy are mainly anecdotal, and ran-
domised, controlled trials have not been performed.

38
Case 07

REFERENCES
1. Nussinovitch M, Prais D, Volovitz B, et al. Post-infectious acute cerebellar ataxia in
children. Clin Pediatr (Phila) 2003; 42:581.
2. Poretti A, Benson JE, Huisman TA, Boltshauser E. Acute ataxia in children: ap-
proach to clinical presentation and role of additional investigations. Neuropediat-
rics 2013; 44:127.
3. Rudloe T, Prabhu SP, Gorman MP, et al.. The Yield of Neuroimaging in Children
Presenting to the Emergency Department With Acute Ataxia in the Post-Varicella
Vaccine Era. J Child Neurol 2015; 30:1333.
4. Ryan MM, Engle EC. Acute ataxia in childhood. J Child Neurol 2003; 18:309.
5. Shiihara T, Kato M, Konno A, et al.. Acute cerebellar ataxia and consecutive cerebel-
litis produced by glutamate receptor delta2 autoantibody. Brain Dev 2007; 29:254.
6. Whelan HT, Verma S, Guo Y, et al.. Evaluation of the child with acute ataxia: a sys-
tematic review. Pediatr Neurol 2013; 49:15.
7. Gieron-Korthals MA, Westberry KR, Emmanuel PJ. Acute childhood ataxia: 10-
year experience. J Child Neurol 1994; 9:381.
8. Go T. Intravenous immunoglobulin therapy for acute cerebellar ataxia. Acta Pae-
diatr 2003; 92:504.

39
08 Case
A 7-year-old girl, whose eyelids began drooping one year ago, now
presented with horizontal diplopia, right eyelid ptosis, swallowing
dysfunction, easy fatigability, and hypophonation. Her medical history was
significant for persistent asthma and recurrent pneumonia requiring two recent
hospitalisations and one admission to the ICU where she was intubated. Over
the following twelve hours, the patient developed hypoxia with respiratory
distress and bilateral ptosis with facial muscle weakness. She was transferred
to the intensive care unit on 50% oxygen where she deteriorated and required
continuous positive airway pressure.
Physical examination revealed that she was well nourished. Her speech
was noted to be slurred after continuous talking associated with significant
drooling with mild exophthalmos. She also had extraocular weakness on
sustained upward gaze and mild-to-moderate weakness of neck flexion and
extension. She was awake and alert. Recent and remote memory was intact.
Visual acuity and fundoscopic examination was normal bilaterally, but there
was nearly complete right eyelid ptosis. Both pupils were 4 mm and reacted
briskly to light, both directly and consensually. She complained of diplopia
on extreme right and left upgaze as well as horizontal medial and lateral gaze.
There was no ataxia, dysmetria, or tremor. Her gait, including tandem, reverse
tandem, skipping, running, tone, bulk of all group muscles were all within
normal limits. Her deep tendon reflexes were 1+. Fine motor coordination
was intact, symmetrical, and within normal limits. Computed tomography
and MRI were normal. An ANA titer was 1:40. She had electrodecremental
response on repetitive stimulation with Electromyography (EMG).

Answer the following questions:


• Give the most likely diagnosis.
• Provide the differential diagnosis.
• Discuss the appropriate diagnostic work-up.
• Discuss the management.

40
Case 08

DIAGNOSIS
Juvenile Myasthenia Gravis

SUMMARY
A 7-year-old girl, she presented with drooping of eyelids, for the last one year;
now acute history of horizontal diplopia, right eyelid ptosis, swallowing dys-
function, facial weakness, and hypophonation. Lab tests were unremarkable
with the exception of her acetylcholine receptor anti-body titer, which was 1.5
(normal <0.7). Having positive EMG and acetylcholine receptor anti-body
results, the diagnosis of juvenile myasthenia was made. An edrophonium test
resulted in an immediate resolution of her ptosis.

Where is the Localisation?


The distribution of this patient’s weakness in addition to the absence of long
tract signs suggests a disorder of the peripheral nervous system. The constel-
lation of findings, including ptosis, ophthalmoplegia, diplopia, swallowing
dysfunction, and hypophonation, are consistent with a disorder of the neu-
romuscular junction. That the weakness worsens over the course of the day
(fatigability) further supports the diagnosis of a process involving the neuro-
muscular junction. Hyporeflexia is also suggestive of a motor unit disorder.

DIFFERENTIAL DIAGNOSIS
• Botulism.. History of ingesting food tainted with botulinum toxin.
Descending paralysis begins in the bulbar muscles then the limbs, face,
neck, and respiratory muscles. Respiratory muscles are involved with mild
limb weakness, and reflexes are usually preserved. Ptosis, dilated non-re-
active pupils are present. Constipation is also a characteristic feature of
botulism.
• Miller Fisher Syndrome (MFS). The involvement of CNS is very dis-
tinct in this form of GBS. Ocular motor nerves (oculomotor, trochlear,
and abducens) are affected and produce a triad of ophthalmoplegia, ataxia,
and areflexia and with normal electrophysiology. The characteristic au-
toantibodies are against gangliosides GQ1b and GT1a. GQ1b plays a key
role in the pathogenesis of MFS and diagnosis.
• Kearns-Sayre Syndrome. People with Kearns-Sayre syndrome have pro-
gressive external ophthalmoplegia, which is weakness or paralysis of the
eye muscles that impair eye movement and cause drooping eyelids (ptosis).
Affected children have a short stature and often have multiple endo-
crinopathies including diabetes mellitus, hypoparathyroidism, and
Addison’s disease. Renal tubular acidosis (proximal or distal) has been de-
scribed in numerous cases, with occasional progression to end-stage renal
failure.

41
Child Neurology

• Lambert-Eaton Myasthenic Syndrome (LEMS). Can be difficult to


differentiate because of similar clinical characteristics. However, some
characteristics are more typical for LEMS. These include slower devel-
opment of clinical symptoms, dry mouth, lack of objective sensory loss,
rare involvement of respiratory muscle group, and potentiation of reflexes
after exercise or contraction.
• Brainstem gliomas. Tumours that occur in the region of the brain
between the aqueduct of Sylvius and the fourth ventricle. Brainstem
gliomas account for approximately 10-20% of all childhood brain tu-
mours. Common presenting signs and symptoms include Double vision,
Weakness, Unsteady gait, Difficulty in swallowing, Dysarthria, Headache,
Drowsiness, Nausea, Vomiting, and Deterioration of handwriting.
• Bickerstaff ’s brainstem encephalitis. It is characterised by ophthal-
moplegia, ataxia, and disturbance of consciousness. It is similar to the
Miller Fisher syndrome, a variant of Guillain-Barre syndrome, in that
they share features such as ophthalmoplegia and ataxia. The difference
is that patients with Bickerstaff ’s brainstem encephalitis have impaired
consciousness, whereas patients with Miller Fisher syndrome have alert
consciousness and areflexia.
• Congenital myopathies Centronuclear/Myotubular (CNM) nema-
line (ROD) myopathy. Nemaline (rod) myopathy presents with general
features of all nemaline myopathies including minimally progressive or
non-progressive proximal limb, bulbar, and facial weakness starting in the
neonatal or childhood periods; hypotonia, and respiratory insufficiency,
which is the most common cause of death. CNMX; X-linked mutations
in myotubularin (MTM1): Affected child often present in utero or early
with decreased foetal movements and polyhydramnios. At birth, severe
weakness and hypotonia, feeding difficulty, and respiratory distress are
present. Bilateral ptosis, facial weakness, and ophthalmoplegia are com-
mon. Skeletal features include pectus carinatum, micrognathia, knee and
hip contractures, elongated birth length, narrow face, slender/long digits,
and macrocephaly.

DIAGNOSTIC WORK-UP
JMG is primarily a clinical diagnosis with classical patterns of fluctuating weak-
ness and fatigability and a number of diagnostic tools are available to aid with
diagnosis. In very young children it is particularly important to distinguish
between autoimmune myasthenia and Congenital Myasthenic Syndromes
(CMS) as the treatment options, prognosis, and genetic implications are very
different.
• CMS. Usually present in the first years of childhood with variable dis-
ability. There is often a positive family history, and diagnosis is aided
primarily by electrophysiology and DNA analysis and occasionally by
muscle biopsy. With the exception of the autosomal dominantly inherited

42
Case 08

slow-channel syndrome, the CMS are inherited by autosomal recessive


mutations, which result in loss of function at the neuromuscular junction.
• Serology. Detection of anti-bodies to the AChR supports the diagnosis of
JMG. Some of these children who are negative for AChR anti-bodies will
have “low affinity” anti-bodies to the AChR which were not detectable
using the standard assays. In young children where AChR anti-bodies are
negative, this can lead to difficulty in differentiating from CMS.
• Pharmacological investigation. The diagnosis of myasthenia gravis can
often be confirmed with an edrophonium (Tensilon) test. Edrophonium
is a short-acting (5 minutes) acetylcholinesterase inhibitor that improves
neuromuscular transmission. The patient is observed, and ideally a video
recorded, looking for a transient improvement in previously documented
weakness, for example, ptosis, dysphonia. This test is not without risk
and should only be performed by staff experienced in paediatric resuscita-
tion, due to the cholinergic effects of edrophonium, which can result in
bradycardia, nausea, and excess salivation. Neostigmine can be used as an
alternative to edrophonium in young patients. Another possible alterna-
tive pharmacologic test to edrophonium is a trial of pyridostigmine, with
the response to drug administration observed over time.
• Electrophysiology.. Repetitive nerve stimulation is one test that can be used
to aid in diagnosing MG. Low-frequency stimuli are given, and a decrease in
response from the first to the fifth response of 10% is considered pathologic.
It is not specific for MG, however. Single-fibre Electromyography
(EMG) is considered to be the most sensitive method to detect abnormal
neuromuscular transmission; however, the patient must be cooperative
for the test. Stimulation single fibre EMG of the orbicularis oculi can be
performed on conscious children. It can be done under local or even general
anaesthesia. Sensitivity for a neurotransmission disorder is 97%.
• Other clinical tests. A simple test to perform in the office for ocular
features of MG is to look for fatigability in sustained upgaze. The patient’s
palpebral fissure height and motility are examined first. Then the patient
is asked to look at a target above primary gaze for a prolonged period of
time. Observation during this period may reveal a gradual increase in pto-
sis or a change in ocular alignment. Following this period of sustained
upgaze, the palpebral fissure height and motility examination can be re-
peated, with a change in measurements confirming fatigability.
• Ice pack test. An alternative test suggestive of MG that can be per-
formed in the office is the ice test. In this test, an ice pack or ice-filled
glove is placed over one or both eyes for two minutes, and then removed.
Improvement of ptosis by 2 mm or greater is considered to be a positive
result, and is approximately 80% sensitive.
• Imaging. AChR seropositive MG is frequently associated with chang-
es in the thymus, with histological changes and in vitro effects suggest-
ing that the thymus plays a pathogenic role. Thymus hyperplasia is the

43
Child Neurology

commonest abnormality of the thymus in JMG. Thymoma is particularly


rare in prepubertal children but the thymus must be imaged (usually by
CT) once JMG has been diagnosed.

MANAGEMENT
Management of children with JMG should be delivered by a multidisciplinary
team comprising of a paediatrician with support from a paediatric neurologist,
a physiotherapist, an occupational therapist, a psychologist, a speech therapist
and a dietician. There are few studies looking specifically at interventions in
children, particularly pre pubertal children.
• Acetylcholinesterase inhibitors. Acetylcholinesterase inhibitors are
first-line treatment in JMG and provide symptomatic relief. In mild cas-
es and in some cases of ocular MG, acetylcholinesterase therapy may be
sufficient. Pyridostigmine is a long-acting cholinesterase inhibitor that is
commonly used. Dosing is usually 4-6 times per day and is tailored to
effects.
• Oral steroids are often the initial immunosuppressive agent used in con-
trolling MG, and they work by decreasing autoantibody circulation. The
conversion from OMG to generalised MG may be prevented or delayed
by the use of steroids.
Azathioprine has been used as an immunosuppressant agent for MG;
however, it can cause leucopaenia, pancytopaenia, and is considered car-
cinogenic, which should be taken into account when it is prescribed to
children. Azathioprine’s treatment effect is delayed from the initial dose
by several months.
• Immunomodulatory agents. Rituximab has been reported in sever-
al cases to improve symptoms and to reduce or eliminate the need for
steroids.
Mycophenolate mofetil is another medication that has been used for MG.
Tacrolimus has also been used as a steroid-sparing agent in children who
have MG.
• Other treatment modalities. Use of plasmapheresis has been reviewed,
and little evidence has been found supporting or refuting its use in MG,
despite studies examining this treatment modality.
In contrast, Intravenous Immune Globulin (IVIG) has been suggested
and used for acute exacerbations of MG. A comparison of IVIG to plas-
mapheresis as maintenance therapy for MG in pediatric patients found
that both modalities had high response rates, but plasmapheresis had a
more consistent response.
• Thymectomy. In children, cases of thymoma associated with MG have
been reported but are exceedingly rare. Thymectomy may have a role,
especially in children who have generalised disease that is unrespon-
sive to traditional therapies within one year of diagnosis. The long-term

44
Case 08

consequences of thymectomy in children remain unclear. Additionally,


there may be a protective effect from thymectomy on development of
generalised symptoms in children, and it may control OMG as well.
However, there are no randomised controlled trials evaluating the effec-
tiveness of thymectomy in patients without a thymoma.
• Outcome.. in JMG outcome has improved significantly over the last dec-
ade, with better recognition, diagnosis, and more effective therapies, and
long-term prognosis is good. Children with JMG exhibit higher rates of
remission than adults. This includes spontaneous remission following
a period of drug therapy. Prepubertal children have the highest rates of
spontaneous remission.

REFERENCES
1. Silvestri NJ, Wolfe GI. Myasthenia gravis. Semin Neurol 2012; 32:215.
2. Carr AS, Cardwell CR, McCarron PO, McConville J. A systematic review of pop-
ulation based epidemiological studies in Myasthenia Gravis. BMC Neurol 2010;
10:46.
3. Heldal AT, Eide GE, Gilhus NE, Romi F. Geographical distribution of a seroposi-
tive myasthenia gravis population. Muscle Nerve 2012; 45:815.
4. Mombaur B, Lesosky MR, Liebenberg L, et al.. Incidence of acetylcholine receptor-
anti-body-positive myasthenia gravis in South Africa. Muscle Nerve 2015; 51:533.
5. Breiner A, Widdifield J, Katzberg HD, et al.
al. Epidemiology of myasthenia gravis in
Ontario, Canada. Neuromuscul Disord 2016; 26:41.
6. Werner P, Kiechl S, Löscher W, et al.. Distal myasthenia gravis frequency and clinical
course in a large prospective series. Acta Neurol Scand 2003; 108:209.
7. Mantegazza R, Beghi E, Pareyson D, et al.. A multicentre follow-up study of 1152
patients with myasthenia gravis in Italy. J Neurol 1990; 237:339.
8. Silvestri NJ, Wolfe GI. Myasthenia gravis. Semin Neurol 2012; 32:215.
9. Díaz-Manera J, Rojas García R, Illa I. Treatment strategies for myasthenia gravis:
an update. Expert Opin Pharmacother 2012; 13:1873.
10. Mehndiratta MM, Pandey S, Kuntzer T. Acetylcholinesterase inhibitor treatment
for myasthenia gravis. Cochrane Database Syst Rev 2014.
11. Ionita CM, Acsadi G. Management of juvenile myasthenia gravis. Pediatr Neurol
2013; 48:95.
12. Ebadi H, Barth D, Bril V. Safety of plasma exchange therapy in patients with myas-
thenia gravis. Muscle Nerve 2013; 47:510.
13. Patwa HS, Chaudhry V, Katzberg H, et al. Evidence-based guideline: intravenous
immunoglobulin in the treatment of neuromuscular disorders: report of the Ther-
apeutics and Technology Assessment Subcommittee of the American Academy of
Neurology. Neurology 2012; 78:1009.
14. Evoli A. Acquired myasthenia gravis in childhood. Curr Opin Neurol 2010; 23:536.

45
09 Case
A 5-month-old girl, developed benign neonatal jitteriness soon after birth
that subsequently resolved at 3 months of age. At 5 months of age, she
developed left monocular nystagmus and head shaking in a “no-no” pattern.
She was brought at five months with presents to our office for complaints of
abnormal eye movements, titubation and nystagmus that had persisted and
worsened since she was 4 months. She has otherwise been well without fever,
irritability, or other signs of illness. She is currently able to crawl, pull to a
stand, babble, and wave bye-bye.
On physical examination: No dysmorphic facial features. There was
occasional nodding of the head up and down. Otherwise she was alert and
playful. The fundi were difficult to visualise. Her pupils were equal, round,
and reactive to light. There was no afferent pupillary defect. She could
track objects in all directions and her visual fields appeared to be full. Fine,
intermittent, horizontal nystagmus was noted in the left eye. Her face was
symmetric with palate elevates symmetrically and her tongue was midline
without fasciculations. Normal tone with symmetric limb movement. She
could sit well without support, and crawl. There was no dysmetria grabbing
for toys. Reflexes: 2+ throughout with bilateral plantar flexor responses.

Answer the following questions:


• Give the most likely diagnosis.
• Provide a differential diagnosis.
• Discuss an appropriate diagnostic work-up.
• Discuss the management of the patient.

46
Case 09

DIAGNOSIS
Spasmus Nutans

SUMMARY
A normal 5 months old girl who presented for evaluation of the subacute onset
of monocular nystagmus and titubation. Her vision, as well as the remainder
of her neurologic exam appears to be normal.

DIFFERENTIAL DIAGNOSIS
Spasmus nutans is a paediatric disorder that consists of the triad of ocular oscil-
lations, head nodding, and head turning (torticollis). All three are not required
for the diagnosis; head nodding and nystagmus are most common, with head
turn in only 30 percent. The age at onset is usually 6 to 12 months, but can be
up to several years.
Spasmus nutans was more common at the turn of the century; it was attrib-
uted to poor socioeconomic conditions, inadequate diet, and light deprivation
since it seemed to occur in winter and resolve in the spring. A hereditary com-
ponent is suggested by its occurrence in monozygotic twins.
Delayed maturation of eye movement circuits with prolonged time delays are
hypothesised to underlie this disorder, possibly involving the saccadic system.
However, saccades are normal in most. The nystagmus is a small, fast oscil-
lation; usually horizontal, sometimes oblique. A key feature is disconjugacy.
Spasmus nutans can be unilateral or bilateral with asymmetric size, and when
bilateral it is not synchronized between the two eyes. The nystagmus can even
change within a session. It is often intermittent, with bursts of 5 to 30 seconds.
The head motion is often oblique, with both nodding and shaking. It is slower
and larger than nystagmus. Head nodding in spasmus nutans is not thought to
be pathological but rather a voluntary nod as a neurovisual adaptation to com-
pensate for nystagmus and to improve vision. Head nodding is not specific for
spasmus nutans since it occurs in about 10 percent of children with congenital
nystagmus.
Strabismus and amblyopia are seen in 60 percent of children. These in turn
can be associated with latent nystagmus and dissociated vertical divergence.
Refractive errors such as anisometropia and astigmatism are more frequent.
Some cases are associated with underlying retinal or neurologic problems. The
features of nystagmus do not differentiate more benign spasmus nutans from
that with these other problems.
• Retinal conditions include achromatopsia and congenital stationary night
blindness.
• Neurologic problems consist of underlying tumours in about 1 percent
of cases. These are usually gliomas of the intracranial visual system, but
sometimes can be ependymomas of the fourth ventricle and posterior

47
Child Neurology

thalamic tumours. A vertical or see-saw nystagmus should raise the sus-


picion of tumour. Other rarely reported neurologic associations include
Leigh’s encephalopathy, arachnoid cysts, and porencephalic cysts.
*Clinical features distinguish spasmus nutans from congenital nystagmus. A
positive family history and infantile myopia are clues to congenital station-
ary night blindness; resulting into searching nystagmus, the diagnosis is made
with Electroretinogram (ERG).
• Congenital nystagmus typically begins at an earlier age (within the first
2 months of life). Titubation is much less common in congenital nystag-
mus, occurring in only 10% of patients, and is of smaller amplitude than
that seen in Spasmus Nutans.
• In addition, in contrast to Spasmus Nutans congenital nystagmus is as-
sociated with jerk and pendular eye movements that are conjugate and
constant.
• Diseases causing cerebellar dysfunction, such as meningitis, encephali-
tis, head trauma, posterior fossa tumours, and medication ingestion, may
cause nystagmus.
• Less commonly, patients with inborn errors of metabolism, such as
Pelizaeus-Merzbacher present with nystagmus.
In Spasmus Nutans associated signs that suggest tumours include optic atro-
phy, visual loss, failure to thrive, and hydrocephalus. MRI of the brain or orbits
is required if such features are present. Unlike congenital nystagmus, spasmus
nutans is considered transient, lasting a few months to years. However, clinical
resolution does not always occur; even when it does, residual oscillations are
seen on eye movement recordings.
In the long term,m, visual acuity is good, 20/50 or better in most. Slight reduc-
tions in acuity may reflect strabismic amblyopia and are not correlated with the
nystagmus. Developmental delay is seen in a few children.
• The diagnosis of Spasmus nutans is one of exclusion. Transient monocu-
lar or binocular nystagmus with titubation and head tilt may be seen in
other conditions.
• Most importantly, patients with structural abnormalities in the anterior
visual pathway may present with these symptoms. Specifically, a tumour
of the suprasellar region such as a chiasmatic and/or optic glioma should
be ruled out.
• Spasmus nutans is like nystagmus (condition where children have nystag-
mus, head nodding and abnormal head position similar to SN but symp-
toms are associated with low vision).
• Optic nerve hypoplasia.
• Congenital stationary night blindness.
• Bardet-Biedl syndrome.

48
Case 09

NEUROLOGIC DISEASES
• Optic Pathway Glioma
• Arachnoid Cyst
• Opsoclonus myoclonus syndrome

DIAGNOSTIC WORK-UP
• The cause of spasmus nutans is unknown.
• The evaluation of a patient with spasmus nutans should begin with a com-
plete, dilated eye exam performed by an ophthalmologist. The character
of the nystagmus and visual acuity should be assessed. Examination of the
fundus will rule out retinopathies as well as optic nerve abnormalities.
• Electroretinography (ERG) should be considered in children with spas-
mus nutans and clinical signs of a retinal dystrophy such as myopia or
paradoxical pupils.
• The diagnosis of spasmus nutans warrants neuroimaging, usually MRI
with and without contrast, as this is generally more sensitive in revealing
structural abnormalities pathology than a CT scan.
• Neuroimaging abnormalities, including tumour and aplasia of the cer-
ebellar vermis, have been described in patients with spasmus nutans, but
this is a rare association.

MANAGEMENT
• Spasmus nutans is a self-limiting disorder and the benign form of spasmus
nutans requires no treatment
• If the symptoms are caused by another condition, that condition must be
treated appropriately

REFERENCES
1. Young TL, Weis JR, Summers CG, Egbert JE. The association of strabismus, am-
blyopia, and refractive errors in spasmus nutans. Ophthalmology 1997; 104:112.
2. Jan JE, Groenveld M, Connolly MB. Head shaking by visually impaired children:
A voluntary neurovisual adaptation which can be confused with spasmus nutans.
Dev Med Child Neurol. 1990; 32(12):1061-1066.
3. Wizov SS, Reinecke RD, Bocarnea M, et al. A comparative demographicand so-
cioeconomic study of spasmus nutans and infantile nystagmus. Am J Ophthalmol
2002; 133:256.
4. Alper I. Dai, Oguzhan Saygili. Risk Factors in Spasmus Nutans. Adv Clin Exp Med
2011, 20, 2, 183-186.
5. Quiros PA, Yee RD. Nystagmus, saccadic intrusions, and oscillations. In: Yanoff M,
Duker JS, eds. Ophthalmology. 5th ed. Philadelphia, PA: Elsevier Sunders; 2016:
chap 9.19.
6. Gottlob I, Wizov SS, Reinecke RD. Quantitative eye and head movement record-
ings of retinal disease mimicking spasmus nutans. Am J Ophthalmol 1995; 119:374.

49
Child Neurology

7. Ehrt O. Infantile and acquired nystagmus in childhood. Eur J Paediatr Neurol. 2012
Nov. 16(6):567-72.
8. Akman OE, Broomhead DS, Abadi RV, Clement RA. Components of the neural
signal underlying congenital nystagmus. Exp Brain Res. 2012 Aug. 220(3-4):213-21.
9. Richards MD, Wong A. Infantile nystagmus syndrome: clinical characteristics, cur-
rent theories of pathogenesis, diagnosis, and management. Can J Ophthalmol. 2015
Dec. 50 (6):400-8.
10. American Academy of Ophthalmology. Neuro-Ophthalmology, Basic and Clinical Sci-
ence Course.. San Francisco, Calif: American Academy of Ophthalmology; 2011.
Section 5.
11. Lorenz B, Moore A, eds. Management of Congenital Nystagmus with and without
Stabismus. Pediatric Ophthalmology, Neuro-Ophthalmology, Genetics (Essentials in Oph-
thalmology). New York, NY: Springer; 2010. 153-71.

50
Case 10
A 7-year-old girl presented with a one-year history of gait difficulties, toe-
walking and frequent falls. Her birth was uneventful and development
was normal. The parents noticed abnormal gait at the age of 6 years. Gradually,
she developed instability of posture. She would walk with her eyes fixed on
the ground. The symptoms used to worsen by evening and improve with
sleep. These diurnal variations were observed for several days in the hospital
also. There was no history of cognitive decline, developmental retardation/
regression, behavioural problems, tremors or seizures. There was no family
history for any neurological or liver disease.
Physical examination revealed a mentally normal girl with no pallor or
icterus. There was no hepatosplenomegaly. Central nervous system exami-
nation revealed normal tone and power. Deep tendon reflexes were normal.
Plantar reflexes were however, bilaterally extensor. No sensory deficits were
observed and Romberg was negative. Gait (examined when patient had been
active for several hours) revealed toe-walking and hypertonia of tendoachillis.
Her pupils were equal, round, and reactive to light. Her extraocular muscles
were intact. Her face was symmetric. Investigations showed normal haemo-
gram, liver and renal function tests. Serum ceruloplasmin level was normal.
MRI brain did not reveal any abnormality.

Answer the following questions:


• Give the most likely diagnosis.
• Provide a differential diagnosis.
• Localise the examination findings.
• Discuss an appropriate diagnostic work-up.
• Discuss the management of the patient.

51
Child Neurology

DIAGNOSIS
Dopa-Responsive Dystonia (DRD, Segawa disease)
(Segawa disease, DYT 5, dystonia-parkinsonism with diurnal variation, he-
reditary progressive dystonia with marked diurnal fluctuation)

SUMMARY
A 7-year-old girl presented with gait difficulties, toe walking, instability of
posture, and frequent falls with gait abnormality. There was no history of cog-
nitive decline, developmental retardation/regression, behavioural problems,
tremors or seizures. She was started on 1 mg/kg/d of levodopa as diagnos-
tic trial for possibility of DRD (Dopa Responsive Dystonia) in three divided
doses, in a view of generalised dystonia associated with diurnal variation after
exclusion of Wilson’s disease. Within a few days of therapy, she showed sig-
nificant improvement.

Localisation of Lesion
The patient’s neurologic examination is consistent with a dystonia. A lesion
involving the basal ganglia can account for these signs.

DIFFERENTIAL DIAGNOSIS
Dopa Responsive Dystonia (DRD) manifests primarily as a focal dystonia with
onset in early childhood. Parkinsonism, including rigidity and bradykinesia,
may be present at onset or develop during the course of untreated disease.
Often, children with DRD are initially misdiagnosed as having a primary dys-
tonia or cerebral palsy. In the original description of DRD, the investigators
observed a diurnal fluctuation in symptoms, which worsened over the course
of a day and improved following sleep. However, this diurnal fluctuation may
not be present in all patients with DRD.
The hallmark of DRD is a clinically significant, sustained response to levo-
dopa. The most frequent form of DRD is autosomal dominant DYT5 dystonia
caused by a mutation in the Guanosine Triphosphate (GTP) cyclohydrolase
1 gene (GCH1). The GTP cyclohydrolase 1 protein encoded by this gene is
involved in the biosynthesis of tetrahydrobiopterin, which is a cofactor for
tyrosine hydroxylase, the rate-limiting enzyme in the synthesis of dopamine.
It is also a cofactor for phenylalanine and tryptophan hydroxylase. Numerous
mutations of the GCH1 gene can cause DRD, making gene testing difficult.
An autosomal recessive form of DRD (Segawa syndrome) is caused by muta-
tion in the tyrosine hydroxylase gene. Other rare entities that can present with
a phenotype of DRD include mutations in the Sepiapterin Reductase gene
(SPR gene) and the 6-Pyruvoyl-Tetrahydropterin Synthase gene (PTS gene).
In the clinical setting, the most practical and useful diagnostic test for DRD is
a markedly positive response to a trial of levodopa, slowly increased to doses
of 600 to 1,000 mg daily. However, a positive response to levodopa does not

52
Case 10

differentiate DRD from juvenile onset Parkinson disease. Typically, patients


with DRD will have a sustained benefit from low doses of levodopa without
developing motor fluctuations and dyskinesia, in contrast to juvenile Parkinson
disease, in which these motor complications are a frequent occurrence.
Other proposed diagnostic methods for DRD include assessment of tetrahy-
drobiopterin and neopterin in the cerebrospinal fluid, and the phenylalanine-
loading test. However, most clinicians rely on confirmation with levodopa
and do not undertake more extensive laboratory testing. Genetic testing for
GCH1 is available, although there are many mutations that may give rise to
the disorder.

DIFFERENTIAL DIAGNOSIS OF GENERALIZED DYSTONIA


IN CHILDHOOD
Dystonias are typically classified as primary and secondary disorders. By defi-
nition, primary disorders have dystonia as their only clinical feature. Secondary
dystonias result from well-defined acquired or exogenous causes. In these cas-
es, dystonia is typically accompanied by other neurologic abnormalities.

PRIMARY DYSTONIA
• Primary dystonias (DYT 1-25)
• Dopa-responsive dystonia (DYT 5a, DYT 5b DYT 14)

SECONDARY DYSTONIA
Basal Ganglia Structural Abnormalities
• Tumour
• Trauma
• Haemorrhage
• Infarction
• Arteriovenous malformation
• Central nervous system infection
• Demyelinating disease

Genetic and Hereditary Disorders


• Wilson’s disease
• Huntington disease
• Neuroacanthocytosis
• Juvenile Parkinson disease

Inborn Errors of Metabolism


• Glutaric aciduria type 1
• Pantothenate kinase-associated neurodegeneration

53
Child Neurology

• Mitochondrial disorders
• Leukodystrophies
• Lysosomal storage diseases
• Neuronal ceroid lipofuscinosis
• Lesch-Nyhan syndrome
• Hartnup disease

Other
• Toxins/drugs
• Psychiatric/conversion reaction
• Endocrine abnormalities

DIAGNOSTIC WORK-UP
• DRD usually presents itself in childhood between the ages of 6 and 16 and
is 2-4 times more likely to affect girls than boys. Symptoms of DRD are
often worse later in the day (this phenomenon is called diurnal fluctua-
tion and may increase with exertion. The most commonly identified form
of dopa-responsive dystonia is sometimes referred to as DYT5 dystonia.
• A three-generation family history with attention to other relatives with
neurologic signs and symptoms should be obtained.
• Establishing the specific cause of dystonia for a given individual usually
involves a medical history, physical examination, neurologic examination,
and neuroimaging, and use of molecular genetic testing. It is especially
important to look for treatable causes of dystonia such as dopa-responsive
dystonia (DYT-GCH1, DYT-TH, and DYT-SPR), Wilson’s disease, and
other rare metabolic disorders and toxic or drug-related associations.
• Dopa-responsive dystonia should always be considered in any child with
primary dystonia that worsens during the day with marked benefit from
sleep. Diagnosis of DRD can often be made on clinical grounds.
• Important features are age of onset, site of onset, presence or absence of
other neurologic abnormalities, and presence of non-neurologic abnor-
malities (e.g. developmental delay, dysmorphic features). Presence or ab-
sence of associated findings may help distinguish among isolated dystonia,
and complex dystonia.
• Once the diagnosis of dystonia has been established in an individual, the
following approach can be used to determine the specific cause of dystonia
to aid in discussions of prognosis and genetic counselling.
• Response to levodopa is impressive and a therapeutic trial with this drug
is recommended in suspected idiopathic dystonia.
• Although DRDs are easily treatable, they have a potential to be under di-
agnosed or misdiagnosed when not suspected, and over diagnosed if the
L-dopa response is unclear, for example, in psychogenic dystonia.

54
Case 10

• Lab studies used in the diagnosis and evaluation of Dopamine-


Responsive Dystonia (DRD) include the following:
• Complete Blood Count (CBC) with peripheral smear examination-to
rule out acanthocytosis.
• Serum studies for Blood Urea Nitrogen (BUN), creatinine, liver func-
tion, copper, and ceruloplasmin.
• Cerebrospinal Fluid (CSF) studies: CSF examination is not performed
routinely, but some subjects may show significant reductions in CSF lev-
els of neopterin and biopterin.
• Polysomnography.. Polysomnography in DRD shows a decreased num-
ber of twitch movements during REM sleep (approximately 20% of nor-
mal). The ratio does not decrease with age, and it does not follow the
decremental age variation and incremental nocturnal variation of healthy
subjects.
• Phenylalanine loading test.. Abnormality in phenylalanine metabolism
has been useful in diagnosing DRD for most (50% of patients), but not
all, patients. The basis for this test is that BH4 (Tetrahydrobiopterin) is
required as a cofactor in the breakdown of phenylalanine to tyrosine. In
DRD, BH4 deficiency results in accumulation of phenylalanine.
• Molecular biology. This can confirm the diagnosis in some cases.
• Imaging studies.. MRI Brain magnetic resonance imaging (MRI) may
show abnormalities in the basal ganglia, suggesting Wilson’s disease or
Hallervorden-Spatz disease.
• PET scanning.. PET scan uptake of [18 F] dopamine may be reduced in
early onset Parkinson disease, but it is normal in DRD.
• SPECT scanning. Single-Photon Emission Computed Tomography
(SPECT) scanning with iodine-123 (123 I) 2-beta-carbomethoxy-3beta-(4-
iodophenyl) tropane (b-CIT) can differentiate DRD (normal) from early
onset Parkinson’s disease (reduced).

MANAGEMENT
• The goals of treatment in patients with ITD/DRD are to reduce spasms,
decrease abnormal posturing, prevent contractures, and restore functional
abilities as much as possible. Response to medical management is highly
variable and there are no standard protocols.
• All patients with Dopamine-Responsive Dystonia (DRD) should be treat-
ed with the levodopa/carbidopa combination. Early treatment can prevent
morbidity and contracture formation.
• Patients with Dopamine-Responsive Dystonia (DRD) typically experi-
ence marked, long-term benefit with low-dose levodopa. The optimal
dose differs among patients; while some respond magnificently to small
doses, others require higher doses.

55
Child Neurology

• Other effective medications include the anti-cholinergic agents, such as


trihexyphenidyl, carbamazepine, BH4 (Tetrahydrobiopterin), and 5-hy-
droxytryptophan. The use of Botulinum Toxin (BOTOX) injection for
focal dystonia should be considered in resistant cases as it would be for
any cause of focal dystonia; this recourse rarely is needed in true DRD.
• Surgical care-A fixed equinovarus foot deformity has been corrected sur-
gically after treating the dystonia with levodopa.
• Physical therapy is particularly important if the patient has a contracture
or chronic gait disturbance.
• Follow-up-Regular
Regular outpatient follow-up is required for patients with
DRD to assess the efficacy of treatment and to adjust the dopamine dose
accordingly. Although uncommon, dyskinesias and chorea may develop in
treated patients. Monitor patients carefully for these conditions.

REFERENCES
1. Weber YG, Kamm C, Suls A, et al.. Paroxysmal choreoathetosis/spasticity (DYT9) is
caused by a GLUT1 defect. Neurology 2011; 77:959.
2. Segawa M, Hosaka A, Miyagawa F, et al.. Hereditary progressive dystonia with
marked diurnal fluctuation. Adv Neurol 1976; 14:215.
3. Segawa M, Nomura Y, Nishiyama N. Autosomal dominant guanosine triphos-
phate cyclohydrolase I deficiency (Segawa disease). Ann Neurol 2003; 54 Suppl
6:S32.
4. Nygaard TG, Trugman JM, de Yebenes JG, Fahn S. Dopa-responsive dystonia: the
spectrum of clinical manifestations in a large North American family. Neurology
1990; 40:66.
5. Trender-Gerhard I, Sweeney MG, Schwingenschuh P, et al.
al Autosomal-dominant
GTPCH1-deficient DRD: clinical characteristics and long-term outcome of 34
patients. J Neurol Neurosurg Psychiatry 2009; 80:839.
6. Jan MM. Misdiagnoses in children with dopa-responsive dystonia. Pediatr Neurol
2004; 31:298.
7. Lüdecke B, Dworniczak B, Bartholomé K. A point mutation in the tyrosine hy-
droxylase gene associated with Segawa’s syndrome. Hum Genet 1995; 95:123.
8. Steinberger D, Blau N, Goriuonov D, et al. Heterozygous mutation in 5’-untrans-
lated region of sepiapterin reductase gene (SPR) in a patient with dopa-responsive
dystonia. Neurogenetics 2004; 5:187.
9. Bandmann O, Goertz M, Zschocke J, et al. The phenylalanine loading test in the
differential diagnosis of dystonia. Neurology 2003; 60:700.
10. Saunders-Pullman R, Blau N, Hyland K, et al. Phenylalanine loading as a diag-
nostic test for DRD: interpreting the utility of the test. Mol Genet Metab 2004;
83:207.

56
Case 11
A 14-month-old male child was seen in the clinic for evaluation of
breath-holding which began at 5 months of age, characterised by perio-
ral cyanosis, loss of consciousness, and prolonged stiffening of the body; but
these spells were not associated incontinence. These spells were precipitated
by emotional stimuli; such as anger or frustration. This child typically cries
vigorously but usually for less than 15 seconds, then becomes silent, and holds
its breath in expiration. Most of these episodes occurred after his older brother
took away a favorite toy. These events occurred 4 to 6 times per week. He is
otherwise a healthy boy with normal growth and development. Family history
is unremarkable.
On general examination: Unremarkable. Neurologic Examination: Awake,
alert, and playful. Language: Well developed and produces phrase of two
words and says several words. Cranial Nerves: All his all accessible cranial
nerves are intact. He has normal bulk and tone. He moves all four extremities
equally against gravity. There is no dysmetria on grabbing for toys. His DTR
is 2+ throughout with bilateral plantar flexor responses.
CT scan brain was done in another hospital after his first episode, which
was normal. An EEG was also performed and the record did not show any
epileptiform discharges. He did not have any cardiac arrhythmias on a Holter
monitor.

Answer the following questions:


• Give the most likely diagnosis.
• Provide a differential diagnosis.
• Discuss an appropriate diagnostic work-up.
• Discuss the management of this patient.

57
Child Neurology

DIAGNOSIS
Cyanotic breath-holding spells

SUMMARY
A 14-month-old healthy male child, developmentally normal who presented
with multiple episodes of breath-holding spells provoked episodes of cry-
ing, apnoea with cyanosis, loss of consciousness, and brief stiffening of the
body. His neurologic examination was normal. His EEG record, CT brain and
Holter monitoring did not show any abnormality.

DIFFERENTIAL DIAGNOSIS
Breath-holding spells are a well described phenomenon known to occur most-
ly among children 6 to 18 months of age. Almost 5% of the pediatric popula-
tion might demonstrate such episodes. Breath-holding spells are extremely
frightening to parents. Episodes are described as infants crying, for up to a
minute, and while crying excessively they will hold their breath to a point at
which they might lose consciousness. A significant minority of children (15 to
25 percent) have multiple episodes daily. Most children have one to six spells
per week. On rare occasions a seizure might be witnessed immediately after
the infant loses consciousness; soon thereafter, the infant will usually regain
consciousness and breathe normally. Breath-holding spells are not harmful
and pose no long-term risks for the infant. Many episodes of breath holding
are associated with an inciting incident in which the infant is irritated, is being
disciplined, or is angry. Examples include when infants are having their hair
splashed in the bath, when they insist on holding a toy, or when they experi-
ence a minor injury. While considered by many to be “attention seeking” be-
haviour, these spells are not intentional; they result from an involuntary reflex,
and the child has no ability to control them. This might mean that maturation
delay in myelination of the brainstem could be the cause of breath-holding
spells in children. There are 2 known types of breath-holding spells. The most
common (85%) is a cyanotic breath-holding spell, which occurs when facial
cyanosis is noticed after the child stops breathing. A pallid breath-holding spell
is associated with a sudden scare to the child, and he or she becomes extremely
pale during the spell.
• Breath holding spells primarily need to be differentiated from epileptic
seizures and cardiac phenomena such as syncope or arrhythmias. In chil-
dren with breath-holding spells, there is a positive family history of simi-
lar episodes in 23-38 percent, suggesting a genetic influence.
• Cyanotic breath-holding spells are often precipitated by emotional stimuli,
such as anger or frustration. The child typically cries vigorously but usu-
ally for less than 15 seconds, then becomes silent, and holds the breath in
expiration The apnoea is associated with the rapid onset of cyanosis. Some
episodes may resolve at this point, but there may be loss of conscious-
ness and a brief period of limpness, followed by opisthotonic posturing.

58
Case 11

Recovery is usually within 1 minute, with the child having a few gasping
respirations and then a return to regular breathing and consciousness.
• Epilepsy: The best way to distinguish epilepsy from breath holding is to
search for a trigger: Epilepsy rarely has one and breath-holding almost al-
ways does. Electroencephalograms (EEG) sometimes show abnormalities
between the episodes that support a diagnosis of epilepsy over BHS. EEGs
usually are normal in children with BHS. Therefore, only an abnormal
EEG with changes suggestive of epilepsy is of value for distinguishing
these conditions.
• Congenital long QT syndrome: The clinical diagnosis of long QT syn-
drome is prompted by a high degree of clinical suspicion, which arises
from the presenting complaints, unexplained bradycardia, especially in
the newborn, Syncope, especially when associated with a triggering event
and palpitations or sudden cardiac death in the patient or family history of
sudden cardiac death.
• Orthostatic syncope: Syncope results from a sudden loss of cerebral
perfusion and causes a loss of postural tone as well as a loss of conscious-
ness. Patients lacking sufficient blood volume or having defective sym-
pathetic adrenergic vasoconstriction develop Orthostatic Hypotension
(OH), prohibiting effective upright activities. For the most part, these
episodes are benign and self-limited, although frequent syncope episodes
can be debilitating, and injury may occur from sudden falls.
• Arnold-Chiari malformation: The signs and symptoms can vary great-
ly from one person to another. Specific symptoms can occur in differ-
ent combinations and generally reflect dysfunction of the cerebellum, the
brainstem, the spinal cord and lower cranial nerves. The most common
symptoms associated with a Chiari malformation are occipital headaches
(worsened by coughing, straining or sneezing), double vision (diplopia),
abnormal sensitivity to light (photophobia), blurred vision, involuntary
eye movements (nystagmus) and pain behind the eyes. Vertigo, dizziness,
ringing in the ears (tinnitus) and bilateral hearing impairment can also
develop.
• Familial dysautonomia: Familial Dysautonomia (FD), sometimes
called Riley–Day syndrome and hereditary sensory and autonomic neu-
ropathy type III (HSAN-III) is a disorder of the autonomic nervous sys-
tem. Clinical symptoms include: Insensitivity to pain, inability to produce
tears, poor growth, and labile blood pressure (episodic hypertension and
postural hypotension). People with FD have frequent vomiting crises,
pneumonia, problems with speech and movement, difficulty swallowing,
inappropriate perception of heat, pain, and taste, as well as unstable blood
pressure and gastrointestinal dysmotility.
• Reflex anoxic seizures result from cardiac asystole of vagal origin. Pain
and surprise are common provoking factors for the events. Reflex anoxic
seizures may occur with pallid breath-holding spells but also have been

59
Child Neurology

reported with minor blows to the occiput, expelling hard stools past an
anal fissure, venipuncture, intramuscular injections, and seeing an intra-
venous scalp drip.
• Pallid breath-holding spells, the most common stimulus is a painful
event. The child turns pale (as opposed to blue) and loses consciousness
with little, if any, crying. The provocation of pallid syncope is usually a
sudden, unexpected, painful event such as a bump on the head. The child
rarely cries but instead becomes white and limp and loses consciousness.
After the initial limpness, the body may stiffen, and clonic movements of
the arms may occur. As in cyanotic syncope, these movements represent a
brainstem release phenomenon, not seizure. Observation of children dur-
ing a typical episode reveals marked bradycardia or asystole. Afterward the
child often falls asleep and is normal on awakening.

DIAGNOSTIC WORK-UP
• A detailed history of the event, including the precipitating circumstances,
is essential in making the diagnosis of breath holding spells.
• If the event was not witnessed from onset, important details may not be
available. A video recording by the parents may be helpful in confirming
the diagnosis.
• Usually, no laboratory tests are needed to make the diagnosis. An
Electroencephalogram (EEG) is usually not indicated, unless the convul-
sive activity is prolonged or the clinical description is incomplete and epi-
leptic seizures cannot be ruled out.
• If ocular compression is performed in patients with pallid breath-holding
spells, there may be asystole on cardiac monitoring, and slowing or sup-
pression of voltage on EEG.
• Long QT syndrome is rare but should be considered as part of the dif-
ferential diagnosis in a child with breath-holding spells. Patients with long
QT syndrome have episodes of loss of consciousness that may be induced
by injury, fright, or excitement. An electrocardiogram should be consid-
ered in any patient with breath-holding spells.

MANAGEMENT
• The most important aspect of treatment of breath-holding spells is to
reassure the family of the benign nature of the spells. It is important to
emphasise that the episodes do not lead to mental retardation or epilepsy.
• There is no need to help your child with their breathing. Your child will
start to breathe again on its own after the spell subsides (and will some-
times cry or scream).
• Parents should be instructed to place the child in a lateral recumbent posi-
tion so as not to prolong the period of cerebral anoxia.

60
Case 11

• Although anger and frustration are often precipitants for breath-holding


spells, parents should be encouraged not to alter customary discipline for
fear of triggering an episode.
• Treatment with iron therapy should be initiated in any child who has iron
deficiency anaemia and should be considered in any child with breath-
holding spells because children without anaemia may have improvement
in their breath-holding spells.
• The convulsive movements seen during breath-holding spells are reflex
anoxic seizures, which are not epileptic and do not require anti-epileptic
treatment.
• Treatment with anti-epileptic medication may stop the seizure activity but
not the breath-holding spells. Atropine (0.01 mg/kg two or three times
daily) is effective for pallid breath-holding spells, but its use is rarely
warranted.
• Piracetam, (not approved from the U.S. Food and Drug Administration)
similar to Gamma-Aminobutyric Acid (GABA), has been used to treat chil-
dren with breath-holding spells in doses ranging from 50 to 100 mg/kg/day.
• By 4 years of age, 50% of children will no longer have episodes. Almost
all children will have stopped having episodes by 7 to 8 years of age. Some
children who have had breath-holding episodes will have fainting epi-
sodes later in life.

REFERENCES
1. Lombroso CT, Lerman P. Breathholding spells (cyanotic and pallid infantile syn-
cope) Pediatrics.1967;39(4):563-81.
2. Evans OB. Breath-holding spells. Pediatr Ann. 1997;26 (7) :410-4.
3. DiMario FJ., Jr Breath-holding spells in childhood. Am J Dis Child.
1992;146(1):125-31.
4. Vurucu S, Karaoglu A, Paksu SM, Oz O, Yaman H, Gulgun M, et al. Breath-hold-
ing spells may be associated with maturational delay in myelination of brain stem.
J Clin Neurophysiol. 2014;31(1):99-101.
5. Isikay S, Hızlı S. Frequency of coeliac disease in children with breath-holding
spells. J Paediatr Child Health. 2014;50(11):916-9. Epub 2014 Jun 13.
6. DiMario FJ., Jr Prospective study of children with cyanotic and pallid breath-hold-
ing spells. Pediatrics.2001;107(2):265-9.
7. Mocan H, Yildiran A, Orhan F, Erduran E. Breath holding spells in 91 children and
response to treatment with iron. Arch Dis Child. 1999;81(3):261-2.
8. Yilmaz U, Doksoz O, Celik T, Akinci G, Mese T, Sevim Yilmaz T. The value of
neurologic and cardiologic assessment in breath holding spells. Pak J Med Sci.
2014;30(1):59-64.
9. Daoud AS, Batieha A, al-Sheyyab M, Abuekteish F, Hijazi S. Effectiveness of iron
therapy on breath-holding spells. J Pediatr. 1997;130(4):547-50.
10. Breningstall GN. Breath-holding spells. Pediatr Neurol 1996; 14:91.

61
12 Case
A 10-year-old boy was examined because of frequent falls. He was born at
full term by spontaneous vaginal delivery. There were no complications
during pregnancy and foetal movements were reported normal. There was
no feeding or breathing difficulties after birth. He appeared to be developing
normally until the age of 3 years when he seemed to be getting weaker. His
parents noted that he was having difficulty walking up and down stairs. He
learned to walk but had a history of falling frequently and having trouble
walking up and down stairs at 3-4 years of age. From the age of 5, he could
not keep up with his peers at sport. After 20 minutes walking, he always
complained of pain and weakness in the legs. The legs are more severely
affected than the arms. At the time of referral, he had problems rising from
a sitting position. He did not have cramps. After climbing a third step on
stairs, he experienced difficulty raising his legs. A Couple of years of physical
therapy had no effect. His physician has done the initial work-up, including
electrolytes, CBC, creatine kinase, thyroid function tests, TORCH titers,
head MRI which were reported normal.
On physical examination. His growth and development landmarks were
normal. There were no dysmorphic features and no organomegaly noted. A
Mentally alert child who was visually attentive. His language was normally
developed both in perception and expression. His facial movements were
symmetrical and tongue was midline with fasciculations. There were fine
tremors in the outstretched fingers. He had slightly decreased bulk with little
hypotonia more marked in the lower limbs. Deep tendon reflexes were absent
throughout with bilateral plantar flexor responses.

Answer the following questions:


• What is the most likely diagnosis?
• Briefly summarise the case.
• Localise the lesion.
• What could be the possible differential diagnosis.
• Discuss the appropriate diagnostic work-up.
• Discuss the management of the patient.

62
Case 12

DIAGNOSIS
Juvenile Spinal Muscular Atrophy (SMA 3, Kugelburg Welander Disease)

SUMMARY
A 10-year-old boy was examined because of frequent falls due to weakness
of the legs and trouble walking up and down stairs at 3-4 years of age. After
climbing a third flight of stairs, he experienced difficulty raising his legs. He
had tongue fasciculations and fine tremors in the outstretched fingers. His
DTR were absent with proximal muscles weakness.

Localisation of Lesion
The diminished deep tendon reflexes, proximal muscles weakness and tongue
fasciculations points to a lower motor neuron problem. Disorders of the ante-
rior horn cell, peripheral nerve, neuromuscular junction, and muscle must be
considered. However, tongue fasciculations indicate involvement of the twelve
cranial nerve motor unit and are usually indicative of anterior horn cell disease.

DIFFERENTIAL DIAGNOSIS
Spinal Muscular Atrophy (SMA) disorders are characterised by degeneration of
the anterior horn cells in the spinal cord and motor nuclei in the lower brain-
stem. These diseases are classified as types 1 through 4 depending upon the age
of onset and clinical course. SMA type 1, also known as infantile spinal mus-
cular atrophy or Werdnig-Hoffmann disease, is the most common and severe
type of SMA. It typically presents in the neonatal period. However, mothers
of affected patients may recognise a decrease or loss of foetal movement in late
pregnancy. In these neonatal forms, symptoms progress rapidly, and the major-
ity of infants die before one year of age from respiratory failure. Nevertheless,
long-term survivors have been reported. This is perhaps due, in part, to ad-
vances in the care of chronic respiratory insufficiency and to more aggressive
care. Type 2 (intermediate form) and type 3 (KugelbergWelander disease) have
a later onset and a less severe course. SMA 2 presents between three and 15
months of age, whereas SMA 3, the least severe, typically presents with signs
of weakness at or after one year of age and progresses to a chronic course. In
a study of children and adolescents with SMA 2 and 3, muscle strength was
reduced to a variable extent. Although the muscle weakness affected motor
function, walking, transfer from lying or sitting to the standing position, and
stair-climbing were possible in some children. The outcome depends primar-
ily upon the severity of muscle weakness at presentation rather than the age of
onset, although earlier onset tends to correlate with greater weakness. Adult
onset of SMA (type 4) usually presents in the second or third decade of life and
is otherwise similar to SMA type 3. Kugelberg Welander syndrome is a milder
type of spinal muscular atrophy. It is a rare inherited disorder of anterior horn
cell characterised by wasting and weakness in the muscles of the arms and legs,
leading to walking difficulties in, and eventual loss of ambulation. Symptoms

63
Child Neurology

of Kugelberg Welander syndrome occur after 12 months of age. Patients learn


to walk, may fall frequently and may have trouble walking up and down stairs
at 2-3 years of age; some patients will not show functional changes until the
teens. The legs are more severely affected than the arms. The long-term prog-
nosis depends on the degree of motor function attained as a child.
Kugelberg Welander syndrome is inherited as an autosomal recessive trait.
Molecular genetic testing has revealed that all types of autosomal recessive
SMA (Werdnig-Hoffman disease, juvenile SMA and KugelbergWelander dis-
ease) are caused by mutations in the SMN (Survival Motor Neuron) gene
on chromosome 5. Deletion of the NAIP (Neuronal Apoptosis Inhibitory
Protein) gene that is close to the SMN gene is also associated with SMA.
More patients with Werdnig-Hoffman disease (SMA1) than other types of
SMA have NAIP deletions. The relationship between specific mutations in
the SMN gene and nearby genes and the severity of SMA is still being inves-
tigated, so classification of SMA subdivisions is based on the age of onset of
symptoms and maximum function achieved as opposed to the genetic profile.
• Duchenne Muscular Dystrophy: Children with DMD present present with
delayed motor developmental milestones and they also have mental defi-
cits; association of intellectual impairment in DMD has long been recog-
nized. By age 5 years, awareness increases as the disease is manifested in
all affected children when they experience difficulty with school-relat-
ed activities (e.g., getting to the bus, climbing stairs, reciprocal motions
during activities). Other early features include a gait abnormality, which
classically is a waddling, wide-based gait with hyperlordosis of the lum-
bar spine and toe walking. The waddle is due to weakness in the glu-
teus maximus and gluteus medius muscles. The child then begins having
problems getting up from the sitting or supine position. The Gower sign
is a classic physical examination finding in DMD and results from weak-
ness in the child’s proximal hip muscles. The second important phase in
Duchenne MD is the loss of ambulation. This usually occurs between the
ages of 7 and 13 years, with some patients becoming wheelchair bound
by age 6 years. Cardiopulmonary involvement is present from the begin-
ning of the disease stages, but the findings are not so clinically obvious.
Electrocardiographic (ECG) tracings show right ventricular strain, tall R
waves, deep Q waves, and inverted T waves. A Creatine Phosphokinase
(CPK) determination is the most specific test for Muscular Dystrophy
(MD). Elevated CPK levels are indicative of muscle disease. Early in the
disease process, CPK levels are 50-300 times greater than normal levels.
• Becker Muscular Dystrophy: Becker MD is similar to Duchenne MD,
but because patients have some measure of functioning dystrophin, the
manifestation of Becker MD occurs later and is milder. Patients tend to
live past the fourth or fifth decade's
• Juvenile Metachromatic Leukodystrophy: In this juvenile form,
the first signs of the disorder may be behavioural problems and increas-
ing difficulty with schoolwork. The main clinical presentations are gait

64
Case 12

disturbances, tremors, clumsiness, loss of previously achieved skills,


Intellectual decline, Behavioural changes and seizures (possible). Other
symptoms include muscle wasting and weakness, regression of achieved
developmental milestones, progressive loss of vision leading to blindness.
• Peripheral neuropathies: Neuropathies can be distinguished from
anterior horn cell disease because they tend to primarily affect the distal
muscles and involve the sensory and motor nerves. Affected individuals
have muscle wasting and weakness, as they get older. Weakness in the an-
kle muscles can make walking difficult. Polyneuropathies are character-
ised by prickling or tingling sensations (paresthesias), numbness, and a
reduced ability to feel pain and sense hot and cold
• Juvenile myasthenia gravis: Myasthenia gravis (disease of neuromuscu-
lar junction) is a disorder that causes weakness of the skeletal muscles. The
weakness most often starts in the muscles around the eyes, causing droop-
ing of the eyelids (ptosis) and difficulty coordinating eye movements,
which results in blurred or double vision. Affected individuals may have
unusual facial expressions, difficulty holding up the head, speech impair-
ment (dysarthria), and chewing and swallowing problems (dysphagia) that
may lead to choking, gagging, or drooling
• Mitochondrial Myopathy: The symptoms of mitochondrial myopathies
include muscle weakness or exercise intolerance, heart failure or rhythm
disturbances, dementia, movement disorders, stroke-like episodes, deaf-
ness, blindness, droopy eyelids, limited mobility of the eyes, vomiting, and
seizures. Most mitochondrial myopathies occur before the age of 20, and
often begin with exercise intolerance or muscle weakness. During physical
activity, muscles may become easily fatigued or weak. Muscle cramping is
rare, but may occur
• Primary Carnitine Deficiency: In primary carnitine deficiency, physical
findings may vary depending on the form of presentation. In the myo-
pathic presentation, patients may have mild motor delays, hypotonia, or
progressive proximal weakness. Muscle carnitine deficiency findings are
limited to muscle and can be associated with proximal weakness and signs
of exercise intolerance and cardiomyopathy. This disorder is restricted to
muscles only. Patients with primary carnitine deficiency may present with
cardiomyopathy. Onset may occur with rapidly progressive heart failure or
murmur
• McArdle’s disease (Glycogen storage disease type V): The majority of pa-
tients with McArdle’s disease first present in the first decade of life but are
only diagnosed in the third or fourth decade of life. Clinical presentation
and severity are very variable but patients typically experience reversible
exercise intolerance, fatigue and acute crises. Patients with this disease de-
velop severe muscle cramps and fatigue in the first few minutes of activity.
Following a period of careful pacing and/or rest (the initial pain usually
subsides within less than one minute), after about 8 minutes most patients
achieve a ‘second wind’ and can then continue exercise with less difficulty.

65
Child Neurology

Symptoms tend to worsen with age as weight increases and aerobic fitness
is reduced. About one-half of all patients will have experienced myoglo-
binuria (dark urine) following intense exercise. Clinical findings may be
absent on physical examination. Muscle strength and reflexes may be nor-
mal. In later adult life, persistent proximal weakness and muscle wasting
may be present.

DIAGNOSTIC WORK-UP
• Spinal muscular atrophy is inherited in an autosomal recessive manner.
Molecular genetic testing is used to determine if a mutation is present in
the SMN gene. SMA is caused by a partial or complete loss of the SMN
(survival motor neuron) gene and about 95% of those affected will show
a deletion of both copies of a specific portion (exon 7 or exon 8) of the
gene. About 5% of those affected will show a deletion of exon 7 in one
copy of the SMN gene and a different mutation in the other copy of the
SMN gene.
• Prior to the availability of molecular testing, neurophysiologic studies and
muscle biopsy were used for diagnosis, but these tests are no longer neces-
sary unless SMN gene testing is normal.
• For patients who do not have genetically proven SMA, more conventional
testing is warranted. Serum muscle enzyme, EMG, NCV, and muscle bi-
opsy may help to confirm the diagnosis. Serum creatine kinase may be
mildly elevated but it is usually normal. EMG in severely affected chil-
dren demonstrates fibrillation and positive sharp waves consistent with
acute denervation. In less severely affected children, abnormal sponta-
neous activity is rare, but the duration and amplitude of motor unit po-
tentials are increased, indicating fibre type grouping with reinnervation.
Denervation atrophy causes decreased amplitude of Compound Muscle
Action Potentials (CMAPs) motor nerve conduction testing. Sensory
Nerve Action Potentials (SNAPs) are normal in SMA and help to distin-
guish this condition from polyneuropathies.
• The muscle biopsy in SMA reveals denervation (neurogenic atrophy).
Small fibres are found adjacent to normal and hypertrophied fibres with
type grouping that is seen in reinnervation.

MANAGEMENT
• Physical therapy: Spinal Muscular Atrophy (SMA) has no known cure;
thus, most care for the patient with SMA is focused on symptomatic con-
trol and preventative rehabilitation. Maintaining the patient’s joint mo-
bility is very important, because the goal is to decrease the incidence of
contractures. Plantar flexion contractures are the most common. Aquatic
therapy is an excellent way to maintain mobility, strength, and flexibility.
Because of the progressive weakness associated with SMA, patients may
require the full-time use of a wheelchair. Patients are encouraged to use

66
Case 12

manual wheelchairs rather than electric ones, when possible, to maintain


cardiovascular fitness and upper body strength.
• Occupational therapy: Occupational therapy is useful for teaching the
patient ways to increase his/her independence in Activities of Daily Living
(ADL). Fine motor skills may be affected by fatigue. Affected school-age
patients may benefit from an occupational therapy consultation that ad-
dresses keyboarding and other ways to avoid fatigue from upper extremity
activities in the classroom.
• Orthopaedic: Scoliosis is a major problem in half of the patients with
Spinal Muscular Atrophy (SMA) type III. Spinal orthoses have been
shown to assist in containing the spinal deformity until instrumentation
and fusion can be performed, if necessary.
• Hip subluxation: This is also common. Almost 50% of patients with
SMA type III have hip subluxation or dislocation, with rare improvement
in function from surgical reduction.
• Genetic counseling:: This is for Spinal Muscular Atrophy (SMA)-
Parents, patients, and extended family members may benefit from genetic
counseling. Carrier detection relies on determining the number of exon
7–containing SMN1 gene copies present in an individual. SMA carrier
testing, a polymerase chain reaction based dosage assay, is available on a
limited clinical basis.
• Follow-up: Patients with Spinal Muscular Atrophy (SMA) should
have frequent follow-up care for symptomatic control of their disease.
Respiratory function, nutritional state, orthopaedic status, and equipment
needs should be assessed at each visit. Pain control, preventative medicine,
surgical intervention, and physical therapy are all essential parts of the pa-
tient’s long-term care. The multidisciplinary approach, which includes
family members, social workers, therapists, and physicians, is important
to assist the patient in maintaining a high quality of life.

REFERENCES
1. Wang CH, Finkel RS, Bertini ES, et al.. Consensus statement for standard of care in
spinal muscular atrophy. J Child Neurol. 2007 Aug. 22(8):1027-49.
2. Russman BS, Buncher CR, White M, et al. Function changes in spinal muscular
atrophy II and III. The DCN/SMA Group. Neurology. 1996 Oct. 47(4):973-6.
3. Zerres K, Rudnik-Schöneborn S, Forrest E, et al. A collaborative study on the natu-
ral history of childhood and juvenile onset proximal spinal muscular atrophy (type
II and III SMA): 569 patients. J Neurol Sci. 1997 Feb 27. 146(1):67-72.
4. Ioos C, Leclair-Richard D, Mrad S, et al. Respiratory capacity course in patients
with infantile spinal muscular atrophy. Chest. 2004 Sep. 126(3):831-7.
5. Muscular Dystrophy Campaign sponsored workshop: recommendation for res-
piratory care of children with SMA type II and III. Neuromuscular Disord. 2003.
13:184-189.

67
Child Neurology

6. Moosa A, Dubowitz V. Spinal muscular atrophy in childhood. Two clues to clinical


diagnosis. Arch Dis Child. 1973 May. 48(5):386-8.
7. Simic G. Pathogenesis of proximal autosomal recessive spinal muscular atrophy.
Acta Neuropathol. 2008 Jul 16.
8. Ogino S, Wilson RB. Genetic testing and risk assessment for spinal muscular atro-
phy (SMA). Hum Genet. 2002 Dec. 111(6):477-500.
9. Wang CC, Chang JG, Ferrance J, et al.. Quantification of SMN1 and SMN2 genes
Electrophoresis.
by capillary electrophoresis for diagnosis of spinal muscular atrophy. Electrophoresis
2008 Jul. 29(13):2904-11.
10. Madsen KL, Hansen RS, Preisler N, Thogersen F, Berthelsen MP, Vissing J. Train-
ing improves oxidative capacity, but not function, in spinal muscular atrophy type
III. Muscle Nerve. 2015 Aug. 52 (2):240-4.
11. Swoboda KJ, Kissel JT, Crawford TO, et al.. Perspectives on clinical trials in spinal
muscular atrophy. J Child Neurol.. 2007 Aug. 22(8):957-66.
12. Il’ina NA, Anti-pova RI, Khokhlov AP. [Use of lithium carbonate to treat Kugel-
berg--Welander spinal amyotrophy]. Zh Nevropatol Psikhiatr Im S S Korsakova. 1980.
80(11):1657-60.
13. Younger DS, Gordon PH. Diagnosis in neuromuscular diseases. Neurol Clin. Feb
1 1996. 14:135-68.

68
Case 13
A 12-year-old, right-handed girl presented to the emergency department
with a sudden onset of right-sided body weakness and uncontrolled
twitching of the right eye and cheek. The patient was reportedly walking off
the basketball court when she collapsed to her knees and was observed having
twitching on the right side of her face. Upon arrival to the ED, the patient
exhibited right-sided weakness, slurred speech and she has been very sleepy
for the last 3 hours since the event occurred. Adults who witnessed the event
deny any trauma or falls. She is an otherwise healthy child. There is no family
history of seizures or medical or surgical problems noted in her family.
On physical examination,, the blood pressure was documented as 140/84,
heart rate 102. Sleepy but rousable, and she has difficulty following commands
and attempts to say a few words that were slurred. Cranial nerves II through
XII were intact bilaterally. Her extra-ocular muscles appear intact. Bulk of all
groups of muscles was normal. The right arm and leg were hypotonic and weak
but she had withdrawal response to deep pain stimuli. But she did not have
anti-gravity strength. She was able to move the left arm and leg purposefully
off the bed. Deep tendon reflexes: 2+ on the left and 1+ on the right. There
was a left plantar flexor and a right plantar extensor response.
The patient was noted to have intact sensation to light touch and sharp/dull
discrimination in all extremities.
The laboratory findings were all normal, including: a complete blood cell
count, plasma lactate and pyruvate levels, anti-nuclear anti-body, rheuma-
toid factor, anti-double-stranded DNA anti-body, fasting lipid profiles, anti-
thrombin-III, fibrinogen, protein C and S, prothrombin time, activated partial
thromboplastin time, and cerebrospinal fluid analysis. The electrocardiogra-
phy, echocardiography and abdominal ultrasound were all normal.

Answer the following questions:


• Give the most likely diagnosis and provide a differential diagnosis.
• Provide the summary of the case.
• Localisation of lesion.
• Discuss an appropriate diagnostic work-up.
• Discuss the management of this patient.

69
Child Neurology

DIAGNOSIS
Left carotid artery dissection: (left-middle cerebral artery infarction)

SUMMARY
The patient is a healthy 12-year-old girl who developed a new-onset right focal
seizure and right hamiparesis. Her examination is remarkable for a difficulty fol-
lowing commands and attempts to say a few words that are slurred (aphasia expres-
sive and receptive), and right haemiparesis involving depressed reflexes and a right
plantar extensor response.
The brain Magnetic Resonance Imaging (MRI) showed ill-defined hyper-
intense lesions involving the left temporoparietal lobe on the T2 and diffu-
sion weighted images, consistent with the acute stage of a cerebral infraction
without haemorrhage. The MR angiography and cerebral angiography showed
pre-occlusive irregular scalloped stenosis (99%) in the proximal M1 segment
of the left Middle Cerebral Artery (MCA) and a web like stenosis in the supra-
clinoid portion of the left ICA, suggestive of a dissection of the ICA. In addi-
tion, there was a marked decrease in blood flow at the left MCA.

Localisation of Lesion
The patient demonstrates right-sided focal seizures (involving the right eyelid
and face). This patient has cortical signs (aphasia, encephalopathy, seizure),
as well as right hemiparesis, depressed right-sided reflexes, and right plantar
extensor response, thus most likely the child in this vignette has experienced
a lesion involving the left cerebral hemisphere in the distribution of the left-
middle cerebral artery. Her right focal seizure also localises to the left cerebral
cortex. Thus, the patient has suffered a dissection of the left carotid artery,
possibly caused by neck movement or minor trauma, which has resulted in a
left hemispheric stroke and focal seizure.

DIFFERENTIAL DIAGNOSIS
Carotid artery dissection begins as a tear in one of the carotid arteries of the
neck, which allows blood under arterial pressure to enter the wall of the artery
and split its layers. The result is either an intramural haematoma or an aneu-
rysmal dilatation, either of which can be a source of microemboli, with the
latter also causing a mass effect on surrounding structures. Carotid artery dis-
section is a significant cause of ischaemic stroke in all age groups, but it occurs
most frequently in the fifth decade of life and accounts for a much larger per-
centage of strokes in young patients. Dissection of the internal carotid artery
can occur intracranially or extracranially, with the latter being more frequent.
Internal carotid artery dissection can be caused by major or minor trauma, or
it can be spontaneous, in which case, genetic, familial, or heritable disorders
are likely etiologies. Although in practice, dissections are labeled spontaneous
in the absence of major blunt or penetrating trauma, when they are associ-
ated with minor trauma they may be caused or influenced by an underlying
arteriopathy.

70
Case 13

Sophisticated imaging techniques, which have improved over the past 2 dec-
ades, are required to confirm the presence of dissection. Most ischaemic cer-
ebral symptoms arise from thromboembolic events; therefore, early institu-
tion of anti-thrombotic treatment provides the best outcome. Once diagnosed
and treated, patients with carotid artery dissection require regular follow-up
and imaging studies of both carotid arteries because healing usually takes 3-6
months and the incidence of contralateral dissection is higher than in the gen-
eral population. When the condition is diagnosed early, the prognosis is usually
good. A high index of suspicion is required to make this difficult diagnosis.
• The differential diagnosis in this case is relatively straightforward as this
patient presents with arterial ischaemic stroke. The etiology of sponta-
neous intracranial ICA dissection is not well understood, but it is likely
multifactorial.
• A child presenting with an acute neurologic deficit, before a diagnosis of
ischaemic stroke is made, other cerebrovascular diseases (haemorrhagic
stroke, cerebral venous sinus thrombosis) need to be considered.
However, in general, broad categories of causes of stroke in childhood include:
• Trauma (head or neck trauma, arterial dissection, nonaccidental trauma)
• Congenital heart disease (atrial septal defect, ventricular septal defect)
• Acquired heart disease (arrhythmias, cardiomyopathy, myocarditis)
• Vasculitis (systemic lupus erythematosus, primary cerebral angiitis)
• Vasculopathy (moyamoya disease, Ehlers-Danlos syndrome, Marfan
syndrome)
• Haematologic disorders/coagulopathy (anti-thrombin III deficiency,
oral contraceptive pills, protein C or S deficiency, sickle cell disease)
• Congenital cerebrovascular abnormalities (arteriovenous malfor-
mation)
• Inborn errors of metabolism (homocystinuria, mitochondrial disease)
• Other (migraine, radiation vasculitis)

DIAGNOSTIC WORK-UP
• History should include ethnic origin, the presence of sickle cell disease
or congenital heart disease, head or neck trauma (associated with intrac-
ranial haemorrhage and dissection), recent infection (especially chicken-
pox), vasculitis and blood disorders, as well as cerebrovascular diseases,
coagulopathies.
• Complete physical and neurologic examination, including monitor-
ing of vital parameters, will identify neurologic damages and allow pre-
sumptive diagnosis of the brain vessel involved.
• Imaging studies. Non-contrast Computed Tomography (CT) could be per-
formed promptly and quickly in emergency. It can adequately exclude haem-
orrhagic stroke or parenchymal abnormalities that produce a mass effect, and

71
Child Neurology

it may reveal a low-density lesion in arterial ischaemic stroke and cerebral ve-
nous sinus thrombosis. However, CT is usually normal within the first 12
hours after the onset of symptoms. It should only be requested whenever
Magnetic Resonance (MR) is not available.
• MR, in fact, is the “gold standard” imaging modality for the investigation of
arterial ischaemic stroke in infants and children due to its greater sensitivity
and specificity. The diagnostic efficiency of MR can be further improved by
perfusion techniques, which quantify relative cerebral blood flow, volume,
and transit time by the use of bolus administration of gadolinium-based con-
trast material. MR with diffusion weighting is very useful in accurately iden-
tifying regions of early ischaemia and infarction.
• MR angiography is a non-invasive procedure that detects large vascular
abnormalities, and it is as effective as cerebral angiography in identify-
ing large ischaemic lesions. MR angiography is a reasonable alternative to
conventional arteriography in most patients. If the pattern of brain injury
could be consistent with venous infarction, emergency vascular imaging
should include MR venography.
• Vascular imaging of the extracranial circulation, such as cervical MRA or
Doppler ultrasound, should also be performed, particularly if the history is
suggestive of a cervical arterial dissection.
• Imaging of the cervical and proximal intracranial arterial vasculature
should be performed in all children with arterial ischaemic stroke.
• Investigations after 48 hours: Once stroke has been diagnosed, several
studies may be helpful for the ongoing evaluation and management of the
patient.
• Electrocardiogram and transthoracic or trans-oesophageal echocardio-
gram are always necessary in all children with known or suspected con-
genital heart disease, that have had a stroke.
• The diagnosis of some conditions, including extracranial arterial dissec-
tion, particularly involving the posterior circulation, and small-vessel vas-
culitis, is difficult using MR angiography alone. In these circumstances,
catheter cerebral angiography is sometimes required. However, catheter
cerebral angiography is an invasive procedure, not commonly performed
in children, and it has almost similar diagnostic yield as MR combined
with MR venography and MR arteriography.
• Haemoglobin electrophoresis and urine drug screening, particularly for
sympathomimetics may be indicated. A full evaluation for thrombophilia
is reasonable in all children with stroke. It should include evaluation of
protein C and protein S deficiency, anti-thrombin III, heparin cofactor
II, plasminogen, von Willebrand’s antigen, factor VIII, factor XII, factor
V Leiden, activated protein C resistance, prothrombin, serum homocyst-
eine, methylene-tetra-hydro-folate-reductase, lipoprotein, and anti-phos-
pholipid anti-bodies.

72
Case 13

MANAGEMENT
• Management in emergency: Guidelines based on strong evidence for
the acute care of childhood stroke do not exist, with the exception of sickle
cell disease. Therefore, treatment recommendations for ischamic stroke
in children are extrapolated from adult guidelines.
• Children with early acute ischaemic cerebral stroke need to be admitted
to a critical care unit where continuous monitoring is possible. Only in
selected cases treatment in intensive care units is needed.
• Although anti-coagulation is the recommended (and the most commonly
used) treatment for childhood AIS, there are no randomised controlled
trials comparing anti-platelet therapy versus anti-coagulation in adults or
children with CCAD (Craniocervical Arterial Dissection), and this prac-
tice is based upon less evidence in children with AIS than in adults.
• Considerable variability of treatment recommendations is evident across
international pediatric stroke centres and even within the United States.
Recently published childhood stroke guidelines recommend treatment
of extracranial CCAD with anti-coagulation, such as Unfractionated
Heparin (UFH), Low Molecular Weight Heparin (LMWH), or warfarin.
The American Heart Association (AHA) Scientific Statement says, “In
children with extracranial CCAD, it is reasonable to begin either UFH or
LMWH as a bridge to oral anti-coagulation. It is reasonable to treat a child
with an extracranial [CAD] with either subcutaneous LMWH or warfarin
for 3 to 6 months”.
• Interventional procedures to stent or balloon extracranial dissection are
typically reserved for patients who fail medical therapies. AHA recom-
mendations suggest that surgical procedures are reasonable in children
who continue to have symptoms from an extracranial CCAD (Cranial
Carotid Artery Dissection) despite aggressive medical therapy.
• The use of anti-coagulation in intracranial dissection is discouraged by
AHA guidelines because of the risk of subarachnoid hamorrhage.
• Follow-up imaging is necessary for patients with CCAD, typically at 3 to 6
months after their initial presentation. In addition, children with vertebral
dissections should be evaluated with appropriate neck imaging, to assess
for a cervical skeletal abnormality.
• Pharmacologic treatment: Anti-platelet therapy for CCAD is aimed at
preventing arterial thrombus propagation. The most common anti-plate-
let agent is aspirin, typically dosed at 2 to 5 mg/kg per day. Clopidogrel is
sometimes prescribed, although dosing in paediatric patients older than 2
years is not well established. With aspirin, contraindications include aspi-
rin hypersensitivity and known bleeding disorder.
• Treatment of CCAD in children is challenging and differs for intracra-
nial and extracranial dissections. In extracranial CCAD, we most com-
monly use anti-coagulation for 6 weeks to 6 months in patients with TIA

73
Child Neurology

or AIS. Typically, unfractionated heparin is used in the acutely ill patient at


heightened risk for bleeding (because of its short half-life), whereas Low
Molecular Weight Heparin (LMWH) or warfarin are reserved for the sta-
ble patient. If the history is suspicious for dissection (head and neck trau-
ma, recent cervical chiropractic manipulation, recent car accident, or neck
pain), consider treatment for dissection even with normal MRI/MRA.
• The diagnosis and treatment of intracranial dissection is extraordinarily
challenging in children, in whom inflammatory intracranial arteriopathies
are common. When intracranial arteriopathy is clearly associated with dis-
section, the clinician should look for the presence of subarachnoid haem-
orrhage and/or dissecting aneurysm. A multidisciplinary paediatric stroke
team, given the lack of data in this area, should make treatment decisions.
Intracranial cerebral artery stent placement carries high risk and is not
recommended for intracranial CCAD in children.

REFERENCES
1. Schievink WI. Spontaneous dissection of the carotid and vertebral arteries. N Engl
J Med. 2001 Mar 22. 344(12):898-906.
2. Redekop GJ. Extracranial carotid and vertebral artery dissection: a review. Can J
Neurol Sci. 2008 May. 35(2):146-52.
3. Goyal MS, Derdeyn CP. The diagnosis and management of supraaortic arterial
dissections. Curr Opin Neurol. 2009 Feb. 22(1):80-9.
4. Cothren CC, Moore EE, Biffl WL, Ciesla DJ, Ray CE Jr, Johnson JL. Anticoagula-
tion is the gold standard therapy for blunt carotid injuries to reduce stroke rate.
Arch Surg.. 2004 May. 139(5):540-5; discussion 545-6.
5. Baumgartner RW. Management of spontaneous dissection of the cervical carotid
artery. Acta Neurochir Suppl. 2010. 107:57-61.
6. Tobin J, Flitman S. Cluster-like headaches associated with internal carotid artery
dissection responsive to verapamil. Headache
Headache. Mar 2008. 48(3):461-6.
7. Divjak I, Slankamenac P, Jovicevic M, Zikic TR, Prokin AL, Jovanovic A. A case
series of 22 patients with internal carotid artery dissection. Med Pregl. 2011 Nov-
Dec. 64(11-12):575-8.
8. Patel RR, Adam R, Maldjian C, Lincoln CM, Yuen A, Arneja A. Cervical Carotid
Artery Dissection: Current Review of Diagnosis and Treatment. Cardiol Rev. 2012
Feb 1.
9. Caplan LR. Dissections of brain-supplying arteries. Nat Clin Pract Neurol. 2008
Jan. 4(1):34-42.
10. Arnold M, Baumgartner RW, Stapf C, Nedeltchev K, Buffon F, Benninger D. Ul-
trasound diagnosis of spontaneous carotid dissection with isolated Horner syn-
drome. Stroke. 2008 Jan. 39(1):82-6.
11. Xianjun H, Zhiming Z. A systematic review of endovascular management of inter-
nal carotid artery dissections. Interv Neurol. 2013 Sep. 1(3-4):164-70.

74
Case 14
A previously healthy 11-year-old girl was referred to our hospital (ER)
for generalised status epilepticus that was not associated with fever.
Seizures evolved to include oro-buccal automatism and was secondarily gen-
eralised following with a 3-week history of bizarre psychiatric symptoms that
progressed to behavioural aggression, insomnia and episodic outbursts of ex-
cessive, and distress that were described, “as though she was having night-
mares while awake”. She had delayed sleep onset from her baseline, decreased
appetite, decreased activity level, and emesis. Subsequently she had dysarthria
with progressively decreasing verbal output to the point of mutism with pre-
served comprehension. Following unremarkable neurological examination
and head computerised tomography, the patient was discharged. Since the
patient’s agitation and hallucinations worsened, she underwent psychiatric
consultation, in which a psychotic state had been diagnosed and olanzapine
prescribed at 5 mg/day.
Neurological examination did not disclose any focal signs and routine
laboratory work-up was normal. Owing to the growing agitation and psychosis,
the patient was admitted to PICU care. Subsequent Electroencephalogram
(EEG) revealed intermittent focal frontal bursts, with slow background and
phenytoin loading was initiated.
Cerebrospinal Fluid (CSF) analysis revealed five lymphocytes, no red blood
cells, and normal protein and glucose concentrations. CSF herpes and entero-
virus amplifications were negative. Blood counts, chemistries and urine drug
screen were unremarkable. Anti-streptolysin O, anti-nuclear anti-body, lupus
anti-coagulant, cytoplasmic-anti-neutrophil-cytoplasmic anti-bodies, anti-
neutrophil cytoplasmic anti-bodies and anti-phospholipid anti-body titers,
thyroid function studies, creatine kinase, lactic acid and serum ammonia were
normal. Brain Magnetic Resonance Imaging (MRI) was normal after two days
of extensive laboratory examination. Nevertheless, during the following days
the patient’s confusion and agitation escalated; she spoke in incomprehensi-
ble words, with intermittent bursts of shouting and disorganised behaviour.
Despite phenytoin and diazepam treatment, the patient experienced a second
generalised tonic-clonic seizure, and sodium valproate was added to prevent
further seizures.

Guide to discussion
• Provide the most likely diagnosis and summarise the case.
• Discuss the differential diagnosis.
• Provide the diagnostic work-up.
• Give the management of autoimmune encephalitis.

75
Child Neurology

DIAGNOSIS
Autoimmune Encephalitis (Anti-NMDAR Encephalitis)

SUMMARY
A previously healthy 11-year-old girl presented with Status Epilepticus and
severe psychotic state, with cognitive impairment, fluctuating behavioural
changes, and recurrent seizures despite anti-epileptic treatment. She had posi-
tive results of CSF Anti-NMDA-receptor anti-bodies.

DIFFERENTIAL DIAGNOSIS AND DIAGNOSTIC WORK-UP


The differential diagnosis of anti-NMDA receptor encephalitis is broad and
includes infectious causes of encephalitis, such as herpes simplex virus; Acute
Disseminated Encephalomyelitis (ADEM); inborn errors of metabolism, in-
cluding urea cycle disorders; environmental toxins and medication overdoses;
rheumatologic conditions, such as neuropsychiatric lupus; and primary psychi-
atric conditions, such as schizophrenia. Due to the extensive differential diag-
nosis, the diagnostic work-up of each patient must be individualised based on
the presentation.

Diagnostic Criteria for Possible Autoimmune Encephalitis


Diagnosis can be made when all three of the following criteria have been met:
a. Subacute onset (rapid progression of less than 3 months) of working
memory deficits (short-term memory loss), altered mental status, or psy-
chiatric symptoms
b. At least one of the following:
° New focal CNS findings
° Seizures not explained by a previously known seizure disorder
° CSF pleocytosis (white blood cell count of more than five cells per
mm3)
° MRI features suggestive of encephalitis
c. Reasonable exclusion of alternative causes

Diagnostic Criteria for Anti-NMDA Receptor Encephalitis


Probable anti-NMDA receptor encephalitis
Diagnosis can be made when all three of the following criteria have been met:
a. Rapid onset (less than 3 months) of at least four of the six following major
groups of symptoms:
° Abnormal (psychiatric) behaviour or cognitive dysfunction
° Speech dysfunction (pressured speech, verbal reduction, mutism)
° Seizures
° Movement disorder, dyskinesias, or rigidity/abnormal postures

76
Case 14

° Decreased level of consciousness


° Autonomic dysfunction or central hypoventilation
b. At least one of the following laboratory study results:
° Abnormal EEG (focal or diffuse slow or disorganised activity, epilep-
tic activity, or extreme delta brush)
° CSF with pleocytosis or oligoclonal bands
c. Reasonable exclusion of other disorders
Diagnosis can also be made in the presence of three of the above groups of
symptoms accompanied by a systemic teratoma.

Definite anti-NMDA receptor encephalitis


Diagnosis can be made in the presence of one or more of the six major groups
of symptoms and IgG anti-GluN1 anti-bodies, after reasonable exclusion of
other disorders.

Criteria for autoanti-body-negative but probable autoimmune


encephalitis
Diagnosis can be made when all four of the following criteria have been met:
a. Rapid progression (less than 3 months) of working memory deficits
(short-term memory loss), altered mental status, or psychiatric symptoms
b. Exclusion of well-defined syndromes of autoimmune encephalitis (e.g.,
typical limbic encephalitis, Bickerstaff ’s brainstem encephalitis, acute dis-
seminated encephalomyelitis)
c. Absence of well characterised autoantibodies in serum and CSF, and at
least two of the following criteria:
° MRI abnormalities suggestive of autoimmune encephalitis
° CSF pleocytosis, CSF-specific oligoclonal bands or elevated CSF IgG
index, or both
° Brain biopsy showing inflammatory infiltrates and excluding other
disorders (e.g., tumour)
d. Reasonable exclusion of alternative causes

Diagnostic criteria for definite autoimmune limbic encephalitis


Diagnosis can be made when all four of the following criteria have been met:
a. Subacute onset (rapid progression of less than 3 months) of working
memory deficits, seizures, or psychiatric symptoms suggesting involve-
ment of the limbic system.
b. Bilateral brain abnormalities on T2-weighted fluid-attenuated inversion
recovery MRI highly restricted to the medial temporal lobes.
c. At least one of the following:
° CSF pleocytosis (white blood cell count of more than five cells per
mm3)

77
Child Neurology

° EEG with epileptic or slow-wave activity involving the temporal lobes


d. Reasonable exclusion of alternative causes

Diagnostic criteria for definite acute disseminated encephalomyelitis


Diagnosis can be made when all five of the following criteria have been met:
a. A first multifocal, clinical CNS event of presumed inflammatory demy-
elinating cause.
b. Encephalopathy that cannot be explained by fever.
c. Abnormal brain MRI:
° Diffuse, poorly demarcated, large (>1-2 cm) lesions pre-dominantly
involving the cerebral white matter
° T1-hypointense lesions in the white matter in rare cases
° Deep grey matter abnormalities (e.g., thalamus or basal ganglia) can
be present
d. No new clinical or MRI findings after 3 months of symptom onset
e. Reasonable exclusion of alternative causes

Diagnostic criteria for Bickerstaff’s brainstem encephalitis


Probable Bickerstaff ’s brainstem encephalitis
Diagnosis can be made when both of the following criteria have been met:
a. Subacute onset (rapid progression of less than 4 weeks) of all the following
symptoms:
° Decreased level of consciousness
° Bilateral external ophthalmoplegia
° Ataxia
b. Reasonable exclusion of alternative causes

Definite Bickerstaff ’s brainstem encephalitis


Diagnosis can be made in the presence of positive IgG anti-GQ1b anti-bodies
even if bilateral external ophthalmoplegia is not complete or ataxia cannot be
assessed, or if recovery has occurred within 12 weeks after onset.

Diagnostic criteria for Hashimoto’s encephalopathy


Diagnosis can be made when all six of the following criteria have been met:
a. Encephalopathy with seizures, myoclonus, hallucinations, or stroke-like
episodes
b. Subclinical or mild overt thyroid disease (usually hypothyroidism)
c. Brain MRI normal or with non-specific abnormalities
d. Presence of serum thyroid (thyroid peroxidase, thyroglobulin) anti-bodies

78
Case 14

e. Absence of well characterised neuronal anti-bodies in serum and CSF


f. Reasonable exclusion of alternative causes

DIAGNOSTIC TESTS
Cerebrospinal Fluid (CSF) Evaluation
• CSF will be abnormal in up to 90 percent of patients.
• More than 80 percent of patients will have positive oligoclonal bands,
which are anti-bodies in the CSF that are not found in the serum.
• Up to 75 percent of patients will have elevated CSF white blood cell
counts, usually with lymphocytic pre-dominance, but neutrophilic pre-
dominance can also be seen.
• Approximately 10 to 15 percent of patients will have elevated protein
concentrations.
• Anti-NMDA receptor anti-bodies can be found in the CSF as well wel as the
serum.

MRI of the Brain


• For 50 percent of cases, MRIs will appear normal.
• The most common abnormal finding is nonspecific T2 hyperintense le-
sions involving the grey and/or white matter of the cortex, basal ganglia or
brainstem; spine lesions are rare.
• The changes are usually mild and transient, but can occasionally be
extensive.
• Occasionally, brain MRIs will show white matter lesions with associated
enhancement after contrast that is consistent with demyelination. Patients
with demyelinating lesions and confirmed anti-NMDA receptor enceph-
alitis should be tested for anti-NMO anti-bodies.

Electroencephalogram (EEG)
• EEGs are used to assess the background activity and to evaluate for
seizures.
• Most patients with anti-NMDA receptor encephalitis will have an abnor-
mal EEG at some point in their disease course.
• 90 percent of patients will have slowing of background frequencies.
• 25 to 50 percent will have epileptic features, such as spike and wave dis-
charges or captured seizures.

Infectious Disease Evaluation


• The evaluation for viral encephalitis is individualised, and depends on the
time of the year and geographic location.
• Every patient should be tested for herpes simplex virus (HSV).
• The infectious disease team is often consulted to co-evaluate the patient.

79
Child Neurology

Malignancy Screening
Anti-NMDAR encephalitis usually precedes the discovery of an associated
malignancy which poses a question regarding appropriate screening for fol-
low-up patients. Potential screening imaging modalities include ultrasonogra-
phy or CT abdomen/pelvis. It is also recommended periodic ultrasound and
MRI of the abdomen and pelvis for at least two years following diagnosis,
although there is no good data or guidelines to that effect. Every patient with
anti-NMDA receptor encephalitis should undergo a tumour evaluation.

MANAGEMENT
Treatment of anti-NMDA receptor encephalitis involves dealing with the acute
effects of the disease, removal of any underlying causative tumour, induction
of immunosuppressive therapy and management of long term residual deficits
following remission of the autoimmune process. The treating physician must
remember that features of the disease cross the borders of many medical spe-
cialties and thus management must always be that of a multi-disciplinary ap-
proach. When anti-NMDAR encephalitis is found to be paraneoplastic, early
tumour resection along with immunotherapy is the recommended best prac-
tice and this leads to better outcomes and reduction in symptomatology. The
most reliably documented experience is with ovarian teratomas.
Medications are used to decrease the circulating disease-causing anti-NMDA
receptor anti-bodies and the body’s overactive immune response.

Immunotherapy
• Methylprednisolone is an IV steroid that can be given at high doses and
is often used with IV Immunoglobulins (IVIG) as first-line therapy.
• IVIG are pooled anti-bodies, given by IV, that result in the body degrading
circulating anti-bodies, including the anti-NMDA receptor anti-bodies
that are disease-causing.
• PLEX. (Plasma Exchange).
• Rituximab is a monoclonal anti-body, given by IV, that targets the body’s
anti-body producing cells and stops ongoing production of anti-NMDA
receptor anti-bodies. It is often used as a second-line therapy for those
patients whose recovery after receiving first-line treatments is inadequate.
• Cyclophosphamide can be used as a second line immune-suppressive
agent if no response to other therapeutic agents.
• Other medications are often needed to treat specific symptoms, such
as seizures, abnormal movements or psychosis. Although the follow-up
of CSF and serum anti-body titers may assist in some assessments (e.g.,
relapses or effects of treatment), clinical decisions about changing or dis-
continuing treatments should rely more on clinical assessment.
• Rehabilitation programmes, with intensive physical, occupational and
speech therapy, are important to help patients regain function, and an in-
patient admission to a specialised rehabilitation unit is often required.

80
Case 15
A 6-year-old boy presented to ER with status epilepticus and there was a
history of sudden loss of vision 1 day before admission to our hospital.
He had been admitted to a local hospital for 2 days with episodic attacks of
vomiting and headache. He also had brief attacks of generalised tonic-clonic
seizures during his admission in a primary care hospital.
On physical examination, he was afebrile, but irritable with altered senso-
rium, and mild periorbital swelling. His blood pressure was 180/120 mm Hg
>95th per centile for his age. His pupils were bilaterally equal and reactive to
light. Fundus examination was unremarkable. He had no focal neurological
deficits and there were no signs of meningeal irritation. He was treated with
injection of phenytoin and carbamazepine for the control of seizures and
blood pressure was controlled with anti-hypertensive agents, fluid, and salt
restriction. His blood pressure was carefully monitored and treated.
His investigations revealed Hb 9.5 g/dL, total leukocytes count 14,300 cells/
mm3 polymorphs 80%, lymphocytes 13%, platelets count 370,000 cells/
mm3, ESR 16 mm/hr, C-reactive protein was less than 0.9 mg/L, Blood Urea
Nitrogen (BUN) was raised to 60 mg/dL, and creatinine 0.80 mg/dL. His liver
function test and serum electrolytes were within normal range. His urinalysis
revealed plenty of RBCs with proteinuria of 2+. Anti-streptolysin O-titer was
positive (520 IU/ml) and complement level (C3) was low (38 mg/dL). Blood
and throat swab culture did not show any growth.

Answer the following questions:


• What is the most likely diagnosis?
• Provide the appropriate differential diagnosis.
• Give the appropriate diagnostic work-up.
• What is the management of the patient?

81
Child Neurology

DIAGNOSIS
Posterior Reversible Encephalopathy Syndrome (PRES) (Caused by Acute
Glomerulonephritis)

SUMMARY
A 6-year-old boy presented to ER with status epilepticus and associated with
history of sudden loss of vision 1 day before admission to our hospital. He
was having hypertension, hypocomplementaemia, and haematuria. MRI brain
demonstrated areas of subcortical high signals without mass effect, contrast en-
hancement, or diffusion restriction. The high signal lesions were mainly situ-
ated in the posterior part of the brain, (parieto-occipital lobes).

DIFFERENTIAL DIAGNOSIS
Reversible Posterior Leukoencephalopathy Syndrome (RPLS) is a clinical ra-
diographic syndrome of heterogeneous etiologies that are grouped together
because of similar findings on neuroimaging studies. It is also often referred
to as:
• Posterior Reversible Encephalopathy Syndrome (PRES)
• Reversible posterior cerebral oedema syndrome
• Posterior leukoencephalopathy syndrome
• Hyperperfusion encephalopathy
• Brain capillary leak syndrome
None of these names is completely satisfactory; the syndrome is not always
reversible, and it is often not confined to either the white matter or the pos-
terior regions of the brain. Although described in various specific case reports
for some time, it was first codified as a single named syndrome in a 1996 case
series. This described a clinical syndrome of insidious onset of headache, con-
fusion or decreased level of consciousness, visual changes, and seizures, which
was associated with characteristic neuroimaging findings of posterior cerebral
white matter oedema.
PRLS has been described in a number of medical conditions, with hyperten-
sive encephalopathy, eclampsia, and the use of cytotoxic and immunosuppres-
sant drugs being the most common. Prompt recognition and treatment is im-
portant in preventing the permanent damage that can occur in this otherwise
typically reversible condition.
Hypertensive disorders, renal disease, and immunosuppressive therapies are
risk factors for this disorder.
The differential diagnosis of PRES is broad, and history may be limited. The
pathophysiology of PRES remains controversial. However, the list of condi-
tions known to be associated with PRES is increasing steadily. The non-specific
clinical manifestations and multiplicity of radiological patterns raise diagnostic
challenges. PRES should be considered in patients who present with seizures,

82
Case 15

altered consciousness, visual disturbance, or headache, particularly in the con-


text of acute hypertension. Many conditions may resemble PRES, including:
• Ictal or post-ictal state (with or without status epilepticus)
• Progressive Multifocal Leukoencephalopathy (PML)
• Leukoencephalopathy
• Infectious encephalitis (particularly herpes simplex encephalitis) or
meningitis
• Acute disseminated encephalomyelitis
• Mitochondrial Myopathy Encephalopathy Lactic Acidosis (MELAS)
• Autoimmune encephalitis and metabolic encephalopathies
• Central nervous system vasculitis
• Cerebral venous sinus thrombosis
• Posterior circulation ischaemic stroke (watershed or posterior cerebral ar-
tery territory)
• Cancer chemotherapy or immunosuppressive agents

DIAGNOSTIC WORK-UP
• After a careful history and thorough physical examination, investigations
should be performed as appropriate, starting with the simplest and mov-
ing to the more sophisticated.
• In the acute setting, Computed Tomography (CT) imaging allows for rap-
id assessment and may be easier to obtain first. It can also exclude major
cerebral haemorrhage and space-occupying lesions. Although not 100%
sensitive, CT may also demonstrate venous sinus thrombosis or arterial
ischaemia or thrombosis.
• However, MRI must be performed, either as the first or as the second
imaging study. MRI is considerably better than CT for the diagnosis of
PRES and can provide information regarding many of the causes of PRES.
MRA/MRV must be added to MRI to identify an associated cerebral re-
versible vasoconstriction syndrome. Diffusion-weighted magnetic reso-
nance imaging (DWI-MRI) is the study of choice in PRES to discriminate
between vasogenic and cytotoxic oedema.
• Typical MRI findings in PRES are of bilateral white-matter abnormalities
in vascular watershed areas in the posterior regions of both cerebral hemi-
spheres, affecting mostly the occipital and parietal lobes. Atypical features
including haemorrhage, asymmetrical changes, isolated involvement of
the frontal lobes, and cortical lesions are common.
• Electroencephalography (EEG) should be performed routinely to
look for non-convulsive status epilepticus. Patients most likely to have
non-convulsive status epilepticus are those in a deep coma or prolonged
post-ictal state.

83
Child Neurology

• Lumbar puncture findings are not specific in PRES. However, the


Cerebrospinal Fluid (CSF) must be examined in patients with a fever or
clinical suspicion of meningitis.

MANAGEMENT
• The treatment strategy associates general measures with correction of the
underlying cause of PRES. An early etiologic diagnosis allows prompt cor-
rection of the cause of PRES. Patients may require blood pressure control,
withdrawal of cancer chemotherapy or immunosuppressive agents, dialy-
sis, or other interventions. Prompt correction of the cause is crucial to de-
crease the risk of ischaemia or bleeding and therefore to avoid permanent
disability or death.
• The need for upper airway protection should be evaluated continuously in
patients with marked consciousness impairment or seizure activity.
• In the setting of hypertensive emergencies, Intensive Care Unit (ICU)
admission is a must for continuous haemodynamic, cardiac and neuro-
logical status monitoring.
• Hypoglycaemia should be looked for routinely and corrected.
• Patients should be routinely evaluated for hyperthermia and metabolic
disturbances, in particular hypomagnesaemia, which require prompt cor-
rection. Aspiration pneumonia may complicate the initial consciousness
and overall condition.
• Control of hypertensive emergency, if present, is an important part of the
symptomatic management. The aim is not to normalise the blood pres-
sure but rather to decrease the MAP by 20-25 % within the first 2 hours
and to bring the blood pressure down gradually within the first 6 hours.
More rapid blood pressure reduction is not recommended since it can
adversely affect the cerebral perfusion pressure and promote ischaemia.
Intravenous anti-hypertensive drugs are necessary. Appropriate choices
include labetalol, nicardipine; or other agents can be used according to
availability.
• Anti-epileptic treatment, appropriate for the electrical and clinical pat-
tern of seizures, should be initiated on an emergency basis and accord-
ing to current guidelines. Patients with persistent seizure activity at ICU
admission should be given intravenous benzodiazepines (Lorazepam or
Midazolam or Diazepam) either before ICU admission or in the ICU.
Patients with continuing seizure activity despite intravenous benzodiaz-
epines should receive standard complementary intravenous anti-convul-
sant drugs (phenytoin 18 mg/kg, or phenobarbital 15 to 20 mg/kg or fos-
phenytoin). Patients with refractory status epilepticus need midazolam,
propofol, or thiopental in titrated doses until remission of the clinical
seizure activity. When the EEG reveals electrical status epilepticus, these
anaesthetic drugs are given in titrated doses to induce EEG burst suppres-
sion; then as a continuous infusion for at least 12 hours.

84
Case 15

REFERENCES
1. Hinchey J, Chaves C, Appignani B, et al. A reversible posterior leukoencephalopa-
thy syndrome. N Engl J Med 1996; 334:494.
2. Covarrubias DJ, Luetmer PH, Campeau NG. Posterior reversible encephalopathy
syndrome: prognostic utility of quantitative diffusion-weighted MR images. AJNR
Am J Neuroradiol 2002; 23:1038.
3. Kwon S, Koo J, Lee S. Clinical spectrum of reversible posterior leukoencephalopa-
thy syndrome. Pediatr Neurol 2001; 24:361.
4. Alehan F, Erol I, Agildere AM, et al.. Posterior leukoencephalopathy syndrome in
children and adolescents. J Child Neurol 2007; 22:406.
5. Chen TH, Lin WC, Tseng YH, et al.. Posterior Reversible Encephalopathy Syn-
drome in Children: Case Series and Systematic Review. J Child Neurol 2013;
28:1378.
6. Raj S, Overby P, Erdfarb A, Ushay HM. Posterior reversible encephalopathy syn-
drome: incidence and associated factors in a pediatric critical care population. Pedi-
atr Neurol 2013; 49:335.
7. Fugate JE, Claassen DO, Cloft HJ, et al.. Posterior reversible encephalopathy syn-
drome: associated clinical and radiologic findings. Mayo Clin Proc 2010; 85:427.
8. Staykov D, Schwab S. Posterior reversible encephalopathy syndrome. J Intensive
Care Med 2012; 27:11.
9. Kozak OS, Wijdicks EF, Manno EM, et al.. Status epilepticus as initial manifestation
of posterior reversible encephalopathy syndrome. Neurology 2007; 69:894.
10. Lysandropoulos AP, Rossetti AO. Postictal cortical visual impairment: a symptom
of posterior reversible encephalopathy. Epilepsy Behav 2010; 17:276.
11. Datar S, Singh T, Rabinstein AA, et al.
al. Long-term risk of seizures and epilepsy
in patients with posterior reversible encephalopathy syndrome. Epilepsia 2015;
56:564.
12. Hayes D Jr, Adler B, Turner TL, Mansour HM. Alternative tacrolimus and siroli-
mus regimen associated with rapid resolution of posterior reversible encephalopa-
thy syndrome after lung transplantation. Pediatr Neurol 2014; 50:272.

85
16 Case
A 10-year-old boy referred to the paediatric neurology clinic because he
was suffering from easy fatigability since he started walking. He was the
first child of a consanguineous family. He was born at 35th gestational week,
1400 g and there was no prenatal or post-natal complication. He started to hold
his head at 3-4 months, sat without being supported at 12th months, and was
able to walk at the age of 18 months. His sister had died due to sudden infant
death at the age of 19 months and another 4-year-old sister was suffering from
easy fatigability.
On physical examination his weight and height were on 50th percentile.
His head circumference was below the −2 SD. He was having dysmorphic
features; a long, mask face and protruding large ears, hypertelorism, epicanthal
folds, swollen eyelids, narrow columella, and small nose. His shoulders were
inclined forward while sitting. His muscle strength was 5/5 for four limbs
and deep tendon reflexes were depressed but plantar response was flexor. All
accessible cranial nerves were intact.
Creatine kinase level was 150 u/L (30-200) and electromyography revealed a
myopathic pattern. Tandem MS spectrometry analyses found very low (0.92
uM) free carnitine (>3.8 uM normal) and low (0.13uM) acylcarnitines (>2
normal). He was examined by the Pediatric Cardiology Clinic and diagnosed
as hypertrophic cardiomyopathy.

Answer the following questions:


• What is the most likely diagnosis?
• What is the differential diagnosis?
• Give the appropriate diagnostic work-up.
• Give the appropriate management of this disorder.

86
Case 16

DIAGNOSIS
Systemic Primary Carnitine Deficiency (CDSP) or Carnitine Uptake
Defect (CUD)

SUMMARY
A 10-year-old dysmorphic boy referred to the paediatric neurology clinic
because he was suffering from easy fatigability since he started walking.
He was also diagnosed with hypertrophic cardiomyopathy on echocardiog-
raphy. Tandem MS spectrometry analyses found very low (0.92 uM) free
carnitine (>3.8 uM normal) and low (0.13uM) acylcarnitines (>2 normal).
His molecular genetic testing sequence analysis revealed SLC22A5 muta-
tion and diagnosis CDSP was confirmed.

Systemic Primary Carnitine Deficiency (CDSP) or Carnitine


Uptake Defect (CUD)
Systemic Primary Carnitine Deficiency (CDSP) is a rare metabolic disorder in
which the body cannot properly process fats into energy. Carnitine functions
to carry fatty acids obtained through diet to the energy centres in muscle cells
(mitochondria). A deficiency of Carnitine results in accumulation of fats in
the liver, muscle, and the heart. The Systemic Primary Carnitine Deficiency
(CDSP) phenotype encompasses a broad clinical spectrum including meta-
bolic decompensation in infancy, cardiomyopathy in childhood, fatigability in
adulthood, or absence of symptoms. CDSP has been typically associated with
infantile metabolic presentation in about half of affected individuals and child-
hood myopathic presentation in the other half. Carnitine deficiency results
in defective fatty acid oxidation. When fat cannot be utilised, glucose is con-
sumed without regeneration via gluconeogenesis resulting in hypoglycaemia.
In addition, fats released from adipose tissue accumulate in the liver, skeletal
muscle, and heart, resulting in hepatic steatosis and myopathy.
presentation Affected children can
• Infantile metabolic (hepatic) presentation.
present between age three months and two years with episodes of meta-
bolic decompensation triggered by fasting or common illnesses such as
upper respiratory tract infection or gastroenteritis. These episodes are
characterised clinically by poor feeding, irritability, lethargy, and hepato-
megaly. Laboratory evaluations usually reveal hypoketotic hypoglycaemia
(hypoglycaemia with minimal or no ketones in urine), hyperammonemia,
and elevated liver transaminases.
• Childhood myopathic (cardiac) presentation. The average age of
myopathic presentation is between age two and four years, indicating that
the myopathic manifestations of CDSP may develop over a longer period
of time. Myopathic manifestations include dilated cardiomyopathy, hy-
potonia, skeletal muscle weakness, and elevated serum Creatine Kinase
(CK). Death from cardiac failure can occur before the diagnosis is estab-
lished, indicating that this presentation can be fatal if not treated. Older

87
Child Neurology

children with the infantile presentation may also develop myopathic man-
ifestations including elevated CK, cardiomyopathy, and skeletal muscle
weakness.
• There is known to have asymptomatic patients with absolutely no symp-
toms in CDSP.

DIFFERENTIAL DIAGNOSIS
• Mitochondrial disorders: Mitochondrial diseases are a clinically het-
erogeneous group of disorders that arise as a result of dysfunction of the
mitochondrial respiratory chain. Mitochondrial disorders involve multiple
organ systems and often present with prominent neurologic and myopathic
features. Mitochondrial disorders may present at any age. Common clini-
cal features of mitochondrial disease–whether involving a mitochondrial or
nuclear gene–include ptosis, external ophthalmoplegia, proximal myopa-
thy and exercise intolerance, cardiomyopathy, sensorineural deafness, optic
atrophy, pigmentary retinopathy, and diabetes mellitus. Common central
nervous system findings are fluctuating encephalopathy, seizures, demen-
tia, migraine, stroke-like episodes, ataxia, and spasticity. Diagnosis of mi-
tochondrial disorders can be confirmed by identification of a pathogenic
mitochondrial DNA variant on molecular genetic testing of DNA extracted
from a blood sample.
• Myoadenylate Deaminase Deficiency (MAD): It is an autosomal re-
cessive condition that can manifest as exercise-induced muscle pain, occa-
sionally associated with rhabdomyolysis and/or increased serum creatine
kinase, or even infantile hypotonia. MAD is caused by mutation in the
AMP1 gene on chromosome 1p. The ischaemic forearm test might be
helpful in the diagnosis as ammonia levels fail to rise and lactate remains
normal.
• McArdle disease: Glycogen is the main source of energy during brief ex-
ercise, while free fatty acids are the most important source of fuel during
prolonged exercise. Hence, muscle cramps during strenuous brief exer-
cise are the hallmark of glycogen storage diseases (e.g., McArdle disease).
However, in lipid storage disease (e.g., CPT deficiency), muscle cramps
and exercise intolerance occur only after prolonged exercise and are worse
during fasting.
• Systemic Primary Carnitine Deficiency (CDSP): This needs to be
differentiated from Secondary carnitine deficiency (CDSP) carnitine de-
ficiency seen in the following situations:
° Inherited metabolic disorders: These include organic acidemias
and fatty acid oxidation defects (e.g., very long chain acyl-CoA de-
hydrogenase [VLCAD] deficiency, medium-chain acyl-CoA dehy-
drogenase [MCAD] deficiency, short-chain acyl-CoA dehydroge-
nase [SCAD] deficiency, carnitine-acylcarnitine translocase [CACT]
deficiency long-chain hydroxyacyl-CoA dehydrogenase [LCHAD]

88
Case 16

deficiency and carnitine palmitoyltransferase II [CPT II] deficiency)


° Pharmacologic therapy (e.g., valproate, cyclosporine, and
pivampicillin)
° Malnutrition
° Haemodialysis and renal tubular dysfunction (e.g., renal Fanconi
syndrome)
° Prematurity: Premature neonates may have low plasma carnitine
concentrations due to a lack of carnitine placental transfer in the third
trimester and decreased tissue stores. Moreover, immature renal tu-
bular function in premature neonates could lead to increased renal
carnitine elimination.

DIAGNOSTIC WORK-UP
• Systemic Primary Carnitine Deficiency (CDSP) should be considered in
the following clinical situations: Infant with positive newborn screening,
children with hypoketotic hypoglycaemic episodes that may be associ-
ated with hepatomegaly, elevated transaminases, and hyperammonemia,
children with skeletal myopathy and/or elevated serum concentration of
Creatine Kinase (CK) with unexplained fatigability, Children with cardio-
myopathy and sudden death.
• Newborn screening using tandem Mass Spectrometry (MS/MS) detects
low levels of free carnitine and can identify infants or children with CDSP.
• Extremely reduced plasma free, acylated, and total (i.e. the sum of free
and acylated) carnitine levels (i.e. <10% of controls) are diagnostic of this
disorder.
• Plasma carnitine levels should be measured in all mothers of infants found
to have low free carnitine levels on newborn screening in order to deter-
mine if the mother (rather than the infant) has CDSP, or if both mother
and infant have CDSP.
• After the finding of low plasma carnitine levels on a newborn screening
assay, in a symptomatic individual, or in an asymptomatic at-risk relative,
the diagnosis of CDSP can be confirmed either by identification of bial-
lelic pathogenic variants in SLC22A5 on molecular genetic testing.
• Fibroblast carnitine transport: If molecular and aCGH fail to detect
mutations or large deletions of the gene, then a skin biopsy may be consid-
ered to assess carnitine transport in cultured fibroblasts. Carnitine trans-
port in skin fibroblasts from individuals with CDSP is typically reduced
below 10% of control rates.
• Antenatal diagnosis: Antenatal diagnosis is possible by molecular genet-
ic testing of extracted DNA from chorionic villus sampling or amniocen-
tesis. Measuring carnitine transport in amniocytes obtained from amniotic
fluid can also perform prenatal diagnosis. Antenatal diagnosis for CDSP

89
Child Neurology

has seldom been performed since this condition is treatable and carnitine
supplementation may be initiated at birth while the diagnostic testing is
pending.

MANAGEMENT
Once the diagnosis of CDSP is established in an individual, the following
evaluations should be performed for baseline assessment:
• Echocardiogram to screen for cardiomyopathy
• Electrocardiogram to screen for arrhythmias
• Creatine Kinase (CK) concentration to assess muscle involvement
• Liver transaminases to assess liver function
• Pre-prandial blood sugar to assess hypoglycaemia
• In acute situations, if the patient presents with hypoketotic hypogly-
caemic encephalopathy, insure stabilisation with 10% dextrose in water
at rates of 10 mg/kg/min Intravenous (IV) initially; adjust infusion rate
according to blood glucose concentrations.
• The use of IV carnitine should be considered only when the diagnosis
of primary carnitine deficiency is entertained or confirmed. IV carnitine
may be considered in cases of organic acidemias (e.g., isovaleric acidaemia,
propionic acidaemia, methylmalonic acidaemia) when oral intake is not
feasible.
• Maintaining normal plasma carnitine levels can prevent the metabolic and
myopathic manifestations of CDSP. Primary treatment involves supple-
mentation of oral levocarnitine (L-carnitine) at a dose of 50-300 mg/kg/
day divided into three doses. The exact dose of carnitine supplementation
should be adjusted accordingly based on the individual’s plasma carnitine
level. (While L-carnitine supplementation has relatively few side effects,
high doses may result in increased gastrointestinal motility, diarrhoea, in-
testinal discomfort, and/or the production of trimethylamine, which can
result in a fishy odour. Decreasing the carnitine dose may reduce this po-
tential side effect. If that does not improve the odour, then a course of oral
metronidazole at a dose of 10 mg/kg/day for 7-10 days may be indicated).
• Long-term diet therapy: It is essential to prevent any period of fasting,
which would be sufficient to require the use of fatty acids as a fuel. Simply
ensuring that patients have adequate carbohydrate feeding at bedtime and
do not fast for more than 12 hours overnight can do this. The use of un-
cooked cornstarch at bedtime might be considered as a slowly released
form of glucose, although this has not been formally tested in these disor-
ders. Some authors recommend restricting fat intake. Although this seems
reasonable in patients with severe defects.
• Maintaining plasma carnitine levels will also reduce the risk of hypo-
glycaemic episodes. Other ways to prevent hypoglycaemic episodes in

90
Case 16

individuals with CDSP would include frequent feeding and avoiding


fasting. If an individual with CDSP is hospitalised for hypoglycaemia, or
needs to fast because of a medical or surgical procedure, treatment with
intravenous dextrose is recommended.
• To date, there have been no formal surveillance guidelines for individuals
with CDSP; however, the following screening recommendations are sug-
gested based on experienced neuro-physicians:
° Echocardiogram and electrocardiogram annually during childhood
and less frequently in adulthood. Individuals with cardiomyopa-
thy should be referred to cardiology for further management and
treatment.
° Plasma carnitine levels should be obtained and monitored frequently
(after two weeks of each dose adjustment) until levels reach within
the normal range. Once normalisation is reached, periodic plasma
carnitine analysis should be obtained twice a year in childhood and
annually for adults.
° CK and liver transaminases measurement can be considered during
acute illness.

REFERENCES
1. Burwinkel, B., Kreuder, J., Schweitzer, S., Vorgerd, M., Gempel, 1. Cederbaum,
S. D., Auestad, N., Bernar, J. Four-year treatment of systemic carnitine deficiency.
New Eng. J. Med. 310: 1395-1396, 1984.
2. Chapoy, P. R., Angelini, C., Brown, W. J., Stiff, J. E., Shug, A., Cederbaum, S. D.
Systemic carnitine deficiency--a treatable inherited lipid-storage disease presenting
as Reye’s syndrome. New Eng. J. Med. 303: 1389-1394, 1980.
3. Di San Filippo, C. A., Pasquali, M., Longo, N. Pharmacological rescue of carnitine
transport in primary carnitine deficiency. Hum. Mutat. 27: 513-523, 2006.
4. El-Hattab, A. W., Li, F.-Y., Shen, J., Powell, B. R., Bawle, E. V., Adams, D. J., Wahl,
E., Kobori, J. A., Graham, B., Scaglia, F., Wong, L.-J. Maternal systemic primary
carnitine deficiency uncovered by newborn screening: clinical, biochemical, and
molecular.
5. Lamhonwah, A.-M., Olpin, S. E., Pollitt, R. J., Vianey-Saban, C., Divry, P., Guffon,
N., Besley, G. T. N., Onizuka, R., De Meirleir, L. J., Cvitanovic-Sojat, L., Baric, I.,
Dionisi-Vici, C., Fumic, K., Maradin, M., Tein, I. Novel OCTN2 mutations: no
genotype-phenotype correlations: early carnitine therapy prevents cardiomyopathy.
Am. J. Med. Genet. 111: 271-284, 2002.
6. Lamhonwah, A.-M., Onizuka, R., Olpin, S. E., Muntoni, F., Tein, I. OCTN2
mutation (R254X) found in Saudi Arabian kindred: Recurrent mutation or ancient
founder mutation? J. Inherit. Metab. Dis. 27: 473-476, 2004.
7. Schimmenti, L. A., Crombez, E. A., Schwahn, B. C., Heese, B. A., Wood, T. C.,
Schroer, R. J., Bentler, K., Cederbaum, S., Sarafoglou, K., McCann, M., Rinaldo,
P., Matern, D., di San Filippo, C. A., Pasquali, M., Berry, S. A., Longo, N. Expanded
newborn screening identifies maternal primary carnitine deficiency. Molec. Genet.
Metab. 90: 441-445, 2007.

91
Child Neurology

8. Shibbani, K., Fahed, A. C., Al-Shaar, L., Arabi, M., Nemer, G., Bitar, F., Majdalani,
M. Primary carnitine deficiency: novel mutations and insights into the cardiac phe-
notype. Clin. Genet. 85: 127-137, 2014.
9. Wang, Y., Korman, S. H., Ye, J., Gargus, J. J., Gutman, A., Taroni, F., Garavaglia,
B., Longo, N. Phenotype and genotype variation in primary carnitine deficiency.
Genet. Med. 3: 387-392, 2001.
10. Yamak, A. A., Bitar, F., Karam, P., Nemer, G. Exclusive cardiac dysfunction in fa-
milial primary carnitine deficiency cases: a genotype-phenotype correlation. Clin.
Genet. 72: 59-62, 2007.

92
Case 17
A 6-year-old boy, second by order of birth, product of a non-consanguin-
eous couple, was brought to our hospital with complaints of frequent
falls while walking. He was having difficulty in performing activities which
other children of his age group could easily do. The patient started to sit at 11
months of age and began to walk independently at 19 months. Now he has
difficulty in standing from a sitting position and unable to hop on one foot.
He also has difficulty walking up stairs. His father complains that he runs very
awkwardly and fatigues easily. He also has problem of cognition in school for
the last two years. Birth history was uneventful. There was no history of any
neurological disorders in the family.
Physical examination: He was alert active well oriented with normal
language, he could speak full sentences. There was mild lordosis and tightness
of the heel cords. His calf muscles were hypertrophied with diffuse hypotonia.
He placed his hands on his knees to stabilise himself while getting up from
the floor. There was no ataxia or dysmetria. Deep tendon reflexes were 1+
throughout with bilateral plantar flexor responses. Truncal weakness was
present and neck flexors were weak as compared to extensors. No sensory
deficit was noted. His extraocular muscles were intact and all other accessible
cranial nerves were intact.

Answer the following questions:


• Give the most likely diagnosis.
• Provide the summary of the case.
• How do you localise the lesion?
• Discuss the differential diagnosis.
• Provide the appropriate diagnostic work-up.
• Discuss the management of the case.

93
Child Neurology

DIAGNOSIS
Duchenne Muscular dystrophy

SUMMARY
A 6-year-old boy presented with progressive proximal muscles weakness, dif-
ficulty in climbing up the stairs, easy fatigability and frequent fall. His ex-
amination showed gross hypotonia, mild lordosis, calf muscles hypertrophy,
diminished deep tendon reflexes, and positive Gower’s sign. His CPK level
was very high (25000 IU/L) and genetic study revealed DMD gene mutation.

Localisation of Lesion
Physical examination and history of this patient is suggestive of a primary
neurological problem that resulted in difficulty in getting up from a sitting
position, climbing up stairs, difficulty in walking. Lordosis results due to the
weakness of the pelvic girdle, abdominal muscles and muscles of the back.
Tight heel cords and toe walking are caused by weakness of more distil ante-
rior tibial and peroneal musculature. Thus, the disease is affecting either the
anterior horn cell, peripheral nerve, neuromuscular junction, or the muscles.
However, in this patient, examination and a high CPK level suggests disease of
the muscles. Calf hypertrophy is specific for a myopathic process, in particular
Duchenne muscular dystrophy.

DIFFERENTIAL DIAGNOSIS
• Becker’s muscular dystrophy: Signs and symptoms are similar to those
of Duchenne Muscular Dystrophy (DMD), but the age of onset is usually
later and the clinical involvement is milder. Patients are usually able to
walk throughout their teens and into early adulthood. Occasionally, pa-
tients present with CHF and cardiac arrhythmias before complaining of
muscle weakness and before diagnosis. The diagnosis of Becker’s mus-
cular dystrophy can be established by muscle biopsy that demonstrates a
diminished quantity/quality of dystrophin with at least 3% normal dys-
trophin. Dystrophin is expressed in skeletal muscles, smooth muscles,
cardiac muscles, and the brain. Little or absent dystrophin is identified in
cases of DMD, whereas BMD is associated with dystrophin of abnormal
size and/or quantity.
• Limb-Girdle Muscular Dystrophies (LGMDs): Before DNA and
dystrophin analysis, LGMDs was often misdiagnosed as Duchenne
Muscular Dystrophy (DMD). Initial involvement is of the shoulder and
pelvic girdle muscles; however, the pattern of muscle weakness can be
clinically indistinguishable from that of DMD. Calves can also be hyper-
trophic, and some patients walk only on their toes from infancy. Rate of
progression is highly variable. Severe disability, muscle contractures, skel-
etal deformities, and need for a ventilator are common, and cardiomyo-
pathy can occur in certain forms. CPK levels are usually lower than those

94
Case 17

in DMD. Identification of deletion or mutation of the relevant gene with


DNA analysis confirms the diagnosis.
• Emery-Dreifuss muscular dystrophy: Musculotendinous contractures
tend to be severe by comparison with muscle weakness, and the cervical
spine tends to be hyper-extended with entire spine rigidity. Early con-
tractures are seen at the neck, elbow, and Achilles tendons. Abnormal fat
distribution (lipodystrophy) occurs in some laminopathies, as does hu-
meropelvic or humeroperoneal weakness. Unlike people with Duchenne
Muscular Dystrophy (DMD), most patients are able to walk into the third
decade of life. Cardiac arrhythmias are common and often life-threaten-
ing. This is probably related to the fact that emerin is normally expressed
in the adhesive junctions of the heart. EMD (Emerin) or LMNA lamin
gene mutations are seen in DNA testing. Immunolabelling of emerin is
lost in X-linked cases. Conduction defects are seen on ECG, especially
AV blocks.
• Polymyositis: Polymyositis can occur at 4 to 7 years of age, and muscle
contractures and weakness can become as severe as those of Duchenne
Muscular Dystrophy (DMD). Muscle weakness usually occurs in the
proximal muscles, especially those of the shoulder and pelvic girdles.
Diagnosis is established by characteristic muscle biopsy findings of in-
flammation, including mononuclear invasion of non-necrotic muscle,
CD8+ cytotoxic/suppressor T cells, macrophages, and absence of perifas-
cicular atrophy of dermatomyositis.
• Spinal muscular atrophy. Kugelberg Welander syndrome is a milder
type of spinal muscular atrophy. It is a rare inherited anterior horn cell
disease characterised by wasting and weakness in the muscles of the arms
and legs, leading to walking difficulties, and eventual loss of ambulation.
Symptoms of Kugelberg Welander syndrome occur after 12 months of
age. Patients learn to walk, may fall frequently and may have trouble walk-
ing up and down stairs at 2-3 years of age; some patients will not show
functional changes until the teens. The legs are more severely affected
than the arms. Bulbar dysfunction and tongue fasciculations are some-
times present and may help distinguish SMA from other neuromuscu-
lar disorders. Intelligence is usually normal. KugelbergWelander disease
is caused by mutations in the SMN (Survival Motor Neuron) gene on
chromosome 5.
• Juvenile myasthenia gravis. Symptoms may begin gradually over weeks
or months. The child may become excessively tired after very little activ-
ity, and begin to have problems chewing and swallowing. Drooping eye-
lids may be so severe that the child cannot see. The symptoms tend to be
milder in the morning and worsen throughout the day. JMG is primar-
ily a clinical diagnosis with classical patterns of fluctuating weakness and
fatigability.
• The Juvenile or later-onset forms of GM2 Gangliosidosis. It can
manifest in childhood, adolescence, or even adulthood. Affected individ-
uals develop dysarthria and walking problems, primarily resulting from

95
Child Neurology

spastic paraparesis (in the childhood-onset form, between the ages of 3


and 6 years) or proximal muscle weakness (in the adult form, with symp-
tom onset in the teens). Ataxia with cerebellar atrophy is also a promi-
nent disease sign in the later-onset forms, and peripheral neuropathy has
been described in a subset of patients. Tonic-clonic or myoclonic seizures
are encountered in some children; psychiatric disturbances (i.e. bouts of
psychosis and episodes of depression) are present and may be the initial
manifestation of the disease, particularly among adult-onset patients.
Childhood-onset disease often leads to death by 15 years of age, preceded
by a period characterised by progressive spasticity, rigidity, and dementia,
and ending in the vegetative state. Mutation analyses performed in pa-
tients with late onset Tay–Sachs disease have identified a high prevalence
of HEXA gene on chromosome 15.
• Cerebral palsy: The great majority of children who walk on their toes
have static encephalopathies and at least mild spasticity. In many cases the
children are asymmetrically affected. Children with DMD are hypotonic
and tend to be symmetrically affected. Cerebral palsy is usually diagnosed
on clinical CNS findings marked by spasticity (not seen in neuromuscular
diseases) as well as cognitive, and sensory involvement.

DIAGNOSTIC WORK-UP
• Clinical laboratory tests: The serum creatine kinase level is the most
valuable and universally used screening test for Duchenne muscular dys-
trophy. Levels of creatine kinase muscle enzyme is greatly elevated, typi-
cally from 10,000 to 30,000 International Units/L, early in the disease. A
normal or minimally elevated creatine kinase level effectively excludes the
possibility of DMD. Due to the leakage of intracellular muscle proteins,
other muscle isoenzyme levels also increase in the circulation. These in-
clude Lactate Dehydrogenase (LDH), Alanine Aminotransferase (ALT)
and to a lesser degree, Aspartate Aminotransferase (AST). Similar diag-
nostic confusion may occur in pre-symptomatic BMD patients. Obtaining
a truly “liver-specific” Gamma Glutamyl Transferase (GGT) level elimi-
nates diagnostic error
• Genetic testing: A precise molecular genetics diagnosis is essential in all
patients with DMD and BMD, even in those whose diagnosis has been
confirmed by immunostaining for dystrophin on muscle biopsy. Direct
sequencing of the entire dystrophin gene will define the vast majority of
DMD/BMD patients not identified by deletion/duplication testing. With
more recent technology, it is possible to sequence the entire dystrophin
gene for the specific molecular defects responsible in the other 30-40 per-
cent of patients with DMD and BMD in whom a genetic abnormality has
not been detected. Several commercial laboratories offer gene sequencing
for the diagnosis of suspected dystrophinopathy patients, as well as for
carrier detection and prenatal diagnosis.

96
Case 17

• Muscle biopsy: Histologic study reveals fibre size variation, degenerating


and regenerating fibres, clusters of smaller fibres, endomysial fibrosis, and
a few scattered lymphocytes. As the disease progresses and degeneration
exceeds regeneration, a decrease in the number of muscle fibres is appar-
ent, with replacement of muscle with fat and connective tissue. Absence
of immunoreactivity for dystrophin, with monoclonal anti-bodies against
the C-terminal, rod domain, and N-terminal, is important for an accurate
diagnosis of dystrophinopathy. However, quantitative dystrophin analysis
by immunoblot correlates more closely with the diagnoses of DMD than
immunostaining, with levels of dystrophin being less than 5 percent of
normal in DMD patients.

MANAGEMENT
• Children with DMD and BMD are best cared for in a multidisciplinary
setting, in which physicians (neurologists, psychiatrists, orthopaedic sur-
geons, cardiologists, and pulmonologists), physical and occupational ther-
apists, nutritionists, exercise physiologists, and social workers can work
together for the overall well-being of the child and the family.
• Parents of the patient with Duchenne dystrophinopathy should be kept
informed on a timely basis about the expected clinical course, the need for
regular evaluations, and available treatment programmes.
• It is often necessary for appropriate team members to contact school au-
thorities or even participate in school team meetings to educate teachers
and peers about how to support the child with DMD and yet let the child
be as independent as possible. Support groups for the family, siblings, and
patient, and special camps should be made available, and participation
encouraged.
• Pharmacologic treatment: Daily corticosteroid therapy stabilises or
improves the strength of children with DMD, and it is the only treatment
proven to be effective for this disease. To date, the most effective, well-
studied prednisone dose is 0.75 mg/kg/day. There is a dose response effect,
the lowest proven effective dose being 0.3 mg/kg/day. Effects on strength
are seen as soon as 10 days after treatment starts, with a peak at 3 months,
followed by slowing of disease progression. Steroids may help prevent
exercise-induced apoptosis and thus lessen exercise-related muscle dam-
age. Deflazacort, an oxazoline derivative of prednisone, is also used in the
countries in which it is available. Deflazacort yields a similar benefit in
improving and maintaining strength, with a decreased incidence of exces-
sive weight gain related to prednisone-treated patients. It carries an in-
creased risk of asymptomatic cataract formation. The suggested dosage
is 0.9-1.2 mg/kg/day. Deflazacort is not Food and Drug Administration-
approved and is not available in the United States. Other immunosup-
pressive medications have produced mixed results. Whereas azathioprine
yields no benefit, there is uncontrolled evidence that cyclosporine is effi-
cacious in DMD. Prednisone remains the only drug that has been proven
effective in 80 percent of children with DMD. An American Academy of

97
Child Neurology

Neurology Practice Parameter paper on the use of steroids in treating and


a Cochrane Database review of all support the use of corticosteroids for
DMD. Despite the many potential side effects of corticosteroid treatment,
the benefits of steroid therapy outweigh the risks. There is accumulating
evidence that treatment should start as soon as the diagnosis is made.
• Experimental therapies
• Approximately 15 percent of DMD patients have stop codon mutations
as the molecular genetic basis of their dystrophinopathy. In an attempt to
repair mutant dystrophin genes in vivo, gentamicin, an aminoglycoside
anti-biotic that binds the ribosome and causes “read-through” of prema-
ture stop codon (nonsense) mutations, was tried first in the mdx murine
model of DMD, and then in clinical trials in patients with DMD or BMD.
• Other approaches involving gene repair mechanisms are being evaluated
in experimental models. These include delivery to dystrophin-deficient
cells of RNA–DNA oligonucleotides that target the specific mutation and
cause it to revert to the normal sequence (chimeraplasts) and the delivery
of anti-sense RNA molecules to dystrophin-deficient cells so that semi
functional dystrophin can be produced. This method forces the cell-splic-
ing machinery to skip the dystrophin gene exon that contains the gene
mutation, which results in the full translation of dystrophin messenger
RNA (minus the mutant exon) into an “in-frame” semi-functional dys-
trophin protein.
• Currently, the most promising therapies for DMD are arguably gene
therapy and exon skipping, both aiming to restore the expression of
dystrophin.
• Drisapersen, an exon skipping drug for Duchenne Muscular Dystrophy
(DMD), failed to meet the primary endpoint of a statistically significant
nt.
improvement.

RESPIRATORY CARE
• With the progression of muscle weakness, loss of respiratory muscle
strength is associated with ineffective cough and hypoventilation which
leads to pneumonia, atelectasis, and respiratory insufficiency, initially in
sleep and later in the waking state. These complications are often treatable
with careful monitoring and assessments of respiratory functions.
• Patients with DMD should have routine immunisations. In addition,
these patients should receive the pneumococcal vaccine and an annual
influenza vaccine. Maintaining good pulmonary toilet is essential; hence,
these patients should be taught strategies to improve airway clearance and
how to employ these techniques early and aggressively.
• The use of assisted cough technologies should be recommended when
peak cough flow is below 270 L/min or when the maximal expiratory pres-
sures are less than 60 cm H2O, or both.

98
Case 17

REHABILITATION
• Clinical experience in boys with DMD. There is general consensus that
high-resistance exercises, especially those involving eccentric contrac-
tions, such as weight lifting, are damaging to the muscle cell membrane
and should be avoided.
• Therefore, active non-resistive exercises are encouraged. Swimming is
widely recommended for these children. Maintaining an active lifestyle will
also help prevent excessive weight gain. Regular periods of daily walking
enhance maintenance of strength and may retard contracture formation.

CONTRACTURES
Contractures of the Achilles tendons and, later, development of more wide-
spread contractures occur in all DMD patients. Active range-of-motion exer-
cises supplemented by passive stretching are important for preventing contrac-
tures. Nighttime stretching orthoses are probably useful and are recommended
at age 5-6. A standing board tilted up 20 degrees may be used for 20 minutes
twice per day to provide constant stretching of the Achilles tendons. Keeping
the heel cords stretched through vigorous passive stretching by parents and
physical therapists helps the patient maintain better gait mechanics.

CARDIAC MANAGEMENT
DMD and BMB patients are facing symptomatic cardiomyopathy as their
cause of death. Although electrocardiographic abnormalities are common in
DMD, a better correlation of cardiac involvement and prognosis is obtained
with echocardiography. It is recommended that DMD patients should un-
dergo electrocardiography and echocardiography at the time of diagnosis, and
then be screened every 2 years up to age 10 and subsequently at yearly inter-
vals. Early preventive use of Angiotensin-Converting Enzyme(ACE) inhibi-
tors and, later, beta blockers may be needed.

GENETIC COUNSELLING
• DMD and BMD are inherited as X-linked recessive traits, and the risk to
siblings of a patient depends on the carrier status of the mother. Male pa-
tients with DMD rarely reproduce. Child carriers have a 50 percent chance
of transmitting the dystrophin mutation in each pregnancy. Sons who inherit
the abnormal gene are affected, whereas daughters who inherit it are carriers.
• Today, the specific molecular characterisation of a patient makes genetic
counseling and prenatal diagnosis far more accurate. If a specific mutation
is identified in a DMD or BMD patient, genetic testing of the mother or
sister for the same mutation determines whether she is or is not a carrier;
appropriate counseling for future pregnancies can be performed.

DRUG PRECAUTIONS
Use of anti-cholinergic drugs and ganglionic blocking agents should be avoid-
ed because of their tendency to decrease muscle tone. Patients with DMD

99
Child Neurology

have increased susceptibility to malignant hyperthermia, and proper evalua-


tion and preparation before administration of general anaesthesia are recom-
mended. Cardiotoxic drugs, such as halothane, should not be used, and cau-
tion is advised in inducing general anaesthesia.

Emotional and Behavioural Abnormalities


Depression is often associated with intellectual limitation, which may induce
low tolerance for frustration. Psychologic evaluation and counseling may be
necessary.

REFERENCES
1. Darras BT, Menache-Stroninki CC, Hinton V, Kunkel LM. Dystrophinopathies. In:
Neuromuscular Disorders of Infancy, Childhood and Adolescence: A Clinician’s Ap-
proach, 2nd ed, Darras BT, Jones HR Jr, Ryan MM, De Vivo DC (Eds), Academic
Press, San Diego 2015. p.551-592.
2. Aartsma-Rus A, Van Deutekom JC, Fokkema IF, et al.. Entries in the Leiden Duch-
enne muscular dystrophy mutation database: an overview of mutation types and
paradoxical cases that confirm the reading-frame rule. Muscle Nerve 2006; 34:135.
3. Takeshima Y, Yagi M, Okizuka Y, et al.. Mutation spectrum of the dystrophin gene
in 442 Duchenne/Becker muscular dystrophy cases from one Japanese referral
centre. J Hum Genet 2010; 55:379.
4. Pegoraro E, Hoffman EP, Piva L, et al.. SPP1 genotype is a determinant of disease
severity in Duchenne muscular dystrophy. Neurology 2011; 76:219.
5. Flanigan KM, Ceco E, Lamar KM, et alal.. LTBP4 genotype predicts age of ambula-
tory loss in Duchenne muscular dystrophy. Ann Neurol 2013; 73:481.
6. Bello L, Kesari A, Gordish-Dressman H, et al.. Genetic modifiers of ambulation in
the Cooperative International Neuromuscular Research Group Duchenne Natu-
ral History Study. Ann Neurol 2015; 77:684.
7. van den Bergen JC, Hiller M, Böhringer S, et al. Validation of genetic modifiers for
Duchenne muscular dystrophy: a multicentre study assessing SPP1 and LTBP4
variants. J Neurol Neurosurg Psychiatry 2015; 86:1060.
8. Centres for Disease Control and Prevention (CDC). Prevalence of Duchenne/
Becker muscular dystrophy among child aged 5-24 years-four states, 2007. MMWR
Morb Mortal Wkly Rep 2009; 58:1119.
9. Romitti PA, Zhu Y, Puzhankara S, et al. Prevalence of Duchenne and Becker mus-
cular dystrophies in the United States. Pediatrics 2015; 135:513.
10. Dooley J, Gordon KE, Dodds L, MacSween J. Duchenne muscular dystrophy: a
30-year population-based incidence study. Clin Pediatr (Phila) 2010; 49:177.
11. Wood CL, Straub V, Guglieri M, et al. Short stature and pubertal delay in Duch-
enne muscular dystrophy. Arch Dis Child 2016; 101:101.
12. Banihani R, Smile S, Yoon G, et al. Cognitive and Neurobehavioral Profile in Boys
With Duchenne Muscular Dystrophy. J Child Neurol 2015; 30:1472.
13. Spurney CF. Cardiomyopathy of Duchenne muscular dystrophy: current under-
standing and future directions. Muscle Nerve 2011; 44:8.

100
Case 18
A 9-month-old girl was referred to the paediatric neurology clinic with
developmental delay, hypotonia, inability to take food orally, and
failure to thrive. Her family history was unremarkable and antenatal course
was uneventful. At 2 months of age, she was admitted for an episode of
unprovoked seizures.
On physical examination, she was noted to have axial hypotonia, hyporeflexia,
esotropia, and dysmorphic features including a high arched palate, narrow
palpebral fissures and inverted nipples. There were prominent fat pads on her
supra gluteal regions and global developmental delay. Camptodactyly of the
3rd and 4th fingers was noted without other abnormal features. Length, weight,
and head circumference were between the 50th and 75th centiles. At admission,
her respiratory and myocardial functions were deteriorated and there was an
escalating pericardial effusion and we had to twice perform pericardiocentesis.
The pericardial effusion was serous in nature and the biopsy revealed mild
inflammation and fibrous thickening of the peritoneum. Investigations for
a metabolic etiology showed normal levels of urine amino acids, plasma
amino acids, plasma organic acids, urine organic acids, basic chemistry panel,
serum transaminases, acylcarnitine profile, and very long chain fatty acid
profile. There was no documented episode of hypoglycaemia, presence of
urinary ketones and/or metabolic acidosis. Abdominal ultrasound revealed
hyperechogenicity of the renal parenchyma. Magnetic resonance imaging of
the brain revealed a cerebellar vermis hypoplasia and a generalised reduction
in myelination.

Answer the following questions:


• What is most likely diagnosis?
• Provide the summary of this case.
• Discuss the differential diagnosis.
• Discuss the diagnostic approach.
• Provide the appropriate management.

101
Child Neurology

DIAGNOSIS
Congenital Disorder of Glycosylation (CDG-1a)

SUMMARY
A 9-month-old girl was referred with developmental delay, hypotonia, and
failure to thrive. She also had one episode of seizures at 2 months of age. Her
physical examination revealed, central hypotonia, hyporeflexia, esotropia, and
dysmorphic features including a high arched palate, narrow palpebral fissures
and inverted nipples. Camptodactyly of the 3rd and 4th fingers was noted with-
out other abnormal features.
The child was felt to have a phenotype classic of the CDG-1a. Isoelectric
focusing of transferrin showed a pattern consistent with CDG-1a. Genetic
analysis revealed that she was homozygote for pathogenic mutation in phos-
phomannomutase 2 (PMM2) gene, consistent with a diagnosis of CDG-1a.

DIFFERENTIAL DIAGNOSIS
• The differential diagnosis of developmental delay is broad. However, in
a patient like ours who has many systems affected, a multisystem disease
should be suspected.
• Most infants with global developmental delay have a static encephalopathy
caused by an antenatal or perinatal disturbance. However, 1% of infants
with developmental delay and no evidence of regression have an inborn
error of metabolism and 3.5% to 10% have a chromosomal disorder.
• Chromosomal Disturbances: Abnormalities of autosomal chromosomes
are always associated with infantile hypotonia and in addition, multiple
minor face and limb abnormalities are usually associated features. These
abnormalities in themselves are common, but they assume diagnostic sig-
nificance in combination.
• The differential diagnosis is then mostly limited to metabolic diseases that
produce brain, cardiac, and muscle involvement. There is also significant
clinical overlap in many of these syndromes, further complicating the di-
agnosis in many children. Typically, these symptoms are produced by the
accumulation of small-molecule metabolites, insufficient amounts of a
necessary enzymatic pathway product, or abnormal transport of certain
molecules.
• Numerous different metabolic disorders share similar signs and symp-
toms to the CDG. Such disorders include congenital muscle disorders
(myopathies), urea cycle disorders, inborn errors of bile metabolism, fatty
acid oxidation disorders, organic acidurias, peroxisome biogenesis disor-
ders, and sphingolipidoses. Additional disorders that have similar signs
and symptoms include cerebral palsy, Prader-Willi syndrome, congeni-
tal coagulation disorders, and ataxia-telangiectasia and other hereditary
ataxias.

102
Case 18

• Congenital Disorders of Glycosylation (CDGs) are a genetically het-


erogeneous group of autosomal recessive disorders caused by enzymatic
defects in the synthesis and processing of asparagine (N)-linked glycans
or oligosaccharides on glycoproteins. CDG typically present as multisys-
temic disorders with a broad clinical spectrum including developmental
delay, hypotonia, with or without neurological abnormalities, abnormal
magnetic resonance imaging findings, skin manifestations, and coagulopa-
thy. There is considerable variation in the severity of this group of diseases
ranging from a mild presentation in adults to severe multiorgan dysfunc-
tions causing infantile lethality. CDG should be suspected in all patients
with neurological abnormalities including developmental delay and sei-
zures, brain abnormalities such as cerebellar atrophy or hypoplasia as well
as unexplained liver dysfunction. Abnormal subcutaneous fat distribution
and chronic diarrhoea each may or may not be present. The differential
diagnosis of abnormal transferrin patterns also includes liver disease not
related to CDG including uncontrolled galactosaemia, hereditary fruc-
tose intolerance in acute crisis, and liver disease of unexplained etiology.
CDG1a is the most common form of CDG and was the first to be charac-
terised at the molecular level.

DIAGNOSTIC WORK-UP
• A diagnosis
osis of a CDG may be suspected based upon the identification of
characteristic symptoms, a detailed patient history and a thorough clinical
evaluation.
• A variety of specialised tests may be necessary to confirm a diagnosis of
CDG and/or to determine the specific subtype. CDG should be consid-
ered and ruled out in any unexplained syndrome.
• A simple blood test to analyse the glycosylation status of transferrin can di-
agnose CDG due to N-glycosylation. Another test known as electrospray
ionisation-mass spectrometry may be used to detect abnormal transferrin.
IEF (isoelectric focusing) is the standard test for diagnosing CDG due to
a defect of N-glycosylation.
• Some subtypes of CDG can be diagnosed through an enzyme assay, a
test that measures the activity of a specific type of enzyme in Leukocytes
or Fibroblasts. However, for many subtypes no enzyme assay has been
developed.
• Molecular genetic testing is needed to confirm a diagnosis of CDG.

MANAGEMENT
• Treatment may require the coordinated efforts of a team of specialists.
Paediatricians, neurologists, surgeons, cardiologists, speech pathologists,
ophthalmologists, gastroenterologists, and other healthcare professionals.
• Although there is no specific therapy for most forms of CDG. Individuals
with MPI-CDG are treated with oral mannose. This therapy bypasses the

103
Child Neurology

underlying genetic defect in glycosylation that causes the disorder. Some


individuals have experienced a near complete resolution of symptoms fol-
lowing mannose therapy.
• Some individuals have been treated with fucose. Fucose therapy can be
beneficial in treating recurrent infections associated with this form of
CDG and improving health. However, fucose therapy does not help with
other symptoms of this disorder.
• Some individuals with PIGM-CDG have been treated with butyrate,
which increases the transcription of PIGM and is able to help manage
seizures associated with this form of CDG.
• Maintaining proper nutrition and caloric intake is critical for infants with
chronic disorders and often a particular challenge for infants and children
with CDG. Symptomatic therapies are common for infants and children
with CDG including nutritional supplements to ensure maximum caloric
intake. In addition, some children may require gastrostomy or a nasogas-
tric tube. A variety of therapies may be necessary to ensure proper feeding
including agents to thicken food, antacids, and maintaining an upright
position when eating.
• Most affected children will benefit from occupational, physical and speech
therapy. Additional medical, social, and/or vocational services including
special remedial education may also be beneficial. Ongoing counseling
and support for parents is beneficial as well.
• Additional therapies for CDG depend upon the specific abnormalities
present and generally follow standard guidelines. For example, anti-con-
vulsants may be used to treat seizures, thyroid hormone to treat hypothy-
roidism, and surgery may be used to treat certain skeletal malformations.

Investigational Therapies
• Researchers are studying enzyme replacement therapy for the treatment
of CDG.
• Gene therapy is also being studied as another approach to therapy for in-
dividuals with CDGs. In gene therapy, the defective gene present in a pa-
tient is replaced with a normal gene to enable the production of the active
enzyme and prevent the development and progression of the disease in
question.
• Researchers are also studying whether simple pharmacological agents can
be developed that would bypass the underlying genetic defect in CDG
allowing proper glycosylation to occur. Such agents may increase the syn-
thesis or activity of alternative enzymes that could carry out the functions
normally performed by the deficient enzyme in question.

104
Case 18

REFERENCES
1. Haeuptle MA, Hennet T. Congenital disorders of glycosylation: an update on de-
fects affecting the biosynthesis of dolichol-linked oligosaccharides. Hum Mutat.
2009 Dec;30(12):1628-41.
2. Krasnewich D, O’Brien K, Sparks S. Clinical features in adults with congenital dis-
orders of glycosylation type Ia (CDG-Ia). Am J Med Genet C Semin Med Genet.
2007 Aug 15;145C(3):302-6.
3. Monin ML, Mignot C, De Lonlay P, Héron B, Masurel A, Mathieu-Dramard M,
Lenaerts C, Thauvin C, Gérard M, Roze E, Jacquette A, Charles P, de Baracé C,
Drouin-Garraud V, Khau Van Kien P, Cormier-Daire V, Mayer M, Ogier H, Brice
A, Seta N, Héron D. 29 French adult patients with PMM2-congenital disorder of
glycosylation: outcome of the classical pediatric phenotype and depiction of a late-
onset phenotype. Orphanet J Rare Dis. 2014 Dec 11;9:207.
4. de Lonlay P, Seta N, Barrot S, Chabrol B, Drouin V, Gabriel BM, Journel H, Kretz
M, Laurent J, Le Merrer M, Leroy A, Pedespan D, Sarda P, Villeneuve N, Schmitz
J, van Schaftingen E, Matthijs G, Jaeken J, Korner C, Munnich A, Saudubray JM,
Cormier-Daire V. A broad spectrum of clinical presentations in congenital disor-
ders of glycosylation I: a series of 26 cases. J Med Genet. 2001 Jan;38(1):14-9.
5. Brodsky MC, ed. Pediatric Neuro-Ophthalmology, 2nd ed. New York, NY: Spring-
er; 2010: 493-494.
6. Patterson MC. Congenital Disorders of Glycosylation. In: NORD Guide to Rare
Disorders. Lippincott Williams & Wilkins. Philadelphia, PA. 2003:457-458.
7. Ohba C, Okamoto N, Murakami Y, et al.. PIGN mutations cause congenital anom-
alies, developmental delay, hypotonia, epilepsy, and progressive cerebellar atrophy.
Neurogenetics. 2014; 15:85-92.
8. Maydan G, Noyman I, Har-Zahav A, et al.. Multiple congenital anomalies-hypo-
tonia-seizures syndrome is caused by a mutation in PIGN. J Med Genet. 2011;
48:383-389.
9. Jaeken J. Congenital disorders of glycosylation. Ann NY Acad Sci. 2010; 1214:190-
198.
10. Vodopiutz J, Bodamer OA. Congenital disorders of glycosylation–a challenging
Di 208;31:267-269.
group of IEMs. J Inherit Metab Dis.
11. Matthijs G. Euroglycanet: a European network focused on congenital disorders of
glycosylation. Eur J Hum Genet. 2005;13:395-397.

105
19 Case
A 5-year-old boy was brought to the emergency department after his
parents witnessed him having generalised convulsions occurring shortly
after the child went to bed. They reported hearing gurgling noises coming
from his room. When they found him, he was sitting up in bed, irritable, and
was not able to speak. He had had a cold but otherwise had been well recently.
Discussion with the family revealed a past history of several nocturnal events
with right facial and tongue clonic movements with drooling, lasting less than
2 minutes.
Physical examination revealed normal mental Status: Alert and cooperative.
Normally developed language: All accessible cranial nerves were intact. Motor
examination showed normal bulk and tone with 5/5 strength throughout. Gait
was also normal. Deep tendon reflexes: 2+ throughout with bilateral plantar
flexor responses. Cerebellar examination was also unremarkable. No focal
neurological deficits were noted.
A Computed Tomography (CT) scan of the head in the emergency room was
unremarkable. Electroencephalography (EEG) performed on the next day of
admission showed prominent epileptiform activity with high-amplitude sharp
waves with following slow waves, predominantly in the left central-temporal
regions but occasionally seen independently on the left. The spike activity was
activated (increased in frequency) during sleep state.

Answer the following questions:


• Give the most likely diagnosis.
• Discuss the differential diagnosis.
• Discuss the diagnostic work-up.
• Provide the management of this patient.

106
Case 19

DIAGNOSIS
Benign Childhood Epilepsy with Centrotemporal Spikes, (BECTS) (Rolandic
epilepsy)

SUMMARY
A 5-year-old boy had recurrent episodes of seizures shortly after going to sleep
with gurgling sounds, drooling of saliva from the mouth, and facial twitching.
Electroencephalography (EEG) result showed prominent epileptiform activity
with high-amplitude sharp waves with following slow waves, predominantly
in the left central-temporal regions.

Localisation of Clinical Findings


Right facial twitching and speech arrest localise to the left centrotemporal,
Rolandic region.

DIFFERENTIAL DIAGNOSIS
• Benign childhood epilepsy with centrotemporal spikes: BECTS is
an electroclinical syndrome characterised as focal, genetically determined,
and age-related epilepsy. Patients usually present predominantly with noc-
turnal seizures along with unilateral or bilateral centrotemporal diaphasic
spike waves. Simple febrile seizures may occur before Rolandic seizures
in approximately 10 to 20% of cases. Seizures typically begin between 3
and 12 years and resolve spontaneously until 16 years. It is usually diag-
nosed by the association of clinical findings and the Electroencephalogram
(EEG), which has a normal background cerebral activity with high voltage
sharp waves in the centrotemporal (Rolandic) region, followed by slow
waves activated by sleep. BECTS was always considered a benign develop-
mental disorder, because of the absence of obvious anatomic lesions, pre-
dictable spontaneous remission of seizures, and evidences of no cognitive
and language impairments when compared to normal children. However,
in the last 10 years, the “benign” term has been questioned. Studies have
shown some degrees of cognitive deficits during the active epileptic phase.
Neuropsychological alterations have been described for language, impair-
ment in verbal and attention skills, executive functions and memory, and
phonological awareness.
• Nocturnal paroxysmal events are commonly misdiagnosed or under
diagnosed. Differentiating epileptic seizures from sleep disorders can
be difficult based on history alone. Absence of epileptiform activity on
a scalp EEG interictally and ictally makes the diagnosis a further chal-
lenge. However, following differential diagnosis must be considered:
Nocturnal frontal lobe epilepsy, Dysembryoplastic Neuroepithelial
Tumour (DNET), Panayiotopoulos syndrome, Cortical dysplasia (focal
structural abnormality), and parasomnias.

107
Child Neurology

• Frontal Lobe Epilepsy (NFLE): Nocturnal frontal lobe epilepsy is a


condition which occurs only during sleep and which originates in the
frontal lobes. Nocturnal frontal lobe seizures typically: (1) have an abrupt,
explosive, onset awakening the patient from NREM 2 sleep; (2) are ac-
companied by sustained asymmetric dystonic, tonic posturing, and hy-
permotor behaviours including thrashing, pedaling and kicking; (3) tend
to be stereotyped in appearance for the individual patient; (4) are brief,
typically lasting 20 to 30 seconds; (5) are associated with preserved aware-
ness; (6) have no postictal confusion or amnesia; (7) typical onset occurs
between 7 and 12 years, and (8) have no scalp-recorded ictal EEG activity
accompanying them. Nocturnal sleep EEGs with video monitoring is a
reliable method for diagnosing it.
• Parasomnias: Parasomnias are undesirable physical events or experi-
ences that occur during entry into sleep, within sleep, or during arous-
als from sleep.” As delineated by the International Classification of Sleep
Disorders, Second Edition, parasomnias are classified as: 1) disorders of
arousal (from non-rapid eye movement, or NREM, sleep); 2) parasom-
nias are usually associated with REM sleep; and 3) other parasomnias. In
most cases the child will not remember any details of what transpired.
Parents should also be asked to describe in detail the movements and be-
haviours that are typically seen. Disorders of arousal (sleep walking, con-
fusional arousals, sleep terrors) can be thought as differential diagnosis of
epilepsy. Night terrors is a disorder of arousal, a common type of para-
somnia. Classic sleep terrors occur in the first half of the night, usually
in the first 60 to 90 minutes of sleep, and the child arouses suddenly out
of deep non-REM sleep. Typically, the child has no memory of the sleep
terror episode.
• Panayiotopoulos syndrome: Seizures usually occur in sleep, and
autonomic and behavioural features predominate. Age at onset of
Panayiotopoulos syndrome is 3 to 6 years, but the range extends from 1
to 14 years. These include vomiting, pallor, sweating, irritability, and tonic
eye deviation. The seizures last for hours in one-third of patients. Seizures
are infrequent, and the overall prognosis is good, with remission occur-
ring in 1 to 2 years. One-third of children have only one seizure. Interictal
EEG shows runs of high amplitude 2-to 3-Hz sharp-and slow-wave com-
plexes in the posterior quadrants.
• Rolandic epilepsy should be differentiated from several other conditions,
especially centrotemporal spikes without seizures, centrotemporal spikes
with local brain pathology, central spikes in Rett syndrome and fragile
X syndrome, malignant Rolandic epilepsy, temporal lobe epilepsy and
Landau-Kleffner syndrome.
• Other localisation-related epilepsies caused by underlying structural le-
sions (trauma, cerebrovascular disease, neuronal migration abnormalities,
mesial temporal sclerosis) should also be considered.

108
Case 19

DIAGNOSTIC WORK-UP
• A normal developmental history and normal examination support the diag-
nosis, but abnormal findings should not exclude the diagnosis. In addition
to seizure semiology, the diagnosis of BECTS rests on the presence of typi-
cal EEG findings. The interictal EEG will have a normal background. Light
sleep can almost always be recorded in an appropriate outpatient setting.
Epileptiform activity consisting of high-voltage diphasic or surface-negative
spikes with following slow wave is usually present and is usually activated
by light sleep.
• MRI (Magnetic Resonance Imaging): For the patient with classic features
of BECTS, including normal examination, typical seizures, and CTS,
neuroimaging is not usually indicated. This test is usually normal in chil-
dren with BREC/BECTS.
• Children with atypical clinical or EEG features should have imaging per-
formed because treatable CNS abnormalities may be present, including
malformations of cortical development.
• Nocturnal polysomnography may be suggested by the occurrence of
events during sleep, but careful history and typical interictal EEG will rule
out a primary sleep disorder.

MANAGEMENT
• Given the benign nature of the condition and the low seizure frequency,
treatment is often unnecessary, withholding AEDs is reasonable if the
child and family are comfortable with this approach. If treatment is indi-
cated, carbamazepine or Oxcarbazepine is often the first medication to be
tried, and seizures usually are well controlled. Once-a-day (HS) adminis-
tration may be the only regimen needed to control seizures.
• Other reportedly effective AEDs include phenobarbital, phenytoin, valp-
roic acid, clonazepam, clobazam, gabapentin and sulthiame. Levetiracetam
may also be effective and well tolerated.
• Treatment can be short and drugs can almost certainly be discontinued
after two years without seizures and with normal EEG findings; perhaps
even earlier. Parental education about Rolandic epilepsy is the cornerstone
of correct management. The traumatising, sometimes long-lasting effect
on parents is significant.
• By general consensus, the prognosis of BECCT is excellent, as almost all
patients achieve remission by adolescence.

109
Child Neurology

REFERENCES
1. Muhammad Saeed, M Azam, Nadeem Shabbir, Qamar ali Shair. Is "benign Child-
hood Epilepsy with Centrotemporal Spikes" Always Benign? Iran J Child Neurol.
2014 Summer; 8(3): 38-43.
2. Schneebaum-Sender N, Goldberg-Stern H, Fattal-Valevski A, Kramer U. Does a
normalizing electroencephalogram in benign childhood epilepsy with centrotem-
poral spikes abort attention deficit hyperactivity disorder?. Pediatr Neurol. 2012 Oct.
47(4):279-83.
3. Lopes AF, Simões MR, Monteiro JP, Fonseca MJ, Martins C, Ventosa L, et al al. In-
tellectual functioning in children with epilepsy: Frontal lobe epilepsy, childhood
Seizure. 2013
absence epilepsy and benign epilepsy with centro-temporal spikes. Seizure
Aug 12.
4. Kramer U, Nevo Y, Neufeld MY, Fatal A, Leitner Y, Harel S. Epidemiology of
epilepsy in childhood: a cohort of 440 consecutive patients. Pediatr Neurol.
Neurol 1998
Jan. 18(1):46-50.
5. Vannest J, Tenney JR, Gelineau-Morel R, Maloney T, Glauser TA. Cognitive and
behavioral outcomes in benign childhood epilepsy with centrotemporal spikes.
Epilepsy Behav. 2015 Apr. 45:85-91.
6. Kavros PM, Clarke T, Strug LJ, Halperin JM, Dorta NJ, Pal DK. Attention impair-
ment in rolandic epilepsy: systematic review. Epilepsia.
Epilepsia. 2008 Sep. 49(9):1570-80.
7. Connolly AM, Northcott E, Cairns DR, McIntyre J, Christie J, Berroya A, et al.
Quality of life of children with benign rolandic epilepsy. Pediatr Neurol. 2006 Oct.
35(4):240-5.
8. Tan HJ, Singh J, Gupta R, de Goede C. Comparison of anti-epileptic drugs, no
treatment, or placebo for children with benign epilepsy with centro temporal
spikes. Cochrane Database Syst Rev. 2014 Sep 5.

110
Case 20
A 5-year-old boy was admitted with a complaint of recurrent attacks of
febrile seizures. He was the first child of non-consanguineous parents.
He was delivered full term by cesarean section with unremarkable antenatal
maternal history and there were no post-natal complications. He had repeated
episodes of convulsions for the last four months. He had been treated sympto-
matically with anti-convulsant drugs. At the 5th month of age he was admitted
with protracted fever. A full sepsis work-up showed no positive results and
cause of the fever was not found. He had unexplained recurrent fever con-
stantly, especially during the summer months, he was unable to tolerate ex-
posure to the sun. After his teeth started to grow, he began to chew his fingers
and bit off the tip of his tongue. At that time, his parents noticed that he was
insensitive to painful stimuli. At 2½ years of age he was admitted with pain-
less ulceration of soles of both feet, not healing for the previous six months.
Some other relatives who reside in his home country, had similar neurologic
problems but no definitive diagnosis has ever been made.
On physical examination, the child had self-inflicted injuries to the lips and
a mutilated tongue, the tips of his toes and his fingers were missing, there were
multiple ulcers on both feet. His body temperature was 39.2 C, but his skin
was dry and warm. On neurologic examination the patient was lethargic and
could be temporarily awakened with verbal stimuli. There was no evidence of
meningism and the optic fundi were normal. There was a global delay in the
developmental milestones and the child was mentally retarded. He was not
able to walk.

Answer the following questions:


• What is the most likely diagnosis?
• Discuss the differential diagnosis.
• What should be the diagnostic approach?
• Discuss the appropriate management.

111
Child Neurology

DIAGNOSIS
Type IV Hereditary Sensory Autonomic Neuropathy (HSAN type IV)

SUMMARY
A 5-year-old male child had multiple admissions with febrile convulsions. At
a very early age he also started to have intolerance to heat associated with self-
mutilation. On physical examination the child had self-inflicted injuries to
the lips and a mutilated tongue, the tips of his toes and his fingers were miss-
ing, there were multiple ulcers on both feet. His sural nerve biopsy from the
calf of the leg revealed characteristic findings including reduced numbers of
myelinated and unmyelinated small-diameter fibres with normal numbers of
large-diameter fibres. His genetic study revealed HSAN IV, which is caused by
mutations in the NTRK1 gene on chromosome 1q21-22.

DIFFERENTIAL DIAGNOSIS
The differential diagnosis of polyneuropathy in childhood and adolescence
is broad. Both hereditary and acquired conditions must be considered. It
is helpful to determine the distribution of the neuropathy (symmetric ver-
sus asymmetric), the destructive process (demyelinating versus axonal), the
nerves involved (motor versus sensory), the location (distal versus proximal)
and the time course of development of the neuropathy (acute versus chronic).
However, as in this case, a positive family history can help narrow the differ-
ential diagnosis significantly.
• The Hereditary Sensory and Autonomic Neuropathies (HSAN), also
known as the hereditary sensory neuropathies, include at least six similar,
but distinct inherited neurodegenerative disorders of the nervous system
that frequently progress to loss of feeling, especially in the hands and feet.
Some of these disorders have several subtypes based upon the specific
associated genes. Some types of HSAN are related to or identical with
some forms of Charcot-Marie-Tooth disease, and others are related to, or
identical with, familial dysautonomia (Riley-Day syndrome). The clas-
sification of the HSANs is complicated, and the experts too do not always
agree on it. Furthermore, HSANs are classified more broadly as peripheral
neuropathies, which encompasses all of the nerves outside of the central
nervous system. HSAN IV is also known as Congenital Insensitivity to
Pain with Anhidrosis (CIPA)
• Hereditary Sensory and Autonomic Neuropathy type IV (HSAN4 or
HSAN IV) is a rare genetic disorder that usually begins in infancy and
is characterised by an inability to feel pain and an inability to sweat (an-
hidrosis). Affected individuals also cannot feel temperature and cannot
distinguish between hot and cold. The sensory loss in individuals with
HSAN IV is due to abnormal functioning of the sensory nerves that con-
trol responses to pain and temperature. Anhidrosis can cause recurrent
episodes of fever and high body temperature. An inability to feel pain can

112
Case 20

lead to unintentional self-mutation, repeated fractures, and joint damage.


Affected individuals and especially children or infants may be unaware
of injury delaying treatment. HSAN IV is caused by mutations in the
NTRK1 gene. The disorder is inherited in an autosomal recessive manner.
• HMSN1 is the most common form of hereditary neuropathy and should
also be considered. Most of the patients with this condition have weak-
ness, and a reduced ability to feel pain and sense hot and cold. As the
disorder progresses, the sensory abnormalities can affect the hands, arms,
shoulders, and abdomen. Affected individuals may also experience mus-
cle wasting and weakness as they get older, but this varies widely within
families. Affected patients typically get ulcers on their feet or hands or
infections of the soft tissue of the fingertips that are slow to heal. Because
affected individuals cannot feel the pain of these sores, they may not seek
treatment right away.
• HSAN II (Morvan’s disease): Symptoms occur in infancy or early
childhood. Affected individuals have acral anhidrosis; ulcers, paronychia,
whitlows, or other trophic changes of the fingers and toes; and other au-
tonomic dysfunction including tonic pupils, oromotor incoordination,
constipation from gastrointestinal dysmotility, bladder dysfunction, in-
termittent fevers, impaired sensory perception, hypotonia, and apnoea.
Unrecognised injuries and neuropathic arthropathy (Charcot joint) oc-
cur. Except for decreased or absent tendon reflexes, the general neurologic
examination is normal.
• Riley-Day syndrome: Riley-Day syndrome is another heterogeneous
group of disorders characterised by autonomic and sensory findings.
Newborns have absent or weak suck reflex, hypotonia and hypothermia.
Delayed physical development, poor temperature and motor incoordina-
tion are seen in early childhood. Other features include reduced or absent
tears, depressed deep tendon reflexes, absent corneal reflex, postural hy-
potension and relative indifference to pain. Pain fibres are typically af-
fected. Four forms of the disease are recognised.
• Chronic Inflammatory Demyelinating Polyneuropathy (CIDP):
Acquired neuropathy also needs to be considered in the differential di-
agnosis, particularly when the family history is negative. Special attention
should be given to any treatable conditions that may be responsible for an
affected patient’s symptoms. First, Chronic Inflammatory Demyelinating
Polyneuropathy (CIDP) must be considered in any child presenting with
a subacute polyneuropathy. CIDP is an immune-mediated demyelinating
polyneuropathy and is distinguished from Guillain-Barre syndrome by
its clinical course. Weakness of the lower extremities is the predominant
symptom although cranial, autonomic, and sensory neuropathies can also
occur in CIDP.
• There are a wide variety of metabolic diseases that can present with pe-
ripheral neuropathies, but this is rarely the primary manifestation. For

113
Child Neurology

example, leukodystrophies (metachromatic leukodystrophy, Krabbe dis-


ease, Pelizaeus-Merzbacher disease), lipid storage diseases (Tay-Sachs,
Fabry disease, Gaucher disease), peroxisomal disorders (Refsum disease,
adrenoleukodystrophy), and mitochondrial disorders can manifest with
peripheral neuropathy. Inborn errors of metabolism would be unusual in
the patient presented in this case, however.

DIAGNOSIS
• A diagnosis is based upon identification of characteristic symptoms, a de-
tailed patient history, a thorough clinical evaluation and a variety of spe-
cialised tests. Characterised symptoms proposed as necessary for a diagno-
sis are insensitivity to pain, anhidrosis and intellectual disability. However,
the severity of these symptoms is highly variable. The following studies
have been used for clinical diagnosis prior to the availability of NTRK1
molecular genetic testing:
• Pain tests: Caution: (These tests should not be performed routinely
for ethical reasons) Painful stimuli that fail to evoke either withdrawal
or emotional change in persons with CIPA include pin prick; vigorous
pressure on the Achilles tendons, the testes, the stylomastoid processes,
and the superior orbital rim; burning heat; immersion of the limbs in ice
water; galvanic electrical stimulation of the skin; and prolonged ischaemia
of the limbs.
• Standardised tests of quantitative thermal perception. Decreased percep-
tions of hot and cold assessed quantitatively using standardised tests of
thermal perception are also observed.
• Histamine test: An intradermal injection of histamine can stimulate
the axon reflex in wheal formation and diffuse erythema in the following
manner. A stimulus applied to one branch of a nerve induces an impulse
that moves centrally to the point of division of the nerve and then down
another branch to blood vessels. The activated neurons release neuropep-
tides that induce vasodilation and extravasation of plasma. An intradermal
injection of histamine produces in controls both a local wheal and flare,
but in individuals with CIPA only produces a local wheal and no flare.
Additionally, controls-but not individuals with CIPA-note burning pain
or pruritus in the axon flare area.
• Sweating tests: Individuals with CIPA lack sweating. Methods available
in clinical practice to evaluate autonomic sweating function.
• Cold pressor test. Submersion of the forearm in ice-cold water usually
causes an increase in blood pressure as an autonomic response; however,
no increase in blood pressure is observed in individuals with CIPA.
• Skin biopsy. The skin appears normal, except for nonspecific changes
secondary to traumatic injuries. Histologic studies demonstrate normal
sweat glands, sebaceous glands, hair follicles, and nerve receptors.

114
Case 20

• Electron microscopic studies: They reveal lack of innervation of the


eccrine sweat glands with loss of unmyelinated sudomotor. No nerve
endings are demonstrated in the epidermis.
• Immunohistochemistry: It demonstrates almost complete absence of
innervation to sweat glands, blood vessels, and erector pilomotor muscles.
• Nerve biopsy: Electron microscopic studies of cutaneous nerves reveal
complete absence of small-diameter myelinated and unmyelinated nerve
fibres without degenerative or regenerative changes. Morphometric anal-
ysis confirms the reduced numbers of these small-diameter fibres com-
pared with normal numbers of large-diameter myelinated fibres.
• The diagnosis of CIPA is confirmed by identification of biallelic path-
ogenic variants in NTRK1 (TRKA).. Furthermore, because of the large
number of mutations that have been identified in HSAN IV, molecular
genetic testing is not routinely used.

MANAGEMENT
• Treatment is directed toward the specific symptoms that are apparent in
each individual. Treatment may require the coordinated efforts of a team
of specialists. Paediatricians, dermatologists, neurologists, dental special-
ists, orthopaedists, ophthalmologists, and other healthcare profession-
als may need to systematically and comprehensively plan for an affected
child’s treatment.
• Psychosocial support for the entire family is essential as well. Genetic
counseling is of benefit for affected individuals and their families.
• Affected individuals may be treated with acetaminophen or ibuprofen
when fevers are present. Direct cooling in a bath or with a blanket de-
signed to lower body temperature (cooling blanket) may also be used.
• Various orthopaedic measures may be necessary to treat abnormalities af-
fecting the bones and joints including surgery or the use of braces or or-
thopaedic devices.
• Various dental procedures may be used to treat individuals. Smoothing
over or grinding down the sharp edges of teeth, prophylactic use of
crowns, the use of a night-guard and other orthodontic treatments may be
considered. Extracting teeth to prevent self-mutilation has also been done.
• Treatment of neurotrophic keratitis can include a procedure in which the
eyelids are sewn together to narrow the opening (tarsorrhaphy), plastic
surgery to repair the cornea (keratoplasty), replacement of a part or all
of an affected cornea with healthy corneal tissue from a donor (scleral
corneal graft), and special contact lenses that protect the cornea (scleral
bandage lens). These contact lenses create a space between the front of the
cornea and the back of the lenses that fills with a sterile saline solution.
• Behavioural issues tend to improve with age. These issues have been
treated with behaviour modification techniques along with anti-psychotic
medications or attention deficit hyperactivity disorder medications.

115
Child Neurology

• Therapies under research: A group of researchers in Germany is in-


terested in HSAN disorders and offers genetic testing on a research ba-
sis (free of charge) for individuals meeting the criteria for a diagnosis of
HSAN.

REFERENCES
1. Axelrod FB: Autonomic and Sensory Disorders. Principles and Practice of Medical
Genetics. Volume3. 5th edition. Edited by: Rimoin DL, Connor JM, Pyeritz RE,
Korf BR. Edinburgh: Churchill Livingstone; 2007:2802-2816.
2. Dyck PJ: Neuronal atrophy and degeneration predominantly affecting peripheral
sensory and autonomic neurons. Peripheral Neuropathy Volume 2. Edited by:
Dyck PJ, Thomas PK, Griffin JW, Low PA, Podulso JF. Philadelphia: W.B. Saun-
ders; 1993: 1065-1093.
3. Hilz MJ: Assessment and evaluation of hereditary sensory and autonomic neu-
ropathies with autonomic and neurophysiological examinations. Clin Auton Res.
2002, 12 (Suppl 1): I33-I43.
4. Zaenglein AL, Chang MW, Meehan SA, Axelrod FB, Orlow SJ: Extensive Riga-
Fede disease of the lip and tongue. J Am Acad Dermatol. 2002, 47 (3): 445-447.
5. Oddoux C, Wang J, Clayton CM, Hilz M, Cilio MR, Bertini E, Maayan Ch, Blu-
menfeld A, Axelrod F, Ostrer H: Genetic heterogeneity in hereditary and auto-
nomic sensory neuropathy (HSAN4) [abstract]. Society of Human Genetics. 1999.
6. Rosenberg S, Nagahashi Marie SK, Kliemann S: Congenital insensitivity to pain
with anhidrosis(hereditary sensory and autonomc neuropathy type IV). Pediatr
Neurol. 1994, 11: 50-56.
7. Goebel HH, Veit S, Dyck PJ: Confirmation of virtual unmyelinated fibre absence
in hereditary sensory neuropathy type IV. J Neuropathol Exp Neurol. 1980, 39:
670-675.
8. Hilz MJ, Stemper B, Axelrod FB: Sympathetic skin response differentiates heredi-
tary sensory autonomic neuropathies III and IV. Neurology. 1999, 52: 1652-1657.

116
Case 21
A 6-year-old boy was presented at the emergency room with asymmetric
flaccid weakness in his upper and lower extremities. He also had a
history of constipation for more than three days. Symptoms appeared during
the last 24 hours and the parents also reported an antecedent upper respiratory
tract infection with cough and low-grade fever 10 days before presentation.
The patient had an unremarkable past medical history and he achieved
developmental milestones normally.
On physical examination he was cooperative with a normal state of
consciousness. The chest was clear on auscultation and there was no heart
murmur. Neurological examination revealed sensory loss in the upper and
lower extremities with an incomplete cervical sensory level. Tendon reflexes
were reduced mainly in the lower extremities. Muscle strength was diminished
and the patient had marked difficulty standing and walking. Initially extensor
plantar responses were not elicited in both legs and sphincter tone also seemed
to be reserved. An urgent CT scan of the brain was normal and a lumbar
puncture revealed a leukocyte count of 24 cells/µL (67% lymphocytes),
84 mg/dl protein and 55 mg/dl glucose). Polymerase chain reaction of the
cerebrospinal fluid for herpes simplex virus types 1 and 2, coxsackie virus,
echovirus and M. pneumoniae were negative.

Answer the following questions:


• Give the most likely diagnosis.
• Summarise the case briefly.
• Where do you localise the lesion?
• Provide a differential diagnosis.
• Discuss an appropriate diagnostic work-up.
• Discuss the management of this patient.

117
Child Neurology

DIAGNOSIS
Acute Transverse Myelitis

SUMMARY
A 6-year-old boy presented with acute asymmetric upper and lower extremities
flaccid weakness. He had an antecedent upper respiratory tract infection with
cough and low-grade fever 10 days before. Neurological examination revealed
sensory loss in the upper and lower extremities with an incomplete cervical
sensory level. Tendon reflexes were reduced mainly in the lower extremities.
Muscle strength was diminished and the patient had marked difficulty stand-
ing and walking. Spinal Magnetic Resonance Imaging (MRI) showed a high
T2 signal and swelling extending from C4 to T5. Brain MRI and visual evoked
potentials were reported as normal.

Localisation of the Lesion


History and clinical findings (lower-extremity paralysis, anaesthesia, areflex-
ia, autonomic dysfunction) localised to the spinal cord. There is no apparent
cortical involvement. The cranial nerves and cerebellar function was normal.
Anaesthesia to sensation is characteristic of a complete transverse myelitis.
A lesion involving the descending corticospinal tracts below the level of the
lesion can explain his asymmetric lower and upper extremity weakness. His
bowel dysfunction is a result of involvement of the autonomic pathways that
also pass through this portion of the spinal cord. Absent reflexes could suggest
a lower motor neuron process. However, given the relatively acute onset and
associated symptoms, areflexia is more likely a result of an acute upper motor
neuron (spinal shock) process.

DIFFERENTIAL DIAGNOSIS
• Acute Idiopathic Transverse Myelitis: Inflammatory transverse my-
elitis (CSF inflammation with usual pleocytosis and occasionally elevated
IgG index/OCBs) in the absence of a specific cause (such as MS, NMO,
ADEM, connective tissue disease, etc.) is the most common cause of
acute myelitis. However, the idiopathic TM is a diagnosis of exclusion.
A preceding nonspecific fever, nausea, or muscle pain, possibly indicating
a prior viral infection, is common, although one or more of these symp-
toms may also precede attacks of MS and NMO. The lesion length varies
from less than one segment to the entire cord. Many of these large series
of patients were reported before NMO-IgG was identified, and it is pos-
sible that many such patients may have an NMO spectrum disorder. The
proportion of ‘‘idiopathic’’ inflammatory transverse myelitis is likely to
decline with the increasing availability of newer autoimmune markers,
imaging techniques, and microbiological tests capable of defining a spe-
cific etiology.

118
Case 21

• Guillain-Barré syndrome: Guillain-Barré syndrome can present simi-


larly to acute transverse myelitis, with depressed reflexes, weakness, bowel
and bladder dysfunction, and autonomic dysregulation. Differentiating
factors of Guillain-Barré syndrome include lack of a prominent sensory
component and/or the presence of cranial neuropathies (excluding the
optic nerve). Magnetic resonance imaging can also help to distinguish
the conditions by demonstrating enhancement of spinal nerve roots in
Guillain-Barré syndrome and the absence of an intramedullary disease.
While both acute transverse myelitis and Guillain-Barré syndrome may
have elevation of cerebrospinal fluid protein, Guillain-Barré syndrome
will not demonstrate pleocytosis as is often seen in acute transverse myeli-
tis. In cases that remain unclear, nerve conduction studies can be of use to
demonstrate an acquired neuropathy.
• Compressive myelopathy: Neurological symptoms caused by ex-
tramedullary mass effect on the spinal cord can be the first signs of the
underlying pathology. Depending on the underlying etiology, additional
constitutional symptoms can include back pain and fever. Traumatic in-
jury can result in vertebral body compression, intervertebral disk her-
niation, and epidural haematoma. Compressive extramedullary tumours
presenting in childhood include Ewing sarcoma, neuroblastoma, granu-
locytic sarcoma, T cell and malignant lymphoma, and Hodgkin disease.
In addition, spinal cord abscesses may result in symptoms of compressive
myelopathy.
• Acute Disseminated Encephalomyelitis (ADEM): Acute
Disseminated Encephalomyelitis (ADEM) is a monophasic disorder that
affects the brain and occasionally the spinal cord. Often there is a history
of preceding viral or other infectious illness. The brain and spinal cord
show demyelinating lesions that are generally of the same age. ADEM
may evolve over the course of up to 3 months. ADEM is more common in
children, and is only reliably diagnosed in individuals who have concomi-
tant encephalopathy. Follow-up of individuals with a clinical diagnosis of
ADEM reveals that 25% of cases eventually meet clinical criteria for MS.
• Post-vaccine myelitis: An acute transverse myelitis occurring in the 3
weeks following a vaccination has been linked to an immunological reac-
tion to the vaccine, such as smallpox or rabies. In recent years vaccines
such as hepatitis B, typhoid, influenza, rubella, and tetanus have been
implicated, but a causal relationship has not been established. Such cases
may reflect chance occurrences of idiopathic transverse myelitis in pa-
tients who incidentally have had a vaccination.
• Ischaemic myelopathy: Ischaemia of the spinal cord is uncommon
in children. Anterior spinal artery infarction presents with motor, auto-
nomic, and spinothalamic-related sensory deficits localised to the anterior
two thirds of the spinal cord, with sparing of vibration and propriocep-
tion. Anterior spinal artery infarction is reported after both mild and ma-
jor trauma, aortic dissection, anterior spinal artery dissection, embolism,

119
Child Neurology

and primary thrombus. The most sensitive imaging results are diffusion-
weighted sequences demonstrating restricted diffusion in the distribution
of a T2 hyperintense lesion. However, patterns of ischaemia on spinal MRI
are more difficult to detect and interpret, especially in children, secondary
to the smaller diameter of the cord. Specific MRI modalities can improve
visualisation at 7 to 10 days. Repeat imaging on day 2 may demonstrate le-
sions in a previously normal cord. Absence of cerebrospinal fluid, pleocy-
tosis and normal protein are more typical with infarct and may distinguish
this from an inflammatory myelopathy.
° Fibrocartilaginous Embolism: Fibrocartilaginous embolism causes
the same symptoms as anterior spinal artery infarction and can occur
after mild trauma or straining in children. The intervertebral disc’s
nucleus pulposus material embolises and obstructs arterial flow to the
associated spinal cord. Imaging may demonstrate ischaemic changes
in the anterior spinal cord with unusual T2 isointensity and narrow-
ing of the associated disc(s), which are normally hyperintense on T2-
weighted images.
• Spinal arteriovenous malformation: Arteriovenous malformation or
fistula of the spine classically presents with fluctuating motor, sensory,
and/or autonomic symptoms related to a vascular steal phenomenon.
A vascular bruit can sometimes be auscultated over the back. The flow
voids, which may or may not be visualised on MRI, represent engorged
veins. Increased T2 signal and cord swelling can mimic acute transverse
myelitis. Spinal angiography is the modality of choice but, in small chil-
dren and infants, is often not possible secondary to the very small vessel
caliber. Time of flight and phase contrast magnetic resonance angiography
can prove useful.
• Infectious myelitis: Presentation and MRI findings of direct infection
of the spinal cord can be similar to acute transverse myelitis, with cord
swelling, focal T2 hyperintensity, and enhancement. Cerebrospinal fluid
protein and cell count are classically markedly elevated, typically with pro-
tein elevation above 100 to 500 mg/dL, cell count greater than 50 cells/ml,
and decreased glucose. Exceptions to these findings are numerous, and
acute transverse myelitis can have similar white cell count and protein
findings. Fever can occur as part of autonomic dysfunction in acute trans-
verse myelitis, further confusing the picture. A variety of pathogens have
been reported to include human herpes virus-1 and-6; varicella zoster
virus; Epstein-Barr virus; cytomegalovirus; influenza viruses; enterovi-
ruses; hepatitis A, B, and C; and bacteria such as Mycoplasma pneumonia,
Bartonella henselae, and Borrelia burgdorferi.
• Disease-associated acute transverse myelitis: While idiopathic acute
transverse myelitis is a diagnosis of exclusion, it accounts for 89% of cases
in pediatric studies, compared to 36% of adult cases. The purpose of in-
vestigating the presence of disease-associated acute transverse myelitis is
primarily to identify those at risk for recurrence and to initiate appropriate

120
Case 21

treatment and surveillance to improve outcomes. Diseases associated with


pediatric acute transverse myelitis include multiple sclerosis, acute dis-
seminated encephalomyelitis, neuromyelitis optica, and rheumatological
conditions such as systemic lupus erythematosus and anti-phospholipid
anti-body syndrome. The results of brain and spinal cord MRI as well as
serum and cerebrospinal fluid studies are useful in such diagnoses.
• Intramedullary tumour: Intramedullary tumours, such as gliomas,
can present similarly to acute transverse myelitis. Astrocytomas, the most
common childhood primary cord neoplasm, typically demonstrate an
enhancing infiltrating mass with fusiform expansion of the cord, usually
extending less than 4 vertebral segments. A syrinx above and below the
lesion is often present secondary to obstruction of the central canal by
the tumour. Sometimes, hyperintense proteinaceous cerebrospinal fluid
(Froin’s syndrome) may be present below the mass, occasionally associ-
ated with haemorrhage. Pilocytic astrocytomas may have a cyst adjacent to
the tumour. Ependymomas are well circumscribed with cord expansion.
These tumours may be cystic or be associated with a syrinx, and frequent-
ly haemorrhage. Occasionally, it can be difficult to distinguish acute trans-
verse myelitis from an intramedullary mass in the setting of diffuse cord
swelling with increased T2 signal, gadolinium enhancement, and elevated
protein. Inflammatory markers such as cerebrospinal fluid pleocytosis can
help to distinguish the two. However, patients with large expansile lesions
associated with cysts or those who do not improve with standard therapy
for acute transverse myelitis or continue to worsen over days should be
reimaged. Biopsy should be considered if there is progressive disease
• Previous spinal radiation: With corresponding clinical history, pos-
tradiation myelopathy may be considered and range from acute transient
myelitis, to chronic progressive necrotic myelitis with severe paralysis.
Latent period can be 3 months to 6 years. Treatments include steroids and
hyperbaric oxygen.

DIAGNOSTIC APPROACH
• This boy presented with an acute spinal cord process that is a neurologic
emergency. He first requires an urgent MRI of the entire spine with and
without gadolinium. The MRI of the spine first helps to exclude extramed-
ullary processes that might require immediate surgical intervention.
• Longitudinally extensive transverse myelitis is defined as acute transverse
myelitis involving 3 or more consecutive vertebral levels and is clinically
most often associated with acute complete transverse myelitis. Similar to
adults, longitudinally extensive transverse myelitis is common in paedi-
atric idiopathic acute transverse myelitis. In addition, longitudinally ex-
tensive transverse myelitis is more typically seen in children with acute
disseminated encephalomyelitis associated transverse myelitis and in
neuromyelitis optica. Multiple sclerosis is more typically associated with
segmental transverse myelitis occupying fewer than 3 vertebral segments

121
Child Neurology

• Once the diagnosis of transverse myelitis is confirmed, a brain MRI with


and without gadolinium should be obtained to investigate possible in-
volvement of the cerebral white matter that can occur with a demyelinat-
ing disease such as ADEM or multiple sclerosis. Also, the optic nerves
should be carefully evaluated with contrast in the event that the patient
has Devic’s disease. Depending on other clinical and lab findings, optic
nerve involvement also may support a diagnosis of multiple sclerosis. In
addition, leukodystrophies may demonstrate characteristic findings on
brain MRI that might suggest a specific underlying diagnosis.
• Once MRI has ruled out a structural cause and the diagnosis is more clear,
a lumbar puncture should be performed to help distinguish inflamma-
tory from non-inflammatory processes. CSF should be sent for cell count,
glucose, protein, myelin basic protein, oligoclonal bands, IgG synthesis,
and cytology. CSF findings in ATM are abnormal in more than 60% of pa-
tients and often show a mild lymphocytic pleocytosis, elevated total pro-
tein, and myelin basic protein with occasional oligoclonal bands. If there
is concern for an infectious or post-infectious process, extra CSF can be
obtained to evaluate for infections such as Lyme disease, Epstein-Barr vi-
rus, or Mycoplasma pneumoniae, by using anti-body titers or PCR testing
for specific pathogens.
• Serum studies are also typically done to exclude other possible diagno-
sis. Initial screening labs might include CBC, basic chemistry panel, liver
function tests, erythrocyte sedimentation rate, ANA, vitamin B12 level,
and thyroid function tests. If Devic’s disease is suspected, serum testing
for the presence of the Neuromyelitis Optica (NMO) anti-body can be
clinically performed. It is positive in as many as 70% of cases and helps
to distinguish Devic’s disease from multiple sclerosis. If an autoimmune
vasculitis is suspected, testing with angiotensin-converting enzyme, anti-
double-stranded DNA anti-body, SS-A (Ro), SS-B (La), lupus anti-co-
agulant, complement levels, and beta glycoprotein 1 can be done. Under
certain circumstances, additional testing might include HIV, HTLV-1, and
very-long-chain fatty acids (adrenomyeloneuropathy). Serum anti-body
titers might also be done to evaluate for the infections mentioned above
• Electro-diagnostic studies (SSEPs, VERs, BAERs) may have value in de-
termining the extent of neural injury and prognosis for recovery. In par-
ticular, VERs may detect subtle optic nerve involvement, consistent with
several demyelinating diseases.

MANAGEMENT
• The initial treatment of patients suspected to have spinal cord lesions in-
cludes evaluation of airway, breathing, and circulation. A history of trauma
requires initial immobilisation of the spinal cord until imaging studies and
neurological evaluations rule out trauma-related myelopathy. Attendance
to acute urinary retention should be managed with catheterisation.

122
Case 21

• Class I evidence for treatment in children does not exist. However, re-
cently published guidelines following typical practice suggest first-line
treatment of noninfectious immune mediated acute transverse myelitis
to be intravenous methylprednisolone 30 mg/kg/d for 5 to 7 days, with a
maximum dose of 1 g/d. Use of high-dose corticosteroids can reduce the
length of disability and improve outcomes. An oral corticosteroid taper
starting at 1 mg/kg patient weight per day over 3 to 4 weeks should follow
intravenous steroid treatment. If clinical improvement does not begin or
symptoms are worsening within 24 to 48 hours of beginning corticoster-
oid treatment, consideration should be given for initiation of plasma ex-
change therapy, especially for longitudinally extensive transverse myelitis
• Limited evidence exists for additional therapies but can include intrave-
nous immunoglobulin and cyclophosphamide. Intravenous immuno-
globulin is typically dosed at 2 g/kg divided over 2 to 5 days.
• Cyclophosphamide 500 to 750 mg/m2 once may be another alternative.
Patients with acute transverse myelitis and having brain MRI consistent
with multiple sclerosis and meeting the 2010 McDonald multiple sclero-
sis criteria can be considered for initiation of typical immunomodulatory
agents after the acute period. Those patients meeting diagnostic criteria
for neuromyelitis optica should likewise be started on prophylactic disease
treatment.
• Rehabilitation should begin as soon as possible through consultation
with physical medicine and rehabilitation specialists as well as physical
and occupational therapy. Typical goals include maintaining range of mo-
tion, strengthening, prescribing adaptive equipment to include orthotics,
and bowel and bladder continence programs. The latter can addition-
sultation with urology specialists. Patients continuing to
ally require consultation
demonstrate recovery from severe deficits after completion of their acute
treatment with intravenous steroids and plasmapheresis can benefit from
a more intensive inpatient rehabilitation program as directed by physi-
cal medicine consultants. Depending on the amount of recovery, ongoing
life-long follow-up with physical medicine and rehabilitation specialists
may be needed.

123
Child Neurology

REFERENCES
1. Kaplin AI, Krishnan C, Deshpande DM, et al. Diagnosis and management of acute
myelopathies. Neurologist 2005; 11:2.
2. Baxter R, Lewis E, Goddard K, et al. Acute Demyelinating Events Following Vac-
cines: A Case-Centered Analysis. Clin Infect Dis 2016; 63:1456.
3. Jeffery DR, Mandler RN, Davis LE. Transverse myelitis. Retrospective analysis of
33 cases, with differentiation of cases associated with multiple sclerosis and parain-
fectious events. Arch Neurol 1993; 50:532.
4. de Seze J, Lanctin C, Lebrun C, et al.. Idiopathic acute transverse myelitis: applica-
tion of the recent diagnostic criteria. Neurology 2005; 65:1950.
5. Beh SC, Greenberg BM, Frohman T, Frohman EM. Transverse myelitis. Neurol
Clin 2013; 31:79.
6. Berman M, Feldman S, Alter M, et al.. Acute transverse myelitis: incidence and
etiologic considerations. Neurology 1981; 31:966.
7. Scott TF, Frohman EM, De Seze J, et al.. Evidence-based guideline: clinical evalua-
tion and treatment of transverse myelitis: report of the Therapeutics and Technol-
ogy Assessment Subcommittee of the American Academy of Neurology. Neurol-
ogy 2011; 77:2128.
8. Pidcock FS, Krishnan C, Crawford TO, et al.. Acute transverse myelitis in child-
hood: centre-based analysis of 47 cases. Neurology 2007; 68:1474.
9. Transverse Myelitis Consortium Working Group. Proposed diagnostic criteria and
nosology of acute transverse myelitis. Neurology 2002; 59:499.
10. de Seze J, Stojkovic T, Breteau G, et al.. Acute myelopathies: Clinical, laboratory
and outcome profiles in 79 cases. Brain 2001; 124:1509.
11. Krishnan C, Kaplin AI, Calabresi P, Kerr DA. Clinical characteristics and prognos-
tic factors in 170 patients with idiopathic transverse myelitis. Neurology 2004; 62
(Suppl 5):A231.
12. Wolf VL, Lupo PJ, Lotze TE. Pediatric acute transverse myelitis overview and dif-
ferential diagnosis. J Child Neurol 2012; 27:1426.
13. Bigi S, Banwell B, Yeh EA. Outcomes after early administration of plasma ex-
change in pediatric central nervous system inflammatory demyelination. J Child
Neurol 2015; 30:874.
14. Greenberg BM. Treatment of acute transverse myelitis and its early complications.
Continuum (Minneap Minn) 2011; 17:733.
15. Defresne P, Hollenberg H, Husson B, et al. Acute transverse myelitis in children:
clinical course and prognostic factors. J Child Neurol 2003; 18:401.

124
Case 22
A 7-year-old boy came to the Outpatient Department (OPD) with
complaints of fever, recurrent respiratory infections, and increasing
redness of the eyes.
His 5 years’ past history was significant of recurrent diarrhoea since the age
of 9 months, meningitis at the age of 3 years, redness of eyes since the age of
2 years, recurrent ear discharge and recurrent chest infections since the age
of 4 years. Patient had been treated symptomatically for all these recurrent
infections. He was a product of a consanguineous couple, a full term normal
vaginally delivered baby with normal developmental milestones till the age of
9 months.
On physical examination, his vitals were stable. He had dilated blood vessels
on sclera, but no cutaneous involvement. His height and weight were less
than the 50th percentile of normal. His developmental milestones were mildly
delayed. Central Nervous System (CNS) examination showed a conscious,
oriented, and cooperative child but there was significant dysmetria on finger to
nose testing bilaterally. He could speak full sentences but with little dysarthria.
His face was symmetric and his tongue was midline without fasciculations.
He had normal tone; bulk with 5/5-muscle strength throughout and his deep
tendon reflexes were present. He had marked truncal and appendicular ataxia
while sitting and walking. He also had a wide based gait. Rest of the CNS
examination was unremarkable. His fundus examination was also normal.
His serum immunoglobulin A (IgA) was markedly reduced to 3 mg/dl
(normal-35-250 mg/dl), IgE and IgG (G2 and G4) levels were also reduced,
however tissue transglutaminase IgA anti-body was normal, this ruled out
the celiac disease. Serum alfa-fetoprotein (AFP) was elevated to 65 IU/ml
(normal-<5.8 IU/ml), and serum lactate dehydrogenase (LDH) was elevated
to 960 U/L (normal-440 U/L). MRI brain revealed cerebellar atrophy.

Answer the following questions:


• Give the most likely diagnosis.
• Briefly summarise the case.
• Discuss the differential diagnosis.
• Discuss the diagnostic approach.
• Discuss the management of this case.

125
Child Neurology

DIAGNOSIS
Ataxia Telangiectasia

SUMMARY
This patient, a seven-year-old boy presented with a history of recurrent res-
piratory infections with delayed developmental milestones, dysarthric speech
and chronic progressive ataxia. His neurological examination is remarkable for
dysarthria, dysmetria, and ataxic gait. Bulbar telangiectasias were present on
eye examination making a diagnosis of ataxia telangiectasia highly likely.

DIFFERENTIAL DIAGNOSIS
Ataxia telangiectasia: Ataxia-Telangiectasia (A-T) is an autosomal recessive,
complex multisystem disorder characterised by progressive neurologic im-
pairment, cerebellar ataxia, variable immunodeficiency with susceptibility to
sinopulmonary infections, impaired organ maturation, X-ray hypersensitivity,
ocular and cutaneous telangiectasia, and a predisposition to malignancy. The
responsible gene (ATM gene) has been mapped to band 11q22-23.
There are several other disorders with similar symptoms or laboratory features
that physicians may consider when diagnosing A-T. The three most common
disorders that are sometimes confused with A-T are:
• Ataxic cerebral palsy: Ataxia is a less common type of cerebral palsy,
occurring between 5% and 10% of all cases. Some of these individuals
have hypotonia and tremors. Motor skills such as writing, typing, or using
scissors might be affected, as well as balance; especially while walking. It
is common for individuals to have difficulty with visual and/or auditory
processing. They usually have an awkward gait with some dysarthria.

Inherited Ataxias
• Friedreich Ataxia (FA) is the most common genetic cause of ataxia in
children. Most often the ataxia appears between 10 and 15 years of age, and
differs from A-T by the absence of telangiectasia and oculomotor apraxia,
a normal alpha-fetoprotein, and the frequent presence of scoliosis, absent
tendon reflexes, and abnormal features on the EKG. Individuals with FA
manifest difficulty standing in one place that is much enhanced by closure
of the eyes (Romberg sign) that is not so apparent in those with A-T–even
though those with A-T may have greater difficulty standing in one place
with their eyes open.
• Progressive hereditary ataxias (Spinocerebellar degenerations): A
genotypic classification of the dominantly inherited ataxias has replaced
the phenotypic classification. The clinical features overlap and genetic
testing is the best method for diagnosis. Treatment is symptomatic and,
depending on the disease, may include anti-convulsants, muscle relaxants,
and assistive devices.

126
Case 22

• Ataxia with occulomotor apaxia type 2: This disorder may be mis-


taken for ataxia-telangiectasia, but it is a different genetic disorder. The
onset of symptoms is between ages 3 and 30 years. The features include
ataxia caused by cerebellar atrophy, an axonal sensorimotor neuropathy,
and oculomotor apraxia. The serum concentration of alpha-fetoprotein is
increased. Molecular genetics testing is clinically available to confirm the
diagnosis.
• Abetalipoproteinaemia (Ataxia with Vit E deficiency): Other terms
for the disorder are acanthocytosis and Bassen-Kornzweig syndrome. The
clinical features are fat malabsorption and a progressive deficiency of vita-
mins A, E, and K. Fat malabsorption is present from birth, and most new-
borns come to medical attention because of failure to thrive, vomiting,
and large volumes of loose stools. A cerebellar ataxia develops in one-third
of children during the first decade and in almost every child by the end
of the second decade. Tendon reflexes are usually lost by 5 years of age.
Proprioceptive sensation in the hands and feet is lost, whereas pinprick
and temperature sensations are less severely affected. Sensory loss results
from demyelination in the posterior columns of the spinal cord and the
peripheral nerves. Retinitis pigmentosa is an almost constant feature.
The age at onset varies, but is usually during the first decade. The initial
symptom is night blindness. Nystagmus is common, and its cause may
be either the cerebellar disturbance or the loss of central vision. Chronic
vitamin E deficiency is the cause of the neurological complications of
abetalipoproteinaemia.

Congenital Structural Malformations


• Arnold chiari malformations: Chiari malformations, types I-IV, refer
to a spectrum of congenital hindbrain abnormalities affecting the struc-
tural relationships between the cerebellum, brainstem, the upper cervi-
cal cord, and the bony cranial base. The signs and symptoms of Chiari
malformation can vary greatly from one person to another. Some indi-
viduals may not have any symptoms (asymptomatic) upon diagnosis as an
incidental finding; others may have serious manifestations such as neuro-
logical deficits. Abnormalities affecting the eyes can also affect individuals
with a Chiari malformation including double vision (diplopia), abnormal
sensitivity to light (photophobia), blurred vision, involuntary eye move-
ments (nystagmus) and pain behind the eyes. Vertigo, dizziness, ringing
in the ears (tinnitus) and bilateral hearing impairment can also develop.
Other symptoms associated with a Chiari malformation may include poor
coordination and balance problems, muscle weakness, difficulties swal-
lowing (dysphagia) or speaking (dysarthria), palpitations, fainting episodes
(syncope) and tingling or burning sensations in the fingers, toes or lips
(paresthesia).
• Joubert syndrome: The signs and symptoms of this condition vary
among affected individuals, even among members of the same family.
The hallmark feature of Joubert syndrome is a structural abnormality of

127
Child Neurology

brain involving under development of the cerebellar vermis which appear


as molar tooth sign, seen on brain Magnetic Resonance Imaging (MRI).
Most infants with Joubert syndrome have weak muscle tone (hypotonia)
in infancy, which evolves into difficulty coordinating movements (ataxia)
in early childhood. Other characteristic features of the condition include
episodes of unusually fast or slow breathing in infancy and abnormal eye
movements. Most affected individuals have delayed development and in-
tellectual disability, which range from mild to severe.
• Dandy-Walker malformation: Dandy-Walker malformation affects
brain development, primarily development of the cerebellum. In the
majority of individuals with Dandy-Walker malformation, signs and
symptoms caused by abnormal brain development are present at birth or
develop within the first year of life. Some children have hydrocephalus
that may cause macrocephaly. Children often have delayed development,
particularly a delay in motor skills such as crawling, walking, and coordi-
nating movements. In 10 to 20 percent of people with Dandy-Walker mal-
formation, signs and symptoms of the condition do not appear until late
childhood or into adulthood. These individuals typically have a different
range of features than those affected in infancy, including headaches, an
unsteady walking gait, facial palsy, increased muscle tone, muscle spasms,
mental and behavioural changes.

Inborn Error of Metabolism


• Hartnup disease: Hartnup disease is a rare metabolic disorder belong-
ing to the neutral amino acidurias and characterised by abnormal renal
and gastrointestinal transport of neutral amino acids (tryptophan, alanine,
asparagine, glutamine, histidine, isoleucine, leucine, phenylalanine, ser-
ine, threonine, tyrosine and valine). In few symptomatic subjects, clini-
cal symptoms usually appear in childhood (3-9 years of age), but some-
times manifest as early as 10 days after birth, or as late as early adulthood.
Symptomatic subjects usually present with skin photosensitivity (pellagra-
like skin eruption), neurological symptoms (intermittent cerebellar ataxia,
spasticity, delayed motor development, trembling, headaches, and hypoto-
nia), and psychiatric symptoms (anxiety, emotional instability, delusions,
and hallucinations). Ocular manifestations may occur (double vision,
nystagmus, photophobia, and strabismus). Intellectual deficit and short
stature have been described in a few patients. Exacerbations are seen most
frequently in the spring or early summer after sunlight exposure. Fever,
drugs, and emotional or physical stress may also trigger symptoms.
• Refsum disease: This autosomal recessive disorder is associated with
impaired oxidation of phytanic acid. Elevated phytanic acid levels in the
nervous system are associated with neurotoxicity. Clinical features in-
clude; Onset in the second to third decade of life with Cerebellar ataxia,
early presentation may be of night blindness, pigmentary degeneration of
the retina, polyneuropathy with elevated CSF protein, sensorineural deaf-
ness, skin (ichthyosis) and cardiac arrhythmia.

128
Case 22

• Leukoencephalopathy with vanishing white matter (Van der


Knapp syndrome): Leukoencephalopathy with vanishing white matter
(VWM) has an autosomal recessive inheritance with an age-dependent
penetrance. Clinical features include prominently cerebellar ataxia and
spasticity, chronic progressive neurologic deterioration and episodic exac-
erbation follow in late infancy or early childhood; episodes of deteriora-
tion occur after minor infection and head trauma, leading to periods of
lethargy or coma, cognitive ability may show decline but is relatively pre-
served compared to the severity of motor deficit, initial motor and mental
development is normal or mildly delayed, optic atrophy and epilepsy may
be additional features.
• 4H syndrome: 4H syndrome is a recessively inherited phenotype with
distinctive clinical features and a hypomyelinating leukodystrophy. To
date, no gene locus or mutations have been identified. Clinical features
include early onset progressive ataxia, Short stature, hypodontia and de-
layed puberty, secondary to gonadal dysfunction.
• Biotinidase deficiency: Because of the lack of free biotin, biotinidase
deficiency results in dysfunction of 3 mitochondrial carboxylases. It is re-
cessively inherited, and the underlying defect involves mutations of BTD
gene where the locus is at 3p25. Clinical features include; delayed presen-
tation (second year of life), Intermittent ataxia, sensorineural hearing loss,
myoclonic seizures, developmental delay, skin rashes, and alopecia.
• L-2-hydroxyglutaricaciduria: This autosomal recessively inherited de-
fect is characterised by excessive excretion of L-2-hydroxyglutaric acid in
the urine. The precise molecular basis is not well established. The clinical
course is of slowly progressive neurodegenerative disorder. Clinical fea-
tures include age of onset from 6-20 years, presence of cognitive delay and
epileptic seizures, progressive ataxia, dysarthria, extrapyramidal dysfunc-
tion, short stature and macrocrania.
• Late-onset sphingolipidoses: These complex biochemical defects are re-
lated to specific deficiencies of lysosomal enzymes. The presentation is a
combination of cognitive deterioration, seizures, and gait abnormalities due
to a combination of pyramidal features (spasticity), cerebellar dysfunction
(ataxia), extrapyramidal features (e.g., dystonia), choreoathetosis, and oph-
thalmologic abnormalities. Ataxia almost never is the sole clinical symptom.
Other systemic features can include coarse facies, organomegaly, and dysos-
tosis multiplex. Definitive diagnosis can be established by lysosomal enzyme
assay in leukocytes or cultured skin fibroblasts.
• Congenital disorders of glycosylation: The Congenital Disorders
of Glycosylation (CDG) represent a new class of disorders that result
from abnormalities of carbohydrate-deficient glycoproteins, particularly
transferrin. Glycoproteins are important constituents of the developing
brain; CNS involvement and multisystem manifestations are frequent.
Clinical features include; stage of ataxia; mental deficiency during infancy.
Childhood stage includes delayed development, failure to thrive, and hy-
potonia. Dysmorphic facial features, including prominent ears and nose,

129
Child Neurology

fat pads over buttocks, abnormal patches of skin over thighs (orange peel
skin), and inverted nipples (considered as a characteristic clinical feature).

Ataxias Associated with Mitochondrial Cytopathies


• Neuropathy,
athy, ataxia, retinitis pigmentosa, and peripheral neuropa-
thy syndrome: Neuropathy, Ataxia, Retinitis pigmentosa, and Peripheral
neuropathy (NARP) syndrome is a mitochondrial disorder that displays
maternal inheritance. Affected individuals present with features of cerebel-
lar ataxia, seizures, cognitive impairment, and peripheral neuropathy. The
diagnosis can be confirmed by mitochondrial DNA mutation analysis.
• Leigh disease: This disorder has distinct neuropathologic findings, highly
variable clinical presentation, and can be caused by multiple biochemical
and molecular genetic defects. Autosomal recessive inheritance and mater-
nal inheritance (mutations in mitochondrial DNA) patterns exist. Clinical
features include manifestations due to multifocal lesions in the brainstem,
thalamus, and cerebellum; the most important of these are as follows:
Oculomotor-nuclear or supranuclear ophthalmoplegia, central nystagmus
with rotary and horizontal components. Respiratory problems are charac-
terised by unexplained hyperventilation, apnoea, and irregular respiration
(air hunger); and neurologic issues include truncal ataxia, incoordination,
and intention tremors are evident as the child begins to walk.
• Coenzyme-Q10–associated ataxia: CoQ-10 is involved in facilitation
of electron transfer between the various dehydrogenases and cytochromes
participating in the respiratory chain and oxidative phosphorylation reac-
tion. Clinical features include; a slowly progressive ataxia in childhood,
associated with cerebellar atrophy. Associated features include develop-
mental delay, mental retardation, and seizures.
• Brain tumours are always an initial concern when progressive ataxia devel-
ops in previously normal children, especially if headache is present as well.
Approximately 85% of primary brain tumours in children between 2 to 12
years of age are located in the posterior fossa. The four major tumours of
the posterior fossa are cerebellar astrocytoma, brainstem glioma, ependymo-
ma, and primitive neuroectodermal tumour (medulloblastoma). Although
this discussion is limited to tumours of the posterior fossa, it is impor-
tant to remember that supratentorial brain tumours may also cause ataxia.
Approximately one-fourth of children with supratentorial brain tumours
have gait disturbances at the time of their first hospitalisation, and many
show signs of cerebellar dysfunction. Gait disturbances occur with equal
frequency whether supratentorial tumours are in the midline or the hemi-
spheres, whereas cerebellar signs are more common with midline tumors.

DIAGNOSTIC APPROACH
• An initial diagnosis of ataxia-telangiectasia is normally made on the ba-
sis of clinical evidence. Neurological symptoms, particularly progressive
cerebellar ataxia. This is normally apparent from an early age, often when

130
Case 22

children first start to sit or stand and is often manifested in a wobbly gait
or swaying while sitting or standing. Although the age when they start to
walk is normal, they remain unsteady, as if they had only just achieved this
milestone. Unlike ataxia due to cerebral palsy, the problems will worsen
over time with chorea, dystonia, and tremor.
• Abnormalities of the eye movements. Movements normally become re-
stricted and following objects becomes difficult.
• Telangiectasia in the whites of the eyes or other areas of the face and ears,
though this may not occur until four or five years of age.
• Recurrent sinopulmonary infections, although these only affect around
50% of children with A-T.
• Serum concentration of Alpha-Fetoprotein (AFP). Serum AFP con-
centration is elevated above 10 ng/ml in more than 95% of individuals
with A-T. Note: Serum AFP concentration may remain above normal in
unaffected children until age 24 months.
• Immunoblotting for ATM protein. Intracellular ATM protein is se-
verely depleted in most patients with A-T. To date, this is the most sen-
sitive and specific clinical test for establishing a diagnosis of A-T. Small
amounts of ATM protein have been occasionally associated with a milder
prognosis, although there are many exceptions to this and the association
needs further validation.
Of individuals with A-T:
• About 90% have no detectable ATM protein
• About 10% have trace amounts of ATM protein
• About 1% have a normal amount of ATM protein that lacks ATM serine/
threonine kinase activity (so-called “kinase-dead”)
• Chromosome some analysis. A 7;14 chromosome translocation is identified in
5%-15% of cells in routine chromosomal studies of peripheral blood of
individuals with A-T. The break points are commonly at 14q11 (the T-cell
receptor-alpha locus) and at 14q32 (the B-cell receptor [IGH] locus).
ATM is the only gene known to be associated with ataxia-telangiectasia.
More than 99% of individuals with classic A-T have pathogenic variants
in ATM.
• In patients, chronic ataxia whose diagnosis is less clear at presentation, a
screening work-up should begin with a brain MRI and MR spectroscopy.
Neuroimaging rules out structural abnormalities and may point to a spe-
cific category of disease.
• Additional testing screens for other organ involvement, as well as metabol-
ic and genetic abnormalities, might include a basic chemistry panel, CBC,
liver function tests, urine heavy metal screen, thyroid function tests, urine
organic acids, plasma amino acids, lactate/pyruvate battery (mitochondrial
disorders), mitochondrial DNA studies, high-resolution karyotype, vi-
tamin E level, vitamin B12 level, phytanic acid (Refsum disease), choles-
terol/triglycerides (abetalipoproteinaemia), transferrin isoelectric focusing

131
Child Neurology

(congenital disorders of glycosylation), lysosomal enzymes, and very-long-


chain fatty acids (peroxisomal disorders).
• EMG and NCS can also be useful in assessing the presence and/or extent
of peripheral neuropathy.
• In addition, a baseline ophthalmologic examination is helpful in the evalu-
ation of a patient with a suspected genetic or metabolic disease. Findings
such as optic atrophy or retinitis pigmentosa may provide clues to a
diagnosis.

MANAGEMENT
• There is no cure for any of the problems associated with A-T. Treatment
is supportive but should be proactive. There are many parts of this dis-
ease, and a team approach, including the patient and family, primary care
provider, immunologist, pulmonologist and neurologist is essential. A
nutritionist as well as physical, occupational and speech therapists, will
have important contributions to offer for specific problems that patients
encounter. There is neither a cure for A-T nor is there a specific therapy
for the neurological problems associated with the disease.
• Special attention needs to be paid to nutritional status as eating becomes
more challenging. Nasogastric and subsequent gastrostomy tube feedings
are inevitable. Input from a gastroenterologist and nutritionist is typical-
ly required. Dysphagia and tremor can make meals last a long time and
be very fatiguing, can interfere with nutritional intake, and can lead to
aspiration. Once recognised, early placement of a gastrostomy tube can
provide supplemental nutrition that allows growth, improves stamina and
decreases the risk of lung damage from aspiration.
• Treatment of manifestations: IVIG replacement therapy for individu-
als with frequent and severe infections and low IgG levels; aggressive pul-
monary hygiene for those with chronic bronchiectasis; supportive therapy
for drooling, choreoathetosis, and ataxia. Patients with A-T who do not
need immunoglobulin should receive all standard immunisations, as well
as yearly influenza vaccine and a pneumococcal (pneumonia) vaccine eve-
ry five to ten years.
• Prevention of secondary complications: Early and continued physical
therapy to minimise contractures and scoliosis.
• Surveillance: Monitoring for early signs of malignancy (e.g., weight
loss, bruising, localised pain or swelling). Monitoring of immune status
in those with severe recurrent infections or undergoing immunomodula-
tory therapy.
• Agents/circumstances to avoid: Excessive ionising radiation including
those from diagnostic procedures such as repeated or high-resolution CT
scans; attention to risks associated with general anaesthesia.
• Any child presenting with A-T should be evaluated by a geneticist to aid in
counseling with the family.

132
Case 22

REFERENCES
1. Woods CG, Taylor AM. Ataxia telangiectasia in the British Isles: the clinical and
laboratory features of 70 affected individuals. Q J Med 1992; 82:169.
2. Nissenkorn A, Levy-Shraga Y, Banet-Levi Y, et al. Endocrine abnormalities in ataxia
telangiectasia: findings from a national cohort. Pediatr Res 2016; 79:889.
3. Moin M, Aghamohammadi A, Kouhi A, et al.. Ataxia-telangiectasia in Iran: clinical
and laboratory features of 104 patients. Pediatr Neurol 2007; 37:21.
4. Stewart E, Prayle AP, Tooke A, et al.. Growth and nutrition in children with ataxia
telangiectasia. Arch Dis Child 2016; 101:1137.
5. Crawford TO. Ataxia telangiectasia. Semin Pediatr Neurol 1998; 5:287.
6. Perlman SL, Boder Deceased E, Sedgewick RP, Gatti RA. Ataxia-telangiectasia.
Handb Clin Neurol 2012; 103:307.
7. Vinck A, Verhagen MM, Gerven Mv, et al.. Cognitive and speech-language perfor-
mance in children with ataxia telangiectasia. Dev Neurorehabil 2011; 14:315.
8. McGrath-Morrow SA, Gower WA, Rothblum-Oviatt C, et al. Evaluation and
management of pulmonary disease in ataxia-telangiectasia. Pediatr Pulmonol 2010;
45:847.
9. Soresina A, Meini A, Lougaris V, Cattaneo G, Pellegrino S, Piane M, et al. Different
clinical and immunological presentation of ataxia-telangiectasia within the same
family. Neuropediatrics.. 2008 Feb. 39(1):43-5.
10. Azarsiz E, Karaca NE, Gunaydin NC, Gulez N, Ozturk C, Aksu G, et al. Do elevat-
ed serum IgM levels have to be included in probable diagnosis criteria of patients
with ataxia-telangiectasia?. Int J Immunopathol Pharmacol.
Pharmacol. 2014 Jul-Sep. 27(3):421-
427.
11. Chaudhary MW, Al-Baradie RS. Ataxia-telangiectasia: future prospects. Appl Clin
Genet. 2014. 7:159-67.

133
23 Case
An 8-year-old girl presented with generalised tonic-clonic seizures and
an altered mental state. One month before admission, she displayed
episodes of an abnormal behaviour such as anxiety, stereotypes, verbal aggres-
siveness toward the parents. During the night she was agitated and continu-
ously talking while sleeping. In the morning of the admission she was found
comatose and having generalised tonic and clonus movement, had a tongue
bite, and was incontinent. Seizures were controlled with anti-convulsants.
When the seizures had stopped, the patient woke up and the family as well
as the paramedics noticed an aggressive behaviour and speech dysfunction,
which was stopped by intravenous administration of midazolam first, fol-
lowed by prophylactic treatment with sodium valproate. When she woke up,
she was responsive but displayed an incoherent language. She was perfectly
well before this ailment.
Physical examination was normal. No abnormal movements are noted
during the exam. Neurologic Examination: Alert and cooperative. The pupils
are equal, round, and reactive to light. Her extraocular muscles are intact. Her
face was symmetric. The tongue was midline without fasciculation. Normal
bulk and tone of muscles with 5/5 strength throughout. Coordination: No
dysmetria, tremor, dystonia, or choreoathetosis noted. Normal heel, toe,
flat, and tandem gait. Reflexes: 2+ throughout with bilateral plantar flexor
responses.
A spinal tap was performed which showed a mild pleocytosis (40/mm3) with
78% lymphocytes and 3% atypical lymphocytes, elevated lactate (2.5 mmol/L,
normal range <1.7), and mildly elevated protein (60 mg/dL). A complete hae-
matological work-up failed to reveal the cause of her symptoms; C-Reactive
Protein was normal (CRP <0.5 mg/L). Brain MRI was normal. An electro-
encephalogram recording showed a continuous, slow, rhythmic activity in the
delta-theta activity, which was consistent with encephalitis. The patient’s neu-
rologic condition worsened five days after admission and she developed mood
changes, irritability and fluctuation in consciousness, hallucination, expressive
dysphasia, and facial dyskinesia, diaphoresis, and choreoathetotic movements.

Answer the following questions:


• Give the most likely diagnosis.
• Provide a differential diagnosis.
• Discuss an appropriate diagnostic work-up.
• Discuss the management of this patient.

134
Case 23

DIAGNOSIS
Anti-N-Methyl-D-Aspartates Receptor Encephalitis (ANMDARE)

SUMMARY
An 8-year-old girl presented with generalised tonic-clonic seizures and epi-
sodes of an abnormal behaviour such as anxiety, stereotypes, verbal aggressive-
ness behaviour toward the parents for the last one month before admission.
The clinical evolution and the symptoms support the diagnosis of autoim-
mune encephalitis. Serum and CSF samples from the day of admission sent
for Anti-NMDA-receptor anti-bodies were significantly high in both.

DIFFERENTIAL DIAGNOSIS
Encephalitis refers to an inflammatory disorder of the brain resulting in al-
tered mental status, seizures, or focal neurologic deficits, usually accompanied
by signs of inflammation in the cerebrospinal fluid and Magnetic Resonance
Imaging (MRI) findings ranging from normal to extensive abnormalities. The
causes of encephalitis are numerous, and most patients undergo extensive test-
ing for infectious etiologies without discovery of a causative agent.
• Anti-NMDA receptor encephalitis: The disorder most frequent in
the paediatric population is anti-N-Methyl-D-Aspartate (anti-NMDA)
receptor encephalitis
ncephalitis in which the anti-bodies target the NR1 subu-
nit of the receptor. This disorder has become a leading cause of auto-
immune encephalitis in children and adolescents, with 40% of patients
being younger than age 18 years. The syndrome is highly predictable in
adults and teenagers and usually evolves in stages, including a prodromal
phase of fever, headache, or viral-like symptoms that often goes unno-
ticed. This is followed within a few days or weeks by the onset of psy-
chiatric and behavioural problems including anxiety, bizarre behaviours,
paranoid thoughts, grandiose or hyper religious delusions, and insomnia
that progress to decreased levels of consciousness, seizures, dyskinesias,
choreoathetoid movements or postures, and breathing or autonomic in-
stability. The behavioural changes included new-onset temper tantrums,
agitation, aggression, and changes in mood or personality. Many parents
report changes in speech, including reduced speech, mutism, echolalia,
or perseveration. Regardless of the initial presentation, more than 90% of
patients develop at least 3 of the following groups of symptoms within 1
month of disease onset: psychiatric features, memory disturbance, speech
disorder, seizures, dyskinesias, decreased level of consciousness, auto-
nomic instability, or hypoventilation. Therefore, one should be cautious
diagnosing anti-NMDA receptor encephalitis in patients who after several
weeks of disease onset have developed only 1 or 2 of the above symp-
toms. MRI of the brain shows abnormality in cortical and subcortical T2-
fluid-attenuated inversion recovery signal abnormalities, sometimes with
transient cortical-meningeal enhancement. Similar findings can occur in

135
Child Neurology

the brainstem and cerebellum, despite that the symptoms related to these
structures are rare. In children, the frequency of MRI abnormalities is
less than in adults. Cerebrospinal fluid showing lymphocytic pleocytosis
and less frequently increased protein synthesis or oligoclonal bands. In
patients with normal initial cerebrospinal fluid studies (cell count, pro-
tein, and glucose concentration), oligoclonal bands may be identified,
and repeat analysis performed a few days later often reveals pleocytosis.
The Electroencephalogram (EEG) usually shows diffuse slowing of the
background in the delta-theta range, although some patients may have fo-
cal slowing. The diagnosis of the disorder is confirmed by demonstrating
NMDA receptor anti-bodies in serum or cerebrospinal fluid.
• Limbic encephalitis: This disorder refers to an inflammatory process
of the limbic system, including the medial temporal lobes, amygdala, and
cingulate gyrus. The limbic area of the brain controls many types of activi-
ties including memory, learning, and emotions such as sexual desire, love,
anger, sadness and jealousy. The symptoms of limbic encephalitis include
memory loss, seizures, confusion, disturbances of sleep and psychological
disturbances such as altered personality or behaviour. Causes of limbic
encephalitis and most forms of encephalitis fall into 2 main categories:
° Infectious encephalitis–Caused by direct invasion of the limbic area
of the brain, usually by a virus.
° Autoimmune encephalitis–Caused by the person’s own immune sys-
tem reacting against a component of the limbic area. The most fre-
quent cell surface target antigen of limbic encephalitis is LGI1. Other
cell surface antigens related to limbic encephalitis include AMPA and
GABAB receptors. There are broadly two forms of autoimmune lim-
bic encephalitis: Paraneoplastic Limbic Encephalitis (PLE) and Non-
Paraneoplastic Limbic Encephalitis (NPLE).
° Vascular Limbic Disorders: Differentiation between primary vascu-
litis and LE may represent a real challenge under certain conditions
of subacute presentation. Abnormal vessels on angiography and cyto-
toxic oedema on DWI that usually extends throughout the compro-
mised vascular territory and is not restricted to the limits of the limbic
system are helpful to confirm imaging suspicions. Transient global
amnesia also affects the hippocampal formation, but its clinical and
imaging presentation is rather typical.
° Seizure-related limbic disorders. Hippocampal sclerosis associated
with temporal lobe abnormalities is the multi factorial hallmark of
mesial temporal sclerosis. This condition could be a consequence of
prolonged unilateral febrile seizures or status epilepticus, which oc-
curs mainly in children when the hippocampus is more vulnerable to
convulsion-induced excitotoxic damage and involves the sectors of
the hippocampus rich in kainate or N-methyl-D-aspartate receptors
and, therefore, that lack protection against calcium overload.

136
Case 23

• Hashimoto encephalopathy: This is an ill-defined disorder in which


the syndrome and immunological findings are unclear, but it has been
seen in one study; 52% with encephalopathy had hypothyroidism and
48% normal thyroid function; however, some patients can develop
hyperthyroidism.
• Rasmussen encephalitis: This is an inflammatory encephalopathy char-
acterised by progressive refractory partial seizures, cognitive deterioration,
and focal deficits that occur with gradual atrophy of one brain hemisphere.
The disorder frequently presents in 6-to 8-year-old children, although ado-
lescents and adults can be affected. The etiology is unknown and, therefore,
multiple theories have been proposed.
• Other autoimmune disorders: Autoimmune systemic disorders are as-
sociated with LE. Sjogren syndrome, lupus erythematosus, Behcet’s dis-
ease, primary angiitis of the CNS, and anti-phospholipid syndrome can
occasionally cause clinical and/or radiologic abnormalities in the limbic
system that are not anti-body mediated but that are accompanied by histo-
pathologic evidence of cellular inflammation. Hashimoto encephalopathy
or steroid-responsive encephalopathy associated with autoimmune thy-
roiditis manifests as a diffuse progressive AME characterised by demen-
tia, psychiatric disturbances, and seizures; there also is a vasculitic type
characterised by multiple stroke like episodes, seizures, and fluctuating
consciousness. This disorder is more common in women and is associated
with autoimmune anti-thyroid anti-bodies.

DIAGNOSTIC APPROACH
• Currently it is suggested that any rapidly progressive encephalopathy of
unclear etiology, particularly if accompanied by lymphocytic CSF pleocy-
tosis (although routine CSF studies can be normal), and multifocal symp-
toms with or without MRI changes should raise concern for an immune
mediated process.
• Anti-body testing cannot replace clinical evaluation. Determination of an-
ti-bodies should be considered as a supportive test to confirm the etiology
of a disorder clinically suspected to be immune mediated.
• Autoantibody testing is extremely important for the proper diagnosis
of autoimmune encephalitis. Commercial tests for autoantibodies to
NMDAR, LGI1, Caspr2, AMPAR (GluR1, GluR2 subunits), and GABA-
B-R are widely available.
• Neuroimaging: Brain MRI in patients with NMDAR, AMPAR, LGI1,
Caspr2, and GABA-B anti-bodies may be normal or show an increased
T2 signal, especially in the medial temporal lobes. Brain MRI therefore
does not distinguish between infectious and autoimmune causes, and a
normal brain MRI does not exclude these causes. Advanced brain im-
aging with PET or SPECT has shown diverse areas of regional hyper-
or hypo-metabolism in patients with NMDAR, LGI1, Caspr2 or other
autoantibodies.

137
Child Neurology

• Electroencephalography: EEG is useful in patients with autoimmune


or infectious encephalitis for excluding subclinical seizures, for progno-
sis, and sometimes for suggesting particular diagnoses. The extreme delta
brush pattern may be observed in patients with anti-NMDAR encephali-
tis, most often in patients who are comatose. This distinctive EEG pattern
should prompt testing for NMDAR anti-bodies.
• Cancer screening: Paraneoplastic disorders are, in general, autoimmune
disorders that are triggered by tumours. In many cases the target antigen is
expressed by tumour tissue, such as HuD proteins in small cell lung can-
cer and NMDARs in ovarian teratoma. It is important to detect tumours
promptly for several reasons.
° Treating the relevant tumour is thought to be helpful for treating the
autoimmune disorder.
° Tumour therapy and immune therapy may need to be given simulta-
neously and in a coordinated fashion.
° Treatment with steroids, rituximab, or cyclophosphamide could com-
plicate tumour diagnosis in the case of tumours like lymphoma.

MANAGEMENT
• Treatment for suspected autoimmune encephalitis
encephalitis is often given empiri-
cally prior to specific anti-body test results. This may include steroids and/
or IVIG. If a cell-surface/synaptic antibody disorder is diagnosed, initial
treatments may include IVIG, plasmapheresis, and/or steroids. Steroids
may be beneficial in a range of autoimmune disorders but could po-
tentially create problems with the diagnosis of certain disorders such as
CNS lymphoma.
• IVIG offers an important advantage of being unlikely to make infectious
encephalitis worse. Plasmapheresis is also unlikely to significantly worsen
infectious encephalitis.
• In general, prompt treatment, and escalation of treatment in patients who
remain ill, is associated with better outcomes. Although there are no ran-
domised treatment trials, but physicians often use IV solumedrol (30 mg/
kg daily for 3-5 days then a taper over several weeks) and IVIG (0.4 g/kg/
day for 5 days). But some other groups have advocated plasmapheresis
instead of IVIG, and so far, there is no convincing evidence of superiority
for either approach.
• If the patient remains significantly impaired after first-line therapy, sec-
ond-line treatments are typically used. Some groups might wait 2 weeks
or longer to allow first-line therapies time to work, but our group often
proceeds more quickly to second line therapy sooner in patients who are
very ill.
• Second line therapies include Rituximab (often 375 mg/m2 weekly for 4
weeks) or cyclophosphamide (750 mg/m2 IV monthly until improvement

138
Case 23

is noted), or both. Treatment strategies are similar in children and adults,


but physicians may be more reluctant to use cyclophosphamide, relying
more on Rituximab as a second line treatment.

REFERENCES
1. Dalmau J, Lancaster E, Martinez-Hernandez E, Rosenfeld MR, BaliceGordon R.
Clinical experience and laboratory investigations in patients with anti-NMDAR
encephalitis. The Lancet Neurology 2011;10:63-74.
2. Florance NR, Davis RL, Lam C, et al.. Anti-N-methyl-D-aspartate receptor (NM-
DAR) encephalitis in children and adolescents. Annals of Neurology 2009;66:11-
18.
3. Titulaer MJ, McCracken L, Gabilondo I, et al.. Treatment and prognostic factors for
long-term outcome in patients with anti-NMDA receptor encephalitis: an obser-
vational cohort study. The Lancet Neurology 2013;12:157-165.
4. Adang, L. A., Lynch, D. R., & Panzer, J. A. Pediatric anti-NMDA receptor enceph-
alitis is seasonal. Annals of Clinical and Translational Neurology, (2014). 1(11), 921-925.
5. Armangue, T., Titulaer, M. J., Málaga, I., Bataller, L., Gabilondo, I., Graus, F., &
Dalmau, J. Pediatric anti-NMDAR encephalitis: Clinical analysis and novel find-
ings in a series of 20 patients. Journal of Pediatrics, (2013). 162(4), 850-856.
6. Hong, S., Klein-Gitelman, M., & Wainwright, M. S. Recognition and treatment of
N-methyl-D-aspartate receptor encephalitis. Clinical Pediatric Emergency Medicine,
(2015)16, 3-10.
7. Byrne, S., & Lim, M. N-methyl D-aspartate receptor anti-body encephalitis: How
much treatment is enough? Developmental Medicine & Child Neurology, (2015).
57(1), 14-15.
8. Dalmau, J., & Rosenfeld, M. R. Paraneoplastic syndromes of the CNS. The Lancet
Neurology, (2008). 7(4), 327-340.
9. DeSena, A. D., Greenberg, B. M., & Graves, D. ‘’Light switch’’ mental status
changes and irritable insomnia are two particularly salient features of anti-NMDA
receptor anti-body encephalitis. Pediatric Neurology, (2014). 51(1), 151-153.

139
24 Case
An 8-month-old male baby of non-consanguineous parents presented
from birth with generalised hypotonia and respiratory distress,
requiring artificial ventilation. He was delivered by normal spontaneous
vaginal delivery. The pregnancy was uncomplicated and foetal movements in
utero seemed normal. The patient remained in the nursery for several weeks
after birth because he required oxygen and feeding support. Because he had a
poor suck and swallow, he required a nasogastric tube for the first 2 months of
life. On day 11, 5 days after introduction of gavage feeding, he also developed
bilateral pleural effusions, requiring drainage. Now he is just starting to sit
up, but has not attempted to crawl. He babbles appropriately. He can eat some
pureed foods and drink from a bottle, but is not gaining weight well. He often
has a cough and was hospitalised once at 6 months, for pneumonia.
On physical examination: He has no dysmorphic features. There is no
hepatosplenomegaly. Alert look but lying in bed with hypotonic posture.
The pupils are equal, round, and reactive to light. He tracks objects in all
directions. His facial movements are weak but symmetric. A gag is present
but there is excessive drooling. The tongue is midline without fasciculation.
There is normal bulk with moderate diffuse hypotonia. He is able to move all
four extremities against gravity but has some mild difficulty holding his head
up against gravity. He sits only with support. He picks up small objects with a
raking motion and does not use a pincer grasp. There is no dysmetria. Deep
tendon reflexes absent with bilateral plantar flexor responses.

Answer the following questions:


• Give the most likely diagnosis.
• Discuss the differential diagnosis.
• Discuss an appropriate diagnostic work-up.
• Discuss the management of this patient.

140
Case 24

DIAGNOSIS
X-linked myotubular myopathy

SUMMARY
The patient is an 8-month-old boy with a history of respiratory and feeding
problems at birth who presents with motor delays, as well as persistent respira-
tory and feeding difficulties. His exam is remarkable for myopathic facies, neck
flexor weakness, oropharyngeal weakness with excessive drooling, hypotonia,
and areflexia. Muscle biopsy disclosed typical features of X-linked myotubular
myopathy. The MTM1 gene was sequenced and revealed a C.1261-10 A>G
mutation, confirming the diagnosis.

DIFFERENTIAL DIAGNOSIS
X-Linked Myotubular Myopathy (XLMTM) is a rare genetic neuromuscular
disorder that is characterised by muscle weakness that is most typically se-
vere but can range from mild to severe. Symptoms are often present at birth,
though may develop later in infancy or early childhood. Rarely, symptoms
may not present until adolescence or adulthood. Common symptoms include
mild to profound muscle weakness, diminished muscle tone (hypotonia or
“floppiness”), feeding difficulties, and potentially severe breathing complica-
tions (respiratory distress). Feeding difficulties and respiratory distress devel-
op because of weakness of the muscles that are involved in swallowing and
breathing. The overall severity of the disorder can range from mildly affected
individuals to individuals who develop severe, life-threatening complications
during infancy and early childhood. Most affected individuals have a severe
form of the disorder and respiratory failure is an almost uniform occurrence.
XLMTM is caused by Mutations To Myotubularin (MTM1) gene. The disor-
der is inherited as an X-linked condition. The disorder predominantly affects
child, but child carriers, while typically asymptomatic, can develop a range of
symptoms. In rare specific cases, childs can develop a severe form similar to
that seen in child.
Important clinical differences exist between central core disease; nemaline
myopathy and centronuclear myopathy deserve mention. Central core disease
is not associated with facial weakness, oculomotor palsies, or cardiomyopathy,
but does carry a susceptibility to malignant hyperthermia. Nemaline myopa-
thy frequently causes facial weakness, but oculomotor abnormalities and car-
diomyopathy occur only rarely. Finally, centronuclear myopathy often causes
facial and oculomotor weakness, but no cardiomyopathy.

Cerebral Hypotonia
• Chromosome disorders
° Prader-Willi syndrome
° Trisomy

141
Child Neurology

• Chronic non-progressive encephalopathy


° Cerebral malformation
° Perinatal distress
° Postnatal disorders
• Infantile GM1 gangliosidosis
• Pyruvate carboxylase deficiency

Other Genetic Defects


a. Familial dysautonomia
b. Oculocerebrorenal syndrome (Lowe syndrome)

Peroxisomal Disorders
a. Cerebrohepatorenal syndrome (Zellweger syndrome)
b. Neonatal adrenoleukodystrophy

Spinal Cord Disorders


Spinal muscular atrophies

Polyneuropathies
a. Congenital hypomyelinating neuropathy
b. Giant axonal neuropathy
c. Hereditary motor-sensory neuropathies

Disorders of Neuromuscular Transmission


a. Familial infantile myasthenia
b. Infantile botulism
c. Transitory myasthenia gravis

Congenital Myopathies
a. Fibre-type disproportion
b. Central core disease
c. Congenital fibre-type disproportion myopathy
d. Myotubular (centronuclear) myopathy
e. Nemaline (rod) myopathy

Muscular Dystrophies
a. Bethlem myopathy
b. Congenital dystrophinopathy
c. Congenital muscular dystrophy

142
Case 24

d. Primary merosin deficiency


e. Secondary merosin deficiency
f. Merosin positive
g. Congenital myotonic dystrophy

Metabolic Myopathies
a. Acid maltase deficiency
b. Cytochrome-c oxidase deficiency

DIAGNOSTIC APPROACH
• The first step in diagnosis is to determine whether the disease location
is in the brain, spine, or motor unit. Careful neurological examination
should, in most cases, localise the site of the lesion to the Upper Motor
Neuron (UMN) or Lower Motor Neuron (LMN) unit.
• Poor spontaneous movements and a frog-like posture are characteristic of
LMN conditions.
• The brain and the peripheral nerves are concomitantly involved in some
lysosomal and mitochondrial disorders.
• Both brain and skeletal muscles are abnormal in infants with acid maltase
deficiency and congenital myotonic dystrophy.
• Diagnosis of cerebral hypotonia: There are many clues that exist for
the diagnosis of cerebral hypotonia which are; abnormalities of other
brain functions, dysmorphic features, fisting of the hands, malformations
of other organs, movement through postural reflexes, normal or brisk ten-
don reflexes and scissoring on vertical suspension. Most important is the
presence of other abnormal brain functions: Decreased consciousness and
seizures. Cerebral malformation is the likely explanation for hypotonia in
an infant with dysmorphic features or with malformations in other organs.
• A tightly fisted hand in which the thumb is constantly enclosed by the
other fingers and does not open spontaneously (fisting) and adduction of
the thighs so that the legs are crossed when the infant is suspended verti-
cally (scissoring) are early signs of spasticity and indicate cerebral dysfunc-
tion. Eliciting postural reflexes in newborns and infants when spontane-
ous movement is lacking indicates cerebral hypotonia.
• Motor unit disorders: Disorders of the motor unit are not associated
with malformations of other organs except for joint deformities and the
mal-development of bony structures. The face sometimes looks dysmor-
phic when facial muscles are weak or when the jaw is underdeveloped.
Tendon reflexes are absent or depressed. Loss of tendon reflexes that are
out of proportion to weakness is more likely caused by neuropathy than
myopathy, whereas diminished reflexes that are consistent with the de-
gree of weakness are more often caused by myopathy than neuropathy.

143
Child Neurology

Muscle atrophy suggests motor unit disease but does not exclude the pos-
sibility of cerebral hypotonia. Failure of growth and even atrophy can be
considerable in brain-damaged infants. The combination of atrophy and
fasciculation is strong evidence of denervation. However, the observation
of fasciculation in newborns and infants is often restricted to the tongue,
and distinguishing fasciculation from normal random movements of an
infant’s tongue is difficult unless atrophy is present.
• A serum CK level may initially be sent to screen for muscle diseases. In
congenital myopathies, the CK level is usually normal or only mildly el-
evated. A markedly elevated CK is more indicative of a congenital mus-
cular dystrophy.
• EMG is also usually normal but may show mild and nonspecific myo-
pathic features. NCSs are normal and help rule out peripheral neuropa-
thies. Obtaining reliable EMGs and NCSs in infants and young children
can be very difficult. But when performed by an experienced clinician,
they can help refine the differential diagnosis in a hypotonic patient.
• However, the diagnosis is usually clarified most easily with a muscle bi-
opsy. In recent times, as these diseases are better characterised, a genetic
diagnosis is becoming easy. Molecular genetic testing can detect a muta-
tion in approximately 60%-98% of affected individuals.

MANAGEMENT
• There are no specific treatments for congenital myopathies at this time.
Patients require varying degrees of supportive care, depending on the se-
verity of their disease.
• Treatment may require the coordinated efforts of a team of specialists with
expertise in treating neuromuscular disorders. Paediatricians, pulmonolo-
gists, neurologists, orthopaedists, eye specialists, dental specialists, and
other healthcare professionals who can systematically and comprehen-
sively plan an affective supportive treatment.
• Genetic counseling will be of benefit for affected individuals and their
families.
• Some affected individuals will require prolonged, constant ventilation
support. There are different methods for ventilation including non-in-
vasive and invasive techniques. The family should be involved in making
decision about the duration of respiratory support in careful consultation
with the patient’s physicians and other members of the healthcare team
based upon the specificity of their case.
• In some individuals feeding difficulties will require the insertion of a
feeding tube (gastrostomy).
• Physical and occupational therapy is recommended to improve muscle
strength and prevent contractures. Special measures may be necessary
to allow ventilator-dependent individuals to communicate. Additional

144
Case 24

therapies are symptomatic and supportive. For example, scoliosis may re-
quire surgical intervention.

INVESTIGATIONAL THERAPIES
• Gene therapy is being studied as an approach to therapy for XLMTM.
Given the permanent transfer of the normal gene, which is able to pro-
duce active enzyme at all sites of disease, this form of therapy is theoreti-
cally most likely to lead to a “cure”. Studies of MTM1 gene therapy in
both mouse and dog models of MTM have shown great promise, and
clinical trial of this treatment strategy is planned in the near future.
• Another consideration is protein replacement therapy. Studies utilising
the mouse model of MTM have demonstrated that systemic injection of a
genetically engineered (recombinant) MTM1 protein can prevent disease
onset and progression in this model.

REFERENCES
1. Romero NB, Clarke NF. Congenital myopathies. Handb Clin Neurol 2013;
113:1321.
2. North KN. Clinical approach to the diagnosis of congenital myopathies. Semin
Pediatr Neurol 2011; 18:216.
3. Sarnat HB. New insights into the pathogenesis of congenital myopathies. J Child
Neurol 1994; 9:193.
4. Bartsch O, Kress W, Wagner A, Seemanova E. The novel contiguous gene syn-
drome of myotubular myopathy (MTM1), child hypogenitalism and deletion in
Xq28:report of the first familial case. Cytogenet Cell Genet 1999; 85:310.
5. Laporte J, Kress W, Mandel JL. Diagnosis of X-linked myotubular myopathy by
detection of myotubularin. Ann Neurol 2001; 50:42.
6. Laporte J, Biancalana V, Tanner SM, et al.. MTM1 mutations in X-linked myotubu-
lar myopathy. Hum Mutat 2000; 15:393.
7. Bijarnia S, Puri RD, Jain M, Kler N, Roy S, Urtizberea JA, Biancalana V, Verma
IC. Mutation studies in X-linked myotubular myopathy in three Indian families.
Indian J Pediatr. 2010 Apr;77(4):431-3.
8. Wallgren-Pettersson C, Clarke A, Samson F, et al. The myotubular myopathies:
differential diagnosis of the X linked recessive, autosomal dominant, and auto-
somal recessive forms and present state of DNA studies. J Med Genet. 1995 Sep.
32(9):673-9.

145
25 Case
A 6-year-old female child without a significant past medical history was
referred for evaluation of frequent unusual episodes for the past 3
months. The unusual episodes consist of sudden arrest of activity with staring
and minimal eyelid flutter for 10 to 15 seconds occurring 5 to 10 times per
day. She was unresponsive to voice or tactile stimulation during the episodes.
She was able to resume activities immediately without any recollection of
the event once the episode finished. Her teachers have noted that she stares
off and frequently daydreams in class repeatedly and does not seem to be
remembering instructions and classroom material.
On physical examination: She was alert and active and her speech was fluent
without dysarthria. All accessible cranial nerves were intact. She had normal
bulk and tone with 5/5 strength throughout. Her cerebellum examination was
also unremarkable. There were no sensory deficits and had a normal heel,
toe, flat, and tandem gait. Reflexes were normally present throughout with
bilateral plantar flexor responses.

Answer the following questions:


• Give the most likely diagnosis.
• Briefly summarise this case.
• What are the differential diagnoses?
• Discuss an appropriate diagnostic work-up.
• Discuss the management of this patient.

146
Case 25

DIAGNOSIS
Childhood absence epilepsy

SUMMARY
A 6-year-old child referred for evaluation of frequent unusual episodes of
sudden arrest of activity with staring and minimal eyelid flutter for 10 to 15
seconds occurring 5 to 10 times per day. The patient is unresponsive to voice
or tactile stimulation during the episodes. She is able to immediately resume
activities without any recollection of the event once the episode finishes. Her
EEG record showed typical 3 Hz spikes and waves complex and exaggerated
with hyperventilation.

DIFFERENTIAL DIAGNOSIS
• The differential diagnosis of a child with difficulties in school and day-
dreaming includes various types of absence, as well as complex partial sei-
zures. In certain cases, typical absence seizures may need to be differenti-
ated from eyelid myoclonia with absence seizures when parents report
associated rhythmic eye blinking or fluttering. In this seizure type, chil-
dren experience rhythmic eyelid myoclonia with a minor impairment of
consciousness during seizures. These patients are highly photosensitive.
Absence seizures with perioral myoclonia, as well as myoclonic absence,
are additional generalised epilepsies that may need to be considered.
• Complex partial seizure: A complex partial seizure starts focally with-
in the brain and causes impairment of consciousness. In most patients,
complex partial seizures represent underlying temporal lobe epilepsy.
Complex partial seizures typically last 30 seconds to 2 minutes. Complex
partial seizures arise in the cortex, most often the temporal lobe, but can
originate in the frontal and parietal lobes as well. Impaired consciousness
without generalised tonic-clonic activity characterises complex partial sei-
zures. Amnesia and complete lack of awareness of the event are essential
features. Less than 30% of children report an aura. The aura is usually a
nondescript unpleasant feeling, but may also be a stereotyped auditory
hallucination or abdominal discomfort. The first feature of the seizure can
be staring, automatic behaviour, tonic extension of one or both arms, or
loss of body tone. The seizure usually terminates with a period of postic-
tal confusion, disorientation, or lethargy. Transitory aphasia is sometimes
present.
• Attention Deficit Hyperactivity Disorder (ADHD): According to the
Diagnostic and Statistical Manual of Mental Health Disorders, (DSM-5),
the 3 types of Attention Deficit Hyperactivity Disorder (ADHD) are (1)
predominantly inattentive, (2) predominantly hyperactive/impulsive, and
(3) combined. In children, problems paying attention may result in poor
school performance. ADHD predominantly inattentive type (ADHD-
PI) presents with symptoms including being easily distracted, forgetful,

147
Child Neurology

daydreaming, disorganisation, poor concentration, and difficulty complet-


ing tasks.
• Delirium is defined as a transient, usually reversible, cause of cerebral
dysfunction and manifests clinically with a wide range of neuropsychiatric
abnormalities. The clinical hallmarks of delirium are decreased attention
span and a waxing and waning type of confusion.
• Daydreaming: Daydreaming is a pleasant escape for people of all ages.
Children feel the need for escape most acutely when in school and may
stare vacantly out of the window to the place where they would rather be.
Daydreams can be hard to interrupt, and a child may not respond to verbal
commands.

DIAGNOSTIC WORK-UP
• When evaluating a child for staring spells, laboratory tests for metabolic
abnormalities or toxic or drug ingestion (especially in older children) may
be indicated. If a clear history of the episodic nature of the attacks is ob-
tained, then the EEG can be diagnostic and laboratory tests may not be
necessary.
• A child with suspected absence seizures may be hyperventilated in the
office, which will often provide the diagnosis. However, an EEG with
hyperventilation and photic stimulation should be performed to confirm
the diagnosis.
• The only diagnostic test for absence seizures is the EEG. The EEG re-
veals a generalised spike wave occurring ictally, interictally, and in re-
sponse to hyperventilation. The spike wave is superimposed on a normal
background.
• When evaluating a child with a developmental delay, or if the EEG re-
veals atypical absences, then a full work-up for the underlying cause of a
symptomatic generalised epilepsy is indicated. Atypical absence seizures
are characterised by slow spike-and-wave paroxysms, classically 2.5 Hz.
• Neuroimaging findings are normal in idiopathic epilepsies by definition,
and therefore, neuroimaging is not indicated if the typical clinical pattern
is present. A normal result helps to support the diagnosis of idiopathic epi-
lepsy. For cryptogenic and symptomatic generalised epilepsies, neuroim-
aging can help in the diagnosis of any underlying structural abnormality.

MANAGEMENT
• The course remains relatively benign, with few cases remitting but 80 per-
cent responding well to first-line therapy. Only 2 first-line AEDs have
approval from the US Food and Drug Administration (FDA) to be indi-
cated for absence seizures: ethosuximide and valproic acid. Ethosuximide
has efficacy for absence seizures only and valproic acid has efficacy for
absence, generalised tonic-clonic, and myoclonic seizures.

148
• Ethosuximide is preferred because of its lower incidence of serious side
effects. If neither drug alone provides seizure control, use them in com-
bination at reduced doses or substitute another drug. Lamotrigine is more
effective than levetiracetam. The EEG becomes normal if treatment is
successful, and repeat EEG is useful to confirm the seizure-free state.
Clonazepam is sometimes useful in the treatment of refractory absence
epilepsy. Carbamazepine and phenytoin may accentuate the seizures and
cause absence status epilepticus.

REFERENCES
1. Berg AT, Berkovic SF, Brodie MJ, et al.. Revised terminology and concepts for or-
ganization of seizures and epilepsies: report of the ILAE Commission on Classifi-
cation and Terminology, 2005-2009. Epilepsia 2010; 51:676.
2. Proposal for revised classification of epilepsies and epileptic syndromes. Commis-
sion on Classification and Terminology of the International League Against Epi-
lepsy. Epilepsia 1989; 30:389.
3. Loiseau P, Duché B, Pédespan JM. Absence epilepsies. Epilepsia 1995; 36:1182.
4. Fong GC, Shah PU, Gee MN, et al.. Childhood absence epilepsy with tonic-clonic
seizures and electroencephalogram 3-4-Hz spike and multispike-slow wave com-
plexes: linkage to chromosome 8q24. Am J Hum Genet 1998; 63:1117.
5. Yalçın O. Genes and molecular mechanisms involved in the epileptogenesis of idi-
opathic absence epilepsies. Seizure 2012; 21:79.
6. Bai X, Vestal M, Berman R, et al.. Dynamic time course of typical childhood absence
seizures: EEG, behavior, and functional magnetic resonance imaging. J Neurosci
2010; 30:5884.
7. Callenbach PM, Bouma PA, Geerts AT, et al.. Long-term outcome of childhood ab-
sence epilepsy: Dutch Study of Epilepsy in Childhood. Epilepsy Res 2009; 83:249.
8. Shinnar S, Cnaan A, Hu F, et al.. Long-term outcomes of generalized tonic-clonic
seizures in a childhood absence epilepsy trial. Neurology 2015; 85:1108.
9. Dlugos D, Shinnar S, Cnaan A, et al. Pretreatment EEG in childhood absence epi-
lepsy: associations with attention and treatment outcome. Neurology 2013; 81:150.
10. Mariani E, Rossi LN, Vajani S. Interictal paroxysmal EEG abnormalities in child-
hood absence epilepsy. Seizure 2011; 20:299.
11. Glauser TA, Cnaan A, Shinnar S, et al. Ethosuximide, valproic acid, and lamo-
trigine in childhood absence epilepsy: initial monotherapy outcomes at 12 months.
Epilepsia 2013; 54:141..
12. Grosso S, Galimberti D, Vezzosi P, et al. Childhood absence epilepsy: evolution and
prognostic factors. Epilepsia 2005; 46:1796.
13. Franzoni E, Matricardi S, Di Pisa V, et al. Refractory absence seizures: An Italian
multicentre retrospective study. Eur J Paediatr Neurol 2015; 19:660.
26 Case
An 8-year-old boy presented to the emergency department with
headache and vomiting for the last one week, and focal seizures of
1-day duration. He started complaining of severe headache one week before
and developed multiple episodes of left sided focal seizures. The seizures were
of simple partial type and consisted of left sided clonic jerks. He also had a
history of recurrent tonsillitis over the past couple of months for which he
had been treated with anti-biotics by his family physician. Now he again had a
history of flu, low-grade fever and mild cough for the last one week. His birth
and development periods were uneventful and there was no past history of
convulsions. Parents were second-degree cousins and there was no history of
epilepsy or stroke at young age in siblings or relatives.
His physical examination revealed an alert and oriented child. Vital signs were
normal. There were no signs of systemic illness. The neurological examina-
tion showed slurred speech. Cranial nerve examination showed left sided fa-
cial paralysis of upper motor neuron type. His pupils were equal, round, and
reactive to light. His extraocular muscles are intact, except he was unable to
completely abduct his right eye. His fundi are difficult to visualise but there
appears to be blurring of the right disc margin.
Motor examination revealed left sided hypotonia, power grade of 3/5, hypore-
flexia, equivocal planters, and absent clonus. Motor findings were normal on
the right side. There was no sensory deficit on examination.
Brain Magnetic Resonance Imaging (MRI), performed, showed hypointense
signals within the venous structures with clear hyperintensity on T1 within
the superior sagittal sinus. The post contrast images showed a filling defect in
superior sagittal sinus. The MR Venogram (MRV) confirmed non-visualisa-
tion of the superior sagittal sinus.

Answer the following questions:


• What is the most likely diagnosis?
• Provide a differential diagnosis.
• Discuss an appropriate diagnostic work-up.
• Discuss the management of this patient.

150
Case 26

DIAGNOSIS
Cerebral sinus venous thrombosis resulting in left sided hemiparesis with fa-
cial nerve palsy and Right abducens nerve palsy.

SUMMARY
An 8-year-old boy presented with headache, emesis and focal seizures. He
started complaining of severe headache one week before and developed multi-
ple episodes of left sided focal seizures. There were hypointense signals within
the venous structures with clear hyperintensity on T1 within the superior sag-
ittal sinus. The post contrast images showed a filling defect in the superior
sagittal sinus. The MR Venogram (MRV) confirmed non-visualisation of the
superior sagittal sinus.

DIFFERENTIAL DIAGNOSIS
Several mechanisms can cause thrombosis in the cerebral venous system
such as venous stasis, hypercoagulable conditions, and endothelial damage.
Impaired venous drainage leads to ischaemia as a consequence of congestion
of the underlying white matter. Superior sagittal sinus thrombosis can cause
bilateral cortical infarcts, whereas, thrombosis involving the deep venous sys-
tem can cause cerebellar or basal ganglia infarcts. Neurologic deficits caused by
venous infarction may less clearly localise than those caused by arterial stroke.
A wide variety of local and systemic conditions predispose to the development
of CSVT in childhood. Cerebral hypoperfusion as a result of dehydration,
shock, or congestive heart failure may result in CSVT. Infectious processes,
such as mastoiditis, otitis media, and bacterial meningitis, predispose children
to developing CSVT, and sepsis in neonates can also be a risk factor. A hyper-
coagulable state is found in many children with CSVT and may be related to
a broad range of conditions. Autoimmune diseases (systemic lupus erythema-
tosus, rheumatoid arthritis), malignancy, hereditary thrombophilia (protein C
deficiency, protein S deficiency, anti-thrombin III deficiency, factor V Leiden
gene mutation, prothrombin gene mutation), and medications (oral contracep-
tive pills) should be considered. Anti-phospholipid syndrome occurs in child-
hood, predisposing to both arterial and venous infarction. Classically, affected
children experience headaches, livedo reticularis, pulmonary hypertension,
and recurrent deep vein thrombosis in addition to strokes. A variety of other
conditions deserve to be mentioned as potential causes of CSVT in childhood,
including head trauma, nephrotic syndrome, and liver disease. Nevertheless,
the underlying cause of CSVT remains idiopathic in as many as 25% of cases.
Once a CSVT is diagnosed, risk factors for CSVT in childhood should be
assessed. In many cases, the cause is obvious. For example, an otitis media
might be seen on examination; mastoiditis might be noted on neuroimaging;
the patient already might have a diagnosis of an autoimmune disease, malig-
nancy, or kidney disease; gastroenteritis with vomiting and diarrhoea might be

151
Child Neurology

responsible for dehydration. In other cases, the underlying cause is less clear
and these patients deserve a thorough evaluation for potential etiologies.

Risk Factors for CSVT


The majority of signs and symptoms of stroke are nonspecific, and can be eas-
ily attributed to other causes. One way to avoid delays or misdiagnosis would
be to identify risk factors for stroke that would prompt more aggressive and
timely investigation.
Long-standing cyanotic lesions cause polycythaemia and anaemia, which both
increase the risk of thromboembolism and cerebral infarction. Embolic clots
can arise in children with cardiomyopathies, rheumatic heart disease, pros-
thetic valves, or valvular vegetation from endocarditis.

Haematologic Causes
Sickle Cell Disease (SCD) is a very common cause of pediatric CSVT.
Prothrombotic disorders have been identified in 30 to 76% of patients expe-
riencing arterial or venous events, and should be suspected if there is a family
history of early onset disease. Acquired prothrombotic disorders secondary to
deficiencies in proteins C and S may occur in children with renal and liver dis-
ease; including nephrotic syndrome with loss of coagulation factors. Protein C
deficiency has also been reported in children taking valproate.

Infection
Five to twelve percent of children with bacterial meningitis, TB meningitis,
and viral encephalitis will have a stroke due to local vasculitis, or CSVT. Head
and neck infections, such as mastoiditis or periorbital infections, remain im-
portant causes of CSVT.

Oncologic
Children with cancer are at increased risk for AIS or CSVT as a result of their
disease, subsequent treatment, and susceptibility to infection. Leukemia and
lymphoma create a hypercoagulable and hyperviscous state. Treatment with
L-asparaginase decreases anti-thrombin levels, and may trigger venous throm-
bosis in leukemic children concurrently receiving prednisone.

DIAGNOSTIC WORK-UP
The diagnosis of CSVT is typically based on clinical suspicion and imaging
confirmation. Clinical findings in CSVT usually fall into 2 major categories,
depending on the mechanism of neurological dysfunction: (1) Those that
are related to increased intracranial pressure attributable to impaired venous
drainage and (2) those related to focal brain injury from venous ischaemia/
infarction or haemorrhage.

152
Case 26

CSVT is an important diagnostic consideration in patients with headache and


papilledema or diplopia (caused by sixth nerve palsy) even without other neu-
rological focal signs suggestive of idiopathic intracranial hypertension.
A complete blood count, chemistry panel, sedimentation rate, and measures of
the prothrombin time and activated partial thromboplastin time are indicated for
patients with suspected CSVT. These studies may demonstrate abnormalities
suggestive of an underlying hypercoagulable state, an infectious process, or an
inflammatory state, all of which may contribute to the development of CSVT.
Lumbar puncture: Unless there is clinical suspicion of meningitis, exami-
nation of the Cerebrospinal Fluid (CSF) is typically not helpful in cases with
focal neurological abnormalities and radiographic confirmation of the diagno-
sis of CSVT. Elevated opening pressure is a frequent finding in CSVT and is
present in >80% of patients. An elevated opening pressure may be a clue for
diagnosing CSVT in patients who present at the emergency department with
headaches. Elevated cell counts (found in ≈50% of patients) and protein levels
(found in ≈35%) are often present, but their absence should not discourage
consideration of the diagnosis of CSVT. There are no specific CSF abnormali-
ties in CSVT.

Imaging in Diagnosis of CSVT


Diagnostic imaging has played an increasing role in the diagnosis and man-
agement of CSVT. CT is widely used as the initial neuroimaging test in pa-
tients who present with new-onset neurological symptoms such as headache,
seizure, mental alteration, or focal neurological signs. CT without contrast is
often normal but may demonstrate findings that suggest CSVT. The primary
sign of acute CSVT on a non-contrast CT is hyperdensity of a cortical vein or
dural sinus. Acutely thrombosed cortical veins and dural sinuses appear as a
homogenous hyperdensity that fills the vein or sinus and are most clearly visu-
alised when CT slices are perpendicular to the dural sinus or vein However,
only approximately one-third of CVT demonstrates direct signs of a hyper-
dense dural sinus. Thrombosis of the posterior portion of the superior sagittal
sinus may appear as a dense triangle, the dense or filled delta sign.
Contrast-enhanced CT may show enhancement of the dural lining of the si-
nus with a filling defect within the vein or sinus. Contrast-enhanced CT may
show the classic “empty delta” sign, in which a central hypointensity due to
very slow or absent flow within the sinus is surrounded by contrast enhance-
ment in the surrounding triangular shape in the posterior aspect of the supe-
rior sagittal sinus. This finding may not appear for several days after onset of
symptoms but does persist for several weeks.
Magnetic Resonance Imaging: MRI is more sensitive for the detection of
CSVT than CT at each stage after thrombosis. The magnetic resonance signal
intensity of venous thrombus varies according to the time of imaging from the
onset of thrombus formation. In the first week, venous thrombus frequently
appears as isointense to brain tissue on T1-weighted images and hypointense

153
Child Neurology

on T2-weighted images owing to increased deoxyhaemoglobin. By the second


week, thrombus contains methemoglobin, which results in hyperintensity on
T1-and T2-weighted images. With evolution of the thrombus, the paramagnet-
ic products of deoxyhemoglobin and methemoglobin is present in the sinus.
The principal early signs of CSVT on non-contrast-enhanced MRI are the
combination of absence of a flow void with alteration of signal intensity in the
dural sinus. MRI of the brain is suggestive of CSVT by the absence of a fluid
void signal in the sinus; T2 hypointensity is suggestive of a thrombus, or a
central isodense lesion in a venous sinus with surrounding enhancement. This
appearance is the MRI equivalent of the CT empty delta sign. An acute venous
thrombus may have a hypointense signal that mimics a normal flow void. Thus,
contrast-enhanced MRI and either CTV or MRV may be necessary to establish
a definite diagnosis. The secondary signs of MRI may show similar patterns to
CT, including cerebral swelling, oedema, and/or haemorrhage. Occasionally,
Diffusion-Weighted Imaging (DWI) and perfusion-weighted MRI may assist
in making the diagnosis. DWI may show high signal intensity as restricted dif-
fusion and perfusion-weighted MRI with prolonged transit time.
CT Venography: CTV can provide a rapid and reliable modality for detecting
CVT. CTV is much more useful in subacute or chronic situations because of the
varied density in thrombosed sinus. Because of the dense cortical bone adjacent
to dural sinus, bone artifact may interfere with the visualisation of enhanced
dural sinus. CTV is at least equivalent to MRV in the diagnosis of CVT.

Magnetic Resonance Venography


The most commonly used MRV and contrast-enhanced magnetic resonance.
Phase-contrast MRI is used less frequently, because defining the velocity of
the encoding parameter is both difficult and operator-dependent. The 2-di-
mensional TOF technique is the most commonly used method currently for
the diagnosis of CVT, because 2-dimensional TOF has excellent sensitivity to
slow flow compared with 3-dimensional TOF.
Invasive cerebral angiographic procedures are less commonly needed to estab-
lish the diagnosis of CVT given the availability of MRV and CTV.
Transfontanellar ultrasound may be used to evaluate pediatric patients, includ-
ing newborn or young infants with open anterior or posterior fontanelles.
Ultrasound, along with transcranial Doppler, may be useful to support the
diagnosis of CVT and for ongoing monitoring of thrombus and parenchymal
changes.

MANAGEMENT
Treatment of CSVT has historically involved general supportive or sympto-
matic measures, such as hydration, anti-biotics for septic cases, control of sei-
zure activity with anti-convulsants, and measures aimed at decreasing intrac-
ranial pressure.

154
Case 26

There are currently no well-designed clinical trials in children to support acute


or chronic antithrombotic therapy with anticoagulants or antiplatelet agents
once the diagnosis of CSVT is made.
Treatment regimens vary between centres, but many older infants and chil-
dren receive anticoagulation in the acute setting with parenteral unfractionated
heparin, subcutaneous Low Molecular Weight Heparin (LMWH), or oral war-
farin (Coumadin). Some centres prefer to use unfractionated heparin acutely,
as the effects of heparin can be reversed if intracranial haemorrhage occurs.
This regimen is often followed by chronic anticoagulation with LMWH or
Coumadin for 3 to 6 months.
Anticoagulation should be carefully monitored, with Activated Partial
Thromboplastin Time (APTT) for unfractionated heparin, anti-Xa for
LMWH, or international normalised ratio for Coumadin, to achieve adequate
levels for efficacy while preventing over dosage. However, anticoagulation
may be terminated sooner than this if recanalization of the affected vessel(s) is
demonstrated on follow-up neuroimaging with MR or CT venography.

FOLLOW-UP
All children with CSVT require close monitoring for neurologic and ophthal-
mologic symptoms and signs related to increased intracranial pressure and op-
tic nerve compression. As visual impairment and failure may go undetected by
parents, particularly in nonverbal children, ophthalmology follow-up is war-
ranted in the first year after diagnosis. Persistent headache, nausea, or vomiting
(particularly if nocturnal or early morning) mandate repeat neuroimaging to
exclude hydrocephalus, CSVT propagation, and/or recurrence.
Chronically elevated intracranial pressure may respond to treatment with ster-
oids or acetazolamide, or may require lumbo-peritoneal shunting.
Follow-up neuroimaging with MR or CT venography should be undertaken
in the acute phase and during the first year of follow-up to look for evidence
of extension or persistence or recanalization of venous occlusion, or the devel-
opment of venous stenosis. Some centres perform this at 3, 6, and 12 months
after diagnosis.

REFERENCES
1. Viera JP, Luis C, Monteiro JP, et al. Cerebral sinovenous thrombosis in children:
clinical presentation and extension, localization and recanalization of thrombosis.
Eur J Paediatr Neurol; 2010;14:80-5.
2. Wasay M., Dai A.I., Ansari M. Cerebral venous sinus thrombosis in children: a
multicentre cohort from the United States. J Child Neurol. 2008;23:26-31.
3. Kenet G., Kirkham F., Niederstadt T. Risk factors for recurrent venous thrombo-
embolism in the European collaborative paediatric database on cerebral venous
thrombosis: a multicentre cohort study. Lancet Neurol. 2007;6:595-603.
4. Sébire G., Tabarki B., Saunders D.E. Cerebral venous sinus thrombosis in chil-
dren: risk factors, presentation, diagnosis and outcome. Brain. 2005;128:477-489.

155
Child Neurology

5. Kenet G., Waldman D., Lubetsky A. Paediatric cerebral sinus vein thrombosis: a
multi-centre, case-controlled study. Thromb Haemost. 2004;92:713-718.
6. deVeber G., Andrew M. The Canadian Paediatric Ischemic Stroke Study group.
The epidemiology and outcome of sinovenous thrombosis in pediatric patients. N
Engl J Med. 2001;345:417-423.
7. Narayanan J.T., Murthy J.M. Non-convulsive status epilepticus in a neurological
intensive care unit: profile in a developing country. Epilepsia. 2007;48:900-906.
8. Cakmak S., Derex L., Berruyer M. Cerebral venous thrombosis: clinical outcome
and systematic screening of prothrombotic factors. Neurology. 2003;60:1175-1178.
9. Wong I., Kozak F.K., Poskitt K. Pediatric lateral sinus thrombosis: retrospective
case series and literature review. J Otolaryngol. 2005;34:79-85.
10. Royal College of Physicians Paediatric Stroke Working Group. Stroke in Child-
hood: clinical guidelines for diagnosis, management and rehabilitation. Royal Col-
lege of Physicians, London, November, 2004. Available at: http://www.rcplondon.
ac.uk/pubs/books/childstroke/childstroke_guidelines.pdf. Accessed March 10,
2010.
11. Roach E.S., Golomb M.R., Adams R. Management of stroke in infants and chil-
dren: a scientific statement from a Special Writing Group of the American Heart
Association Stroke Council and the Council on Cardiovascular Disease in the
Young. Stroke. 2008;39:2644-2691.

156
Case 27
A 6-year boy was brought to the outpatient department with a history of
seizures that started at around the age of 4 years. He was born of a non-
consanguineous couple and had normal antenatal, perinatal and developmental
histories. By the time when he was 4 years of age, his parents noticed brief jerky
movements affecting both upper and lower limbs, individually and at times
simultaneously. These increased in frequency and severity, over 2-3 months
and became a daily occurrence. By this time, he also started having episodes
of abrupt loss of contact with his surroundings with prolonged staring (3-5
minutes) and few eye blinks. At times, he also had abnormal behaviour with
irrelevant talking, aggression and he also started to fall. His falls were at times
secondary to a sudden jerky movement throwing him forward while at other
times he slumped to the ground usually preceded by a jerky movement. There
were no recognisable precipitating factors such as a hot bath, cold, sleep or
sleep deprivation, fever, infection, bright lights or noise for any of the seizure
types. No other family members were affected.
On physical examination, he had a normal head size, and no neurocutaneous
stigmata. He was noted to have hyperactivity and decreased attention span.
Subtle autistic features were also observed. Cranial nerves, motor, sensory
and cerebellar examinations were unremarkable. The rest of the general and
systemic examination was normal. An EEG was done after and the findings
were generalised spike and wave discharges on a slow background.

Answer the following questions:


• Give the most likely diagnosis.
• Briefly summarise this case.
• Provide a differential diagnosis.
• Discuss an appropriate diagnostic work-up.
• Discuss the management of this patient.

157
Child Neurology

DIAGNOSIS
Doose syndrome (Myoclonicastatic epilepsy)

SUMMARY
A 6-year-old previously healthy boy was brought to the neurology outpatient
with a history of seizures that started at around the age of 4 years. He devel-
oped seizures of different semiology over a two-year period. At times, he also
had abnormal behaviour with irrelevant talking, aggression and he also started
to fall. Subtle autistic features were also observed. EEG findings were general-
ised spike and wave discharges on a slow background.

DIFFERENTIAL DIAGNOSIS
• This child has a diagnosis of Myoclonic Astatic Epilepsy (MAE), also
known as Doose syndrome; named after the initial describing author.
MAE is classified as a generalised epilepsy, which develops in normal chil-
dren between the ages 2 to 3 years (rarely as early as at 1 year of age).
MAE is more common in boys than in girls by about a 2:1 to 3:1 ratio.
It is characterised by multiple seizure types, predominantly myoclonic,
astatic (drop attacks), myoclonic-astatic seizures, generalised tonic-clonic
seizures, absences, and myoclonic absences seizures. Myoclonic-astatic
seizures are the characteristic seizure type in this syndrome and typically
consist of a symmetric myoclonic jerk followed by atonia causing a fall, if
standing. Generalised tonic-clonic seizures usually herald the syndrome,
and every child with MAE will develop myoclonic-astatic seizures. Rarely,
nocturnal generalised tonic seizures may develop.
The differential diagnosis includes severe myoclonic epilepsy of infancy
(SMEI; also known as Dravet syndrome), myoclonic Lennox–Gastaut
Syndrome (LGS), and progressive myoclonic epilepsies. The differen-
tiation between these disorders can be difficult and is based on develop-
mental status prior to seizures, age of onset, seizure types and EEG pat-
terns, as well as the absence of metabolic findings and a normal Magnetic
Resonance Imaging (MRI).
• Myoclonic LGS is the most significant differential diagnosis. There are
several differences that should help distinguish between the two syn-
dromes. Children with LGS do not have a normal developmental history
before the onset of the seizures. Tonic seizures predominate, and they are
partial with secondary generalisation when viewed on EEG. Atypical ab-
sences are prominent with LGS, which are not seen with MAE. Children
with LGS also may have partial discharges and activation of generalised
discharges during slow-wave sleep, which is not a characteristic of MAE.
LGS is classically associated with slow spike–wave activity on EEG, al-
though this activity may occasionally be seen in MAE. Both LGS and
Doose syndrome are associated with multiple seizure types. Like Doose

158
Case 27

syndrome, LGS is classified by the International League Against Epilepsy


as an electroclinical syndrome and an epileptic encephalopathy (previous-
ly as a symptomatic generalised epilepsy); however, magnetic resonance
imaging abnormalities are more common in individuals with LGS. A high
rate of seizures and EEG traits resembling those seen in Doose syndrome
have also been found in the relatives of affected children. In Doose syn-
drome, individuals have typically normal cognition before the onset of
seizures (and may maintain normal cognition); however, in LGS, there
is often cognitive delay from the start. Tonic seizures, although seen in
both LGS and Doose syndrome, occur while awake and asleep in LGS but
only infrequently during sleep in individuals with Doose syndrome. EEG
findings also differ in the two epilepsy syndromes. LGS is characterised
by electrical status epilepticus during slow-wave sleep, whereas normal
background activity is more likely in Doose syndrome. Considering all
of these similarities, it is possible that the two syndromes are different
manifestations of a single epilepsy syndrome, with Doose syndrome at
the mild end of the spectrum and LGS at the more severe end. Some have
theorised that other epilepsy syndromes may be on a spectrum of a single
disorder, including, most notably, benign epilepsy with centrotemporal
spikes and Landau–Kleffner syndrome.
• SMEI or Dravet syndrome is also generalised epilepsy, but can be dif-
ferentiated from MAE by an earlier onset of seizures (in the first year of
life), often associated with fever. Although myoclonic seizures predomi-
nate, they may also have generalised tonic–clonic convulsions and hae-
miconvulsions. Children with SMEI often have a defect in the Sodium
Channel Neuronal gene (SCN1A). The cognitive outcome is almost uni-
versally affected negatively, although approximately half of the children
with MAE have “normal” cognitive function once their seizures abate
Occasionally, progressive encephalopathies (e.g., mitochondrial disor-
ders, late infantile neuronal ceroid lipofuscinosis) may mimic the clinical
course, but in general, myoclonic seizures predominate. These disorders
may also be distinguished through biochemical or other testing that iden-
tifies the underlying etiology.

DIAGNOSTIC WORK-UP
• A good history focusing on the development and the seizure types as they
emerge is essential to make the correct diagnosis. Video-EEG monitoring
can be very helpful in documenting the seizure types. Initial EEGs are
often normal or may show a characteristic bi-central or parietal theta be-
fore the epileptiform discharges emerge. Generalised 2-3 Hz or irregular
polyspike discharges then evolve. Myoclonic seizures are associated with a
burst of 2-4 Hz spike–waves or polyspike activity and involved symmetric
myoclonus, most often involving the proximal upper extremities. Partial
discharges on initial EEG are exclusion criteria, as should be onset of sei-
zure before age 1 and daytime tonic seizures.

159
Child Neurology

• In younger individuals, the EEG may show continuous irregular activity,


which looks similar to hypsarrhythmia. During status epilepticus, rhythms
consisting of continuous spike–wave activity with interposed slow waves
can be seen. This type of activity can lead to clinically unpredictable myo-
clonus occurring in multiple parts of the individual’s body.
• Children with MAE often become encephalopathic when their seizures
are frequent. Progressive encephalopathies and myoclonic seizures due
to inborn errors of metabolism should be ruled out by metabolic screen-
ings (serum lactate and pyruvate, plasma amino acids, urine organic acids,
and acylcarnitine esters). MRI/Magnetic Resonance Spectroscopy (MRS)
imaging is normal in MAE and remains so during their disease course.
• Genetics play an important role in Doose syndrome and may become an
additional method for differentiating it from other disorders. Individuals
with Doose syndrome were some of the first to be diagnosed with Sodium
Channel Neuronal type 1 alpha subunit (SCN1A) mutations within the
Generalised Epilepsies with Febrile Seizures plus (GEFS+) disorders. In
the recent issue of the American Journal of Human Genetics, the true
gene for Doose syndrome has been discovered and that is SLC6A1 (GAT-
1). SLC6A1 is the first proper gene for Doose syndrome. Doose syndrome
is likely to be genetically heterogeneous and in contrast to Dravet syn-
drome, caused by various different genes.

MANAGEMENT
• MAE is a childhood seizure disorder that is often not easily controlled
by conventional medications. Since there are varying degrees of sever-
ity of the condition, some children may easily respond to the prescribed
first line AEDs (Anti-Epileptic Drugs). Others may need second or even
third medications added to achieve complete control, but there will still
be some children who fail to respond even to this level of therapy. In some
cases a polytherapy regime (more than one medication used at a time) or
even each medication used on its own, may actually lead to the onset of
increased or paradoxical seizures.
• The optimal treatment of MAE is not known and Doose syndrome is
historically described as difficult to treat. There has been no randomised
clinical trial comparing our current treatment options. Carbamazepine,
phenytoin, and vigabatrin have all been reported to worsen seizures in
Doose syndrome
• Anti-epileptic drugs that have shown promise in small clinical series in-
clude the following: VPA, ethosuximide, topiramate, lamotrigine, and le-
vetiracetam. These Anti-epileptic Drugs (AEDs) are often combined with
benzodiazepines, though side effects and tolerance may limit their utility.
Various studies over recent years have shown that the ketogenic diet is ex-
tremely effective in controlling seizures in children with MAE, and is now
considered one of the most successful treatments for the disorder. Some
children require a combination of the ketogenic diet as well as medication,

160
Case 27

others respond completely to the diet alone. KD, a 90% fat, 7% protein
and 3% carbohydrate diet that mimics the metabolic changes that occur
with fasting, has shown promising efficacy in the MAE population, offer-
ing 30-50% seizure freedom.
• Steroids have also been successfully used as treatment for MAE in some
children, either with or without accompanying AEDs. Oral prednisone/
prednisolone has also shown tremendous success in seizure control, ei-
ther as a therapy on it’s own, or as a maintenance/weaning agent following
a course of ACTH.
• Long-term prognosis in the MAE group is variable, and ranges from sei-
zure freedom with normal development to severe retardation and contin-
ued refractory epilepsy. Doose syndrome may have a favorable or unfa-
vorable prognosis. It is not usually possible to predict the outcome in the
first year of disease, either clinically or from the EEG findings. However,
disease progression resulting in episodes of status epilepticus, including
tonic vibratory seizures and myoclonic status, as well as cognitive decline
reflects an unfavourable prognosis.

REFERENCES
1. Doose H, Gerken H, Leonhardt R, Volzke E, Volz C. Centrencephalic myoclonic–
astatic petit mal. Clinical and genetic investigations. Neuropediatrie 1970; 2: 59-78.
2. Berg AT, Berkovic SF, Brodie MJ, et al. Revised terminology and concepts for or-
ganization of seizures and epilepsies: report of the ILAE commission on classifica-
tion and terminology, 2005-2009. Epilepsia 2010; 51: 676-85.
3. Neubauer BA, Hahn A, Doose H, Tuxhorn I. Myoclonic–astatic epilepsy of early
childhood–definition, course, nosography and genetics. Adv
4. Commission on Classification and Terminology of the International League
Against Epilepsy. Proposal for revised classification of epilepsies and epileptic syn-
dromes. Epilepsia 1989; 30: 389-99.
5. Ewen J, Comi A, Kossoff E. Myoclonic–astatic epilepsy in a child with Sturge–We-
ber syndrome. Pediatr Neurol 2007; 36: 115-7.
6. Oguni H, Fukuyama Y, Tanaka T, et al. Myoclonic–astatic epilepsy of early child-
hood–clinical and EEG analysis of myoclonic–astatic seizures, and discussions of
the nosology of the syndrome. Brain Dev 2001; 23: 757-64.
7. Oguni H, Tanaka T, Hayashi K, et al. Treatment and long-term prognosis of myo-
clonic–astatic epilepsy of early childhood. Neuropediatrics 2002; 33: 122-32.
8. Doose H. Myoclonic–astatic epilepsy. Epilepsy Res 1992; 6 (Suppl.) 163-8.
9. Dimova P, Yordanova I, Bojinova V, Jordanova A, Kremenski I. Generalized epi-
lepsy with febrile seizures plus: Novel SCN1A mutation. Pediatr Neurol 2010;
42: 137-40.
10. Kilaru S, Bergqvist AG. Current treatment of myoclonic astatic epilepsy: clinical
experience at the Children’s Hospital of Philadelphia. Epilepsia 2007; 48: 1703-7.

161
28 Case
A 5-year-old boy was brought to our hospital with complaints of slurring
of speech and difficulty in swallowing and fever for 3 days. He was
evaluated at emergency and his GCS was 10/15.
On physical examination, he was in altered sensorium; tone was increased
in upper and lower limb with brisk deep tendon reflexes. Plantars were up
going and cranial nerve examination was normal. No meningeal sign was pre-
sent. He was started on IV anti-biotic (ceftriaxone), acyclovir and supportive
measures with suspicion of meningoencephalitis. His problem started with
cough and cold like symptoms followed by fever three days later. On fourth
day of fever the child had one episode of brief generalised seizure for which
he was admitted in the intensive care unit of a local hospital and managed.
MRI of the brain was done which was unremarkable except for mildly dilated
lateral and third ventricle. CSF cytobiochemistry was normal. Gradually he
improved and was discharged in a stable condition after 6 days. Two days after
discharge he again developed mild fever along with slurring of speech and
difficulty in swallowing for which he was brought to our hospital. Initial in-
vestigations revealed Hb-11.8 gm/dl, TLC-15200/mcl, platelet-3.13 lac. RFT
and LFT were normal. CRP was 17 mg/L, Typhi dot, Widal test, Blood C/S
and Malarial antigen tests were negative. MRI of the brain was done which
showed inhomogeneous area of increased signal in T2 weighted images in ba-
sal ganglia, pons, midbrain, medulla, thalami, and frontal cortex. Fundoscopy
was normal. Lumbar puncture revealed CSF cell count of 3 cells (100% lym-
phocytes), Protein-32 mg/dl and Glucose-56 mg/dl (capillary blood glucose
was 86 mg/dl) LDH-12 U/L. CSF Bacterial antigen test for causative agents
for encephalitis viz; Streptococcus Group B, H. influenzae b, S. pneumoni-
ae, N. meningitidis, E. coli, herpes IgM anti-bodies and herpes PCR were
negative. EEG revealed generalised cerebral dysfunction with no epileptiform
discharge.

Answer the following questions:


• Give the most likely diagnosis.
• Briefly summarise this case.
• Provide a differential diagnosis.
• Discuss an appropriate diagnostic work-up.
• Discuss the management of this patient.

162
Case 28

DIAGNOSIS
Acute Disseminated Encephalomyelitis (ADEM)

SUMMARY
A 5-year-old boy was brought to the hospital with complaints of altered sen-
sorium, difficulty in swallowing and fever for 3 days. He had been treated for
meningoencephalitis at a primary hospital. Two days later after discharge, the
patient developed slurring of speech and difficulty in swallowing with fever for
which he was admitted again in hospital. Lumbar puncture revealed CSF cell
count of 3 cells (100% lymphocytes), Protein-32 mg/dl and Glucose-56 mg/dl.
First MRI of the brain done earlier was unremarkable except for mildly dilated
lateral and third ventricle. Second MRI of the brain showed inhomogeneous
area of increased signal in T2 weighted images in basal ganglia, pons, midbrain,
medulla, thalami, and frontal cortex. So clinical information and radiological
findings were suggestive of ADEM.

DIFFERENTIAL DIAGNOSIS
Acute Disseminated Encephalomyelitis (ADEM) is classically described
as a uni-phasic syndrome occurring in association with an immunisation or
vaccination (post-vaccination encephalomyelitis) or systemic viral infection
(para-infectious encephalomyelitis). Pathologically, there is perivascular in-
flammation, oedema, and demyelination within the CNS. Clinically, patients
present with the rapid development of focal or multifocal neurologic dysfunc-
tions. In its classic form, ADEM is a monophasic disease. However, relapsing
forms of ADEM have been described and can be characterised as either re-
current ADEM or multiphasic ADEM, depending upon the clinical features.
Relapsing variants of ADEM can occur more than three months after the ini-
tial event and more than one month after discontinuation of steroids.
• Recurrent ADEM describes patients with a relapse of similar symptoms
and MRI findings
• Multiphasic ADEM refers to patients presenting with new neurologic
symptoms and MRI lesions
The hallmark clinical feature of the disorder is the development of a focal or
multifocal neurologic disorder following exposure to a virus or vaccination. In
some, but not all cases, a prodromal phase of several days of fever, malaise, and
myalgia occur. The onset of the CNS disorder is usually rapid (abrupt or up to
several hours), reaching peak dysfunction within several days. Initial features
include encephalopathy ranging from lethargy to coma, seizures, focal and or
multifocal signs reflecting cerebral (hemiparesis), brain stem (cranial nerve
palsies), and spinal cord (paraparesis) involvement. Other reported findings
include movement disorders and ataxia. Each of these findings may occur as
isolated features or in various combinations. Multifocal CNS lesions are gen-
erally evident on MRI that are initially indistinguishable from those observed
in MS. After several weeks, ADEM lesions show at least partial resolution
without the appearance of new lesions, unlike MS.

163
Child Neurology

In a child who presents with neurologic abnormalities, including signs of en-


cephalitis, nonspecific CSF findings, and MRI evidence of white matter ab-
normalities, other inflammatory demyelinating disorders should also be con-
sidered. The differential diagnosis of ADEM includes acute Meningitis, acute
Viral Encephalitis, Autoimmune Encephalitis and acute Multiple Sclerosis.
Differentiation of these diseases is not easy, especially in the early stages. In viral
encephalitis the CSF is often abnormal and a rise in specific viral anti-body may
occur. To distinguish ADEM from multiple sclerosis in the initial phase may be
more difficult. Magnetic resonance imaging and CSF examination may help.
• Multiple sclerosis: The most important alternative diagnosis to ADEM
is Multiple Sclerosis (MS). At the initial presentation, the two disorders
cannot be distinguished with absolute certainty. Because subsequent at-
tacks of MS in children may not occur for months or years, prolonged
follow-up is required to establish a diagnosis. Certain clinical features may
be helpful in supporting the diagnosis of ADEM or MS. ADEM typically
follows a prodromal viral illness, while MS does not. ADEM patients of-
ten have fever and stiff neck, which is unusual in MS.
• ADEM usually produces widespread CNS disturbance with impaired
consciousness, while MS typically is mono symptomatic (e.g., optic neu-
ritis or a subacute myelopathy) and has a relapsing, and remitting course.
Optic neuritis is more often bilateral when it occurs in ADEM and unilat-
eral in MS. Myelopathy is usually complete and accompanied by areflexia
in ADEM and partial in MS. Ataxia is a common presenting feature of
ADEM, but rare in MS. MRI features may also be helpful in distinguish-
ing ADEM from MS, although complete differentiation is not possible on
the basis of a single examination. ADEM usually has more lesions than
MS, with larger bilateral but asymmetrical white matter abnormalities.
Lesions tend to have poorly defined margins in ADEM and but better-
defined ones in MS. The age of the lesions is usually similar in ADEM and
different in MS. Thalamic lesions are common in ADEM and are rare in
MS. Periventricular lesions are less common in ADEM than MS.
• Devic’s syndrome: Devic’s syndrome (also known as neuromyelitis op-
tica) is a combination of bilateral optic neuropathy and traverse myelitis
without any other neurologic signs. Optic neuritis and transverse myelitis
can occur concurrently, or one may follow the other. Unlike ADEM there
are no white matter lesions in the brain. This condition is closely related
to MS, although the disorders might be differentiated by application of
stricter diagnostic criteria. Patients with Devic’s syndrome have a poorer
prognosis for recovery of vision. The myelopathy tends to be more severe
than MS, with less likelihood of recovery. The neuropathologic features at
autopsy are those of a much more severe necrotic lesion of the cord rather
than the incomplete demyelination seen in MS. A serum IgG autoanti-
body (NMO-IgG) can be a specific marker for neuromyelitis optica and
a further aid in the distinction of this disorder from MS. A case-control
study reported that NMO-IgG had good sensitivity and specificity for the
detection of neuromyelitis.

164
Case 28

• Central nervous system angiitis: Central nervous system angiitis, also


known as primary angiitis of the central nervous system, is a rare dis-
order in children that can have a similar presentation to demyelinating
disease, including multifocal T2-hyperintensities on the MRI. Diffusion
weighted MRI sequences may demonstrate restricted diffusion in areas of
acute ischaemic injury. Patients may present with clinical features similar
to ADEM, such as multifocal neurologic impairments, headaches, focal
seizures, and behaviour changes. Cerebral angiography may demonstrate
vascular beading and intraluminal narrowing. Meningeal biopsy is some-
times required to confirm the diagnosis.
• Mitochondrial disease: Mitochondrial disease may present acutely in
the setting of concurrent infection. Typical features on brain MRI include
increased (bright) T2 signal in the basal ganglia or predominately sym-
metric involvement of the parieto-occipital region. Other features of mi-
tochondrial disorders include elevation of serum and cerebrospinal fluid
lactate levels. Symmetry of imaging abnormalities should prompt strong
consideration for an underlying metabolic disease.
• Leukodystrophies: Leukodystrophies are rare disorders that may oc-
casionally be confused with multiple sclerosis or acute disseminated en-
cephalomyelitis. A white matter disease, typically symmetric on MRI im-
ages accompanied with clinical history is the hallmark of these conditions.

DIAGNOSTIC WORK-UP
Laboratory Studies
• Platelet counts
unts are elevated in a number of children with ADEM.
Sedimentation rates are elevated in a third of patients.
• The lumbar puncture is an essential aspect of Acute Disseminated
Encephalomyelitis (ADEM) work-up. It assists in distinguishing ADEM
from various forms of meningoencephalitis, searching especially for the
evidence of various bacteria, viruses, or other infectious agents that may
produce an infectious form of meningoencephalitis.
• Modest-to-moderate elevation of CSF white and red blood cell counts
may be found in childhood ADEM. Red blood cells may be increased due
to modest degrees of AHLE (Acute Haemorrhagic Leukoencephalitis).
Results of CSF immune profile testing (e.g., CSF: serum immunoglobu-
lin G [IgG] index, CNS IgG synthetic rate, oligoclonality) employing age-
appropriate normative data are positive in fewer than 10% of prepubertal
children with ADEM.
• Positivity of studies for CSF oligoclonal bands and immunoglobulin el-
evation favours the diagnosis of MS in individuals younger than 20 years
with first or recurrent bouts of acute CNS demyelinating illness. In such
instances it remains incumbent on those evaluating such individuals to ex-
clude non-MS illnesses with specific biological markers such as systemic
lupus, sarcoid, neuromyelitis optica and alike illnesses.
• CSF myelin basic protein concentration level, reflecting demyelination, is
frequently elevated in ADEM.

165
Child Neurology

Imaging Studies
• The CT scan low-density abnormalities are found in more than half of
childhood or adolescent ADEM cases, but this technique is far less sensi-
tive than MRI for the disclosure of extent and number of lesions.
• T2-weighted and Fluid Attenuated Inversion Recovery (FLAIR), diffu-
sion MRI techniques disclose characteristic high-signal lesions in virtually
all cases of ADEM. T2/FLAIR images are particularly important in the
evaluation of young individuals presenting with inflammatory CNS de-
myelinating illnesses that may be ADEM or MS. Apparent diffusion coef-
ficient maps show high-signal changes consistent with vasogenic oedema.
ADEM lesions are characteristically multiple, bilateral but asymmetric,
and widespread within the CNS.
• The EEG often exhibits a disturbance of normal sleep rhythms and focal
or generalised slowing. Epileptiform discharges are rarely seen in ADEM.
The absence of such abnormalities during the first bout of acute dissemi-
nated demyelinating illness in a child may increase the suspicion for ulti-
mate diagnosis of MS.
• Visual Evoked Potentials (VEP) may prove helpful when optic neuritis is
suspected but not apparent on clinical examination.
• Occasionally, brain biopsy is necessary to distinguish ADEM from other
diagnostic possibilities. The diagnosis of ADEM is confirmed when typical
periventricular demyelinating changes with axonal sparing are observed.

MANAGEMENT
• Acute Disseminated Encephalomyelitis (ADEM) is often treated with
high-dose intravenous corticosteroids, to which it appears to be respon-
sive. One common protocol is 20-30 mg/kg/d of methylprednisolone
(maximum dose of 1 g/d) for 3-5 days. Improvement may be observed
within hours but usually requires several days. An oral taper for 4-6 weeks
or some other interval is sometimes appended.
• The chief alternative therapy is Intravenous Immune Globulin (IVIG). It
is administered as 2 g/kg intravenously as a single dose or over the course
of 3-5 days. IVIG may be preferable in instances where meningoencepha-
litis cannot be excluded, based upon the hypothesis that corticosteroids
might worsen the course of infection.
• There is as yet no convincing evidence that treatment with the combi-
nation of intravenous corticosteroids and IVIG confers any advantage in
such cases, although some clinicians employ this approach.
• Severe ADEM has also been treated, apparently successfully, with such
alternative approaches as (1) combination of intravenous corticosteroids
and IVIG (2) cyclosporine (3) cyclophosphamide or (4) plasma exchange/
plasmapheresis.
• Supportive care includes airway protection in patients with altered men-
tal status and mechanical ventilation if required. Patients with cervical

166
Case 28

myelitis may require mechanical ventilation. Other supportive measures


include; anti-seizure medication in patients with convulsions and correc-
tion of fluid and electrolyte disturbances.

REFERENCES
1. Leake JA, Albani S, Kao AS, et al.. Acute disseminated encephalomyelitis in child-
hood: epidemiologic, clinical and laboratory features. Pediatr Infect Dis J 2004;
23:756.
2. Banwell B, Kennedy J, Sadovnick D, et al.. Incidence of acquired demyelination of
the CNS in Canadian children. Neurology 2009; 72:232.
3. Murthy SN, Faden HS, Cohen ME, Bakshi R. Acute disseminated encephalomy-
elitis in children. Pediatrics 2002.
4. Tenembaum S, Chitnis T, Ness J, et al.. Acute disseminated encephalomyelitis.
Neurology 2007.
5. Brass SD, Caramanos Z, Santos C, et al.. Multiple sclerosis vs acute disseminated
encephalomyelitis in childhood. Pediatr Neurol 2003; 29:227.
6. Fridinger SE, Alper G. Defining encephalopathy in acute disseminated encephalo-
myelitis. J Child Neurol 2014; 29:751.
7. Klein CJ, Wijdicks EF, Earnest F 4th. Full recovery after acute hemorrhagic leu-
koencephalitis (Hurst’s disease). J Neurol 2000; 247:977.
8. Kuperan S, Ostrow P, Landi MK, Bakshi R. Acute hemorrhagic leukoencephalitis
vs ADEM: FLAIR MRI and neuropathology findings. Neurology 2003; 60:721.
9. Mader I, Wolff M, Niemann G, Küker W. Acute haemorrhagic encephalomyelitis
(AHEM): MRI findings. Neuropediatrics 2004; 35:143.
10. Alper G. Acute disseminated encephalomyelitis. J Child Neurol 2012; 27:1408.
11. Callen DJ, Shroff MM, Branson HM, et al.. Role of MRI in the differentiation of
ADEM from MS in children. Neurology 2009; 72:968.
12. Verhey LH, Branson HM, Shroff MM, et al. MRI parameters for prediction of
multiple sclerosis diagnosis in children with acute CNS demyelination: a prospec-
tive national cohort study. Lancet Neurol 2011; 10:1065.
13. Baum PA, Barkovich AJ, Koch TK, Berg BO. Deep gray matter involvement in
children with acute disseminated encephalomyelitis. AJNR Am J Neuroradiol
1994; 15:1275.
14. Krupp LB, Banwell B, Tenembaum S, International Pediatric MS Study Group.
Consensus definitions proposed for pediatric multiple sclerosis and related disor-
ders. Neurology 2007; 68:S7.
15. Krupp LB, Tardieu M, Amato MP, et al. International Pediatric Multiple Sclerosis
Study Group criteria for pediatric multiple sclerosis and immune-mediated central
nervous system demyelinating disorders: revisions to the 2007 definitions. Mult
Scler 2013; 19:1261.

167
29 Case
A 13-year-old boy presented with Generalised Tonic-Clonic Seizures
(GTCS). At the first seizure attack a few months ago, he only suffered
from occasional mild headache. However, during the night before visiting the
ER, he showed inappropriate behaviour, confusion, and disorientation. A few
hours later, he suffered GTCS lasting for 1-2 minutes, and then, remained in
a mild confusional state in the ER.
On physical examination: He was alert and cooperative. The pupils were
equal, round, and reactive to light. His extraocular muscles were intact. His
face was symmetric with normal bulk and tone of muscles with 5/5 strength
throughout. No dysmetria, tremor, dystonia, or choreoathetosis were noted.
Normal sensation for light touch, temperature, and vibration. Normal heels,
toes, and tandem gait. Normal deep tendon reflexes with bilateral plantar
flexor responses.
There were no abnormal findings on the brain MRI and in the CSF analysis.
During the confusion state, on the EEG, there were repetitive, high voltage
generalised polyspikes and wave discharges which subsided after intravenous
injection of lorazepam. Detailed history taking revealed previous occasional
absence-like episodes and myoclonic jerks. The patient’s younger sister
was also suffering from epilepsy since childhood. The ictal EEG’s of the
patient showed continuous bilateral synchronous polyspikes and slow wave
discharges. The epileptic discharges were all abolished on the follow-up EEG.

Answer the following questions:


• Give the most likely diagnosis.
• Briefly summarise this case.
• What is the differential diagnosis.
• Discuss an appropriate diagnostic work-up.
• Discuss the management of this patient.

168
Case 29

DIAGNOSIS
JME (Juvenile Myoclonic Epilepsy)

SUMMARY
A 13-year-old boy presented with Generalised Tonic-Clonic Seizures (GTCS).
At the first seizure attack a few months ago, there was a past history of mild
headache. Detailed history taking revealed previous occasional absence-like
episodes and brief myoclonic jerks. The patient’s younger sister also has epi-
lepsy since childhood. The ictal EEG’s of the patient showed continuous bilat-
eral synchronous polyspikes and slow waves. The epileptic discharges were all
abolished on the follow-up EEG.

DIFFERENTIAL DIAGNOSIS
An afebrile seizure in an adolescent should always raise concern for a pro-
voked seizure, that is, elicited by a trigger, such as sleep deprivation, with-
drawal from benzodiazepines, or use of illicit drugs such as amphetamines or
cocaine. Primary generalised epilepsies that first manifest with seizures during
the teenage years need to be considered, such as Juvenile Myoclonic Epilepsy
(JME) or Juvenile Absence Epilepsy (JAE). Childhood absence epilepsy may
present with a generalised seizure, but it is unusual for associated staring spells
to go undetected by parents and teachers for years, whereas typically seizures
remit during puberty. A seizure can also occur owing to an underlying struc-
tural lesion, such as a cortical dysplasia, a vascular malformation, or a tumour,
which need to be remembered even when acute provocation seems most likely.

DIAGNOSTIC WORK-UP
• As always, the key to the diagnosis lies in the history. Myoclonic seizures
have a propensity to occur in the early morning hours. In combination
with a generalised tonic–clonic seizure, the most likely diagnosis is JME.
Typical EEG findings confirm this suspicion, that is, 4-6 Hz spike–wave
and polyspike–wave complexes, occurring spontaneously out of an other-
wise normal background
• In approximately 30% of patients, these discharges are triggered by photic
stimulation, and some will appear after provocation by sleep deprivation
or drinking caffeinated beverages.
• During a myoclonic seizure, the myoclonic jerk corresponds to a poly-
spike–wave discharge. In contrast, other benign myoclonic jerks occur
when an individual is drifting off to sleep (hypnagogic myoclonic jerk or
sleep myoclonus) or when startled. However benign myoclonic seizures
are not accompanied by epileptiform activity on EEG
• Neuroimaging in a typical case is not required, because the brain is struc-
turally normal in these patients. However, Magnetic Resonance Imaging
(MRI) of the brain is sometimes done to rule out other reasons for seizures

169
Child Neurology

that may mirror the presentation. It should be noted that the EEG of JME
patients may exhibit focal features, thus suggesting a structural abnormal-
ity or lesion in the brain.
• A toxicology screen should be done, if there is a suspicion that drugs
might be involved.

MANAGEMENT
• Patients with JME have greater than 90% chance of experiencing recurrent
seizures. Therefore, lifelong treatment with Antiepileptic Drugs (AEDs)
and avoidance of possibly provoking factors (e.g., alcohol, illicit drugs,
sleep deprivation, and flickering lights) are recommended. Therapeutic
agents effective against seizures in generalised epilepsies should be used
and are usually very effective in controlling seizures.
• Currently, lamotrigine, Levetiracetam, Topiramate, Zonisamide, and
Valproic Acid (VPA, in child), are considered first-line treatment. VPA is
typically avoided in young women because of cosmetic and hormonal ad-
verse effects, teratogenicity, and an increased likelihood of cognitive prob-
lems in children who were exposed to it in utero.
• Long-acting benzodiazepines, such as clonazepam or clobazam, are also
effective, but have the potential risk for tolerance and addiction. Most pa-
tients (80-90%) are controlled on a single agent (i.e. monotherapy), but
some require combination therapy.
• Several drugs used for partial-onset seizures can exacerbate seizures and
even cause status epilepticus in patients with JME (e.g., phenytoin, car-
bamazepine, oxcarbazepine, gabapentin, tiagabine, and vigabatrin), and
should not be used.
• Studies on the natural history of the disorder indicate that virtually all pa-
tients will have recurrent seizures after anti-epileptic medication is discon-
tinued. Unfortunately, this mandates that JME patients need to be treated
for life sometimes. However, despite this poor prognostic fact, they have
otherwise normal intelligence and life expectancy. Generally speaking, it
is estimated that a mother with idiopathic generalised epilepsy has about a
10% risk of having a child with generalised epilepsy.

REFERENCES
1. Jayalakshmi SS, Srinivasa Rao B, Sailaja S. Focal clinical and electroencephalo-
graphic features in patients with juvenile myoclonic epilepsy. Acta Neurol Scand
2010; 122:115.
2. Murthy JM, Rao CM, Meena AK. Clinical observations of juvenile myoclonic epi-
lepsy in 131 patients: a study in South India. Seizure 1998; 7:43.
3. Berg AT, Berkovic SF, Brodie MJ, et al. Revised terminology and concepts for or-
ganization of seizures and epilepsies: report of the ILAE Commission on Classifi-
cation and Terminology, 2005-2009. Epilepsia 2010; 51:676.

170
Case 29

4. Cossette P. Channelopathies and juvenile myoclonic epilepsy. Epilepsia 2010; 51


Suppl 1:30.
5. Badawy RA, Macdonell RA, Jackson GD, Berkovic SF. Why do seizures in general-
ized epilepsy often occur in the morning? Neurology 2009; 73:218.
6. Vollmar C, O’Muircheartaigh J, Barker GJ, et al.. Motor system hyperconnectivity
in juvenile myoclonic epilepsy: a cognitive functional magnetic resonance imaging
study. Brain 2011; 134:1710.
7. Tae WS, Joo EY, Han SJ, et al.. CBF changes in drug naive juvenile myoclonic epi-
lepsy patients. J Neurol 2007; 254:1073.
8. Camfield CS, Camfield PR. Juvenile myoclonic epilepsy 25 years after seizure on-
set: a population-based study. Neurology 2009; 73:1041.
9. Gélisse P, Crespel A. Mixed myoclonic-absence status epilepticus in juvenile myo-
clonic epilepsy. Epileptic Disord 2015; 17:95.
10. Martínez-Juárez IE, Alonso ME, Medina MT, et al. Juvenile myoclonic epilepsy
subsyndromes: family studies and long-term follow-up. Brain 2006; 129:1269.
11. de Araujo Filho GM, Pascalicchio TF, Lin K, et al.. Neuropsychiatric profiles of
patients with juvenile myoclonic epilepsy treated with valproate or topiramate.
Epilepsy Behav 2006; 8:606.
12. Camfield PR, Camfield C. What criteria do adult and pediatric neurologists use to
diagnose juvenile myoclonic epilepsy? Epilepsy Curr 2011; 12:1.154.
13. Destina Yalçin A, Forta H, Kiliç E. Overlap cases of eyelid myoclonia with absences
and juvenile myoclonic epilepsy. Seizure 2006; 15:359.
14. Shahwan A, Farrell M, Delanty N. Progressive myoclonic epilepsies: a review of
genetic and therapeutic aspects. Lancet Neurol 2005; 4:239.
15. Chowdhury A, Brodie MJ. Pharmacological outcomes in juvenile myoclonic epi-
lepsy: Support for sodium valproate. Epilepsy Res 2016; 119:62.
16. Bodenstein-Sachar H, Gandelman-Marton R, Ben-Zeev B, et al. Outcome of
lamotrigine treatment in juvenile myoclonic epilepsy. Acta Neurol Scand 2011;
124:22.
17. Schneider-von Podewils F, Gasse C, Geithner J, et al.al Clinical predictors of the
long-term social outcome and quality of life in juvenile myoclonic epilepsy: 20-65
years of follow-up. Epilepsia 2014; 55:322.

171
30 Case
A 6-year-old boy presented with episodes of generalised tonic-clonic
seizures for 15 minutes. He also complained of transient formed visual
hallucinations for about three to five minutes before the start of seizures. This
was followed by inattentiveness, marked receptive and expressive aphasia and
hyperkinesia. The child became totally non-interactive with his environment
within a week’s time. The history from his mother indicated that he had
normal speech development until 4 years of age. There was no family history
of epilepsy, mental retardation or developmental speech disturbances.
On physical examination, he was a well-appearing and attentive boy but
hyperactive and communication ability was disturbed and interrupted. It was
quickly apparent, however, that he had profound language problems. He was
unable to follow even one-step verbal commands, but he was very observant
to any physical cues to express what the examiner requested, and he would do
his best to comply. When asked what day it was, he responded with “seven.”
His mother indicated that he was guessing that the examiner had asked him to
provide his age. His neurological examination, including head circumference,
was normal. Audiology showed normal hearing by threshold testing. A speech
pathology evaluation found him to have profound deficits in both receptive
and expressive language. Cranial nerves were normal and there were no long
tract signs.
EEG, done in sleep state, showed generalised epileptiform activity. CT
and MRI of the brain were normal. 99m-Technetium brain Single Photon
Emission Computed Tomography (SPECT) study showed left mesial
temporal hypo-perfusion.

Answer the following questions:


• What is the most likely diagnosis.
• Briefly summarise the case.
• Provide the differential diagnosis.
• Discuss an appropriate diagnostic work-up.
• Discuss the management.

172
Case 30

DIAGNOSIS
Landau-Kleffner syndrome (Acquired epileptic aphasia)

SUMMARY
This boy was a previously healthy, developmentally normal child who pre-
sented with a new onset, unprovoked, generalised tonic-clonic seizures in ad-
dition to subacute regression of receptive and expressive language skills. His
examination was significant for expressive language difficulty and articulation.
EEG done in sleep state, showed generalised epileptiform activity.

DIFFERENTIAL DIAGNOSIS
• There is increasing recognition that a subset of LKS patients will have
continuous or nearly continuous spike–wave discharges, usually diffuse or
generalised, particularly during sleep. This EEG pattern is associated with
an electroclinical syndrome of its own, Continuous Spikes and Waves
During Slow Sleep (CSWS), also known as Electrical Status Epilepticus
in Sleep (ESES). However, the overlap of the clinical features and the lack
of clear or specific diagnostic boundaries need to be explicitly acknowl-
edged here. As with LKS, the pathogenesis behind CSWS is unknown. It
has been proposed that LKS is entirely a localisation-specific subset within
CSWS. However, not all children with LKS are shown to have CSWS
(although sampling issues may account for some of this). Clearly, over-
night EEG studies are more likely to capture CSWS compared to routine
outpatient studies.
• There is also a lesser degree of overlap with another, more common,
epilepsy syndrome of childhood, BECTS. BECTS is characterised most
commonly by simple partial seizures with orofacial motor features, often
easy to control, and associated with surprisingly frequent, high-amplitude
spike and after coming slow-wave complexes from the central (Rolandic)
regions bilaterally. The vast majority of these patients have normal higher
cortical function, including, language. However, a small number may de-
velop aphasia or dysphasic features, and may be found to have abundant
spikes during sleep, or even CSWS. The typical clinical phenotype of
BECTS is easily distinguished from LKS, but patients with features com-
mon to both syndromes do exist (and may be even more common than
the classic LKS phenotype).
• Autism is a neurodevelopmental condition typically appearing before the
age of thirty months. It is characterised by impairments in social commu-
nication and severely restricted interests or repetitive patterns of behav-
iour. Between 30 and 50% of affected children have a history of develop-
mental regression affecting language, social and behavioural functioning,
which usually occurs between 18 and 30 months of age. Thirty to fifty
percent of children with autism have low scores on standardised intel-
ligence tests. The clinical expression and severity of autism varies widely.

173
Child Neurology

The functional outlook for those with autism is often improved with in-
tensive behavioural and developmental interventions. Incidence of epilep-
sy in ASD (Autistic Spectrum Disorder) is higher in a normal population,
however, primary presentation as seizures is less common.
• Rett syndrome is a rare neurodevelopmental disorder due to a known gene
mutation. It occurs almost exclusively in child but can occur very rarely
in child. Infants and children with the disorder typically develop normally
until between 7 and 18 months of age, when they experience developmen-
tal regression affecting purposeful hand movements and verbal communi-
cation. Associated abnormalities typically include slowing of head growth
(acquired microcephaly); development of distinctive, repetitive hand move-
ments such as hand clapping, rubbing, or “wringing”; and loss of control of
voluntary movements required for coordination of walking (gait apraxia).
Affected children also typically develop breathing irregularities, feeding and
swallowing difficulties, growth retardation, and seizures.

DIAGNOSTIC WORK-UP
• The syndrome of LK can be difficult to diagnose and and may be misdiag-
nosed as autism, pervasive developmental disorder, hearing impairment,
learning disability, auditory/verbal processing disorder, attention deficit
disorder, intellectual disability, childhood schizophrenia, or emotional/
behavioural problems.
• Structural imaging studies, including high-resolution MRI, are normal
by standard visual analysis. Functional imaging studies have been more
revealing, though inconsistent, with decrease in metabolism in the tem-
poral regions by FDG-PET. Magnetoencephalography (MEG) localises
spike dipoles to the perisylvian or insular regions in some cases. Many
conditions like demyelination and brain tumours can be ruled out by us-
ing Magnetic Resonance Imaging (MRI). In LKS, Fluorodeoxyglucose
(FDG) and Positron Emission Tomography (PET) scanning can show de-
creased metabolism in one or both temporal lobes. Hypermetabolism has
been seen in patients with acquired epileptic aphasia.
• An EEG test is imperative for the diagnosis. Even though an abnormal
EEG reading is common in LKS patients, a relationship has not been
identified between EEG abnormalities and the presence and intensity of
language problems. In many cases however, abnormalities in the EEG
test precede language deterioration and improvement in the readings has
preceded language improvement (this occurs in about half of all affected
children). The most effective way of confirming LKS is by obtaining over-
night sleep EEGs, including EEGs in all stages of sleep.

MANAGEMENT
• Seizures associated with LKS are treated with standard AED therapy.
There is no data to demonstrate the superiority of one AED over another,
or, indeed, any controlled AED trials for LKS. It must be recognised that

174
Case 30

some AEDs, among them carbamazepine, phenytoin, and tiagabine, may


worsen EEG abnormalities in LKS by unmasking or enhancing general-
ised spike–wave changes.
• Accordingly, patients with seizures associated with LKS, particularly
those with generalised spike–wave features (with or without CSWS) and
generalised seizure types, are probably best treated with “broad-spec-
trum” AEDs, of which Valproic Acid (VPA) is the traditional prototype.
Benzodiazepines can be effective, and are sometimes recommended in
combination with VPA, but their use is limited by sedative effects and by
pharmacological tolerance. As noted earlier, the clinical seizure events are
usually relatively easy to control.
• Corticosteroids are used for LKS, and several small open-label treatment
series have been reported. Although recommended by some authors as
a first-line treatment modality for LKS, it appears that most specialists
would use corticosteroids if seizures and/or abundant spike–wave patterns
fail to respond to AED therapy. Since there are no controlled trials, and it
appears that no two series used the same corticosteroid dosing regimen.
A small number of LKS patients have been reported to have favourable
results following treatment with intravenous immunoglobulins.
• A controversial treatment option involves a surgical technique called mul-
tiple subpial transection in which multiple incisions are made through the
cortex of the affected part of the brain beneath the pia mater, severing the
axonal tracts in the subjacent white matter. The cortex is sliced in parallel
lines to the midtemporal gyrus and perisylvian area to attenuate the spread
of the epileptiform activity without causing cortical dysfunction.
• The long-term natural history for LKS is generally such that patients stop
deteriorating after an interval with active regression, usually within 2-4
years of onset. After the syndrome has “run its course,” most patients im-
disappearance of seizures and improved language
prove with complete disappeara
capabilities. A complete resolution of the aphasic features is not expected,
however, and roughly two-thirds of patients will remain significantly disa-
bled on the scale of language impairment. Patients with earlier onset of
symptoms, and those with a more prolonged active phase, have the poor-
est long-term prognosis.

REFERENCE
1. Proposal for revised clinical and electroencephalographic classification of epileptic
seizures. From the Commission on Classification and Terminology of the Interna-
tional League Against Epilepsy. Epilepsia 1981; 22:489.
2. Proposal for revised classification of epilepsies and epileptic syndromes. Commis-
sion on Classification and Terminology of the International League Against Epi-
lepsy. Epilepsia 1989; 30:389.
3. Berg AT, Berkovic SF, Brodie MJ, et al. Revised terminology and concepts for or-
ganization of seizures and epilepsies: report of the ILAE Commission on Classifi-
cation and Terminology, 2005-2009. Epilepsia 2010; 51:676.

175
Child Neurology

4. Myoclonus and epilepsy in childhood. Commission on Pediatric Epilepsy of the


International League Against Epilepsy. Epilepsia 1997; 38:1251.
5. Carvill GL, Regan BM, Yendle SC, et al. GRIN2A mutations cause epilepsy-apha-
sia spectrum disorders. Nat Genet 2013; 45:1073.
6. Turner SJ, Mayes AK, Verhoeven A, et al.. GRIN2A: an aptly named gene for speech
dysfunction. Neurology 2015; 84:586.
7. Caraballo RH, Veggiotti P, Kaltenmeier MC, et al.. Encephalopathy with status
epilepticus during sleep or continuous spikes and waves during slow sleep syn-
drome: a multicentre, long-term follow-up study of 117 patients. Epilepsy Res
2013; 105:164.
8. Maquet P, Hirsch E, Dive D, et al.. Cerebral glucose utilization during sleep in
Landau-Kleffner syndrome: a PET study. Epilepsia 1990; 31:778.
9. Takeoka M, Riviello JJ Jr, Duffy FH, et al.. Bilateral volume reduction of the supe-
rior temporal areas in Landau-Kleffner syndrome. Neurology 2004; 63:1289.
10. Bourgeois BF, Landau WM. Landau-Kleffner syndrome and temporal cortical vol-
ume reduction: cause or effect? Neurology 2004; 63:1152.
11. Mikati MA, Shamseddine AN. Management of Landau-Kleffner syndrome. Pae-
diatr Drugs 2005; 7:377.
12. Kramer U, Nevo Y, Neufeld MY, et al.. Epidemiology of epilepsy in childhood: a
cohort of 440 consecutive patients. Pediatr Neurol 1998; 18:46.
13. Sánchez Fernández I, Chapman KE, Peters JM, et al.. Continuous Spikes and Waves
during Sleep: Electroclinical Presentation and Suggestions for Management. Epi-
lepsy Res Treat 2013; 2013:583:531.

176
Case 31
A 9-year-old boy, who was a product of a non-consanguineous marriage,
presented with pain in his right knee joint and difficulty in walking
for 4 months, 10 months back, which was treated by many physicians with
some symptomatic improvement. After 4 months of the onset of the pain,
he developed abnormal movements of the right upper and lower limbs as
repetitive purposeless movements, which were progressive and involved all
four limbs, over a two month period. These abnormal movements however,
used to subside during sleep. The patient also developed progressive dysarthria.
His elder brother also had a history of very slowly progressive abnormal
movements and dysarthria. There was no history of measles and convulsions
during early childhood. His developmental milestones were normal.
On examination, his vital signs were found to be normal. There was no pal-
lor, icterus or significant lymphadenopathy. His nervous system examination
revealed dystonia, exaggerated deep tendon reflexes, ankle clonus and a posi-
tive Babinski’s sign. His muscle power was >4/5 in all the limbs. His other
systemic examinations did not reveal any abnormality.
The total serum bilirubin was 0.6 mg/dl (direct bilirubin 0.2 mg/dl), the total
serum protein was 6.8 gm/dl (Albumin 4.2 gm/dl) and the serum transaminas-
es (AST, ALT) and Alkaline Phosphatase (ALP) were 38, 24, and 168 IU/L re-
spectively. His Prothrombin Time (PT) and activated Partial Thromboplastin
Time (aPTT) were within normal limits. Serum ceruloplasmin was 90 mg/L
(normal 180-350 mg/L) and 24 hours urine copper excretion was increased to
130 µg/24 hours (normal 24 hours urine excretion <100 µg). Urine routine
and microscopic examinations were normal. Ultrasonography of the abdo-
men was normal. Magnetic Resonance Imaging (MRI) of the brain, on the T2
weighted axial sequence through the pons revealed the “Face of a miniature
Panda”. The T2 weighted image also revealed hyperintensities in the thalami
and the pontine tegmentum. The Kayser–Fleischer ring (K–F ring) was visible
to the naked eye on both sides, which was confirmed by slit lamp examination.

Answer the following questions:


• Give the most likely diagnosis.
• Summarise the case briefly.
• Provide a differential diagnosis.
• Discuss an appropriate diagnostic work-up.
• Discuss the management of the patient.

177
Child Neurology

DIAGNOSIS
Wilson’s Disease

SUMMARY
A 9-year-old boy presented with pain in his right knee joint and difficulty in
walking for the last 4 to 10 months. After 4 months of the onset of the pain,
he had developed abnormal movements of the right upper and lower limbs
in the form of repetitive purposeless abnormal movements. These abnormal
movements subsided during sleep. The patient also had progressive dysarthria.
The Kayser–Fleischer ring (K–F ring) was visible to the naked eye on both
sides. His serum ceruloplasmin was 90 mg/L (normal 180-350 mg/L) and his
24 hours’ urine copper excretion was increased to 130 µg/24 hours (normal 24
hours urine excretion <100 µg). Magnetic resonance imaging (MRI) of the
brain, on the T2 weighted axial sequence through the pons revealed the (face
of a miniature panda). The T2 weighted image also revealed hyperintensities in
the thalami and the pontine tegmentum.

DIFFERENTIAL DIAGNOSIS
Symptoms of the following disorders can be similar to those of Wilson’s dis-
ease. Comparisons may be useful for a differential diagnosis: If the patient
presents with mild liver disease, the most common mistaken diagnosis is vi-
ral hepatitis. Viral antigen and copper studies should differentiate. If the pa-
tient presents with tremor, an incorrect diagnosis of essential tremor, or early
Parkinson’s disease may be made. Again, copper studies should differentiate.
If psychiatric symptoms are pronounced, an incorrect diagnosis of substance
abuse may be made. Again, copper studies should differentiate.
Other disorders which are occasionally mistaken for Wilson’s disease are as
follows:
• Sydenham’s chorea is an acute, usually self-limited disorder that oc-
curs after about 5 to 10 percent of cases of rheumatic fever. The disorder
typically begins with jerky, uncontrollable, non-repetitive muscle move-
ments on one or both sides of the body. Patients develop rapid, involun-
tary movements that can affect the manner or style of walking, arm move-
ments and speech. Clumsiness and facial grimacing are common
• Primary biliary cirrhosis is a chronic, progressive disease of the liver
thought to be related to abnormalities in the immune system. The initial
symptoms of this disorder usually include persistent, generalised itching,
dark urine, pale stools and jaundice. Eventually, excessive amounts of cop-
per accumulate in the liver and fibrous or granular hardening occurs in
the soft tissue of the liver
• Heavy metal poisoning is generally caused by industrial exposure to a
variety of toxins such as copper, aluminum, arsenic or mercury. Depending
on the type and duration of exposure, the injury may occur in the lungs,

178
Case 31

nervous system, the skin or digestive system. The symptoms of the poi-
soning vary according to the type of metal that was involved in the over-
exposure. These include headache, nausea, dizziness, painful joints and
muscles, delirium, seizures and a wide range of other symptoms.
• Levine-Critchley syndrome is a very rare genetic disorder of the neu-
romuscular and blood systems. Abnormal blood cells (acanthocytosis) are
produced and there is atrophy of muscles. The major symptom of this dis-
order is uncontrolled rapid muscular movements (amyotrophic chorea).
Initially there are subtle involuntary movements (tics) of the face, mouth,
and tongue. These slowly progress to severe, uncontrolled, rapid motions
(chorea) of the trunk and limbs. Approximately 50 percent of people with
Levine-Critchley syndrome have seizures.
• Huntington’s disease (Huntington’s chorea) is an inherited, progres-
sively degenerative neurological disorder. Initially there are personality
changes and rapid jerky muscle movements that are involuntary. In time,
speech and memory become impaired and involuntary muscle move-
ments become more frequent and pronounced. As Huntington’s disease
progresses there is a further loss of cognitive abilities and dementia. The
symptoms of this disorder usually begin during adulthood.
• Tourette syndrome is a neurologic movement disorder that is character-
ised by repetitive motor and vocal tics. The first symptoms usually occur
during childhood as rapid eye blinking or facial grimaces. Symptoms may
also include involuntary movements of the extremities, shoulders, face
and voluntary muscles. Some people with Tourette syndrome may vocal-
ise involuntarily; these may be unarticulated sounds or words. Tourette
syndrome is not a progressive or degenerative disorder; symptoms tend to
be variable and follow a chronic waxing and waning course. Onset usually
occurs before the age of 16.
• Cerebral palsy is a neurological disorder that is the result of an injury
to the brain during early development or at birth. The major symptom of
this disorder is a lack of muscle control and coordination. Cerebral palsy
is not a progressive disorder. Generally, infants may exhibit developmental
delays during the first or second year and may have muscle weakness and
abnormal muscle tone. The coordination and speech difficulties associ-
ated with Wilson’s disease can resemble the symptoms of cerebral palsy.

DIAGNOSTIC APPROACH
Although genetic testing may provide definitive proof of the diagnosis of
Wilson’s disease, the multitude of documented mutations identified in
Wilson’s disease make commercial genetic testing impractical. Advances and
refinements in technology may make this possible in the future, but so far, the
diagnosis of Wilson’s disease must be made by the judicious combination of
diagnostic tests. The specific tests necessary differ depending on whether the
mode of clinical presentation implicates dissemination of copper beyond the
confines of the liver.

179
Child Neurology

• Hepatic copper determination: Determination of hepatic copper con-


tent by means of liver biopsy is the single most sensitive and accurate
available test for Wilson’s disease. Hepatic copper content is elevated in
the vast majority of individuals with Wilson’s disease, even those who are
clinically asymptomatic. Elevations greater than 250 mg/g of dry tissue
(normal 15 to 55 mg/g) are typically present. Liver biopsy is usually not
necessary in individuals with neurological or psychiatric dysfunction be-
cause other tests permit diagnosis; its primary use is in individuals pre-
senting with hepatic dysfunction, where copper may not yet have been
discharged from the liver to flood other organs and tissues.
• Slit-lamp examination: In an individual with neurological or psychiat-
ric dysfunction, the presence of Kayser-Fleischer rings strongly supports
a diagnosis of Wilson’s disease. However, the absence of Kayser-Fleischer
rings in individuals with CNS dysfunction has been reported. Kayser-
Fleischer rings are often absent in patients with only hepatic symptoms.
• Ceruloplasmin: Ceruloplasmin is the major copper-carrying protein in
the blood, and in addition plays a role in iron metabolism. Levels of cerulo-
plasmin are abnormally low (<0.2 g/L) in 80-95% of cases. Measurement
of serum ceruloplasmin is safe, simple, and practical as a screening test for
Wilson’s disease, but it is not sufficient by itself. Ceruloplasmin may fall
within or only slightly below the normal range in 5 to 15% of individu-
als with Wilson’s disease. Ceruloplasmin may also be abnormally low in
other conditions (Menkes disease, aceruloplasminemia, sprue, nephritic
syndrome, protein-losing enteropathy) and in chronic liver disease of any
cause. In contrast, as an acute phase reactant, ceruloplasmin may become
transiently elevated into the normal range in Wilson’s disease patients by
infection or inflammation, or steroid ingestion.
• Measurement of 24-hour urinary copper excretion: The 24-hour
urinary copper measurement may be the single best screening test for
Wilson’s disease, especially in individuals with neurological or psychiat-
ric dysfunction. Urine copper levels in symptomatic Wilson’s disease pa-
tients typically exceed 100 mg/d. Heterozygous Wilson’s disease carriers
may have modestly elevated urine copper levels, but not above 100 mg/24
hours. Urine copper levels can also be elevated in obstructive liver disease.
It is important that patients collect their urine in copper-free jugs supplied
by the laboratory to prevent spurious elevations.
• Serum copper and serum free copper (Non-Ceruloplasmin
bound): Routine serum copper levels, which measure total (both bound
and unbound) serum copper, are of little diagnostic value in Wilson’s dis-
ease, even though they are typically reduced. Copper bound to cerulo-
plasmin normally represents 90% of total serum copper. Therefore, the
reduction in total serum copper in Wilson’s disease simply is a reflection
of reduced ceruloplasmin. In contrast, determination of non-ceruloplas-
min bound copper reflects the copper that is free to be deposited in tissue
and, thus, is potentially toxic. This copper fraction is typically elevated in

180
Case 31

Wilson’s disease. It is often difficult to get laboratories to measure non-


ceruloplasmin bound copper, but the level can be calculated by multiply-
ing the number for the ceruloplasmin level (reported in mg/dL) by three
and then subtracting that sum from the total serum copper level (reported
in mg/dL). The normal range for non-ceruloplasmin bound copper is 10
to 15 mg/dL.
• Neuroimaging studies: Recent reports have demonstrated the presence
of Magnetic Resonance Imaging (MRI) abnormalities in virtually 100% of
Wilson’s disease patients with neurological dysfunction. A multitude of
MRI abnormalities has been described in Wilson’s disease; the presence
of increased signal intensity in the basal ganglia on T2-weighted images is
perhaps the most widely recognised, although generalised brain atrophy
may be more common. Abnormalities such as the ‘‘face of a giant panda’’
in the midbrain, the ‘‘face of a miniature panda’’ in the pons, and the
‘‘bright claustrum’’ sign are present in only a relatively small percentage of
individuals with Wilson’s disease. Positron Emission Tomography (PET)
scanning shows abnormalities in Wilson’s disease, but is not routinely
available. Transcranial brain parenchyma sonography has been explored in
the setting of Wilson’s disease.
• Other studies: Incorporation of radioactive copper into ceruloplasmin
may be of value in selected situations for the diagnostic evaluation of sus-
pected Wilson’s disease, but is seldom required. It has been suggested that
Cerebrospinal Fluid (CSF) copper levels may provide the most accurate
reflection of the brain’s copper load, but this test is also not routinely
employed.
• Mutation analysis by whole-gene sequencing is possible and should
be performed on individuals in whom the diagnosis is difficult to establish
by clinical and biochemical testing. Haplotype analysis or specific testing
for known mutations can be used for family screening of first-degree rela-
tives of patients with WD.

TREATMENT
With the exception of liver transplantation, treatment of Wilson’s disease is
only palliative and intended to restore and maintain copper balance. It does
not eliminate the underlying defect responsible for Wilson’s disease. Thus, a
lifelong commitment to treatment is required. Limitation of dietary copper
intake is generally ineffective, and pharmacological management is necessary.
• Penicillamine: Penicillamine, a metabolic byproduct of penicillin that av-
idly chelates copper. Following initiation of treatment, copper is rapidly mo-
bilised from tissues and eliminated in the urine. Functional improvement
may become evident within 2 weeks of initiation of treatment, although
it typically takes somewhat longer. Improvement in virtually all facets of
functions may occur, although psychiatric symptoms improve less con-
sistently than neurological symptoms. The usual dosage of penicillamine

181
Child Neurology

for initial treatment is 250-500 mg four times daily, given on an empty


stomach, although some advocate lower dosages. There is no consensus
on the need for supplemental pyridoxine (penicillamine is a pyridoxine
antagonist). In recent years, increasing attention has been directed toward
the propensity of penicillamine to produce deterioration in neurological
function on initiation of treatment. Penicillamine can produce a variety of
other adverse effects. Acute sensitivity reactions with skin rash, fever, eo-
sinophilia, thrombocytopenia, leukopenia, and lymphadenopathy develop
in 20 to 30% of patients and often necessitate abandonment of penicil-
lamine treatment. Penicillamine dermatopathy, with brownish skin discol-
oration, is a consequence of recurrent subcutaneous bleeding during inci-
dental trauma. With chronic penicillamine administration, a host of other
drug-induced complications may occur, including nephritic syndrome,
Goodpasture’s syndrome, a lupus-like syndrome, a myasthenia-like syn-
drome, acute polyarthritis, thrombocytopenia, retinal haemorrhages, and
loss of sense of taste.
• Zinc: The use of zinc in the treatment of Wilson’s disease has gradually
assumed an increasingly important role in the management of the dis-
ease. Zinc has primarily been used as maintenance therapy following ini-
tial treatment with more potent ‘‘decoppering’’ agents. The usual dosage
regimen for zinc is 50 mg of elemental zinc three times daily (zinc sulfate
tablets contain 220 mg of zinc sulfate salt, which translates to 50 mg of
elemental zinc; zinc acetate is labeled by its elemental zinc content). Zinc
is generally well tolerated, although gastric discomfort may occur.
• Trientine: Trientine is a copper chelating agent with a mechanism of ac-
tion similar to penicillamine. As concerns have grown regarding the po-
tential complications of penicillamine, more attention has been focused
on trientine because it provokes a less precipitous decoppering and, thus,
may be safer to use. As with penicillamine, trientine should be taken on
an empty stomach. The usual daily dose is 750 to 2000 mg, divided into
three doses.
• Tetrathiomolybdate: It currently remains an experimental agent and
is unavailable for general use, and waiting approval for commercial use.
Tetrathiomolybdate has a distinct, dual mechanism of action in that, it
both limits gastrointestinal absorption of copper by forming a nonabsorb-
able tripartite complex with copper and albumin within the gut lumen,
and also forms the same complex within the bloodstream, preventing cel-
lular uptake of free copper. However, taking advantage of this dual capabil-
ity requires a somewhat complicated dosing scheme. Tetrathiomolybdate
binds copper in the gut when it is given with food and is absorbed into
the bloodstream when it is given without food. Therefore, a 20 mg dose is
given six times per day—three times daily with meals and three times daily
between meals. Tetrathiomolybdate is not intended for long-term treat-
ment, but only for an initial 8-week period, to be followed by long-term
maintenance therapy with zinc.

182
Case 31

• Liver transplantation currently remains reserved for patients who have


developed hepatic failure despite optimal medical management.
• In individuals with Wilson’s disease who are still asymptomatic, treat-
ment can be initiated and maintained with zinc alone. Most investigators
believe that individuals who have developed symptoms initially require
more vigorous removal of copper, although the use of zinc monotherapy
in this situation has strong advocates. Either penicillamine or trientine can
be used in such patients, but the danger of initial deterioration in neuro-
logical function hovers above both these treatment modalities, especially
penicillamine.
• For individuals with Wilson’s disease being managed medically, treat-
ment is a lifelong necessity, and compliance must be zealously moni-
tored. Compliance with zinc therapy can be assessed by measurement
of 24-hour zinc and copper levels. A 24-hour urinary zinc level of less
than 2 mg indicates inadequate compliance. Monitoring compliance with
penicillamine or trientine therapy is a bit more difficult, but a spike in a
previously receding or stable 24-hour urinary copper level may indicate
noncompliance. Monitoring serum non-ceruloplasmin bound copper can
also be used. Prolonged treatment with both zinc and chelating agents can
induce copper deficiency. Anaemia may be the first sign of this. In patients
on zinc maintenance therapy, a 24-hour urinary copper level below 35 mg
is suggestive of copper deficiency due to overtreatment. For individuals
on trientine or penicillamine, a serum non-ceruloplasmin bound copper
level below 5 mg suggests overtreatment.

REFERENCES
1. European Association for Study of Liver. EASL Clinical Practice Guidelines: Wil-
son’s disease. J Hepatol 2012; 56:671.
2. Lorincz MT. Neurologic Wilson’s disease. Ann N Y Acad Sci 2010; 1184:173.
3. Moores A, Fox S, Lang A, Hirschfield GM. Wilson disease: Canadian perspectives
on presentation and outcomes from an adult ambulatory setting. Can J Gastroen-
terol 2012; 26:333.
4. Merle U, Schaefer M, Ferenci P, Stremmel W. Clinical presentation, diagnosis and
long-term outcome of Wilson’s disease: a cohort study. Gut 2007; 56:115.
5. Bruha R, Marecek Z, Pospisilova L, et al. Long-term follow-up of Wilson disease:
natural history, treatment, mutations analysis and phenotypic correlation. Liver Int
2011; 31:83.
6. Walshe JM. The liver in Wilson’s disease. In: Diseases of the liver, 6th ed, SchiffL,
Schiff ER. (Eds), Lippincott, Philadelphia 1987. p.1037-1050.

183
32 Case
A 11-year-old boy with a several-day history of fever, malaise, diarrhoea,
and emesis developed Convulsive Status Epilepticus (CSE) requiring
diazepam, phenobarbital, and phenytoin. Cranial Computed Tomography
(CT) was unremarkable, and Cerebrospinal Fluid (CSF) contained six white
blood cells/mm3 (95% lymphocytes, 5% monocytes), with normal sugar and
protein. Electroencephalogram (EEG) revealed diffuse slowing with occasion-
al temporal spikes on continuous Electroencephalography (EEG) monitoring.
He continued to have frequent seizures described as sudden staring episodes
with head and eye deviation to the left, associated with cyanosis. He was
intubated and transferred to the ICU where he was treated with a pentobarbital
infusion. Pentobarbital was weaned 2 days later, but he then had a recurrence
of his seizure. Sodium thiopental was given, followed by midazolam. High-
dose steroids, and intravenous immunoglobulin were also given.
On Physical examination in the ICU, he was intubated, sedated, and uncon-
scious. His vital signs were normal. Pupillary was isochoric and light reflexes
were positive bilaterally. His tone was increased, deep tendon reflexes were
brisk, and the Babinski response was bilaterally extensor. Other system ex-
aminations were normal.

Answer the following questions:


• Give the most likely diagnosis.
• Provide a differential diagnosis.
• Discuss an appropriate diagnostic work-up.
• Discuss the management of the patient.

184
Case 32

DIAGNOSIS
Refractory Convulsive Status Epilepticus

DIFFERENTIAL DIAGNOSIS AND DIAGNOSTIC WORK-UP


This case involves Refractory Status Epilepticus (RSE), which occurs when
seizure activity persists despite adequate therapy. In children, cases of RSE not
responding to first-line therapy usually occur secondary to an acute symp-
tomatic SE or related to an underlying progressive neurological disorder.
Psychogenic seizures (or nonepileptic seizures) must also be considered when
seizures persist despite treatment, but are very unusual in younger children.
Unusual motor movements, an “on/off ” pattern of movements, a poor re-
sponse to treatment, or a lack of metabolic abnormalities, particularly after a
long seizure, suggest pseudostatus.
Following are causes of RSE:
1. Inadequate Anti-epileptic Drug (AED) therapy
2. Failure to start maintenance AED therapy
3. Failure to reverse ongoing metabolic derangements
4. Failure to identify and treat the underlying cause
5. Failure to identify and treat other medical complications
6. Misdiagnosis, especially of pseudoseizures. Therefore, when RSE occurs,
it is mandatory to exclude underlying etiologies that may require a spe-
cific treatment; anti-epileptic treatment may suppress seizure activity, but
it does not treat the acute precipitating cause. The American Academy of
Neurology (AAN) practice parameter did not specifically evaluate RSE.

Treatment
Refractory Status Epilepticus (RSE) is a common and life-threatening neuro-
logic emergency in Intensive Care Units (ICUs), characterised by high mor-
bidity and mortality. It heralds a prolonged hospitalisation and worse prognosis
than treatment-responsive Status Epilepticus (SE).
Although to date, there is no single, accepted operational definition of SE a
consensus has developed that SE should be considered refractory if it contin-
ues after adequate treatment for 30 minutes. However, RSE is mostly defined
as a persistent seizure activity after initiation of a first-line ( IV benzodiaz-
epines) and one second-line AED (mostly phenytoin, valproate, levetiracetam,
or phenobarbital), while others suggest a duration of SE of more than 60 min-
utes. The variability in definitions makes it difficult to estimate the incidence,
but studies suggest that refractory GCSE occurs in 9% to 44% of patients with
GCSE.
Refractory status epilepticus is typically treated with medical therapy, and a
small number of children may be candidates for urgent surgical therapy. The
failure of some children to respond to these therapeutic options combined

185
Child Neurology

with the high morbidity and mortality of persistent status epilepticus have al-
lowed other therapies to emerge.

Monitoring
• Implement continuous EEG monitoring
• Ensure patient is normovolumic
• Ensure cerebral perfusion pressure is >70 mm Hg
• Administer volume as needed and begin vasopressors to achieve adequate
mean arterial pressure (>120 mm Hg)
• Consider administration of pyridoxine, magnesium sulfate, and IVIG
Administering continuous infusions of medications at anesthetic doses in
RCSE is considered after failure of previous treatments. Current evidence
supports benzodiazepines as the most efficacious and safe continuous infusion
in children.

First-Line RCSE
• Midazolam: 0.2-0.4 mg/kg (maximum dose 10 mg) bolus at a rate of 2 mg/
min, followed by 0.05-2 mg/kg/hr.
• Pentobarbital: 10-20 mg/kg bolus at a rate of ≤50 mg/min followed by
1-5 mg/kg/hr (if hypotension occurs, slow the rate of delivery or add
vasopressors).
• Propofol: 1-2 mg/kg bolus followed by <4 mg/kg/hr in children; larger
dose may be used in adolescents.

Emerging Therapies
Several alternative therapies for RCSE have been reported as effective in in-
dividual patients or small series, but larger confirmatory trials are not avail-
able in children. Topiramate loading was able to control seizures in a series
of three children with refractory SE. The ketogenic diet has been reported as
efficacious in a limited number of children with refractory non-convulsive and
convulsive SE. Several cases of refractory SE have responded to hypothermia
both in adults and children.
Preliminary case series suggest that epilepsy surgery may be an effective alter-
native in children with refractory SE not responsive to conventional treatment
and with an identifiable focus.
Brain stimulation is emerging as a potentially useful therapy for refractory epi-
lepsy. Invasive brain stimulation methods such as vagus nerve stimulation and
deep brain stimulation are presently options to be considered along with non-
invasive transcranial magnetic stimulation. A common limitation to the proper
evaluation of emerging therapies is the rarity of their availability and use.

186
Case 32

REFERENCES
1. M. Sahin, C.C. Menache, G.L. Holmes, J.J. Riviello. Outcome of severe refractory
status epilepticus in children Epilepsia, 42 (2001), pp. 1461-1467.
2. S.A. Mayer, J. Claassen, J. Lokin, F. Mendelsohn, L.J. Dennis, B.-F. Fitzsimmons.
Refractory status epilepticus. Arch Neurol, 59 (2002), pp. 205-210.
3. S. Shorvon, M. Ferlisi. The outcome of therapies in refractory and super-refractory
convulsive status epilepticus and recommendations for therapy. Brain, 135 (2012),
pp. 2314-2328.
4. A. Rossetti, G. Logroscino, E. Bromfield. Refractory status epilepticus. Effect of
treatment aggressiveness on prognosis. Arch Neurol, 62 (2005), pp. 1698-1702.
5. R. Wilkes, R.C. Tasker. Pediatric intensive care treatment of uncontrolled status
epilepticus. Crit Care Clin, 29 (2013), pp. 239-257.
6. N.S. Abend, D.J. Dlugos. Treatment of refractory status epilepticus: literature re-
view and a proposed protocol. Pediatr Neurol, 38 (2008), pp. 377-390.

187
33 Case
A 2.5-year-old boy who was completely well until 6 months ago when
he experienced two brief (<2 minute) generalised convulsions 15 days
apart, both associated with fever. Although these were diagnosed as simple
febrile seizures, he was referred to a paediatric neurologist for further evalua-
tion, who ordered a routine brain Magnetic Resonance Imaging (MRI) scan.
This study was interpreted as normal. However, his Electroencephalogram
(EEG) study revealed generalised background slowing and generalised atypi-
cal spike-wave discharges occurring at a frequency of 2.0-2.5Hz. A follow-up
24-hour video-EEG study captured multiple generalised myoclonic and tonic
seizures, both associated with generalised spike-wave discharges, followed by
voltage suppression. No focal or lateralising features were noted.
This patient was placed on valproic acid monotherapy. Despite serum levels
(in the 100-120 range), he continued to have frequent seizures and then
developed “drop” attacks (seizures) that became progressively more frequent.
Further medication trials with topiramate, zonisamide, levetiracetam, and
clonazepam were unsuccessful, and he was ultimately placed on the ketogenic
diet. Though his generalised myoclonic seizures improved substantially, but
he was still experiencing several drop seizures per day. Notably, his speech and
attention became slowly impaired, and he developed difficulty walking, with
mild ataxia. A comprehensive metabolic/genetic work-up (including genetic
testing for severe myoclonic epilepsy of infancy) failed to reveal an etiology. As
his neurological condition steadily worsened, he underwent an anterior two-
thirds corpus callosotomy as a palliative procedure, to which he responded
favourably. His gait and ataxia improved, and his “drop” seizures got reduced
to 1-2 per day, with no myoclonic or tonic seizures.

Answer the following questions:


• Give the most likely diagnosis.
• Provide a differential diagnosis and diagnostic work-up.
• What is appropriate management of the patient.

188
Case 33

DIAGNOSIS
Lennox–Gastaut Syndrome (LGS)

DIFFERENTIAL DIAGNOSIS
LGS can be differentiated from other epilepsy syndromes based on history and
EEG characteristics, but achieving an accurate diagnosis can be challenging.
Not all patients with LGS display the characteristic triad of features, particu-
larly at onset. This syndrome comprises a clinical triad consisting of (1) dif-
fusely slow spike–wave discharges (occurring at a frequency of 1.5-2.5 Hz), (2)
psychomotor retardation, and (3) multiple electroclinical seizure types refrac-
tory to medical therapy (including generalised tonic, atonic, atypical absence,
myoclonic, and tonic–clonic seizures). The hallmark seizure type is the gener-
alised tonic seizure that most often occurs while falling asleep. In most cases,
LGS manifests between 2-8 years of age, represents 3-10% of all paediatric
epilepsies and this condition affects child more frequently than child. There
are two general subtypes: (1) cryptogenic (i.e. there is no identifiable cause)
in approximately a third of patients, and (2) symptomatic (i.e. associated with
a remote brain injury, usually acquired during the perinatal period or early
infancy). It is not uncommon for LGS to be preceded by a history of infantile
spasms.
• The major differential diagnostic consideration is myoclonic–astatic epi-
lepsy (Doose syndrome), which appears between 2-5 years of age, often
with generalised tonic–clonic seizures. The affected child, however, is
developmentally normal prior to onset of seizures, but developmental
decline can occur if they are not controlled adequately. Within several
months of onset, the characteristic drop attacks occur, along with atypical
absence seizures.
• There is also the syndrome of continuous spike–wave discharges in slow-
wave sleep (electrical status epilepticus in slow-wave sleep, or ESES),
which can result in speech/language regression. Slow spike–wave dis-
charges occur with atonic and atypical absence seizures. However, gener-
alised tonic seizures do not occur with ESES.
• Dravet syndrome, also known as Severe Myoclonic Epilepsy of Infancy
(SMEI), is a rare and catastrophic form of intractable epilepsy that be-
gins in infancy. Patients experience frequent seizures, poor seizure con-
trol, and developmental delays. Initial seizures are most often prolonged
events and in the second year of life other seizure types begin to emerge.
Development remains on track initially, with plateaus, with however, a
progressive decline typically beginning in the second year of life.
• Finally, epileptic spasms (i.e. spasms that persist beyond infancy) can also
produce drop attacks. Careful video-EEG monitoring of episodes can
help distinguish between LGS and these other rare entities. If the clini-
cian obtains a generic history of drop attacks without other ancillary data
(most importantly, the EEG), then other diagnostic considerations would

189
Child Neurology

include syncope (neurogenic or cardiogenic), cataplexy (seen in narco-


leptic patients), and hyperekplexia (or “startle disease”). It is important
to note, however, that there are several epileptic syndromes and epileptic
encephalopathies that share some but not all clinical and/or EEG features
of LGS, and many LGS patients may lack all of the characteristic features,
especially during the early stages of disease evolution.
• Despite clear parameters and distinguishing clinical features, significant
overlap exists between LGS and other early-onset epileptic encephalopa-
thies. Drop attacks, which are seen frequently in LGS patients, occur in
many other syndromes. Epilepsies that share this or other characteris-
tics of LGS include focal epilepsies with secondary bilateral synchrony,
myoclonic–astatic epilepsy (Doose syndrome), Dravet syndrome, West
syndrome, and atypical benign partial epilepsy of childhood. There is a
tendency to incorrectly diagnose LGS whenever there are multiple sei-
zure types or drop attacks. It is important to differentiate LGS from other
forms of epilepsies because of differences in prognosis and approaches to
treatment.

Steps for Evaluating LGS: A Proposed Algorithm


In June 2012, a group of LGS experts were convened to discuss approaches
to the differential diagnosis of LGS and identification of a possible under-
lying etiology. Although there were areas of consensus, some advocated few
tests which are not used by others. Experts asked for, at a minimum, awake
and asleep EEGs, physical examination (in particular neurologic deficits, skin
changes, fibromas, heart murmur) and ophthalmologic examination (retinal
abnormalities, visual impairment) together with medical and social history and
brain MRIs. Some of the neurologists also advocated for utilisation of video-
EEGs with Electromyogram (EMG) electrodes, and genetic/chromosomal mi-
croarrays, although these were not used universally.
If the underlying etiology was not identified following these investiga-
tions, some experts called for further testing, in particular DNA microarray,
SLC2A1 (glucose transporter defect), CLN2 (Late Infantile Neuronal Ceroid
Lipofuscinosis), and TSC 1, 2 (Tuberous Sclerosis). Some of these tests are
considered when the condition deteriorates. It is observed that different cen-
tres have different approaches for doing the tests on a regular basis. Genetics
and metabolism consultations were recommended by some experts, as well
as ophthalmology (e.g., retinal hamartomas in tuberous sclerosis, visual loss
in lipofuscinosis) and cardiology (e.g., rhabdomyomas in tuberous sclerosis).

Treatment Strategy
• Managing Lennox-Gastaut Syndrome really represents a challenge. The
number one challenge is to individualise treatment.
• The polymorphism of different seizure types, and the type of etiology for
what is the cause of the Lennox-Gastaut Syndrome, really needs to be

190
Case 33

taken into consideration when choosing any treatment modalities. The


three main forms of treatment of Lennox-Gastaut syndrome are Anti-
Epileptic Drugs (AEDs), dietary therapy (typically the ketogenic diet) or
surgery (VNS therapy or corpus callosotomy).
• Valproic acid has traditionally been the drug of choice, as it can provide
beneficial effects against all seizure types, but caution is advised when us-
ing valproic acid in young patients (under 2 years of age) and on polyphar-
macy (as this substantially increases the risk of hepatotoxicity).
• There have been several new drugs recently approved by the Food and
Drug Administration (FDA) for treatment of seizures associated with
LGS. Until 2011, only four medications were designated for the treat-
ment of LGS:
° Topiramate can help control the multiple seizure types of LGS. It
has been shown to reduce “Drop Attacks”, and has a safety profile
that many consider better than lamotrigine or felbamate. Side effects
may include sleepiness (somnolence), mental slowing and unstable
gait (ataxia) in addition to word-finding difficulties and weight loss.
° Lamotrigine is effective with multiple seizure types, as well as being
mood stabilising. It is a challenging drug to use because of the varying
ways it works in a person’s body, and the large number of medicinal
interactions. It is rarely related to a life-threatening rash, as long as
the drug is introduced slowly, and drug interactions (particularly with
valproate) are taken into account.
° Clonazepam is the most commonly used benzodiazepine. It is very
effective in myoclonic seizures, but it is limited by the common prob-
lem of tolerance, as well as withdrawal seizures. In addition, sedation
and drooling stop many providers from prescribing it initially. It is
rarely used as a long-term option.
° Felbamate was very successful for LGS when it was first introduced,
and one of the first medications that promoted alertness as well as
weight loss. However, the risk of aplastic anaemia and hepatic failure
have reserved its use to a final drug choice for difficult-to-mana
difficult-to-manage
seizures.

First-line drugs
• Rufinamide
• Valproates (valproic acid, sodium valproate and valproate semisodium)
• Felbamate
• Benzodiazepines, specifically (clonazepam, nitrazepam, and clobazam)

Second-line drugs
• Topiramate

191
Child Neurology

• Lamotrigine
• Felbamate

Unapproved, off-label, and investigational drugs


• Although
gh there are numerous uncontrolled studies of other medications
being effective for patients with LGS (including zonisamide, vigabatrin,
prednisone, IVIG, etc.), the reported benefits have been variable, limited,
or short-lived.
• Medical cannabis and/or specifically Cannabidiol. Cannabidiol (CBD) is
a compound found in cannabis that may have potential therapeutic use
for individuals living with Lennox-Gastaut Syndrome. The compound is
said to quiet the chemical and electrical activity in the brain without the
drawback of effects from cannabis that it typically includes.
Nonpharmacological options, such as the ketogenic diet, vagus nerve stim-
ulation, and corpus callosotomy (especially for the atonic seizures), have been
shown to be beneficial to varying degrees. In practice, the majority of children
with LGS need combination therapy, with both anti-epileptic medications and
non-pharmacological options. Each patient should be considered individually,
with a careful assessment of potential benefits of the chosen therapy weighed
against the risks of adverse effects.

Long-term outcome
Long-term prognosis of LGS is generally poor, as medically intractable sei-
zures persist in over 90% of patients, and recurrent episodes of status epilep-
ticus are not uncommon. Not only do patients experience significant neuro-
cognitive impairment, they are also exposed to a 3-7% risk of mortality (often
coming from accidents) and as such, protective helmets are often advised for
patients with drop attacks. Studies have demonstrated a steady worsening of
IQ, and behavioural and psychiatric symptoms. Without significant improve-
ment from medical and/or surgical treatment strategies. LGS can be appro-
priately considered a progressive epileptic encephalopathy. Unlike infantile
spasms, where the majority (>80%) of patients outgrow their seizures (but of-
ten followed later in life by other seizure types), patients with LGS may evolve
to a more refractory form of epilepsy, that is, epileptic spasms, or continue to
exhibit electroclinical features of LGS.

REFERENCES
1. Trevathan E, Murphy CC, Yeargin-Allsopp M. Prevalence and descriptive epi-
demiology of Lennox-Gastaut syndrome among Atlanta children. Epilepsia. 1997
Dec. 38(12):1283-8.
2. Arzimanoglou A, French J, Blume WT, Cross JH, Ernst JP, Feucht M, et al. Len-
nox-Gastaut syndrome: a consensus approach on diagnosis, assessment, manage-
ment, and trial methodology. Lancet Neurol. 2009 Jan. 8(1):82-93.

192
Case 33

3. Al-Banji MH, Zahr DK, Jan MM. Lennox-Gastaut syndrome. Management up-
date. Neurosciences (Riyadh). 2015 Jul. 20 (3):207-12.
4. Ng YT, Conry JA, Drummond R, Stolle J, Weinberg MA. Randomized, phase III
study results of clobazam in Lennox-Gastaut syndrome. Neurology. 2011 Oct 11.
77(15):1473-1481.
5. Ferrie CD, Patel A. Treatment of Lennox-Gastaut Syndrome (LGS). Eur J Paediatr
Neurol. 2009 Nov. 13(6):493-504.
6. Ohtsuka Y, Yoshinaga H, Shirasaka Y, Takayama R, Takano H, Iyoda K. Rufinamide
as an adjunctive therapy for Lennox-Gastaut syndrome: A randomized double-
blind placebo-controlled trial in Japan. Epilepsy Res. 2014 Nov. 108(9):1627-36.
7. Freeman JM, Vining EP, Kossoff EH, Pyzik PL, Ye X, Goodman SN. A blinded,
crossover study of the efficacy of the ketogenic diet. Epilepsia
Epilepsia. 2009 Feb.
50(2):322-5.

193
34 Case
A formerly healthy 8-year-old boy presented with a two-month history
of behavioural disturbances, mainly manifested by emotional lability
and apathy associated with impairment of educational performance. Parents
also gave a history of repeated falls due to abnormal jerky movements of
his whole body (myoclonus). A week later, he developed acute onset non-
fluctuating bilateral horizontal diplopia unaccompanied by ptosis or other
cranial nerve involvement. On the third day of diplopia, he developed rapidly
progressive bilateral painless vision loss leading to only hand perception at
one-meter distance over a span of 24 hours. Earlier he had left schooling due
to inattentiveness, inability to remember his studies. There was no history of
generalised seizures, limb weakness, gait abnormalities, or altered sensorium.
Family history was unremarkable. There was no previous history of measles
infection. Vaccination status of measles was doubtful.
General physical examination revealed that the patient was conscious,
oriented with normal conversation. Cranial nerve examinations revealed
bilateral decreased visual acuity of finger counting at 1-meter distance.
Examination of fundi showed bilateral papilloedema. Pupils were bilaterally
equal and reacting to light. There was bilateral lateral rectus palsy. Rest of the
cranial nerves examination was normal. Motor system examination showed
wide based gait, hypertonia, and increased deep tendon reflexes with bilateral
extensor plantar response. There were no signs of meningeal irritations.
MRI of the brain revealed bilateral symmetrical confluent white matter hyper
intensities involving both cerebral hemispheres with a frontal lobe predilection
with negative diffusion restriction. EEG showed generalised high amplitude
periodic sharp wave complexes diffuse discharges. CSF studies showed that
the opening pressure was 22 cm H2O. CSF sugar and protein were normal,
with acellular back ground. CSF Gram stain, Ziehl-Neelsen stain, India ink
and culture for pyogenic, tubercular and fungal organisms were negative.
CSF PCR for mycobacterium tuberculosis was negative. Screening for HIV,
neurosyphilis, hepatitis B and C were negative.

Answer the following questions:


• Give the most likely diagnosis.
• Summarise the case briefly.
• Where do you localise the lesion?
• Provide a differential diagnosis.
• Discuss an appropriate diagnostic work-up.
• Discuss the management of this patient.

194
Case 34

DIAGNOSIS
Subacute Sclerosing Panencephalitis (SSPE)

SUMMARY
A formerly healthy 8-year-old boy presented with behavioural disturbances,
myoclonic seizures and apathy associated with impairment of educational
performance. He developed rapidly progressive bilateral painless vision loss
and he left schooling due to inattentiveness. MRI of the brain revealed bi-
lateral symmetrical white matter hyper intensities involving both cerebral
hemispheres with a frontal lobe predilection with negative diffusion restric-
tion. EEG showed generalised periodic high voltage sharp wave complexes of
diffuse discharges. Serum and CSF measles immunoglobulin G anti-body was
positive in high titer.

Localisation of Lesion
Symptoms of behavioural disturbances, cognitive problem showed involve-
ment of the cortical gray matter. Increased tone, hyperreflexia, and bilateral
plantar extensor responses localise to the corticospinal tracts. Rigidity suggests
involvement of the extrapyramidal tracts. A wide-based, clumsy gait is a non-
specific finding but it could be caused by spasticity or cerebellar involvement.
Myoclonic seizures are a form of generalised epilepsy and localise to the cortex
bilaterally.

DIFFERENTIAL DIAGNOSIS
A chronic meningitis (tuberculosis), encephalitis, HIV encephalopathy, brain
tumours, central nervous system vasculitis, or toxic exposure may present with
a chronic encephalopathy. However, when myoclonus becomes apparent, a
broad range of neurodegenerative myoclonic conditions need to be consid-
ered. These disorders can be divided into, progressive myoclonic encepha-
lopathies, and progressive myoclonic ataxias. The progressive myoclonic epi-
lepsies include Lafora body disease, Neuronal Ceroid Lipofuscinosis (NCL),
Myoclonic Epilepsy with Ragged Red Fibres (MERRF), Unverricht-Lundborg
syndrome, Sialidosis, and Dentatorubral-pallidoluysian atrophy.
• Subacute sclerosing panencephalitis is a rare progressive neurological
disorder that occurs mostly in children under the age of 12 years. Subacute
sclerosing Panencephalitis (SSPE) is also known as Dawson Disease.
Dawson encephalitis and measles encephalitis is a rare and chronic form of
progressive brain inflammation caused by a persistent infection with mea-
sles virus. The symptoms begin gradually and may include behavioural
changes, loss of previously acquired intellectual skills, involuntary muscle
contractions or spasms (myoclonus) of the trunk, arms, and/or legs, and
the inability to communicate verbally. Later symptoms usually include loss
of vision and hearing, muscle rigidity, and/or dementia.

195
Child Neurology

• Progressive rubella panencephalitis is a rare slowly progressive neu-


rological disorder that closely resembles subacute sclerosing panencepha-
litis. It is caused by the rubella virus and develops because of congeni-
tal rubella syndrome or childhood rubella infection (German measles).
Symptoms usually include behavioural changes, the loss of previously
acquired intellectual skills, inability to coordinate movement (ataxia),
involuntary muscle movements (spasticity), and/or seizures. There is no
known treatment for this disorder.
• Progressive multifocal leukoencephalopathy is a rare progressive
neurological disorder which occurs in people with certain forms of cancer,
Acquired Immunodeficiency Syndrome (AIDS), or those who are receiv-
ing immunosuppressant drugs. The disease develops during adulthood and
symptoms may include paralysis of one side of the body (hemiplegia), loss
of vision in one eye (hemianopsia), loss of verbal communication skills
(aphasia), inability to coordinate movement (ataxia), stupor, and/or coma.
Progressive multifocal leukoencephalopathy may be the result of reactiva-
tion of a slow virus (polyoma).
• Myoclonic Epilepsy with Ragged Red Fibres (MERRF) is a form of
mitochondrial cytopathy associated most frequently with myoclonic sei-
zures and ataxia. However, affected individuals also may exhibit dementia,
deafness, optic atrophy, peripheral neuropathy, or dystonia. A muscle bi-
opsy is required to make a definitive diagnosis.

DIAGNOSTIC APPROACH
Diagnosis is primarily based on clinical presentation supported by high titres
of serum anti-bodies against measles virus and the presence of oligoclonal
measles virus anti-bodies in CSF, or detection of viral RNA, expressing multi-
ple mutations, from brain biopsy or at autopsy. Ancillary findings such as those
obtained by EEG, neuroimaging, and tissue analysis may also be used. On
EEG, periodic complexes are characteristic features of SSPE. Neuroimaging
(magnetic resonance imaging and computed tomography) have limited roles
in the early diagnosis of SSPE but can be used to follow disease progression
(may show progressive atrophy and white matter lesions). Brain biopsy or
post-mortem tissue examination show typical histopathological findings.

Diagnostic Criteria of SSPE


a. Clinical Progressive, subacute metal deterioration with typical signs like
myoclonus
b. EEG periodic, stereotyped, high voltage discharges
c. Cerebrospinal fluid raised gammaglobulin or oligoclonal pattern
d. Measles anti-bodies raised titer in serum (>1:256) and/or cerebrospinal
fluid (>1:4)

196
Case 34

e. Brain biopsy suggestive of panencephalitis (in particular, intranuclear and


cytoplasmic inclusion bodies in neurons and glial cells) or detection of
viral RNA
Definitive: Criteria 5 probable: Three of the five criteria.

MANAGEMENT
• Currently, there is no cure for SSPE. Clinical trials of anti-viral (isoprino-
sine and ribavirin) and immunomodulatory (interferon alpha) drugs have
suggested that these types of therapies given alone or in combination halt
the progression of the disease and can prolong life, but their long-term
effects on individuals, and eventual outcome, are unknown.
• The pharmacologic management of SSPE includes symptomatic therapies
and disease modifying agents. Many different anti-convulsants have been
tried and no single agent has emerged as the best. With current evidence,
carbamazepine has been effective in few settings. There are case reports
of successful treatment of myoclonus with trihexyphenidyl and ketogenic
diet. In the disease modifying drugs, Isoprinosine (inosiplex) and inter-
feron have been studied in detail.
• Isoprinosine is an immune modulating substance which promotes lym-
phocyte proliferation, production of immunoglobulin and lymphokines
that facilitate lymphocyte immune function once triggered by a viral anti-
gen. It has been found to be effective in numerous trials.
• Interferon acts by activating Natural Killer cells and directly inhibiting vi-
rus replication. IFN alpha is administered intraventricularly and has been
found to be effective. Beta-interferon has also been successfully used in
treating seven patients in a case series. Combined use of both Isoprinosine
and IFN alpha has been found to be effective in large multicentre trials,
although conflicting reports of it being ineffective in the early stages of
SSPE are present.
• Ribavirin has been used as adjunct therapy, in addition to intra ventricu-
lar IFN alpha, with minimal success. There are isolated reports claiming
success with IVIF, amantadine, steroids and acyclovir, but none proven
in multicentric trials. Rituximab (anti- CD 20 anti-body) has been tried
in SSPE with no success. Flupirtine, an anti-apoptotic agent which has
been used with limited success in Alzheimer’s and prion diseases has been
hypothesised to halt the disease progression in SSPE, but no clinical data
is available yet.
• Good nursing care is the most important aspect of treatment for SSPE,
along with anti-convulsant and anti-spasmodic drugs when needed.
Therefore, the most successful strategy in the management of SSPE is the
prevention of the primary disease by means of immunisation.

197
Child Neurology

REFERENCES
1. P. S. Dimova and V. S. Bojinova, “Case of subacute sclerosing panencephalitis with
atypical absences and myoclonic-atonic seizures as a first symptom,” Journal of
Child Neurology, , 2004; 548-552.
2. Goraya J, Marks H, Khurana D, et al.. Sub acute sclerosing pan encephalitis (SSPE)
presenting as acute disseminated encephalomyelitis in a child. J Child Neurol
2009; 2013:899-903.
3. Comert S, Vitrinel A, Gursu HA, et al.. Sub acute sclerosing pan encephalitis pre-
senting as acute disseminated encephalomyelitis. Indian J Pediatr 2006;2013:1119-
21.
4. S. Praveen-kumar, S. Sinha, A. B. Taly et al.,., “Electroencephalographic and imaging
profile in a subacute sclerosing panencephalitis (SSPE) cohort: a correlative study,”
Clinical Neurophysiology, 2007; 1947-1954.
5. Campbell C, Levin S, Humphreys P, et al.. Subacute sclerosing panencephalitis:
results of the Canadian pediatric surveillance program and review of the literature.
BMC Pediatr. 2005;5(1):47.
6. Honarmand S, Glaser CA, Chow E, et al.. Subacute sclerosing panencephalitis in
the differential diagnosis of encephalitis. Neurology. 2004;63:1489:1493.

198
Case 35
A 7-year-old child presented with a history of fever, vomiting, abdominal
pain which was treated as a case of gastroenteritis. Two weeks later
she developed abnormal eye movements, head nodding, unsteady gait and
behavioural changes. Abnormal eye movements were random, saccadic,
present during rest and increased by movement. Head nodding increased by
movement but stopped only during sleep. She lost her ability to walk, feed and
dress despite having normal muscular power. She had emotional disturbances
with irritability, frequent crying and aggressiveness.
Physical examination of ocular motor nerves revealed conjugate, arrhythmic,
chaotic large amplitude eye movements, predominantly in a horizontal
direction. Her pupils were equal, round, and reactive to light. She tracked
objects in all directions, but on close observation she had rapid, conjugate jerks
of her eyes in all directions (opsoclonus). Voluntary ocular movements were
full but tended to exacerbate the opsoclonus. She also had severe truncal and
appendicular ataxia, along with truncal and limb myoclonus. Her myoclonic
movements tended to exacerbate on movements. The sensations were intact.
Remaining neurological and systemic examination was within normal limits.
Tremulousness of both upper extremities is noted in addition to truncal ataxia.
Due to ataxic gait she needed help to walk. Reflexes: 2+ throughout with
bilateral flexor plantar responses.
VMA (Vanillylmandelic Acid), MIBG (Metaiodobenzylguanidine), brain
MRI, abdomen and chest CT scan were normal. Anti-body titer (IGM)
against cytomegalovirus was elevated.

Answer the following questions:


• Give the most likely diagnosis.
• Briefly summarise the case.
• Localise the examination findings.
• Provide a differential diagnosis.
• Provide the diagnostic work-up.
• Discuss the treatment of the patient.

199
Child Neurology

DIAGNOSIS
Post-infectious (Idiopathic) Opsoclonus Myoclonus Syndrome

SUMMARY
The patient is a previously healthy, developmentally normal seven-year-old
girl who presented with the subacute onset of irritability and abnormal chaot-
ic, conjugate eye movements (opsoclonus) in addition to tremulousness with
truncal and gait ataxia. She lost her ability to walk, feed and dress despite hav-
ing normal muscular power. She had emotional disturbances with irritability,
frequent crying and aggressiveness.

Localisation
Opsoclonus may localise to the cerebellum or to the brainstem (especially the
midbrain). Tremulousness, as well as truncal and gait ataxia, localise to the
midline cerebellum. Irritability typically denotes a generalised illness that may
or may not involve the central nervous system.

DIFFERENTIAL DIAGNOSIS
Symptoms of the following disorders can be similar to those of OMS.
Comparisons may be useful for a differential diagnosis:
• About 95% of children with OMS are misdiagnosed as acute cerebellar
ataxia because the ataxia may appear before the eye findings. This diagno-
sis causes delay in recognition and treatment of OMS. Once opsoclonus
is present, however, the diagnosis cannot be acute cerebellar ataxia. Also,
children with disproportionately small amounts of opsoclonus or myo-
clonus are harder to recognise and may carry the wrong diagnosis for years
• Myoclonus can occur in other contexts without opsoclonus and with or
without ataxia. It may be epileptic or non-epileptic, rhythmical or arrhyth-
mic, generalised or localised. Myoclonus may accompany a number of
neurologic diseases, including seizure disorders, brain injuries, hereditary
brain disorders, viral infections, and metabolic or toxic disorders.
• In cases where opsoclonus is either absent or delayed in onset, acute cer-
ebellar ataxia or acute cerebellitis of childhood can be easily confused with
OMS. Acute cerebellar ataxia represents cerebellar inflammation that typ-
ically occurs following infection or immunisation. Patients present with
acute truncal and gait ataxia, which is usually benign and self-limited. Eye
movement abnormalities may be present in patients with acute cerebellar
ataxia as well; however, they will have nystagmus rather than opsoclonus.
This is a helpful diagnostic difference and is important because whereas
acute cerebellar ataxia is self-limited, OMS requires prompt therapeutic
intervention.
• Other differential diagnosis presenting with the subacute onset of ir-
ritability and cerebellar signs includes toxic ingestion, central nervous

200
Case 35

system infections (meningitis, encephalitis), acute cerebellar ataxia, Acute


Disseminated Encephalomyelitis (ADEM), head trauma, hydrocephalus,
and posterior fossa tumours. Inborn errors of metabolism, such as mito-
chondrial diseases, urea cycle disorders, and aminoacidopathies, can pre-
sent with intermittent ataxia and abnormal eye movements. An inborn
error of metabolism is a less likely consideration in this patient as she has
been developmentally normal and her neurologic problems have not been
intermittent.

DIAGNOSTIC WORK-UP
Opsoclonus Myoclonus Syndrome (OMS), also known as Opsoclonus-
Myoclonus-Ataxia (OMA), is a rare neurological disorder of unknown causes
which appears to be the result of an autoimmune process involving the nerv-
ous system. It is an extremely rare condition, affecting as few as 1 in 10,000,000
people per year. It affects 2 to 3% of children with neuroblastoma and has been
rarely reported to occur with celiac disease.
• The diagnosis is clinical; there is no diagnostic test yet, as the antigen re-
mains unidentified. The presence of the ‘dancing eyes’, the shock-like
muscle spasms, and the stumbling gait, especially if accompanied by ir-
ritability, are highly reliable indicators of this syndrome.
• Any child presenting with suspected OMS needs a very thorough evalu-
ation to search for an underlying neuroblastoma. The urine should be
screened for catecholamines (HMA, VMA, dopamine). However, nega-
tive urine catecholamine studies do not rule out neuroblastoma because
tumours may be small and/or non-secreting. MRI and/or CT scanning
of the chest and abdomen should be performed but may miss small tu-
mours. In these cases, more sensitive radiologic studies may be required.
Scintigraphic testing with the radioisotope Metaiodobenzylguanidine
(MIBG) may be helpful. MIBG acts as an analogue of norepinephrine. It
is taken up and stored in neural crest tumours. Octreotide, an analogue of
somatostatin, may also be useful in identifying small lesions.
• If these tests are initially negative, follow-up repeat studies may also be
recommended several weeks to months later to ensure that a tumour has
not been missed, which becomes evident with time.
• To detect a tumour in children, both a CT scan of the neck, chest, abdo-
men, and pelvis (with oral and IV contrast) need to be done. In addition, a
spinal tap to detect neuroinflammation is necessary and may help rule out
an infectious or parainfectious process if a neuroblastoma is not identified.
Besides routine tests for infection, recommended CSF studies include ol-
igoclonal bands (with paired serum sample), which are autoantibodies se-
creted by B cells in the CSF. Autoantibodies in some children with OMS
have been detected in research laboratories, but commercial autoantibody
testing is not cost-effective and is best reserved for atypical cases.

201
Child Neurology

• If the diagnosis is less clear, additional testing may be requested. An EEG


may help rule out epileptic myoclonus.
• OMS patients may acutely demonstrate a mild pleocytosis. In addition,
a brain MRI with and without contrast should be performed to rule out
brainstem and/or cerebellar structural lesions.

TREATMENT
• Patients with OMS and neuroblastoma obviously need to be followed and
managed by an oncologist. If a tumour is present, surgical resection is the
standard. The tumours are usually low stage (I or II), and tumour chemo-
therapy or radiation therapy are not indicated. Tumour resection does not
usually provide sufficient clinical benefit for OMS, however.
• The main tenet of treatment for OMS is early and sufficient immuno-
therapy with the goal of gaining a durable complete neurological remis-
sion. OMS treatment, which is usually continued over 1-2 years, should
involve combined immunotherapies as soon as possible after diagnosis.
• FLAIR therapy is a three-agent protocol involving front-loaded high-
dose ACTH (corticotropin), IVIG, and rituximab that was developed by
the National Paediatric Myoclonus Centre for paediatric OMS, and has
the best-documented outcomes for moderately severe and severe cases.
Rituximab is a monoclonal anti-body against B cells (anti-CD20). Almost
all patients (80-90%) show improvement with this treatment. Over time,
treatment with ACTH may have mostly cortisol-related adverse effects
that must be monitored carefully.
• Pulse dose dexamethasone instead of ACTH is an option in mild and
more moderate cases. The use of oral steroids like prednisone is not rec-
ommended, because they are the least effective of the steroids for paedi-
atric OMS.
• For OMS relapse, low-dose IV cyclophosphamide (3-6 cycles) or more
rituximab (1-2 cycles) are given. Oral weekly methotrexate may be a use-
ful steroid sparer in chronic relapses.
• A biomarker-guided approach, which led to the introduction of anti-B-
cell monoclonal anti-bodies in OMS, promises to give rise to new thera-
peutic targets and strategies.

REFERENCES
1. Pang KK, de Sousa C, Lang B, Pike MG. A prospective study of the presentation
and management of dancing eye syndrome/opsoclonus-myoclonus syndrome in
the United Kingdom. Eur J Paediatr Neurol 2010; 14:156.
2. Tate ED, Allison TJ, Pranzatelli MR, Verhulst SJ. Neuroepidemiologic trends in
105 US cases of pediatric opsoclonus-myoclonus syndrome. J PediatrOncolNurs
2005; 22:8.
3. Hasegawa S, Matsushige T, Kajimoto M, et al. A nationwide survey of opsoclonus-
myoclonus syndrome in Japanese children. Brain Dev 2015; 37:656.

202
Case 35

4. Blaes F, Fühlhuber V, Korfei M, et al. Surface-binding autoantibodies to cerebellar


neurons in opsoclonus syndrome. Ann Neurol 2005; 58:313.
5. Krasenbrink I, Fühlhuber V, Juhasz-Boess I, et al. Increased prevalence of autoim-
mune disorders and autoantibodies in parents of children with opsoclonus-myo-
clonus syndrome (OMS). Neuropediatrics 2007; 38:114.
6. Matthay KK, Blaes F, Hero B, et al.. Opsoclonus myoclonus syndrome in neuro-
blastoma a report from a workshop on the dancing eyes syndrome at the advances
in neuroblastoma meeting in Genoa, Italy, 2004. Cancer Lett 2005; 228:275.
7. Wells EM, Dalmau J. Paraneoplastic neurologic disorders in children. Curr Neurol
Neurosci Rep 2011; 11:187. Wong A. An update on opsoclonus. Curr Opin Neu-
rol 2007; 20:25.
8. Mitchell WG, Brumm VL, Azen CG, et al.. Longitudinal neurodevelopmental eval-
uation of children with opsoclonus-ataxia. Pediatrics 2005;116(4):901: 907.
9. Bell J, Moran C, Blatt J. Response to rituximab in a child with neuroblastoma and
opsoclonus-myoclonus. Pediatr Blood Cancer 2008; 50:370.
10. Tate ED, Pranzatelli MR, Verhulst SJ, et al.. Active comparator-controlled, rater-
blinded study of corticotropin-based immunotherapies for opsoclonus-myoclonus
syndrome. J Child Neurol 2012; 27:875.
11. Dalmau J, Rosenfeld MR. Paraneoplastic syndromes of the CNS. Lancet Neurol
2008; 7:327.

203
36 Case
A 20-month-old boy, was referred because of concerns about a language delay.
He was born at full-term by spontaneous vaginal delivery to a 30-year-old
G3P2 mother. The pregnancy was uncomplicated and his birth weight was
3 Kg. He crawled at 7 months and walked at 12 months. His parents were
concerned because he was not saying any word. He was not interested to play
with his brothers; rather preferred to play alone. His parents also noted he had
no eye contact, did not look when his name was called, was not interested in
people and rocked forward and backward in a sitting position and lived in his
own world.
Physical examination showed no dysmorphic facial features. He was unco-
operative during the examination and did not show any interest in toys used
to try to get his attention. His pupils were equal in size, reactive to light, and
extraocular muscles were intact. His gross motor development was acceptable.
He didn’t have any dysmetria or ataxia. Reflexes were 2+ throughout with bi-
lateral plantar flexor responses. His developmental assessment revealed a scat-
tered performance. There was severe impairment in language development
and social-emotional reciprocity, and repetitive behaviours in his psychiatric
evaluation. He had no verbal expression and limited non-verbal communica-
tion with poor eye contact and his response to instructions was inconsistent.
He had an unusual preoccupation with holding saliva in his mouth and play-
ing with it.
Extensive metabolic work-up, EEG, thyroid functions, MRI, and MRS were
all concluded normal.

Answer the following questions:


• Give the most likely diagnosis.
• Briefly summarise the case.
• Discuss the differential diagnosis.
• Provide the appropriate diagnostic work-up.
• What is the management of the patient?

204
Case 36

DIAGNOSIS
Autism

SUMMARY
This 18-month-old boy, presented with delayed language and he was not able
to say any word. He was also not interested to play with other children; rather
preferred to play alone. He also had no eye contact, did not look when his
name was called, was not interested in people and rocked forward and back-
ward in a sitting position and lived in his own world.

DIFFERENTIAL DIAGNOSIS
• Autism and Autism Spectrum Disorder (ASD) are both general terms for a
group of complex disorders of brain development. Autism is clinically char-
acterised by abnormalities in three main areas: social interaction, language
and communication, and interests and activities. Symptoms of these abnor-
malities must be present before the age of three, although the development
during the first year may be relatively normal in many cases. Additional
findings are mental retardation in 75% of the cases and epilepsy in 30%, as
well as hyperactivity and other neurological signs. About 10% of cases of
autism are due to or associated with specific etiologies such as congenital
rubella, chromosomal abnormalities, or single gene disorders.
• Chromosomal disorders: Chromosomal abnormalities have been iden-
tified in individuals with autism. Various kinds of abnormalities have been
described in different regions of almost all chromosomes in individuals
with autism. Concerning chromosomal abnormalities in general, trisomy
21 or Down syndrome is the most common. About 10% of patients with
Down syndrome exhibit autistic behaviour and trisomy 21 can be detected
in almost 2% of individuals thought to be autistic.
• In reference to other chromosomal disorders, particular attention should
be given to the proximal long arm of chromosome 15. There are several
descriptions of autistic individuals showing abnormalities on this region,
including the critical region for Prader-Willi and Angelman syndromes. A
frequency of 1% to 3% of abnormalities in this chromosome is found in
individuals with autism. It is also suggested that maternal origin of the ab-
normal chromosomes 15 in autistic patients is more frequent.
• Fragile X syndrome is the most frequent cause of inherited mental re-
tardation, with some studies indicating an incidence of 6% of all mentally
retarded child and 2% of child with mild mental impairment. Although
its name refers to the X chromosome, a genetic disorder that follows an
X-linked mode of inheritance, caused by the FMR-1 gene. It is clinically
characterised by mental retardation, usually moderate to severe in child
and mild to moderate in 1/3 of carrier females. Behavioural and emotional
abnormalities are also described, including repetitive motor behaviours

205
Child Neurology

(hand-biting and hand-flapping), repetitive speech patterns (echolalia,


dysfunctional verbal communication), impaired socialisation, persevera-
tion, and gaze aversion. Since the detection of the fragile X chromosome
in autistic individuals, several reports have mentioned high prevalence of
fragile X syndrome in samples of people with autism, varying from 5% to
16%. For this reason, cytogenetic or molecular tests for the diagnosis of
the fragile X syndrome have been included in the investigation of indi-
viduals with autism.
• Tuberous sclerosis is an autosomal dominant disorder characterised by
hamartomatous growth in multiple organs. Clinical findings are variable
among affected individuals
ividuals and include hypopigmented skin macules, fa-
cial angiofibromas, seizures, and mental retardation. Over 50% of patients
with this condition present with autistic behaviour and about 3% to 9% of
all cases of autism are estimated to be due to tuberous sclerosis
• Neurofibromatosis type I is another disorder with autosomal dominant
inheritance. About 8% to 11% of people with this disorder present with
mild mental retardation, learning disabilities, and behavioral problems.
Some cases of neurofibromatosis are also suggested to be associated with
autism.
• Hypomelanosis of Ito is characterised by abnormal skin pigmentation,
most frequently asymmetric areas of hypo-or hyperpigmentation, mental
retardation, seizures, and/or other congenital abnormalities. It is suggested
that about 10% of patients with these signs present autistic-like behaviour
and that 0.5% of autistic individuals have hypomelanosis of Ito.
• Rett syndrome is a disorder that affects only females. Rett syndrome
have an apparently normal development until the age of six to eighteen
months. Then, the neurologic acquisitions stop and later regress. There is
simultaneous deceleration of ponderal (weight) and head growth, with ac-
quired microcephaly. Other characteristics are social isolation, absence of
language (or other non-verbal communication), and behavioural changes
like cry-crisis, and at least at this age, the clinical picture nearly resembles
autism. Comparing with autism, some differences may be noted. First of
all, Rett syndrome occurs only in females, while autism has a strong child
preponderance. Besides, loss of meaningful hand movements is seen in
Rett syndrome but not in autism.
• Landau-Kleffner Syndrome (LKS) is a rare, childhood neurological
disorder characterised by the sudden or gradual development of aphasia
(the inability to understand or express language) and an abnormal electro-
Encephalogram (EEG). The disorder usually occurs in children between
the ages of 5 and 7 years. Typically, children with LKS develop normally
but then lose their language skills for no apparent reason. While many of
the affected individuals have seizures, some do not. The disorder is dif-
ficult to diagnose and may be misdiagnosed as autism, pervasive develop-
mental disorder, hearing impairment, learning disability, auditory/verbal

206
Case 36

processing disorder, attention deficit disorder, childhood schizophrenia,


or emotional/behavioural problems.
• Metabolic disorders: Phenylketonuria is a well-known autosomal re-
cessive disorder of phenylalanine metabolism. Untreated phenylketonuria
was also reported among people with autism, but its frequency has been
significantly reduced in developed countries since the introduction of
early diagnosis and treatment.
• Neurodegenerative disorders: Metachromatic leukodystrophy, adreno-
leukodystrophy, and GM1 gangliosidosis are examples of neurodegenera-
tive diseases that should be included in the differential diagnosis of autism.
These disorders are generally characterised by a period of normal develop-
ment followed by progressive neuromotor and cognitive involution, result-
ing in social and communicative isolation. The diagnosis is usually facilitated
by a distinct clinical picture and known biochemical basis.

DIAGNOSTIC WORK-UP
• There are no specific laboratory findings in autism. However, several
diagnostic scales were created on the basis of the clinical aspects, in or-
der to define its diagnosis. The most useful are the criteria listed in the
Diagnostic and Statistical Manual of Mental Disorders (DSM-V) from
the American Psychiatric Association (2013), and the criteria of the
International Classification of Diseases and Health Problems (ICD) 10th
Revision Volume 2 Instruction manual 2010 Edition. This evaluation may
be done by a multidisciplinary team that includes a psychologist, a neu-
rologist, a psychiatrist, a speech therapist, or other professionals who diag-
nose children with autism.
• Because ASDs are complex disorders and may involve other neurological
or genetic problems, a comprehensive evaluation should entail neurologic
and genetic assessment, along with in-depth cognitive and language test-
ing. In addition, measures developed specifically for diagnosing autism
are often used.

Autism Spectrum Disorder: Diagnostic Criteria


a. Persistent deficits in social communication and social interaction across
multiple contexts, as manifested by the following, currently or by history:
i. Deficits in social-emotional reciprocity, ranging, for example, from
abnormal social approach and failure of normal back-and-forth con-
versation; to reduced sharing of interests, emotions, or affect; to fail-
ure to initiate or respond to social interactions.
ii. Deficits in nonverbal communicative behaviour used for social in-
teraction, ranging, for example, from poorly integrated verbal and
nonverbal communication; to abnormalities in eye contact and body
language or deficits in understanding and use of gestures; to a total
lack of facial expressions and nonverbal communication.

207
Child Neurology

iii. Deficits in developing, maintaining, and understanding relationships,


ranging, for example, from difficulties adjusting behaviour to suit
various social contexts; to difficulties in sharing imaginative play or in
making friends; to absence of interest in peers.
Severity is based on social communication impairments and restricted
repetitive patterns of behaviour
b. Restricted, repetitive patterns of behaviour, interests, or activities, as
manifested by at least two of the following, currently or by history:
i. Stereotyped or repetitive motor movements, use of objects, or speech
(e.g., simple motor stereotypies, lining up toys or flipping objects,
echolalia, idiosyncratic phrases).
ii. Insistence on sameness, inflexible adherence to routines, or ritualised
patterns or verbal, nonverbal behaviour (e.g., extreme distress at small
changes, difficulties with transitions, rigid thinking patterns, greeting
rituals, need to take the same route or eat the same food every day).
iii. Highly restricted, fixated interests that are abnormal in intensity or
focus (e.g., strong attachment to or preoccupation with unusual ob-
jects, excessively circumscribed or perseverative interest).
iv. Hyper-or hypo-reactivity to sensory input or unusual interests in sen-
sory aspects of the environment (e.g., apparent indifference to pain/
temperature, adverse response to specific sounds or textures, exces-
sive smelling or touching of objects, visual fascination with lights or
movement).
Severity is based on social communication impairments and restricted, repeti-
tive patterns of behaviour.
Severity levels for autism spectrum disorder as defined by the new
criteria
(In descending disorder)
Severity level 3 "Requiring intense intervention"
Social communication: Severe deficits in verbal and nonverbal social com-
munication skills cause severe impairments in functioning, very limited ini-
tiation of social interactions, and minimal response to social overtures from
others. For example, a person with few words of intelligible speech who rarely
initiates interaction and, when he or she does, makes unusual approaches to
meet needs only and responds to only very direct social approaches.
Restricted, repetitive behaviors: Inflexibility of behaviour, extreme diffi-
culty in coping with change, or other restricted/repetitive behaviours markedly
interfere with functioning in all spheres. Great distress/difficulty changing fo-
cus or action.
Severity Level 2 "Requiring substantial intervention"
Social communication: Marked deficits in verbal and nonverbal social com-
munication skills; social impairments apparent even with supports in place;
limited initiation of social interactions; and reduced or abnormal responses to

208
Case 36

social overtures from others. For example, a person who speaks simple sen-
tences, whose interaction is limited to narrow special interests, and has mark-
edly odd nonverbal communication.
Restricted, repetitive behaviours: Inflexibility of behaviour, difficulty cop-
ing with change, or other restricted/repetitive behaviours appear frequently
enough to be obvious to the casual observer and interfere with functioning in
a variety of contexts. Distress and/or difficulty changing focus or action.
Severity level 1
Social communication: Without supports in place, deficits in social com-
munication cause noticeable impairments. Difficulty initiating social interac-
tions, and clear examples of atypical or unsuccessful response to social over-
tures of others. May appear to have decreased interest in social interactions.
Restricted, repetitive behaviours: Inflexibility of behaviour causes sig-
nificant interference with functioning in one or more contexts. Difficulty
switching between activities. Problems of organisation and planning hamper
independence.

MANAGEMENT
The National Autism Centre has initiated the National Standards Project,
which has the goal of establishing a set of evidence-based standards for edu-
cational and behavioural interventions for children with autism spectrum dis-
orders. The project has identified established, emerging, and unestablished
treatments.
• In patients, psychiatric care. In December 2015, an expert recom-
mended that children with ASD or Intellectual Disability (ID) can be
treated in general inpatient psychiatric units, with specific accommoda-
tions. The recommendations also set out the information that should be
obtained from children on admission, including the child’s preferences,
means of communication, reinforcement items, sensory sensitivities, etc.
Also emphasised is the importance of screening for co-occurring medical
and psychiatric conditions.
• Special education. Special education is central to the treatment of au-
tistic disorder. Although parents may choose to use various experimental
treatments, including medication, they should concurrently use intensive
individual special education by an educator familiar with instructing chil-
dren who have autistic disorder or a related condition. Intensive behav-
ioural interventions, instituted as early as possible, are indicated for every
child in whom autistic disorder is suspected.
• Speech, Behavioural, Occupational, and Physical Therapies.
Therapies that are reported to help some individuals with autism include
the following:
° Assisted communication-Using keyboards, letter boards, word
boards, and other devices (e.g., the Picture Exchange Communication
System, with the assistance of a therapist.

209
Child Neurology

° Auditory integration training-A procedure in which the individual


listens to specially prepared sounds through headphones.
° Sensory integration therapy-A treatment for motor and sensory mo-
tor problems typically administered by occupational therapists.
° Exercise and physical therapy-Exercise is often therapeutic for indi-
viduals with autistic disorder; a regular program of activity prescribed
by a physical therapist may be helpful.
• Diet. Children with ASD are 6 to 8 times more likely to experience GI
problems (gas/bloating, constipation, diarrhoea) and food allergies. When
these children had frequent GI symptoms, they showed worse irritability,
social withdrawal, stereotypy, and hyperactivity scores compared with those
without frequent symptoms. Dietary consultation may be useful to evalu-
ate the benefits of special diets, including those lacking gluten and casein.
Vitamin B-6 and magnesium are among the vitamins and minerals hypoth-
esised to help some patients.
• Pharmacologic treatment. No pharmacologic agent is effective in the
treatment of the core behavioural manifestations of autistic disorder, but drugs
may be effective in treating associated behavioural problems and comorbid
disorders. The second-generation anti-psychotic agents’ risperidone and
aripiprazole provide beneficial effects on repetitive behaviours in children
with autism spectrum disorder, although these patients may experience
significant adverse effects. Risperidone and aripiprazole have been
approved by the US Food and Drug Administration (FDA) for irritability
associated with autistic disorder. The second-generation anti-psychotic
agent ziprasidone may help to control aggression, irritability, and agitation.
Serotonergic drugs are reportedly beneficial for improving behaviour in
autism. Hyperactivity often improves with methylphenidate therapy.
• Selective Serotonin Reuptake Inhibitors (SSRIs) are widely pre-
scribed for children with autism and related conditions. Beneficial effects
on children and adolescents with autism and other pervasive develop-
mental disorders have been reported with fluoxetine, escitalopram, and
citalopram.

EXPERIMENTAL APPROACHES
i. Secretin therapy. Several anecdotal reports suggested that secretin, a
gastrointestinal hormone that may function as a neurotransmitter, was
an effective intervention for the symptoms of autism. This led to several
scientific studies of secretin for children with autism spectrum disorders
ii. Hyperbaric oxygen therapy. Beneficial effects from hyperbaric oxygen
therapy have been reported in 6 patients with autism. The risks of this
procedure must be weighed against the benefits for individual patients.
Controlled clinical trials and other studies are needed to confirm the po-
tential value of this intervention.

210
Case 36

iii. Intranasal oxytocin. Research suggests that administration of a single


intranasal dose of the hormone oxytocin increases activity in brain regions
associated with reward, social perception, and emotional awareness and
temporarily improves social information processing in children with Au-
tism Spectrum Disorder (ASD).

REFERENCES
1. Akefeldt, A. & Gillberg, C. (1991). Hypomelanosis of Ito in three cases with au-
tism and autistic-like conditions. Developmental Medicine & Child Neurology,
33, 737-743.
2. American Psychiatric Association. Diagnostic and statistical manual of mental dis-
orders. 5th ed. Arlington, VA: American Psychiatric Association; 2013.
3. Gillberg, C. (1993). Autism and related behaviours. Journal of Intellectual Dis-
ability Research, 37, 343-372.
4. Volkmar F, Siegel M, Woodbury-Smith M, King B, McCracken J, State M. Practice
parameter for the assessment and treatment of children and adolescents with au-
tism spectrum disorder. J Am Acad Child Adolesc Psychiatry. 2014 Feb. 53(2):237-
57.
5. Constantino JN, Zhang Y, Frazier T, Abbacchi AM, Law P. Sibling recurrence and
the genetic epidemiology of autism. Am J Psychiatry. 2010 Nov. 167(11):1349-56.
6. Developmental Disabilities Monitoring Network Surveillance Year 2010 Principal
Investigators, Centres for Disease Control and Prevention (CDC). Prevalence of
autism spectrum disorder among children aged 8 years-autism and developmental
disabilities monitoring network, 11 sites, United States, 2010. MMWR Surveill
Summ. 2014 Mar 28. 63 (2):1-21.
7. Marcus RN, Owen R, Kamen L, Manos G, McQuade RD, Carson WH, et al. A
placebo-controlled, fixed-dose study of aripiprazole in children and adolescents
with irritability associated with autistic disorder. J Am Acad Child Adolesc Psychia-
try. 2009 Nov. 48(11):1110-9.
8. American Academy of Pediatrics. Management of Children with Autism Spectrum
Disorder. Pediatrics. 2007 Nov;120(5):1183-1215. Available at http://pediatrics.
aappublications.org/content/120/5/1162.full.
9. Spencer D, Marshall J, Post B, Kulakodlu M, Newschaffer C, Dennen T, et al.
Psychotropic medication use and polypharmacy in children with autism spectrum
disorders. Pediatrics. 2013 Nov. 132(5):833-40.

211
37 Case
A 4-and-a-half-year-old female child was seen, with the concern of ab-
sence of speech and evaluation of new-onset staring spells. Her parents
reported the onset of 1-to 2-minute long spells of unresponsiveness, staring,
drooling, and body stiffness associated with perioral cyanosis and sleepiness
afterwards. She has otherwise been well with no fever, illness, or head trauma.
She was born to a non-consanguineous parent, through spontaneous vaginal
delivery with a birth weight of 3 kg and head circumference of 33 cm. She
sat at 8 and walked at 14 months. She learned some single words by age 20
to 22 months. Later by about 28 months she was reported to have acquired
a vocabulary of some 20 words, and by 30 months learned to communicate
by joining 3 to 4 words. At around 30 months, she started to lose her speech.
Fine motor abilities were also almost normal until the same period after which
deterioration in hand skills was observed. However, she still managed to eat
with a spoon and retained even some sort of pincer grasp, but she was no
more playing constructively with the use of her hands. There was a delay in
achieving sphincter control and indicated her need to void through vocalisa-
tion or showing uneasiness of being incontinent. There was no family history
of mental retardation or other type of pervasive developmental disorder was
present.
Physical examination: Short stature girl with no dysmorphic features
Her expressive speech included unintelligible sounds and screams, and her
ability to comprehend speech was impaired but not totally lacking. Her head
circumference was 46 cm (micro-cephalic), but had no physical deformity such
as scoliosis or kyphosis. She had typical midline purposeless hand movements,
wringing, clapping and mouthing. She also had episodic hyperventilation, and
grinding of teeth. Her pupils are equal, round, and reactive to light. She tracks
objects in all directions without nystagmus. She could sit comfortably and
independently, walked and ran with a wide based slightly ataxic gait. She did
not show interest in grabbing toys. DTR 3+ throughout with bilateral plantar
flexor responses.

Answer the following questions:


• Give the most likely diagnosis.
• Briefly summarise this case.
• Discuss the differential diagnosis.
• Discuss an appropriate diagnostic work-up.
• Discuss the management of this patient.

212
Case 37

DIAGNOSIS
Rett syndrome

SUMMARY
The patient is a 4 and half year-old girl with a history of developmental regres-
sion and loss of purposeful hand movements. Her examination is remarkable
for microcephaly, hand stereotypies, spasticity, hyperreflexia, seizures, and an
ataxic gait.

DIFFERENTIAL DIAGNOSIS
The presentation and natural history of the disease process in this case is
pathognomonic for the diagnosis of Rett syndrome. In the case of a child with
a chronic, progressive encephalopathy, several other metabolic and genetic
disorders must be considered in the differential diagnosis. It should first be
determined whether there is primary involvement of grey matter or white
matter. Grey matter diseases tend to present with seizures and cognitive de-
cline, whereas white matter diseases cause progressive spasticity and vision
loss. Second, it is important to note whether there is other organ involvement.
However, misdiagnosis of Rett syndrome can occur as the disease progresses
through the stages outlined.
The differential diagnosis varies according to the clinical stage of RS.
Conditions that should receive particular consideration in each of the 4 stages
of the syndrome are as follows:
ollows:
• Stage I-Benign congenital hypotonia, cerebral palsy, Prader-Willi syn-
drome, Angelman syndrome, and metabolic disorders (e.g., foetal alcohol
syndrome and trisomy 13)
• Stage II-Autism spectrum disorder, Angelman syndrome, encephalitis,
hearing or visual disturbance, Landau-Kleffner syndrome, psychoses,
slow virus panencephalopathy, tuberous sclerosis, metabolic disorders
(e.g., phenylketonuria and ornithine transcarbamylase deficiency), and
infantile neuronal ceroid lipofuscinosis
• Stage III-Spastic ataxia, cerebral palsy, spinocerebellar degeneration, leu-
kodystrophies, neuroaxonal dystrophy, Lennox-Gastaut syndrome, and
Angelman syndrome
• Stage IV-Other degenerative disorders
A school-age girl in the stagnant phase of the disease might be diagnosed
with cerebral palsy, mental retardation, epilepsy, and/or an ataxia syndrome.
Adolescent girls might have significant spasticity and be wheelchair bound.
They might be diagnosed with spastic cerebral palsy. The pattern of neurologic
deterioration that occurs over the first several years of life is the key to diagno-
sis and a high suspicion for the disorder must be maintained.
Lysosomal storage diseases, mitochondrial disorders, and peroxisomal disor-
ders can involve organs outside of the central nervous system (liver, spleen,

213
Child Neurology

eyes). Whether there is peripheral nervous system in addition to central nerv-


ous system involvement is also important. Lysosomal storage diseases and mi-
tochondrial disorders may also cause peripheral neuropathy. The inheritance
pattern is also relevant. The child in this case is female and there is no family
history of seizures, autism, or developmental delay. Thus, this may represent
an X-linked, sporadic disorder.
The overall presentation in this case is one of a female child with a neuro-
degenerative process that primarily affects the grey matter but also has white
matter involvement. She does not have dysmorphic facial features and there
is no other apparent organ involvement. Her reflexes are increased, which ex-
cludes a concomitant peripheral neuropathy. Thus, disorders that deserve the
highest consideration include mitochondrial diseases and Neuronal Ceroid
Lipofuscinosis (NCL), in addition to genetic disorders such as Angelman syn-
drome. Mitochondrial disorders vary widely in their clinical presenta
presentations but
may present with developmental regression, seizures, and autistic features.
NCL is caused by the accumulation of lipopigment in neurons and other cells.
It occurs in multiple forms and presents with cognitive decline, epilepsy, vi-
sion loss/blindness, and motor disabilities. Angelman syndrome results from
a mutation on chromosome 15 and demonstrates considerable overlap with
Rett syndrome. Microcephaly, seizures, ataxia, gait abnormalities, and autistic
features may be seen. Finally, given that the patient in this case was not born
in the United States, TORCH infections, especially HIV, are an important ad-
ditional consideration.

DIAGNOSTIC WORK-UP
Rett Syndrome (RS) is a neurodevelopmental disorder that occurs almost ex-
clusively in females and has a typically degenerative course. It is related to
various mutations on the MECP2 gene. The diagnostic approach to the patient
presented in this case involves investigation of both her progressive develop-
mental disability as well as her new-onset epilepsy. The history varies accord-
ing to the clinical stage. Common symptoms reported include the following:
• Stage I-Gross motor development delay, disinterest in play, and loss of eye
contact; hypotonia; hand wringing; unusual placidity and calmness; vague
and nonspecific early symptoms; breath-holding spells.
• Stage II-Deterioration; autistic like behaviour; stereotypic hand move-
ments during wakefulness; breathing irregularities; seizures and vacant
spells that resemble seizures; sleep disorders, intermittent strabismus, and
irritability.
• Stage III-Some improvement in behaviour, hand use, and communication
skills; good eye contact and attempts to communicate intent; continued
mental impairment and hand stereotypies; increasing rigidity, bruxism,
and involuntary tongue movements; motor dysfunction and seizures;
continued breathing irregularities; poor weight gain despite good appetite;
difficult feeding and some degree of oral motor dysfunction.

214
Case 37

• Stage IV-No additional deterioration of cognitive skills, communication


skills, or hand skills; increased motor problems; cessation of walking; pos-
sible reduction of seizure frequency.
• A detailed birth, developmental, medical, and family history is an essential
first step in forming a useful differential diagnosis in these cases. Physical
examination findings such as an abnormal head circumference, dysmor-
phic facial features, neurocutaneous stigmata, and/or hepatosplenomegaly
are relevant. Ophthalmologic, cardiac, or musculoskeletal abnormalities
might also be detected on a routine physical examination, pointing toward
a specific diagnosis.
The presentation in this case is highly suggestive of Rett syndrome and an ex-
haustive work-up is not necessary now that the causative gene has been identi-
fied. Clinical genetic testing for the MECP2 mutation can be performed to
confirm the diagnosis.
If the diagnosis is less clear, a screening metabolic and genetic work-up for
a chronic, progressive encephalopathy may include a basic chemistry panel,
CBC, liver function tests, thyroid function tests, HIV testing, plasma amino
acids, urine organic acids, serum lactate/pyruvate, serum mitochondrial DNA,
serum lysosomal enzymes, and high-resolution karyotype. Specific genetic
testing can be performed to exclude Angelman syndrome. A formal ophthal-
mologic examination can also be very helpful in a screening evaluation.

MANAGEMENT
There are no specific therapies for Rett syndrome. Most people with Rett
syndrome benefit from well-designed interventions no matter what their age,
but the earlier that treatment begins, the better. With therapy and assistance,
people with Rett syndrome can participate in school and community activities.
Referral to a geneticist is appropriate to review the genetic basis of the disease
and to provide genetic counseling where the parents consider having additional
children. Referral to a developmental paediatrician might also be appropriate
to help with anticipated behavioural issues, screaming spells, and sleep distur-
bances. Although this patient currently has no musculoskeletal problems, in
general, early referral to an orthopaedist should be considered as scoliosis and
contractures develop.
These treatments, forms of assistance, and options for medication generally
aim to slow the loss of abilities, improve or preserve movement, and encour-
age communication and social contact. A list of treatment options is present-
ed below; the need for these treatments depends on the severity of different
symptoms.

Physical Therapy/Hydrotherapy
• Improves or maintains mobility and balance
• Reduces mishap in back and limbs
• Provides weight-bearing training for patients with scoliosis

215
Child Neurology

Occupational Therapy
• Improves or maintains use of the hands
• Reduces stereotypic hand movements such as wringing, clapping, rub-
bing, or tapping
• Teaches self-directed activities like dressing and feeding

Speech-Language Therapy
• Teaches nonverbal communication
• Improves social interaction

Feeding Assistance
• Supplements calcium and minerals to strengthen bones and slow scoliosis
• High-calorie, high-fat diet to increase height and weight (over weight
needs to be avoided as well)
• Insertion of a feeding tube

Physical Assistance
• Braces or surgery to correct scoliosis
• Splints to adjust hand movements

Medication
• To reduce breathing problems
• To eliminate problems with abnormal heart rhythm
• To relieve indigestion and constipation
• To control seizures

REFERENCES
1. Rett A. [On a unusual brain atrophy syndrome in hyperammonemia in child-
hood]. Wien Med Wochenschr 1966; 116:723.
2. Neul JL, Kaufmann WE, Glaze DG, et al. Rett syndrome: revised diagnostic criteria
and nomenclature. Ann Neurol 2010; 68:944.
3. Percy AK, Lane JB, Childers J, et al. Rett syndrome: North American database. J
Child Neurol 2007; 22:1338.
4. Percy AK, Neul JL, Glaze DG, et al. Rett syndrome diagnostic criteria: lessons from
the Natural History Study. Ann Neurol 2010; 68:951.
5. Pidcock FS, Salorio C, Bibat G, et al. Functional outcomes in Rett syndrome. Brain
Dev 2016; 38:76.

216
Case 38
A 5-year-old boy was brought in the ER with complaints of seizures
that started as right focal fits, becoming generalised, associated with
deterioration of previously normal functions of vision, hearing and intelligence
of two months’ duration. This boy was the 4th child of second degree parental
consanguinity. The mother had one still birth child and has 2 females and one
male child in normal health. The patient’s neonatal period and developmental
milestones were normal. He was apparently in good health, when he developed
hyperpigmentation of the lips, buccal mucosa, nails and body associated with
occasional history of vomiting.
His physical examination revealed an alert, normally nourished boy with
diffuse hyperpigmentation more marked over lips, toe and finger nails. He
was groping for objects while walking implying impaired vision. He spoke
irrelevantly at times and laughed unprovoked. His cardiovascular, respiratory
and abdominal systems were clinically normal. Power and tone in all muscles
and the sensory system was clinically normal. Funduscopic examination of
both eyes was normal. Visual acuity testing revealed right homonymous
hemianopia.
His investigations showed, sodium of 120 meq/L, potassium of 5 meq/L,
bicarbonate of 18 meq/L and chloride of 88 meq/L. Random blood sugar was
128 mg/dl. X-ray chest showed microcardia with normal lung fields. Serum
cortisol was 3.1 mcg/dl at 8 AM and 2 mcg/dl at 8PM. CT scan brain showed
hypodense areas over the parieto-occipital areas consistent with cerebral white
matter degeneration.

Answer the following questions:


• Give the most likely diagnosis.
• Summarise the case briefly.
• Provide a differential diagnosis.
• Discuss an appropriate diagnostic work-up.
• Discuss the management of the patient.

217
Child Neurology

DIAGNOSIS
Adrenoleukodystrophy (ALD)

SUMMARY
A 5-year-old boy had seizures with deterioration of vision, hearing and intel-
ligence of two months’ duration. The patient’s neonatal period and develop-
mental milestones were normal. He was apparently in good health, when he
developed hyperpigmentation of the lips, buccal mucosa, nails and body asso-
ciated with a history of vomiting occasionally. Skin hyperpigmentation is fairly
specific for adrenal insufficiency. The death of a male child on the maternal
side of the family is consistent with an X-linked inheritance pattern. Thus,
ALD is the most likely diagnosis.

DIFFERENTIAL DIAGNOSIS
Adrenoleukodystrophy is an X-linked recessive genetic disorder caused by an
abnormality in the ABCD1 gene on the X chromosome. This condition affects
the white matter of the nervous system and the adrenal cortex. Some affected
individuals have adrenal insufficiency, leading to abnormalities in blood pres-
sure, heart rate, sexual development and reproduction. Some of those affected
experience serious neurological problems that can affect mental function and
lead to disability and reduced life span.
ALD has several clinical phenotypes in both child and carrier females. The two
primary forms of the disease include cerebral ALD and Adrenomyeloneuropathy
(AMN). It manifests with rapidly progressive inflammatory, demyelinating
changes in the central nervous system white matter. The parietal and occipital
white matter is affected first. Initial symptoms often involve subtle behavioural
problems, social withdrawal, and cognitive decline. Affected boys are occa-
sionally erroneously diagnosed as having attention deficit hyperactivity dis-
order when behaviour and learning problems manifest at school. Neurologic
deterioration inevitably ensues, resulting in spastic paraparesis, gait abnormali-
ties, dysphagia, vision loss, and hearing loss. Seizures occur as a late manifesta-
tion in 30% of patients. Most children progress to a vegetative state within 3
years of the onset of symptoms. Approximately 70% of affected boys will also
have adrenocortical insufficiency. The child in this case presents with a neuro-
degenerative disorder that primarily affects the white matter (pyramidal tract
signs, optic atrophy), but which also dem demonstrates grey matter involvement
(cognitive and behavioural abnormalities).
Symptoms of the following disorders can be similar to those of adrenoleukod-
ystrophy. Comparisons may be useful for a differential diagnosis:
• Addison disease is a rare disorder of the adrenal glands. Symptoms may
result from chronic and progressive low level function of the adrenal gland
resulting in deficiencies of the hormones cortisol and aldosterone. The
deficiency of these hormones leads to low levels of sodium and chloride
and high levels of potassium in the blood. The imbalance of electrolytes

218
Case 38

causes increased water excretion, hypotension, and dehydration. The ma-


jor symptoms of Addison disease may include fatigue, weakness, anorexia,
frequent urination, gastrointestinal discomfort and changes in skin pig-
mentation. However, CNS involvement with white matter demyelination
is not a part of primary Addison’s disease.
• Metachromatic Leukodystrophy (MLD) is an autosomal recessive
leukodystrophy resembling adrenoleukodystrophy in CNS manifesta-
tions and is the closest diagnosis. MLD is characterised by the accumula-
tion of a sulfatide (a sphingolipid) in the brain and other areas of the body
(i.e. liver, gall bladder, kidneys, and/or spleen). Myelin is lost from areas
of the central nervous system due to the buildup of sulfatide. Symptoms
of metachromatic leukodystrophy may include convulsions, seizures,
personality changes, spasticity, progressive dementia, motor disturbances
progressing to paralysis, and/or visual impairment leading to blindness.
Infantile, juvenile, and adult onset forms of metachromatic leukodystro-
phy have been distinguished.
• Krabbe disease, also known as globoid cell leukodystrophy, is an autoso-
mal recessive lipid storage disorder caused by a deficiency of the enzyme
Galactocerebrosidase (GALC), which is necessary for the metabolism of
the sphingolipids galactosylceramide and psychosine. Failure to break
down these sphingolipids results in degeneration of the myelin sheath
(demyelination). Characteristic globoid cells appear in affected areas of
the brain. This metabolic disorder is characterised by progressive neuro-
logical dysfunction such as mental retardation, paralysis, blindness, deaf-
ness and pseudobulbar palsy.
• Alexander disease usually presents in infancy with megalencephaly,
spasticity, seizures, and developmental regression. However, juvenile on-
set with ataxia, spasticity, and bulbar dysfunction can occur. The brain
MRI usually features frontotemporal white matter signal abnormalities.
• Multiple Sclerosis (MS) is a chronic disease affecting the myelin sheath
of the brain and spinal cord. It may be progressive, relapsing and remitting,
or stable. Symptoms may include visual and speech problems, numbness,
walking difficulty and loss of bladder or bowel control. MS affects adults,
and its cause is unknown.
• Other leukodystrophies include Childhood Ataxia with Central Nervous
System Hypomyelination (CACH) or vanishing white matter disease, as
well as Megalencephalic Leukoencephalopathy with Subcortical Cysts
(MLC).
° CACH typically presents with new-onset ataxia in children 1 to 5
years of age. Symptoms can begin spontaneously, but often start after
minor trauma or a febrile illness. Spasticity, tremor, hyperreflexia, and
seizures occur with declining neurologic status. A distinctive pattern
of diffuse white matter T1-weighted hypointensity, and T2-weighted
hyperintensity is seen on brain MRI.

219
Child Neurology

° Finally, MLC is an autosomal recessive disorder caused by a muta-


tion on chromosome 22. Affected children experience macrocephaly,
ataxia, spasticity, and cognitive deterioration. The brain MRI demon-
strates diffuse white matter signal change and subcortical cysts.

DIAGNOSIS
The child in this case presents with a neurodegenerative course and deserves a
thorough diagnostic evaluation. ALD should be considered high in the differ-
ential diagnosis of any school-age boy presenting with behavioural and cogni-
tive decline, long tract signs, and vision loss. Serum VLCFAs should be done
early in his work-up. The presence of elevated VLCFAs is consistent with the
diagnosis. Confirmatory mutational analysis can now be performed and is
most useful for genetic counseling.
• The concentration of Very Long Fatty Acids (VLFA) in blood plasma is
elevated in 99% of child with ALD and in approximately 85% of female
carriers of the abnormal ABCD1 gene.
• Molecular testing for the ABCD1 gene is available and is used primarily
to confirm a diagnosis if other testing is not conclusive, to provide genetic
counseling to family members and for prenatal diagnosis.
• Adrenal function tests are abnormal in 90% of boys with ALD who
have neurologic symptoms and in approximately 70% of men with
adrenomyeloneuropathy.
• A brain MRI with and without contrast is indicated. The classic pattern
demonstrates parieto-occipital white matter demyelinating lesions with
a leading edge of contrast enhancement. With time, the changes spread
anteriorly. Once an MRI is performed, the diagnosis is usually clear and
an exhaustive work-up to exclude other conditions is unnecessary. MRI
changes are typically seen prior to the onset of symptoms, which has prog-
nostic value. The presence of contrast enhancement of white matter le-
sions is associated with rapid progression of the disease. Thus, brain MRI
is a useful tool for selecting appropriate children for treatments such as
bone marrow transplant. The NAA-to-choline ratio on MR spectroscopy
has also been used as a diagnostic and predictive tool in ALD.
• Once the diagnosis of ALD is recognised, evaluation by an endocrinolo-
gist is necessary to assess adrenal insufficiency. An impaired cortisol re-
sponse to Adrenocorticotropic Hormone (ACTH) stimulation is seen.
Consultation by an ophthalmologist also is required to evaluate this
child’s vision loss and to document a baseline eye exam. VERs also may be
helpful in assessing the visual pathway.

MANAGEMENT
• The management of children with ALD involves treating the underly-
ing metabolic abnormality and controlling the anticipated complications.

220
Case 38

All boys with adrenal insufficiency should receive replacement adrenal


hormone therapy, although this does not alter the neurologic course of
the disease.
• Bone Marrow Transplant (BMT) is now being used in boys in the ear-
ly inflammatory stages of the disease. BMT stabilises symptoms of the
disease. Candidates must be chosen carefully, however, as BMT carries
significant risks. Dietary restriction of VLCFAs is sometimes prescribed.
Lorenzo’s oil consists of a 4:1 mixture of glyceryl trioleate and glyceryl
trierucate. It bypasses the metabolic defect and normalises VLCFA lev-
els in the blood within 4 weeks. Like BMT, Lorenzo’s oil has shown lit-
tle benefit in boys with advanced disease. However, a recent study has
suggested that Lorenzo’s oil might have a preventative effect if started in
asymptomatic boys younger than 6 years of age who have normal brain
MRI scans. Finally, clinical trials have investigated the use of lovastatin to
lower serum VLCFA levels. The effect of lovastatin on the clinical course
of the disease is still unclear.
• Caring for a child with ALD involves multidisciplinary care as the disease
progresses. Feeding difficulties ultimately occur and evaluation by a nutri-
tionist and a gastroenterologist becomes necessary.
• Rehabilitation specialists may become involved in an affected child’s care
to help manage spasticity and to prescribe necessary adaptive equipment.
Ongoing physical and occupational therapy may be beneficial. A geneticist
should be involved early in an affected child’s care to determine mater-
nal carrier status, screen other family members, and help make decisions
about future pregnancies.
• Affected individuals can benefit from supportive care from psychologists,
educators, physical therapists, urologists, and family and vocational coun-
selors. Genetic counseling is recommended for affected individuals and
their family members.
• Investigational therapies: Studies are underway to determine if
Lorenzo’s oil therapy is beneficial in reducing the severity of neurologi-
cal symptoms in individuals who do not yet have neurological problems.
This therapy is not effective in altering the progression of the disease if the
brain is already affected.
• The Kennedy Krieger Institute and the General Clinical Research Centre
at Johns Hopkins Hospital are conducting a large study of Lorenzo’s oil
therapy in patients with Adrenomyeloneuropathy (AMN).
• Studies are underway to determine if lovastatin and 4-phenylbutyrate are
effective therapies for ALD.

REFERENCES
1. Swaiman KF. Lysosomal diseases. In: Swaiman KS, Ashwal S, eds. Pediatric Neu-
rology: Principles and Practice. 5th ed. St. Louis, Mo: Mosby; 2012.

221
Child Neurology

2. Expanded Access for Lorenzo’s Oil (GTO/GTE) in Adrenoleukodystrophy. Study


record detail available at: https://clinicaltrials.gov/ct2/show/NCT02233257 (Ac-
cessed on July 13, 2016).
3. Ferrer I, Aubourg P, Pujol A. General aspects and neuropathology of X-linked adre-
noleukodystrophy. Brain Pathol 2010; 20:817.
4. Moser HW, Loes DJ, Melhem ER, et al.. X-Linked adrenoleukodystrophy: overview
and prognosis as a function of age and brain magnetic resonance imaging abnor-
mality. A study involving 372 patients. Neuropediatrics 2000; 31:227.
5. Percy AK, Rutledge SL. Adrenoleukodystrophy and related disorders. Ment Retard
Dev Disabil Res Rev 2001; 7:179.
6. Dubey P, Raymond GV, Moser AB, et al.. Adrenal insufficiency in asymptomatic
adrenoleukodystrophy patients identified by very long-chain fatty acid screening.
J Pediatr 2005; 146:528.
7. Polgreen LE, Chahla S, Miller W, et al.. Early diagnosis of cerebral X-linked adreno-
leukodystrophy in boys with Addison’s disease improves survival and neurological
outcomes. Eur J Pediatr 2011; 170:1049.
8. Engelen M, Barbier M, Dijkstra IM, et al.. X-linked adrenoleukodystrophy in wom-
en: a cross-sectional cohort study. Brain 2014; 137:693.
9. Vogel BH, Bradley SE, Adams DJ, et al.. Newborn screening for X-linked adreno-
leukodystrophy in New York State: diagnostic protocol, surveillance protocol and
treatment guidelines. Mol Genet Metab 2015; 114:599.
10. Berger J, Pujol A, Aubourg P, Forss-Petter S. Current and future pharmacological
treatment strategies in X-linked adrenoleukodystrophy. Brain Pathol 2010; 20:845.

222
Case 39
A 10-year-old girl, presented with progressive headache, nausea and
vomiting of subacute onset preceded by a 5-month history of convulsive
seizures. It is now described as constant, pounding, severe, and all over the
head. The pain has awakened her up from sleep for the last three nights. Her
seizures are described as staring, drooling, and right-sided jerking. Her birth
history is unremarkable. She walked at 16 months and did not speak in phrases
until she was almost 3 years old. She is now in first grade. The family history
is noncontributory.
General physical examination: Afebrile. There is no nuchal rigidity. Her
margins of the optic discs are blurred. Several small white macules are noted
on the trunk and right leg. Small, raised, erythematous papules are seen on
the nose and cheeks. She was alert and irritable but her speech was fluent.
Her pupils are equal, round, and reactive to light. She tracked objects in all
directions. Her facial movements were symmetric. She had a normal gag
with normal bulk and tone. Her strength was 5/5 throughout. There was no
dysmetria. Reflexes were 3+ symmetrically throughout with bilateral plantar
extensor responses.
A Computed Tomography (CT) scan revealed a voluminous mass in her peri-
lateral ventricular area with similar attenuation to that of cortical gray matter,
following administration of a contrast product, the mass showed marked
enhancement.

Answer the following questions:


• Briefly summarise the case.
• What is the most likely diagnosis.
• Provide a differential diagnosis.
• Discuss an appropriate diagnostic work-up.
• Discuss the management of the patient.

223
Child Neurology

DIAGNOSIS
Tuberous sclerosis complex with subependymal giant cell astrocytoma

SUMMARY
This girl with a history of seizures and cognitive delay has now presented with
the subacute development of headaches and vomiting. Her general exam dem-
onstrated hypopigmented macules and facial angiofibromas that are consistent
with a diagnosis of tuberous sclerosis complex. Her neurologic examination
was significant for bilateral papilloedema, brisk reflexes, and bilateral plantar
extensor responses.

DIFFERENTIAL DIAGNOSIS
• Subependymal Giant cell Astrocytoma (SEGA) is a slowly growing tu-
mour of unknown histogenesis mainly arising in the periventricular
regions adjacent to the foramen of Monro, which causes increased in-
tracranial pressure, seizures, and focal neurologic signs. The incidence of
SEGAs in Tuberous Sclerosis (TSC) varies from 5% to 14%, and may also
be detected prenatally or at birth, although they are much more likely to
arise during childhood or adolescence. Most SEGAs will show avid en-
hancement after contrast administration; however, a growing subependy-
mal lesion even in the absence of enhancement should be considered a
SEGA. Although histologically confirmed SEGA is considered pathog-
nomonic for TSC. This is an autosomal dominant phacomatosis due in
60% of cases to spontaneous mutation in two Tumour Suppressor Genes
(TSC1 and TSC2). It is manifested by the development of benign tu-
mours in many organs (heart, kidney, skin, and brain). The cutaneous and
neurological involvement is almost constant. The central nervous system
is represented by cortical tubers, subependymal nodules, subependymal
giant cell astrocytomas and retinal astrocytoma. The risk of mental retar-
dation is high in this condition especially when associated with seizures
in the first year of life. The clinical features of SEGA are due to hydro-
cephalus, raised intracranial pressure and seizures. Hydrocephalus occurs
due to obstruction of the cerebrospinal fluid pathway by the tumour it-
self. As the association of SEGA and tuberous sclerosis is common, the
characteristic symptoms of this disease are often present and must be in-
vestigated. CT and Magnetic Resonance (MR) imaging characteristics are
usually nonspecific, and the location of the mass and the patient’s age are
useful indicators of the specific tissue diagnosis. Although nonspecific,
the CT and MR findings objectify a well-circumscribed mass at the fo-
ramen of Monro, which frequently exhibits partial calcification or cyst
formation. Enhancement following contrast administration is strong but
inhomogeneous.
• This child’s symptoms are highly suggestive of elevated intracranial
pressure. Her headache is constant, severe, and progressively worsening.

224
Case 39

It is associated with persistent vomiting and awakens her from sleep. In a


patient with TSC, a SEGA obstructing the flow of CSF is the most likely
etiology.
• This may occur with a wide range of pediatric brain tumours, espe-
cially those located in the posterior fossa (brainstem glioma, medulloblas-
toma, cerebellar astrocytoma, ependymoma). Therefore, when signs of
TSC are not present, other etiologies of hydrocephalus would need to be
considered. More specifically, her history is most consistent with non-
communicating hydrocephalus, which is frequently caused by a mass le-
sion obstructing the flow of CSF through the ventricular system.
• Aqueductal stenosis is caused by partial or complete obstruction of the
aqueduct of Sylvius. It may be hereditary or acquired as a consequence of
haemorrhage, infection, a vascular malformation, or a neoplasm. It rep-
resents another type of obstructive hydrocephalus that may occur. The
patient in this case is Mexican and experienced seizures as a young child.
An alternate diagnosis might include neurocysticercosis that resulted in
seizures with a new lesion that is now causing obstructive hydrocep
hydrocephalus.
halus.
• Communicating hydrocephalus causes impairment of CSF absorption
through the arachnoid granulations and is seen in conditions such as menin-
gitis, encephalitis, subarachnoid haemorrhage, and intraventricular haemor-
rhage. The patient has no history of fevers, photophobia, or nuchal rigidity
that would suggest a central nervous system infection.
• The headache described in this case is not consistent with a tension-type
or migraine headache. Pseudotumour cerebri causes headaches and papil-
loedema but typically does not cause other signs of hydrocephalus, such as
vomiting and nocturnal pain.

DIAGNOSTIC APPROACH
• Before the 2012 consensus conference, the diagnostic criteria developed
for TSC during the 1998 consensus meeting were still in use. At the 2012
Washington Consensus Conference, it was decided by the invited expert
panel to document the diagnostic criteria related to TSC brain lesions in
the following manner:
° The presence of tubers (and other types of cortical dysplasia, such as
cortical migration lines), SENs (subependymal nodules), or SEGAs
will each individually be defined as major criteria (two major criteria
will suffice for the diagnosis of TSC as previously defined in 1998).
° For diagnostic purposes, the definition of SEGAs will include a lesion
at the caudothalamic groove with either a size of more than 1 cm in
any direction or a subependymal lesion at any location that has shown
serial growth on consecutive imaging, regardless of size. Most SEGAs
will show avid enhancement after contrast administration; however,
a growing subependymal lesion even in the absence of enhancement
should be considered a SEGA.

225
Child Neurology

• Screening protocols. Hence brain imaging, preferably magnetic reso-


nance imaging with and without contrast, should be performed every 1 to
3 years until the age of 25 years. Because of a lack of knowledge of SEGA
growth behaviour beyond 25 years of age, follow-up magnetic resonance
imaging may not be needed every 3 years but intervals may be prolonged
in the presence of a stable lesion and a stable patient. New onset of symp-
toms such as headaches, visual complaints, nausea or vomiting, or in-
crease in seizure activity should trigger an earlier scan. Similarly, a growing
SEGA should prompt a more frequent clinical and radiological follow-up.
Parents and patients should be educated regarding relevant symptoms that
should prompt referral to medical evaluation.
• This patient should undergo an emergent head CT with and without con-
trast. Neurosurgery should also be contacted immediately. Further imag-
ing with a brain MRI with and without gadolinium will be necessary pre-
operatively if the patient is stable. Basic laboratory studies, such as a basic
chemistry panel, CBC, and PT/PTT, also should be done.
• Because her TSC has not yet been diagnosed, she will require a com-
plete work-up for associated complications. Guidelines for the evaluation
of a newly diagnosed patient with TSC and family members have been
published.
• An ophthalmology evaluation should be performed to monitor for resolu-
tion of papilloedema and for retinal abnormalities seen in TSC.
• She should undergo an ECHO and ECG to investigate possible cardiac
rhabdomyomas and arrhythmias. A renal ultrasound must be performed
to rule out renal angiomyolipomas and cysts.
• Epilepsy is the most common presenting symptom of the disorder and oc-
curs in 90% of patients over a lifetime. All seizure types may develop, with
the exception of pure absence epilepsy. It most often becomes manifest in
childhood, and infantile spasms may be the presenting symptom in one-
third of patients.
• Infantile spasms are the most potentially devastating epileptic compli-
cation of TSC and are a risk factor for neurocognitive impairments and
mental retardation.

MANAGEMENT
There is no cure for TSC. Treatment is aimed at the early identification of
potentially progressive lesions, minimisation of complications, and relief of
symptoms. The presentation in this case represents a neurologic emergency.
• Treatment of SEGAs has been solely surgical because of a lack of respon-
siveness to other strategies such as chemotherapy or radiation. These mo-
dalities may also be associated with an increased risk of secondary malig-
nancies. Generally, it is agreed that small tumours are usually less invasive,
and that resection of non-invasive small tumours could be possible, if

226
Case 39

diagnosed while still asymptomatic, it is associated with excellent clinical


outcomes, with low morbidity and mortality. However, when diagnosed
at a later stage, the tumour more often affects and invades neighbouring
structures such as the fornix, hypothalamus, basal ganglia, and genu of
internal capsule, and resection is associated with higher surgical morbidity
and mortality.
• Recent prospective trials documented successful SEGA shrinkage with
mTOR inhibitors (mTORi). The mTOR inhibitor everolimus sig-
nificantly decreased the volume (>50%) of SEGAs in 35% to 42% at 6
months of treatment.
• Long-term efficacy and safety has been demonstrated for up to 3.5 years in
prospective studies with everolimus. It may be possible to reduce the dose
of mTORi after an initial response with preservation of tumour volume
reduction.
• Also, although usually insignificant, mTORi use is associated with side ef-
fects, most common of which are stomatitis and upper respiratory tract
infections. Additionally, it has been shown that cessation of treatment
may result in tumour regrowth. Current practice still is dependent on the
experience of the individual physician. Despite the growing evidence on
mTORi-induced SEGA shrinkage, many centres still strictly advocate sur-
gical treatment, whereas others prefer medical therapy. The risk of surgi-
cal morbidity must be weighed against a potential lifelong medical therapy
with long-term risks yet to be determined. Incompletely resected SEGA
will grow again.
• Treatment depends on the type of epilepsy encountered; however, car-
bamazepine, oxcarbazepine, lamotrigine, topiramate, and phenytoin are
particularly beneficial in those patients with partial seizures.
• The ketogenic diet may also be beneficial. Patients can additionally benefit
from the early evaluation and provision of surgical interventions for epi-
lepsy. These options incorporate focal and multi-focal cortical resection,
corpus callosotomy, or Vagal Nerve Stimulation (VNS). Best outcomes
were for patients whose seizures began after age 1 year, those who had
unilateral focality on ictal or interictal EEG, and those who underwent
lobectomy. There is credible evidence that vigabatrin, an irreversible
GABA transaminase inhibitor, is particularly effective for the treatment of
infantile spasms in TSC.

REFERENCES
1. Lasarge CL, Danzer SC. Mechanisms regulating neuronal excitability and seizure
development following mTORi pathway hyperactivation. Front Mol Neurosci.
2014; 7:18.
2. Goh S, Butler W, Thiele EA. Subependymal giant cell tumors in tuberous sclerosis
complex. Neurology. 2004; 63:1457.

227
Child Neurology

3. Hallett L, Foster T, Liu Z, et al. Burden of disease and unmet needs in tuberous
sclerosis complex with neurological manifestations: systematic review. Curr Med
Res Opin 2011; 27:1571.
4. Goodman M, Lamm SH, Engel A, et al. Cortical tuber count: a biomarker indicat-
ing neurologic severity of tuberous sclerosis complex. J Child Neurol. 1997 Feb.
12(2):85-90.
5. Northrup H, Krueger DA. Tuberous sclerosis complex diagnostic criteria update:
recommendations of the 2012 Iinternational Tuberous Sclerosis Complex Con-
sensus Conference. Pediatr Neurol. 2013 Oct. 49(4):243-54.
6. Krueger DA, Care MM, Holland K, et al.. Everolimus for subependymal giant-cell
astrocytomas in tuberous sclerosis. N Engl J Med. 2010 Nov 4. 363(19):1801-11.
7. Hancock E, Osborne JP. Vigabatrin in the treatment of infantile spasms in tuberous
sclerosis: literature review. J Child Neurol.. 1999 Feb. 14(2):71-4.
8. Camposano SE, Major P, Halpern E, Thiele EA. Vigabatrin in the treatment of
childhood epilepsy: a retrospective chart review of efficacy and safety profile. Epi-
lepsia. 2008 Jul.
9. Krueger DA, Northrup H, International Tuberous Sclerosis Complex Consensus
Group. Tuberous sclerosis complex surveillance and management: recommenda-
tions of the 2012 International Tuberous Sclerosis Complex Consensus Confer-
ence. Pediatr Neurol 2013; 49:255.
10. Northrup H, Krueger DA, International Tuberous Sclerosis Complex Consensus
Group. Tuberous sclerosis complex diagnostic criteria update: recommendations
of the 2012 Iinternational Tuberous Sclerosis Complex Consensus Conference.
Pediatr Neurol 2013; 49:243.
11. O’Callaghan FJ, Martyn CN, Renowden S, et al.. Subependymal nodules, giant cell
astrocytomas and the tuberous sclerosis complex: a population-based study. Arch
Dis Child 2008; 93:751.

228
Case 40
A 9-year-old right-handed girl presented with a sudden-onset clinical
picture of right-sided weakness and aphasia during bathing, followed
by a left-sided throbbing headache 2 hours later, associated with vomiting
and photophobia. She was adopted during early childhood and did not have a
relationship with her family; so, a more accurate description on her parent’s
history was not available but she had previously suffered similar clinical
episodes 2 years before. She has no brothers or sisters.
Her general physical examination showed a left gaze preference,
predominantly unfluent speech, and right facial palsy with right hemiplegia.
On admission, she developed intermittent headache. There was a mild right-
sided ptosis but movements of the right eye were normal. Her face was
asymmetric with right sided upper neuron type of facial palsy. Her palate
movements were normal and tongue was midline. She had normal bulk and
tone on the left side but there was hypertonia with brisk reflexes on the right
side with a power of 3/5. Blood pressure was normal. Fundoscopy revealed no
evidence of papilloedema.
Urgent unenhanced CT was normal, but perfusion CT revealed increased
Mean Transit Time (MTT) and diminished Cerebral Blood Flow (CBF)
throughout the entire left cerebral hemisphere not confined to a particular
vascular territory including the territory of anterior (ACA), middle (MCA),
and Posterior Cerebral Arteries (PCA). Cerebral Blood Volume (CBV) was
normal. Findings were consistent with hypoperfusion throughout all of the
left cerebral hemisphere. Cerebrovascular study was completed with an
angiography TC and carotid ultrasound with no alterations. Transcranial
Doppler revealed a generalised acceleration in both middle cerebral arteries
with absence of arterial occlusion. A Diffusion-Weighted Brain-Magnetic
Resonance Imaging (DWB-MRI) was performed 3 days after clinical onset
and it showed no restricted diffusion on the region of perfusion abnormality
with normal signal intensity of the brain parenchyma. Her hemiplegia and
facial nerve palsy resolved spontaneously 8 hours after onset. However, the
headache remained on the left side for 3 days.

Answer the following questions:


• Give the most likely diagnosis.
• Briefly summarise the case.
• Provide the possible differential diagnosis.
• Discuss an appropriate diagnostic work-up.
• Discuss the management of the patient.

229
Child Neurology

DIAGNOSIS
Hemiplegic Migraine

SUMMARY
A 9-year-old with right-sided hemiplegia and aphasia, followed by a left-sided
throbbing headache 2 hours later, associated with vomiting and photophobia.
She had previously suffered similar clinical episodes in the past 2 years. Her
brain imaging study was normal. Right-sided hemiplegia resolved spontane-
ously 8 hours after onset. However, the headache remained on the left side
for 3 days. On the basis of these clinical findings, a diagnosis of hemiplegic
migraine was considered most likely.

DIFFERENTIAL DIAGNOSIS
In 2004, the IHS (International Headache Society) issued an overall revision of
the classification of headaches. This revision affects some syndromes unique
to children that historically were considered manifestations of migraine. These
syndromes-cyclical vomiting, abdominal migraine, benign paroxysmal vertigo
of childhood-are usually associated with a positive family history and often
evolve into more characteristic migraine symptoms with age. They are now
considered “childhood periodic syndromes that are commonly precursors of
migraine” in the revised International Headache Society (IHS) classification
system. The hallmark of hemiplegic migraine is unilateral weakness that ac-
companies a migraine headache attack. The weakness is a manifestation of
motor aura and occurs with other forms of aura that impair vision, speech,
or sensation. This form of migraine with aura may occur either in families or
only in one individual (sporadic). The diagnosis of migraine is made on the
basis of a typical history of paroxysmal headaches; the child is normal between
episodes. However, the diagnosis may be difficult in young children who may
not be able to describe their symptoms or in patients with migraine variants
who may not have typical symptoms.
• Migraine with aura. Migraine with aura was formerly known as classic
migraine. Approximately 14 to 30 percent of children report an aura pre-
ceding a migraine, although the incidence may be underestimated. Auras
represent progressive neurologic deficits or disturbances with subsequent
complete recovery. Visual disturbances are the most common type, ac-
counting for the majority of the neurologic symptoms associated with mi-
graine. Numbness and tingling of the lips, lower face, and fingers of one
hand is the second most common type of aura. Auras may also involve
other senses or occasionally cause motor or speech deficits. Some patients
have several types of aura symptoms that vary with attacks.
• Typical aura. In the revised IHS terminology, a typical aura consists of
visual and/or speech and/or sensory symptoms. Additional features of a
typical aura include gradual development, duration no longer than one

230
Case 40

hour, a mix of positive and negative features, and complete reversibility.


If the aura includes motor weakness, the migraine is classified as familial
hemiplegic migraine or sporadic hemiplegic migraine.
• Familial hemiplegic migraine. In the revised IHS classification, fa-
milial hemiplegic migraine is defined as migraine with aura that includes
motor weakness in a patient who has at least one first or second degree
relative who also has migraine aura that includes motor weakness. Familial
Hemiplegic Migraine (FHM) is a rare condition with autosomal domi-
nant inheritance and broad clinical variability. It is associated with muta-
tions in three genes that encode for transmembrane ion channels and has
been subdivided into FHM1, FHM2, and FHM3.
• Sporadic hemiplegic migraine. In the revised IHS classification, spo-
radic hemiplegic migraine is defined as migraine with an aura that in-
cludes motor weakness in a patient who has no first or second degree
relative who also has migraine aura that includes motor weakness.
Because of the unique features of migraine in children, the quality of the
headache and associated symptoms are more important than specific clinical
criteria. In general, three of the following criteria should be present for the
diagnosis of childhood migraine.
• Abdominal pain, nausea, or vomiting with headache
• Hemicrania (unilateral headache)
• Throbbing, pulsating pain
• Complete relief after a brief rest
• An aura, either visual, sensory, or motor
• A history of migraine headaches in one or more family members
However, the revised IHS criteria note that certain features of migraine in
children may differ from typical features in adults.
• Migraine attacks may last one to 72 hours.
• Migraine headache is commonly bilateral in children, and an adult pattern
of unilateral pain usually emerges in late adolescence or early adulthood.
• Occipital headache in children is rare and raises diagnostic caution, as
many cases are attributable to structural lesions.
• Photophobia and phonophobia may be inferred by behaviour in young
children.
• Intracranial haemorrhage. Acute intracranial haemorrhage represents
one of the most devastating causes of weakness. Traumatic epidural or
subdural haematoma is common in children who have sustained severe
head trauma. Less commonly, rupture of a pre-existing arteriovenous mal-
formation or aneurysm may cause subarachnoid haemorrhage. Most of
these patients are asymptomatic prior to rupture. These children present
with sudden weakness, often associated with altered mental status or se-
vere headache.

231
Child Neurology

• Stroke. Stroke is a rare cause of weakness in children. Thrombotic stroke


may be caused by a hypercoagulable state such as factor V Leiden deficien-
cy, protein C or S deficiency, or sickle cell disease. Embolic strokes are less
common and may be considered in children with congenital or acquired
heart disease. Acquired or congenital vasculopathy caused by vasculitis,
moyamoya disease, or specific metabolic disorders (e.g., homocystinuria)
comprise additional unusual causes of stroke in children.
• Brain tumour. Intracranial tumours, either malignant or benign, may
cause weakness. Intracranial tumours can present with or be exacerbated
by haemorrhage into the tumour. These patients usually either have a his-
tory of known brain tumour or a history of clumsiness, weakness, be-
havioural changes, and vomiting. When the acute haemorrhage occurs,
symptoms include weakness, severe headache, and vomiting.
• Seizure. Todd paralysis is a temporary focal weakness following a seizure.
It generally improves rapidly during the postictal period with resolution
in minutes to hours.

DIAGNOSTIC APPROACH
• Rapid onset weakness in association with headache or other signs of in-
creased intracranial pressure in the absence of trauma or seizure suggests
the presence of stroke, subarachnoid haemorrhage, or brain tumour with
haemorrhage.
• Additional history important to elicit includes whether there is a family
history of epilepsy, migraine, or hyper-coagulability. The presence of mi-
graine markers, such as motion sickness and ice cream headache (throb-
bing head pain when eating something cold), should also be sought.
• Given that migraine with prolonged aura is difficult to differentiate from
acute stroke, on first presentation these patients should be imaged with
MRI and intracranial magnetic resonance angiography to rule out ischae-
mia. Imaging also rules out vasculitis or moyamoya disease, which may
present with headache with a migrainous phenotype, probably as a comor-
bid activation of the patient’s underlying tendency to migraine.
• In the emergency room, the patient’s evaluation should begin with a head
CT, with and without contrast, to rule out larger structural abnormali-
ties. A brain MRI with and without gadolinium should be performed in
addition to Magnetic Resonance Angiography (MRA). Although a CT
angiogram may also be helpful in ruling out vascular abnormalities, a
four-vessel catheter angiogram is the gold standard when a posterior com-
municating artery aneurysm is suspected.
• Most patients with a history of migraine headaches and no signs or symp-
toms of neurologic dysfunction or increased intracranial pressure do not
need neuroimaging. However, many parents and children are concerned
that the headaches are caused by a brain tumour or other life-threat-
ening illness. In these cases, brain imaging may be required to relieve
apprehensions.

232
Case 40

• The American Academy of Neurology, American Academy of Family


Physicians, American College of Physicians, American Society of Internal
Medicine, and four other groups formed a consortium that took an ev-
idence-based approach to, among other things, evaluating the need for
brain imaging in patients with headache. This report noted that the odds
of finding a significant abnormality on neuroimaging in patients with no-
nacute headache were increased by the following symptoms:
• Rapidly increasing headache frequency.
• History of lack of coordination.
• History of localised neurologic signs or subjective numbness or tingling
sensation.
• History of headache causing awakening from sleep (although this can oc-
cur with migraine headache.
• Electroencephalogram.. EEG is not indicated in the routine evaluation
of headache. An EEG is performed if seizures are suspected. Rolandic
spike and wave discharges occur more in children with migraine than un-
affected children. These are thought to have no clinical significance in
children without nocturnal seizures. Focal slowing may occur in hemi-
plegic migraine syndromes.
• Laboratory evaluation. Laboratory testing rarely is helpful in the evalu-
ation of childhood headache. In children with complicated syndromes
or migraine variants, testing will depend upon the differential diagnosis.
Lumbar Puncture (LP) may be necessary to diagnose an intracranial infec-
tion or increased intracranial pressure. Other studies might include meta-
bolic testing (amino acids, organic acids, lactic acid), coagulation studies,
echocardiogram, lipid profile, amylase, or gastrointestinal series.

MANAGEMENT
• The management of migraine consists of a-general measures, b-abortive
treatment, and c-preventive treatment. An individual patient may need
all three approaches. Unfortunately, there is a paucity of data from con-
trolled, randomised, and blinded trials regarding treatment of migraine
in children. Thus, most management recommendations are based upon
experience in adults.
• General Measures. Education of the family is important in manage-
ment. Literature describing migraine in children should be provided. The
family should be asked to document the occurrence of headaches on a
calendar to clarify features of the attacks and to help evaluate the effective-
ness of treatment.
• If possible, precipitating factors should be identified. As an example, the
use of caffeine may exacerbate migraine and should be eliminated. Sleep
disturbances, such as snoring or frequent awakenings, may precipitate
headache. Rarely, a temporal relationship is established between diet and
headache; in these cases, an elimination diet may be helpful. Dehydration

233
Child Neurology

and missing meals are also common precipitants of migraine. Stress caused
by school or social situations may increase headache frequency, although
stress does not cause migraine.

Abortive Treatment
• Migraine headaches usually are treated at home. The parents typically
decide when treatment is indicated and when to administer medication.
In general, young children are not able to medicate themselves. In the
school-age child, arrangements must be made for a school professional to
administer medicine.
• When symptoms develop, the child should rest and/or sleep in a quiet
dark room with a cool cloth applied to the forehead. The initial treatment
consists of an analgesic and may include an antiemetic.
• For children with acute migraine headache, we suggest initial abortive
treatment with an analgesic, either acetaminophen or ibuprofen, both of
which have proven efficacy for migraine in children in randomised, con-
trolled trials. The initial choice depends upon individual preference. If the
patient does not respond to one, the other can be tried.
• Ibuprofen is given in an initial dose of 10 mg/kg. This dose may be re-
peated in four to six hours if needed. No more than four doses should be
given in 24 hours (maximum daily dose 40 mg/kg). Alternatively, acetami-
nophen can be given in a dose of 10 to 20 mg/kg (usually as one or two 325
mg tablets), with a maximum dose of 1000 mg. This may be repeated in
two to four hours if symptoms persist but should not exceed three doses
in 24 hours.
• An antiemetic is given to children with nausea and vomiting. Most phy-
sicians prefer promethazine in a dose of 0.25 to 0.5 mg/kg rectally and
repeat as needed at intervals of four to six hours. For children who have
acute migraine headache without vomiting, that is refractory to analgesics,
physicians suggest initial treatment with oral triptans. Preferred agent is
oral sumatriptan starting at 25 mg, with a maximum dose of 50 mg. For
children who do not respond to oral sumatriptan, alternatives include ri-
zatriptan (5 mg wafer), zolmitriptan (2.5 or 5 mg), and almotriptan (6.25
or 12.5 mg).
• In children at least five years of age, if analgesics do not provide relief or
if persistent vomiting precludes the use of oral medications, a trial of su-
matriptan nasal spray due to its proven efficacy in randomised, controlled
trials in adolescents. In addition, it is generally more tolerable to children
than an injection. We prefer to start with 5 mg and repeat once in four
to six hours if initially effective, but the headache returns. If there is no
benefit, 10 mg nasal spray (two 5 mg units given together) may be tried.
We suggest similar doses of nasal spray in older children, with a maximum
daily dose of 20 mg. One limitation to nasal sumatriptan is the associated

234
Case 40

bad taste, which limits its acceptability in children. This problem can be
mitigated by having children suck on a piece of hard candy.
• As an alternative, zolmitriptan 5 mg nasal spray can be used, given the
randomised clinical trial evidence cited earlier, that this agent is safe and
effective for the treatment of migraine in adolescents. In the clinical ex-
perience of some experts, zolmitriptan nasal spray has a less objectionable
taste than sumatriptan nasal spray.

Treatment of Status Migrainosus


Aggressive therapy is needed but can often be administered in an outpatient
infusion centre. There are 5 principles of treatment: hydration, analgesia, spe-
cific anti-migraine medication, anti-emetics, and sedation.
Since vomiting and poor intake are almost always present, treatment begins
with rehydration using glucose-containing fluids. An IV bolus of normal saline
or lactated Ringer’s solution followed by infusion of 5-10% dextrose solution
should be considered.
Analgesic options should include relatively mild pain medications such as
Intramuscular (IM) ketorolac or rectal suppositories of naproxen or indo-
methacin. If possible, narcotics should be avoided.
First-line, migraine-specific regimens include triptans, IV valproic acid, and
IV DHE (Dihydroergotamine mesylate); however, these agents should not be
used together. In addition, the use of IV DHE in patients younger than 12
years is questionable; in this age group, many patients respond to IV fluids
and lorazepam. Subcutaneous (SC) sumatriptan (0.06 mg/kg, maximum dose
6 mg) may be useful in patients unable to tolerate oral medications. Similarly,
in patients weighing less than 50 kg, be sure to appropriately titrate the dose
of IV medications.

Emergency Department Treatment


An adult-sized patient can be pretreated with 0.5-1 mg of lorazepam. IV fluids
can be administered concurrently; hydration often is helpful. After 15-30 min-
utes, the patient is then administered 0.5 mg of haloperidol to prevent nausea.
This is followed in 30 minutes by 0.5 mg of DHE. In many patients, the head-
ache is aborted or significantly relieved with this protocol.

Medications for Migraine Variants


Some forms of familial hemiplegic migraine respond to acetazolamide, and
children with abdominal migraine respond to typical migraine prophylac-
tic medication. Some children with cyclic vomiting respond to anti-mi-
graine drugs (e.g., propranolol, amitriptyline, cyproheptadine, sumatriptan).
However, these children often experience severe fluid and electrolyte distur-
bances that require IV fluid therapy.

235
Child Neurology

Prophylactic Treatment Recommendations


The primary goals of prophylactic drugs are to prevent migraine attacks and
to reduce the frequency and severity of attacks. Half of all patients experience
at most a 50% reduction in migraines. Most prophylactic migraine medica-
tions have potential adverse effects; therefore, consider only patients with 1-2
attacks per week (4 or more headache days monthly) for prophylaxis. Possible
medications for migraine prophylaxis include the following:
• Amitriptyline
• Propranolol
• Selective Serotonin Reuptake Inhibitors (SSRIs).
• Anti-convulsants-e.g., gabapentin, valproate, divalproex, topiramate
• Riboflavin
• Tricyclic Anti-depressants (TCAs).
The agents that seem to be the most effective prophylactic medication in chil-
dren are those that block the 5-HT2 serotonin receptor. These medications in-
clude beta blockers, cyproheptadine, and methysergide (Sansert). Beta block-
ers and cyproheptadine appear to be effective and well tolerated.
• The FDA has approved topiramate (Topamax) for prevention of migraine
headache in adolescents aged 12-17 years. It is the first such approval for
this age group. Frequency of migraine decreased by approximately 72%
in treated patients compared with 44% in participants receiving placebos.
Calcium channel blockers had been used for migraine prophylaxis in chil-
dren, but results have been inconsistent.
• Initially administer drugs at very low dosages and slowly titrate to thera-
peutic efficacy. This approach lessens adverse effects and results in better
long-term patient compliance. Often, several weeks are necessary before
therapeutic gains are observed. No consensus exists on the duration of
prophylactic medication usage, although most neurologists aim for 3-6
months of good symptom control.
• Some patients must be maintained on long-term prophylactic therapy, and
others tolerate drug holidays, particularly during summer when migraine
attacks are less frequent for many children. Occasionally, prophylactic
drugs are effective initially but become ineffective over the long term.
Subsequent prophylaxis with the same agent often is not as effective.
Withdraw drugs slowly to prevent relapse and withdrawal symptoms.
• In children, younger than six years of age, cyproheptadine (Periactin) is
suggested in a dose of 4 to 12 mg per day, given orally once at bedtime.
• In older children, we suggest propranolol in a starting dose of 1 mg/kg in
three divided doses, with a maximum dose of 4 mg/kg per day. Heart rate
and orthostatic blood pressure should be monitored periodically. The heart
rate should be >60 bpm after one minute of exercise. This drug is often
discontinued by children who participate in strenuous physical activities.

236
Case 40

• If propranolol is not well tolerated, we suggest valproate for prophylaxis.


Valproate is given primarily to boys older than five years of age. Prior to
use, the potential side effects are discussed with the parents and child.
Valproate is started in a dose of 10 to 15 mg/kg in two to three divided
doses orally. The dose can be increased in increments of 15 mg/kg to a
maximum dose of 60 mg/kg per day. Serum valproic acid concentration
should be monitored every three to six months to document compliance
and to help adjust doses to avoid toxicity. Complete blood counts, liver
function tests, and electrolyte concentrations also should be monitored
periodically.

REFERENCES
1. Russell MB, Ducros A. Sporadic and familial hemiplegic migraine: pathophysi-
ological mechanisms, clinical characteristics, diagnosis, and management. Lancet
Neurol 2011; 10:457.
2. Lykke Thomsen L, Kirchmann Eriksen M, Faerch Romer S, et al.. An epidemio-
logical survey of hemiplegic migraine. Cephalalgia 2002; 22:361.
3. Thomsen LL, Eriksen MK, Roemer SF, et al.. A population-based study of familial
hemiplegic migraine suggests revised diagnostic criteria. Brain 2002; 125:1379.
4. Thomsen LL, Ostergaard E, Olesen J, Russell MB. Evidence for a separate type of
migraine with aura: sporadic hemiplegic migraine. Neurology 2003; 60:595.
5. Yu W, Horowitz SH. Treatment of sporadic hemiplegic migraine with calcium-
channel blocker verapamil. Neurology 2003; 60:120.
6. Oberndorfer S, Wöber C, Nasel C, et al.. Familial hemiplegic migraine: follow-up
findings of diffusion-weighted magnetic resonance imaging (MRI), perfusion-
MRI and [99mTc] HMPAO-SPECT in a patient with prolonged hemiplegic aura.
Cephalalgia 2004; 24:533.
7. Bosemani T, Burton VJ, Felling RJ, et al.. Pediatric hemiplegic migraine: role of
multiple MRI techniques in evaluation of reversible hypoperfusion. Cephalalgia
2014; 34:311.
8. al. Magnetic resonance angiography evi-
Safier R, Cleves-Bayon C, Vaisleib I, et al
dence of vasospasm in children with suspected acute hemiplegic migraine. J Child
Neurol 2014; 29:789.
9. Stam AH, Louter MA, Haan J, et al. A long-term follow-up study of 18 patients
with sporadic hemiplegic migraine. Cephalalgia 2011; 31:199.
10. Headache Classification Committee of the International Headache Society (IHS).
The International Classification of Headache Disorders, 3rd edition (beta version).
Cephalalgia 2013; 33:629.
11. Peer Mohamed B, Goadsby PJ, Prabhakar P. Safety and efficacy of flunarizine in
childhood migraine: 11 years’ experience, with emphasis on its effect in hemiplegic
migraine. Dev Med Child Neurol 2012; 54:274.
12. Pelzer N, Stam AH, Carpay JA, et al. Familial hemiplegic migraine treated by so-
dium valproate and lamotrigine. Cephalalgia 2014; 34:708.
13. Russell MB, Ducros A. Sporadic and familial hemiplegic migraine: pathophysi-
ological mechanisms, clinical characteristics, diagnosis, and management. Lancet
Neurol 2011; 10:457.

237
41 Case
An 8-year-old girl presented with a history of progressive involuntary
movements of the upper and lower limbs since the last 3-weeks.
Left sided extremities were affected more than her right side. The symptoms
had slightly affected the child’s daily activities, and she had an unstable gait
on walking. Symptoms were aggravated by stress and did not persist during
sleep. Parents do not recall the child suffering from any recent febrile illness
or sustaining any major trauma. No history of any drug intake. No history of
rashes or joint swelling or chest pain. There was no significant past medical
history and no similar history in the family. The child was fully immunised
according to the EPI schedule.
On physical examination, she was afebrile, alert, oriented but appeared
anxious and fidgety. Speech was fluent without element of dysarthria. She
was seen having bilateral involuntary, irregular and purposeless movements,
mostly affecting the left sided extremities and also involving facial muscles
with bilateral facial twitching. Random jerky and writhing movements of the
upper extremities and trunk were noted. She could ambulate independently
without ataxia.
Her pupils were equal, round, and reactive to light. Her extraocular muscles
were intact, with no nystagmus, and the visual fields were full. Her facial
movements were symmetric, gag was normally present and the tongue was
midline. There was normal bulk of muscles but with mild diffuse hypotonia.
However, strength was 5/5 throughout, but she seemed to have trouble
maintaining hand grip. There was no dysmetria.

Answer the following questions:


• Give the most likely diagnosis.
• Briefly summarise the case.
• Localise the examination findings.
• Discuss the differential diagnosis.
• Discuss the diagnostic work-up.
• Discuss the management of the patient.

238
Case 41

DIAGNOSIS
Sydenham chorea

SUMMARY
A previously healthy 8-year-old girl who presented with the subacute onset of
restlessness, labile mood, upper extremity choreoathetosis, hypotonia, and dif-
ficulty maintaining grip strength. She could ambulate independently without
ataxia.

Localisation of Lesion
Chorea most commonly localises to the striatum but can also be caused by
lesions of the subthalamic nucleus and ventral thalamic nucleus. Dysfunction
of the basal ganglia results in extrapyramidal movements (chorea, hemiballis-
mus, dystonia, tics and parkinsonism). In addition to control of movements,
the basal ganglia have an important role in control of behaviour and emotion.
Hypotonia is a nonspecific finding but is likely cerebral in origin, given this
child’s history and physical examination. Difficulty maintaining grip strength
is probably a result of motor impersistence rather than weakness and is hard
to specifically localise.

DIFFERENTIAL DIAGNOSIS
Sydenham chorea is the most common cause of chorea in childhood. Chorea
describes an apparently random, non-rhythmic, purposeless set of movements
of either distal or proximal muscles that appears to flow from one muscle or
muscle group to another without any pattern. Chorea occurs at rest and with
action, and gives the child a “fidgety” appearance and the inability to remain
still. It is associated with motor impersistence (for example, the inability to
maintain the tongue extended). Chorea may worsen or improve with volun-
tary movement, but even very severe chorea may not prevent accurate volun-
tary movement for some children, suggesting that compensatory mechanisms
exist. Many individuals with chorea will incorporate the involuntary move-
ments into voluntary movements in order to mask the movements. However,
the involuntary movements can lead to significant disability, and some chil-
dren will injure themselves or others due to rapid, ballistic, flinging move-
ments of the arms or legs. Tone is normal or reduced in pure chorea. As with
most movement disorders, chorea disappears in sleep but it may be at its worst
when the child is drowsy.
The differential diagnosis of the insidious onset of chorea in childhood is broad.
Symptoms of the following disorders can be similar to those of Sydenham
chorea. The symptoms are extremely similar regardless of the cause.

Static Injury/Structural Disorders


• Stroke

239
Child Neurology

• Trauma
• Moyamoya disease
• Vasculitis
• Tumours

Hereditary/Degenerative Disorders
• Ataxia-telangiectasia
• Ataxia Oculomotor Apraxia (AOA)
• Early-onset cerebellar ataxia and hypoalbuminaemia
• Fahr’s disease
• Pantothenate Kinase-Associated Neurodegeneration (PKAN)
• Huntington disease-like disorder
• HARP syndrome (Hypoprebetalipoproteinaemia, Acanthocytosis,
Retinitis pigmentosa, and Pallidal degeneration)

Metabolic Disorders
• Acyl-coA dehydrogenase deficiency
• Mitochondrial disorders, including Leigh’s syndrome
• Wilson’s disease
• GM1 gangliosidosis
• Metachromatic leukodystrophy
• Lesch–Nyhan disease
• Niemann–Pick disease type C
• Methylmalonic aciduria
• Nonketotic hyperglycaemia
• Propionic academia

Immune-Mediated/Demyelinating Disorders
• Sydenham’s chorea
• Lupus erythematosus
• Paediatric Autoimmune Neuropsychiatric Disorders Associated with
Streptococcal Infections (PANDAS)
• Anti-cardiolipin or anti-phospholipid anti-body syndrome
• Sarcoidosis

Medications
• Anti-convulsants
• Neuroleptic medications

240
Case 41

• Levodopa
• Dopamine agonists
• Lithium
• Methylphenidate

DIAGNOSTIC APPROACH
• The diagnosis of SC essentially is made clinically. No specific laboratory
test exists, and no abnormalities can be detected in the cerebrospinal fluid
during an acute attack. The presence of carditis confirms the diagnosis,
and a careful cardiac examination is mandatory.
• Most patients with pure chorea are afebrile and have little evidence of
acute inflammation (e.g., elevations in white blood cell count or acute
phase reactants). Evidence of an active or recent group A beta haemolytic
streptococcal infection is helpful but not always present. Throat cultures
usually are negative by the time chorea appears, but an attempt should be made
to isolate the organism. Streptococcal anti-bodies are more useful because
they indicate true infection rather than transient carriage, and, by measuring
titers of several different anti-bodies, any significant recent streptococcal
infection can be detected. The anti-streptolysin O titer is measured most
commonly in suspected acute rheumatic fever. However, only 80 percent of
patients with rheumatic fever show a rise in the anti-streptolysin O titer, and
low titers are common in children without rheumatic fever because of the
prevalence of streptococcal infection. The anti-body response peaks four
to five weeks after the onset of pharyngitis, which typically is well before
the appearance of chorea. The anti-deoxyribonuclease (anti-DNAse) B titer
tends to remain elevated longer and may, therefore, be more sensitive in
patients with chorea who typically present later.
• In patients who have clinical, examination, and laboratory features of
Sydenham chorea, neuroimaging usually does not provide additional in-
formation that assists in management decisions or determining prognosis.
However, neuroimaging may be necessary to exclude other causes of cho-
rea in patients with unusual histories, including those with hemichorea.
The use of echocardiography to detect silent valvular disease may be help-
ful in some patients.
• A work-up for an alternative diagnosis is required if there is a family history
of chorea or if the initial work-up demonstrates no prior exposure to
streptococcal infection. A lumbar puncture is often performed to exclude
central nervous system infections or inflammation (not usually unless
clinically indicated). Screening testing to rule out other conditions may be
done by obtaining basic chemistries, liver function tests, CBC, erythrocyte
sedimentation rate, calcium, magnesium, phosphorus, urine toxin screen,
urine heavy metal screen, thyroid function tests, ANA, and anti-cardiolipin
anti-body. Initial testing for an underlying metabolic or genetic disease

241
Child Neurology

might include plasma amino acids, urine organic acids, lactate/pyruvate,


copper, ceruloplasmin, alpha-foetoprotein, and lysosomal enzymes. A
formal ophthalmologic examination can be helpful in evaluating for
Kayser-Fleischer rings associated with Wilson’s disease, as well as for optic
nerve or retinal abnormalities that might suggest a metabolic disorder
• Numerous other conditions associated with chorea, including encephali-
tis, systemic lupus erythematosus, Wilson’s disease, glutaric aciduria (type
2), Huntington disease, Tourette syndrome, familial benign chorea, thy-
rotoxicosis, and attention deficit disorder with choreiform movements,
should be considered. Other testing may include serum copper, ceru-
loplasmin, slit lamp examination, anti-nuclear anti-bodies, and a careful
family history to identify Huntington chorea.

MANAGEMENT
Sydenham chorea typically improves gradually, with a mean duration of 12 to
15 weeks. Full recovery occurs in almost all patients, but symptoms of SC oc-
casionally persist for two years or more. There is no known cure for Sydenham
chorea.
• Pharmacologic therapy of chorea. The patient with chorea should be
treated when significant impairment of motor function and the possibility
of self injury are present. Children who develop chorea as the sole mani-
festation of acute rheumatic fever during the initial episode of illness have
an approximate 50 percent risk of developing rheumatic heart disease with
subsequent infection. Treatments reported to be effective include valproic
acid, phenobarbital, haloperidol, pimozide, diazepam, chlorpromazine,
and carbamazepine. Additional therapies have been used to treat individu-
als with Sydenham chorea include corticosteroids, intravenous immuno-
globulins, and plasma exchange. More research is necessary to determine
the long-term safety and effectiveness of such therapies in treating indi-
viduals with Sydenham chorea. Haloperidol, a centrally acting drug, is
effective in controlling chorea but may be associated with extrapyramidal
reactions. The efficacy of treatment with valproate and carbamazepine is
not well established.
• All patients with acute rheumatic fever, including those whose only mani-
festation is chorea, should receive a 10-day course of penicillin or eryth-
romycin. Prophylaxis with penicillin or sulfadiazine should be started
immediately and continued at least until adulthood because of frequent
reinfection and the risk of rheumatic heart disease with subsequent strep-
tococcal pharyngitis. More specifically, when residual valvular disease ex-
ists, prophylaxis should continue for at least 10 years after the last epi-
sode and at least until age 40. If there is no residual valvular disease, the
duration of treatment beyond 10 years or into adulthood is not clearly
defined. When Sydenham’s chorea is diagnosed and there is no valvular
disease, the duration of prophylaxis should be at least 5 years or until age
21, whichever is longer.

242
Case 41

REFERENCES
1. Eshel G, Lahat E, Azizi E, et al. Chorea as a manifestation of rheumatic fever a 30-
year survey (1960-1990). Eur J Pediatr 1993; 152:645.
2. Zomorrodi A, Wald ER. Sydenham’s chorea in western Pennsylvania. Pediatrics
2006; 117:e675.
3. Barash J, Margalith D, Matitiau A. Corticosteroid treatment in patients with
Sydenham’s chorea. Pediatr Neurol. 2005;32:205-207.
4. Demiroren K, Yavuz H, Cam L, et al.. Sydenham’s chorea: a clinical follow-up of 65
patients. J Child Neurol 2007; 22:550.
5. Harsányi E, Moreira J, Kummer A, et al.. Language Impairment in Adolescents With
Sydenham Chorea. Pediatr Neurol 2015; 53:412.
6. Cardoso F. Chorea: nongenetic causes. Curr Opin Neurol. 2004;17:433-436.
7. Church AJ, Cardoso F, Dale RC, et al.. Anti-basal ganglia anti-bodies in acute and
persistent Sydenham’s chorea. Neurology. 2002;59:227-231.
8. Daoud AS, Zaki M, Shakir R, et al.. Effectiveness of sodium valproate in the treat-
ment of Sydenham’s chorea. Neurology. 1990;40:1140-1141.
9. Punukollu M, Mushet N, Linney M, et al.. Neuropsychiatric manifestations of
Sydenham’s chorea: a systematic review. Dev Med Child Neurol 2016; 58:16.
10. Oosterveer DM, Overweg-Plandsoen WC, Roos RA. Sydenham’s chorea: a practi-
cal overview of the current literature. Pediatr Neurol 2010; 43:1.

243
42 Case
A 26-day-old female baby born to consanguineous parents, through nor-
mal vaginal delivery at term. She presented with brief jerky movements
of all limbs and occasional generalised stiffness secondary to tactile stimula-
tion, which occurred with a frequency of multiple times per day since her
birth. Her Apgar scores were 6 at the first minute and 9 at the fifth min-
ute respectively; no resuscitation was required. The birth weight was 3100 g.
Pregnancy was marked with excessive foetal movements with no associated
polyhydramnios or oligohydramnios. Antenatal screening was normal. The
mother, 20-year-old primigravida, was healthy and had no significant medical
problem.
General physical examination revealed weight 3300 gm, head circumfer-
ence 36 cm. There were no dysmorphic features. Neurologic examination
showed increased tone, spasticity and exaggerated startle reflex. On tapping
the dorsum of the nose or the forehead, jerking of the whole body and exag-
gerated deep tendon reflexes were most noticeable. No other abnormalities
were noted. She was very sensitive to sounds and minimum stimuli provoked
severe jerky movements of all limbs. There was also sustained non-habituat-
ing glabellar tap.
Laboratory investigations showed normal Tandem Mass Spectrometry (TMS)
for metabolic disorders screen was unremarkable. Serum ammonia, lactate
and urine organic acids were normal.
EEG trace consisted of occasional synchronised jerks, mostly myogenic in
origin and head CT scan yielded normal findings. Episodes of seizure-like and
tonic spasms could not be controlled with administration of phenobarbital and
phenytoin. Episodic attacks decreased remarkably after starting clonazepam.
Her feeding pattern improved, irritability decreased, and spontaneous attacks
disappeared. No other seizure-like episodes and generalised stiffening were
noted in the subsequent follow-ups.

Answer the following questions:


• Give the most likely diagnosis.
• Briefly summarise the case.
• Provide a differential diagnosis.
• Discuss an appropriate diagnostic work-up.
• Discuss the management.

244
Case 42

DIAGNOSIS
Hyperekplexia (stiff-baby syndrome)

SUMMARY
A 26-day-old female baby presented with brief jerky movements of all limbs
and occasional generalised stiffness secondary to tactile stimulation, which oc-
curred with a frequency of multiple times per day, since her birth. Examination
showed increased tone, spasticity, exaggerated startle reflex associated with
jerking of the whole body and exaggerated deep tendon reflexes most notice-
able on tapping the dorsum of the nose or the forehead. She was very sensi-
tive to sounds and minimum stimuli provoked severe jerky movements of all
limbs.

DIFFERENTIAL DIAGNOSIS
Hyperekplexia is a rare hereditary, neurological disorder that may affect in-
fants as newborns (neonatal) or prior to birth (in utero). Individuals with this
disorder have an excessive startle reaction (eye blinking or body spasms) to
sudden unexpected noise, movement, or touch. Symptoms include extreme
body stiffness that prevent voluntary movement and can cause the affected
person to fall stiffly, like a log, without loss of consciousness. Exaggerated
Head-Retraction Reflex (HRR) consisting of extension of the head followed
by violent flexor spasms of limbs and neck muscles elicited by tapping the tip
of the nose is observed in most children. Hyperekplexia is usually inherited
as an autosomal dominant trait, but autosomal recessive or rarely, X-linked
inheritance, has also been reported. The genes that cause hyperekplexia are
involved in the production of the glycine protein. It is known as an “inhibitor
transmitter”. If the glycine receptors are interfered in some way or damaged,
the nerve cells lack their inhibitions and thus react to stimuli too easily and
excessively. Sudden Infant Death (SID) has been reported. Intellect is usu-
ally normal; mild intellectual disability may occur. Hyperekplexia is frequently
misdiagnosed as a form of epilepsy so the process of getting an accurate diag-
nosis may be prolonged or missed.
The differential diagnoses in the neonatal period are congenital stiff-person
syndrome, Schwartz–Jampel syndrome, startle epilepsy, myoclonic seizures,
neonatal tetany, and phenothiazine toxicity.
• Schwartz–Jampel syndrome. Schwartz-Jampel syndrome is a rare
condition characterised by myotonia and chondrodysplasia. The signs
and symptoms of this condition become apparent sometime after birth,
usually in early childhood. Either muscle stiffness or chondrodyspla-
sia can appear first. The specific features of Schwartz-Jampel syndrome
vary widely. This sustained myotonia causes stiffness that interferes with
eating, sitting, walking, and other movements. Sustained contraction of
muscles in the face leads to a fixed, “mask-like” facial expression with
blepharophimosis and pursed lips. This facial appearance is very specific

245
Child Neurology

to Schwartz-Jampel syndrome. Two types of the disorder have been iden-


tified that may be differentiated by age of onset and other factors. SJS
type 1, which is considered the classical form of the disorder, may become
apparent after birth or during early to late infancy or childhood. SJS type
2, a rarer form of the disorder, is typically recognised at birth (congenital)
• Stiff-person syndrome. Stiff-Person Syndrome (SPS) is a rare acquired
neurological disorder characterised by progressive rigidity and repeated
episodes of painful muscle spasms. Spasms may occur randomly or be
triggered by a variety of different events including a sudden noise or
light physical contact. The severity and progression of SPS varies from
one person to another. If left untreated, SPS can potentially progress
progres to
cause difficulty walking and significantly impact a person’s ability to per-
form routine, daily tasks. Although the exact cause of SPS is unknown,
it is believed to be an autoimmune disorder and sometimes occurs along
with other autoimmune disorders. A diagnosis of SPS is made based upon
identification of characteristic symptoms, a detailed patient history, and
a thorough clinical evaluation. Additional tests can be used to support a
diagnosis and to rule out other conditions. Such tests include screening
tests to detect the presence of anti-bodies against GAD-65, anti-bodies
against amphiphysin (which are associated with paraneoplastic SPS) and
an Electromyography (EMG), a test that records electrical activity in skel-
etal (voluntary) muscles at rest and during muscle contraction. An EMG
can demonstrate continuous muscle motor unit firing in stiff muscles,
which is characteristic of SPS. High doses of diazepam will suppress the
characteristic EMG results. Drugs known as benzodiazepines, such as di-
azepam and clonazepam, are used to treat muscle stiffness and episodic
spasms. Affected individuals may also benefit from baclofen, usually given
in addition to benzodiazepines.
• Startle epilepsy. Differentiation between startle-induced seizures and
exaggerated startle responses can be challenging. Startle epilepsy is charac-
terised by seizures triggered by unexpected sudden sensory stimuli, usu-
ally sound or touch. Most patients have intractable seizures, evident static
encephalopathy, and neurologic deficits (infantile hemiplegia is common)
with corresponding abnormal imaging. The startle response consists of
axial tonic posturing, frequently causing falls that can often be traumatic.
Concurrent symptoms, such as marked autonomic manifestations, au-
tomatism, laughter, and jerks, may occur. Less commonly, startle-induced
seizures may be atonic or myoclonic, particularly in patients with cerebral
anoxia. Seizures are frequent, occurring many times a day, and sometimes
progress to status epilepticus. The prognosis is often poor, particularly for
those with severe pre-existing encephalopathies. Startle epilepsy is often
resistant to medical treatment. Epilepsy surgery may be beneficial to those
with focal etiology of seizures.
• Neonatal tetany. Hypocalcaemia is a common metabolic problem in
newborns. In term infants or premature infants greater than 1500 g birth

246
Case 42

weight, we define hypocalcaemia as a total serum Ca concentration less


than 8 mg/dL (2 mmol/L) or an ionised fraction of less than 4.4 mg/dL (1.1
mmol/L). Most infants with hypocalcaemia are asymptomatic. Among
those who become symptomatic, the characteristic sign is increased neu-
romuscular irritability. Such infants are jittery and often have muscle jerk-
ing that is induced by environmental noise or other stimuli. Generalised
or focal clonic seizures can occur. The diagnosis is most commonly made
because of the detection of an abnormally low serum Ca value during
routine monitoring.

DIAGNOSIS
Diagnosis of Hereditary Hyperekplexia (HPX) requires the following three
cardinal features:
• Generalised stiffness immediately after birth, normalising during
the first years of life. The stiffness increases with handling and disappears
during sleep.
• Excessive startle reflex to unexpected (particularly auditory) stimuli.
This excessive startle reflex is present at birth. Consciousness is unaltered
during startle responses.
• Short period of generalised stiffness following the startle response
during which voluntary movements are impossible. On rare occasion this
feature is absent.
In general, all laboratory tests are normal in individuals with HPX, including
CT and MRI of the brain.

Molecular Genetic Testing


Genes.. Mutations in one of the following genes are known to cause HPX:
• GLRA1,, the gene encoding the α1 subunit of the glycine receptor, is the
major genetic cause of HPX. Dominant and recessive mutations are iden-
tified in many individuals with the familial form of HPX and occasionally
in simplex cases.
• SLC6A5,
SLC6A5 the gene encoding the presynaptic sodium-and chloride-
dependent glycine transporter 2 (GlyT2), is probably also frequently
involved.
• GLRB, the gene encoding glycine receptor subunit beta, has been associ-
ated with HPX.
• GPHN, the gene encoding the glycinergic clustering molecule gephyrin,
has been associated with HPX.
• ARHGEF9, an X-linked gene encoding collybistin, has been associated
with HPX in one person. These children also have severe epilepsy and
intellectual disability and death usually occurs at three to four years of age.

247
Child Neurology

MANAGEMENT
The drug clonazepam is most effective in reducing symptoms, including stiff-
ness in the neonatal period and stiffness related to the excessive startle reflex;
other drugs with variable results include carbamazepine, phenytoin, diazepam,
valproate, 5-hydroxytryptophan, piracetam, and phenobarbital. Treatment is
most effective with clonazepam (0.1-0.2 mg/kg/day) but, in some cases, symp-
toms may not be totally suppressed. Valproic acid is recommended in cases
of late onset. Phenobarbital, phenytoin, and diazepam have not proved to be
effective. The prognosis is variable.
Neonatal hyperekplexia may result in apnoea, neonatal encephalopathy, cer-
ebral palsy, and unexpected death. Early identification and treatment improve
the outcome in most children. A simple maneuver, like forced flexion of the
head and legs towards the trunk, is known to be life-saving when prolonged
stiffness impedes breathing. Hypertonia and motor delay improve with in-
creasing age, and muscle tone becomes normal by the age of 3 years. In some
families, the exaggerated startle response ameliorates or disappears spontane-
ously by 2 years of age, although hyperekplexia may persist or reoccur in adult
life, resulting in falls. In the minor form of hyperekplexia, there is only an
abnormal startle response without generalised stiffness or tonic spasms. The
minor form is not associated with neurologic or catastrophic sequelae, and in
this group no mutations have been detected in the genes encoding the glycine
receptor.

REFERENCES
1. Visser AM, Jaddoe VW, Arends LR, et al.. Paroxysmal disorders in infancy and their
risk factors in a population-based cohort: the Generation R Study. Dev Med Child
Neurol 2010; 52:1014.
2. Volpe, JJ. Neurology of the Newborn, 4th ed, WB Saunders Co, Philadelphia 2001.
3. Bakker MJ, van Dijk JG, van den Maagdenberg AM, Tijssen MA. Startle syn-
dromes. Lancet Neurol 2006; 5:513.
4. Zhou L, Chillag KL, Nigro MA. Hyperekplexia: a treatable neurogenetic disease.
Brain Dev 2002; 24:669.
5. Giacoia GP, Ryan SG. Hyperekplexia associated with apnea and sudden infant
death syndrome. Arch Pediatr Adolesc Med 1994; 148:540.
6. Praveen V, Patole SK, Whitehall JS. Hyperekplexia in neonates. Postgrad Med J
2001; 77:570.

248
Case 43
A 13-month-old female child, product of second degree consanguineous
marriage, with an uneventful perinatal history presented to our hospital
with status epilepticus, delayed developmental milestones and regression of
the achieved milestones. She was the third living child of a sixth pregnancy
from second-generation, consanguineous parents, and had brothers who died
within their first week of life because of respiratory failure.
On physical examination, she was unconscious (Glasgow Coma Scale-5) and
afebrile. Her weight and height were below the third percentile, but her head
circumference was normal. Pupils were dilated and sluggishly when reacting
to light. She had marked spasticity, having flexor posture on the upper and
extensor posture on the lower extremities, with brisk deep tendon reflexes,
bilateral positive extensor plantar response, and Clonus. Gag reflex and
swallowing were absent. Eye movements were normal, with no nystagmus.
The fundus examination was unremarkable.
She had mild metabolic acidosis, with normal anion gap. Serum amino acid
screening, blood ammonia, lactate, and pyruvate levels were normal; however,
urine amino acid assay showed increased glycine, alanine, and glutamine.
CSF analysis was normal, including lactate and pyruvate levels, with lactate
to pyruvate ratio measured at 1200:100 mmol/L. EEG showed slowing on
background rhythm and no epileptic activity.
Brain MR imaging showed bilateral, symmetrical involvement of the parieto-
occipital periventricular white matter, the posterior aspect of the posterior
crus of the internal capsules, and abnormality of the corpus callosum at the
genu and splenium, with T2 hyperintensity and T1 hypointensity. There were
cystic foci within the involved periventricular white matter. Foci of contrast
enhancement, mostly located at the periphery of the abnormal areas, were
present. The basal ganglia, thalami, brain stem, and cerebellum were not
involved, and the cortex appeared normal.

Answer the following questions:


• Give the most likely diagnosis.
• Summarise the case briefly.
• Provide a differential diagnosis.
• Discuss an appropriate diagnostic work-up.
• Discuss the management of the patient.

249
Child Neurology

DIAGNOSIS
Leigh syndrome (subacute necrotising encephalopathy)

SUMMARY
A 13-month-old female child, presented with status epileptics, delayed devel-
opmental milestones and regression of the achieved milestones. The above
clinical findings were highly suggestive of a neurodegenerative disorder. Thus,
given this child’s clinical picture in addition to her MRI brain findings, a mito-
chondrial disorder such as Leigh disease seems to be the most likely diagnosis.

DIFFERENTIAL DIAGNOSIS
Many metabolic diseases can present with acute or subacute developmental re-
gression with illness in this age group. Leigh syndrome is a severe neurological
disorder that usually becomes apparent in the first year of life. This condition
is characterised by progressive loss of mental and psychomotor regression and
typically results in death within two to three years, usually due to respiratory
failure. A small number of individuals do not develop symptoms until adult-
hood or have symptoms that worsen more slowly.
The first signs of Leigh syndrome seen in infancy are usually vomiting, diar-
rhoea, and dysphagia. These problems often result in failure to thrive. Affected
individuals may develop hypotonia, dystonia, ophthalmoparesis, nystagmus,
optic atrophy and ataxia. Peripheral neuropathy is also common in patients
with Leigh syndrome.
The signs and symptoms of Leigh syndrome are caused by lesions that develop
in the brains in this condition. Magnetic Resonance Imaging (MRI) reveals
characteristic lesions in the basal ganglia, cerebellum, and brainstem. The
brain lesions are a result of demyelination.
In general, progressive encephalopathy in children younger than 2 years of
age can be caused by aminoacidopathies (maple syrup urine disease, phe-
nylketonuria), lysosomal storage diseases (GM1/GM2 gangliosidoses, Krabbe
disease, Gaucher disease, mucopolysaccharidoses, Niemann-Pick disease,
metachromatic leukodystrophy), mitochondrial disorders, leukodystrophies
(Canavan disease, Alexander disease), peroxisomal disorders (Zellweger syn-
drome and variants), and glycogen storage diseases (Pompe disease), as well as
several others (ceroid lipofuscinosis, Menkes disease, biotinidase deficiency,
Rett syndrome, etc.). Fatty acid oxidation defects, such as medium-chain acyl-
coenzyme. A dehydrogenase (MCAD) deficiency, typically present in children
younger than 2 years of age with signs of a metabolic crisis. Vomiting, lethargy,
and coma occur with fasting or intercurrent illness. Developmental delay, epi-
lepsy, and proximal weakness are frequent long-term sequelae of this group
of disorders. These diseases can be differentiated based on the clinical pres-
entation, presence or absence of dysmorphic features, other organ involve-
ment (ophthalmologic abnormalities, organomegaly), and the results of initial
screening tests.

250
Case 43

DIAGNOSTIC WORK-UP
The evaluation of a child with a suspected neurodegenerative disorder begins
with a detailed history of pregnancy, birth, developmental, and family. A his-
tory of miscarriages in the mother or consanguinity may be relevant. It is im-
portant to inquire whether there is a family history of epilepsy, mental retar-
dation, unexplained death in infants, developmental delay, autism, myopathy,
peripheral neuropathy, migraine, diabetes, deafness, vision loss, or any other
neurologic disease. Lactic acidosis can represent a primary defect of anaerobic
metabolism or a secondary defect caused by other medical conditions such
as multiorgan system failure or sepsis. Persistent lactic acidosis in a medically
stable child suggests a primary disorder. Several other inborn errors of metabo-
lism can be associated with lactic acidosis (fatty acid oxidation disorders, maple
syrup urine disease, organic acidurias, biotinidase deficiency) but abnormali-
ties are also seen on serum amino acid and urine organic acid studies in these
conditions whereas CSF changes are hall marks of mitochondrial cytopathies
such as leigh disease.
• Initial lab work should include a CBC, basic chemistry panel, glucose,
liver function tests, and creatine kinase (given her apparent weakness).
The presence of hypoglycaemia, elevated liver function tests, elevated cre-
atine kinase, or metabolic acidosis is important. A lumbar puncture should
be considered to rule out central nervous system infection, although the
child does not appear to have overt signs of meningitis or encephalitis
(headache, photophobia, meningismus). If a lumbar puncture is done,
CSF lactate should be measured.
• Screening for metabolic diseases should begin with serum lactate/pyru-
vate, urine organic acids, plasma amino acids, ammonia, and urine ke-
tones. Serum and CSF lactate are elevated in Leigh disease, as well as in
other mitochondrial disorders. Persistent metabolic acidosis is also com-
mon. Lactate and pyruvate are in equilibrium with each other.
• Children with primary lactic acidosis either have normal or elevated pyru-
vate. The lactate-to-pyruvate ratio is calculated, with a normal value con-
sidered to be less than 20. Patients with proportionally elevated serum
pyruvate and lactate values will also have an elevated ratio? These findings
suggest a disorder of pyruvate metabolism like pyruvate dehydrogenase
deficiency. If serum lactate is elevated and pyruvate is normal or decreased,
the lactate-to-pyruvate ratio will be greater than 20. This suggest a dis-
order of the mitochondrial respiratory chain. Blood can also be sent for
commercially available analysis when a mitochondrial DNA mutation is
suspected.
• Given the head CT findings, a follow-up brain MRI with contrast should
be promptly performed. As discussed above, Leigh disease demonstrates
bilaterally symmetric signal abnormalities of the basal ganglia as well as
brainstem involvement.

251
Child Neurology

• Brain Magnetic Resonance Spectroscopy (MRS) can also be performed.


The presence of a lactate peak and reduced N-Acetylaspartate (NAA) in
affected areas supports the diagnosis. MRS can help differentiate mito-
chondrial disorders from other diseases that affect the basal ganglia (ma-
ple syrup urine disease, Wilson’s disease) as these other disorders do not
produce a lactate peak.
• After the initial work-up is complete, histochemical and biochemical
studies on a muscle biopsy specimen or skin fibroblasts are often re-
quired to evaluate for specific abnormalities of mitochondrial metabolism.
However, a normal serum/CSF lactate and normal muscle biopsy do not
rule out a diagnosis of a mitochondrial cytopathy because of the variable
tissue distribution of abnormal mitochondria in these conditions.
• A formal ophthalmologic examination, as many neurodegenerative disor-
ders are associated with retinal pigmentary abnormalities, optic atrophy,
and abnormal eye movements. Consultation with a geneticist early in the
hospital course is also essential to help guide the work-up.
• The diagnosis of Leigh syndrome may be confirmed by a thorough clini-
cal evaluation and a variety of specialised tests, particularly advanced im-
aging techniques. Magnetic Resonance Imaging (MRI) or Computed
Tomography (CT) scans of the brain may reveal abnormal signals in basal
ganglia, brain stem, and grey matter. Small or large cysts may be present in
the cerebral cortex of the brain.
• Laboratory tests may reveal high levels of lactate as well as elevated levels
of pyruvate and alanine. Blood glucose may be slightly lower than normal.
The enzyme pyruvate carboxylase may be absent from the liver and an
inhibitor of Thiamine Triphosphate (TTP) production may be present
in the blood and urine of affected individuals. Some children with Leigh
syndrome may have detectable deficiencies of the enzymes pyruvate de-
hydrogenase complex or cytochrome C oxidase.

MANAGEMENT
• There are no proven therapies of any type for Leigh Syndrome. The treat-
ment of Leigh syndrome is directed toward the specific symptoms that are
apparent in each individual. Vitamin replacement therapy with thiamine,
riboflavin, coenzyme Q, and/or L-carnitine often is provided in an at-
tempt to maximise oxidative abilities of the mitochondria. Children with
pyruvate dehydrogenase deficiency often benefit from treatment with the
carbohydrate-deficient ketogenic diet. Other treatment is symptomatic
and supportive.
• Treatment may require the coordinated efforts of a team of specialists.
Paediatricians, cardiologists, neurologists, specialists who assess and treat
hearing problems (audiologists), eye specialists, and other health care pro-
fessionals may need to systematically and comprehensively plan an effec-
tive child’s treatment.

252
Case 43

• Genetic counseling in Leigh disease is complex and depends on the in-


heritance pattern. Thus, after a diagnosis is confirmed, the geneticist
plays an important role in screening other family members and providing
counseling.

REFERENCES
1. Baertling F, Rodenburg RJ, Schaper J, et al.. (June 2013). “A guide to diagnosis and
treatment of Leigh syndrome”. J. Neurol. Neurosurg. Psychiatr.
2. Genetics Home Reference. National Institute of Health. 23 September 2013.
3. Rahman S, Blok RB, Dahl HH, Danks DM, Kirby DM, Chow CW, Christodou-
lou J, Thorburn DR. Leigh syndrome: clinical features and biochemical and DNA
abnormalities. Ann Neurol. 1996 Mar;39(3):343-51.
4. Leigh’s Disease Information Page. National Institute of Neurological Disorders
and Stroke. December, 2011.
5. Arii J, Tanabe Y. Leigh syndrome: serial MR imaging and clinical follow-up. AJNR
Am J Neuroradiol. 2000;21 (8): 1502-9.

253
44 Case
A 10-year-old child presented with a 3-week history of headache, weak-
ness of the right upper and lower limbs and a one-day history of in-
ability to talk and irrational behaviour. There was associated anorexia, vomit-
ing, fever, neck stiffness and photophobia. She had no seizures or blurring
of vision. There was no personal or family history of hypertension, diabetes
mellitus or sickle cell disease or blood transfusion. Her father was receiving
malaria treatment because it was an endemic infection in the region. She did
not have any recent travel. There was no family history of TB.
Physical examination revealed pyrexia (temperature of 39 °C) and altered
level of consciousness (Glasgow coma score = 8/15). The signs of meningeal
irritation were present and she had anisocoria and right spastic hemiparesis.
The pupils were bilaterally dilated (6 mm) and were unresponsive to light.
There was no papilloedema but she had right facial paresis (upper motor
neuron type). At the same time, she had hypertension and bradycardia
suggesting increased intracranial pressure. The scar for BCG vaccination was
not present.
Initial investigations showed an Erythrocyte Sedimentation Rate (ESR) of
112 mm/hour (Westergren method). The Mantoux test was non-reactive.
Cerebrospinal Fluid (CSF) examination revealed a pleocytosis of 350/cell/
mm3, which were 90% lymphocyte, protein 720 mg/dl, glucose 15 mg/
dl. Mycobacterium tuberculosis was not shown by neither cultures nor
stains. Chest X-ray did not show evidence of pulmonary tuberculosis, but
enlarged mediastinal nodes were detected by Computerised Tomography
(CT). A brain Computed Tomography (CT) scan showed basal meningeal
enhancement associated with moderate dilatation of all ventricles with
associated visualisations of the temporal horns of the left lateral ventricle, in
keeping with early obstructive hydrocephalus.

Answer the following questions:


• Give the most likely diagnosis.
• Summarise the case briefly.
• Provide a differential diagnosis.
• Discuss an appropriate diagnostic work-up.
• Discuss the management of the patient.

254
Case 44

DIAGNOSIS
Tuberculous meningitis

SUMMARY
A 10-year-old child had headache, weakness of the right upper and lower
limbs, right sided facial palsy, and inability to talk, with irrational behaviour.
There was associated anorexia, vomiting, fever, neck stiffness and photopho-
bia. She had pyrexia with an altered level of consciousness. The signs of me-
ningeal irritation were present and she had anisocoria. At the same time, she
had hypertension and bradycardia suggesting increased intracranial pressure.

DIFFERENTIAL DIAGNOSIS
CNS tuberculosis accounts for about one percent of all cases of TB and six
percent of all extrapulmonary infections in immunocompetent individu-
als. Central Nervous System (CNS) Tuberculosis (TB) includes three clini-
cal categories: meningitis, intracranial tuberculoma, and spinal tuberculous
arachnoiditis. All three forms of CNS infection are encountered frequently
in regions of the world where the incidence of TB is high and the prevalence
of post-primary dissemination is common among children and young adults.
The differential diagnosis of CNS TB is that of a subacute or chronic menin-
gitis syndrome with a CSF picture characterised by a lymphocytic pleocytosis,
lowered glucose concentration, and a high protein content. This is seen most
commonly with cryptococcosis, occasionally with other deep-seated granu-
lomatous fungal infections, brucellosis, and neurosyphilis. A similar syndrome
may be encountered in patients with a parameningeal suppurative infection,
(e.g., sphenoid sinusitis, brain abscess, or spinal epidural space infection).
Patients with herpes encephalitis may exhibit a similar CSF picture, including
mild lowering of CSF glucose concentration.
• Fungal meningitis (cryptococcosis, histoplasmosis, blastomycosis,
coccidio-idomycosis)
• Viral meningoencephalitis (herpes simplex, mumps)
• Parameningeal infection (sphenoid sinusitis, brain abscess, spinal epidural
abscess)
• Partially treated bacterial meningitis
• Acute disseminated encephalomyelitis
• Neurosyphilis
• Neoplastic meningitis (lymphoma, carcinoma)
• Neurosarcoidosis
• Neurobrucellosis
• Vasculitis: Isolated Central Nervous System (CNS) angiitis, systemic gi-
ant cell arteritis, Wegener granulomatosis, polyarteritis nodosa, noninfec-
tious granulomatosis, lymphomatoid granulomatosis

255
Child Neurology

DIAGNOSTIC APPROACH
Although early and rapid identification of TBM is crucial for successful dis-
ease management, in most of the cases, diagnosis is significantly delayed.
Initial signs and symptoms of disease are non-specific and the suspicion of
TBM usually arises only some days or weeks after the disease’s onset and is
not different in children who have or have not been vaccinated with Bacille
Calmette-Guerin.
Fever, headache, anorexia, and vomiting characterise the prodrome of disease
in older children, whereas failure to thrive, poor appetite, vomiting, and sleep
disturbances are more common in younger ones. TBM is more easily sus-
pected when these symptoms are associated with a history of recent contact
with a case of documented TB or when, after the first days of disease, relevant
neurological manifestations, such as cranial nerve palsy, occur.
The usual patient with tuberculous meningitis presents with a subacute febrile
illness that progresses through three discernible phases.
• The prodromal phase, lasting two to three weeks, is characterised by the
insidious onset of malaise, lassitude, headache, low-grade fever, and per-
sonality change.
• The meningitic phase follows with more pronounced neurologic features,
such as meningismus, protracted headache, vomiting, lethargy, confusion,
and varying degrees of cranial nerve and long-tract signs.
• The paralytic phase, is the stage during which the pace of illness may ac-
celerate rapidly; confusion gives way to stupor and coma, seizures, and
often hemiparesis. For the majority of untreated patients, death ensues
within five to eight weeks of the onset of illness.
Atypical presentations. Cases with atypical features that mimic other neu-
rologic conditions are important to recognise. As an example, patients may
present with an acute, rapidly progressive, meningitic syndrome suggesting
pyogenic meningitis or with a slowly progressive dementia over months or
even years characterised by personality change, social withdrawal, loss of li-
bido, and memory deficits. Less common is an encephalitic course manifested
by stupor, coma, and convulsions without overt signs of meningitis.
Clinical stages. It is useful for prognosis and therapy to categorise patients
with tuberculous meningitis on presentation by the stage of illness, based
upon the mental status and focal neurologic signs.
• Stage I patients are lucid with no focal neurologic signs or evidence of
hydrocephalus.
• Stage II patients exhibit lethargy, confusion; may have mild focal signs,
such as cranial nerve palsy or hemiparesis.
• Stage III represents advanced illness with delirium, stupor, coma, seizures,
multiple cranial nerve palsies, and/or dense hemiplegia.

256
Case 44

Probable or possible TBM


Diagnosis of probable or possible TBM requires signs and symptoms of men-
ingitis in association with clinical, CSF and cerebral imaging findings sugges-
tive of M. tuberculosis infection. The evidence of TB outside the CNS can
further contribute to probable or possible diagnosis. A score that includes the
most common findings in children with TBM and in which single findings are
assigned a point according to the frequency with which they are usually dem-
onstrated has been created by Marais et al.. According to these authors, prob-
able TBM is defined by a score between 10 and 12, whereas possible TBM is
defined by a score higher than 6 (Table 44.1).
Table 44.1: Diagnostic criteria for classification of definite, probable or
possible tuberculous meningitis.

Clinical criteria Maximum category


score=6

Duration of symptom more than 5 days 4

Systemic symptoms suggestive of TB 2


(one or more of the following): weight
loss or poor weight gain, night sweats or
persistent cough for more than 2 weeks

History of recent contact (within the past year) 2


close contact with an individual with pulmonary TB or a
positive tuberculin skin test or interferon-
interferon-ϒ release assay

Focal neurologic deficit (excluding facial nerve palsies) 1

Cranial nerve palsy 1

Altered consciousness 1
CSF criteria Maximum category score 4

Clear appearance 1

Cells 10-500 per µL 1


Lymphocytic predominance (>50%) 1

Protein concentration >1 g/L 1

CSF to plasma glucose ratio of less than 50% or an abso- 1


lute CSF glucose concentration <2.2 mmol/L

Cerebral imaging criteria 6


Maximum category score

Hydrocephalus 1

Basal meningeal enhancement 2

257
Child Neurology

Tuberculoma 2

Infarct 1
Pre-contrast basal hyperdensity 2
Evidence of tuberculosis elsewhere 4
Maximum category score

Chest X-ray suggestive of active TB: 2/4


(signs of TB = 2; miliary TB = 4)
CT/MR/ultrasound evidence for TB 2

outside the CNS 4


M. tuberculosis cultured from another source
(i.e. sputum, lymph node, gastric lavage,
urine, blood culture)
Positive commercial M. tuberculosis NAAT 4
(Nucleic Acid Amplification Test) from extra-neural
specimen
From marais et al., modified

CEREBROSPINAL FLUID EXAMINATION


The examination of CSF SF specimens is of critical importance to early diagno-
sis of TB meningitis. Typically, the CSF formula shows elevated protein and
lowered glucose concentrations with a mononuclear pleocytosis. CSF protein
ranges from 100 to 500 mg/dL in most patients; however, patients with suba-
rachnoid block may show extremely high levels in the range of 2 to 6 g/dL,
associated with xanthochromia which carries poor prognosis. The CSF glu-
cose is less than 45 mg/dL in 80 percent of cases. The usual CSF cell count is
between 100 and 500 cells/microL.
Early in the course of illness, the cellular reaction is often atypical with only a
few cells or with Polymorphonuclear Leukocyte (PMN) predominance. Such
cases usually rapidly change to a lymphocytic cellular response on subsequent
CSF examinations. Upon initiation of anti-tuberculous chemotherapy, the
CSF of some patients briefly reverts to a PMN cellular reaction, associated
(“therapeutic paradox”).
with transient clinical deterioration (“therapeu

BACTERIOLOGY
The importance of repeated, careful examination and culture of CSF speci-
mens for M. tuberculosis cannot be overemphasised. The demonstration of
acid-Fast Bacilli (AFB) in the CSF remains the most rapid and effective means
of reaching an early diagnosis. It is recommended that a minimum of three
lumbar punctures be performed at daily intervals, bearing in mind that empiric
therapy need not be delayed during this time.

258
Case 44

The sensitivity of the AFB smear may be enhanced by attention to the follow-
ing principles:
• It is best to use the last fluid removed at lumbar puncture, and recovery of
the organism improves if a large volume (10 to 15 ml) is removed.
• Organisms can be demonstrated most readily in a smear of the clot or
sediment. If no clot forms, the addition of 2 ml of 95 percent alcohol gives
a heavy protein precipitate that carries bacilli to the bottom of the tube
upon centrifugation.
• 0.02 ml of the centrifuged deposit should be applied to a glass slide in an
area not exceeding one centimeter in diameter and stained by the standard
Kinyoun or Ziehl-Neelsen method.
• Between 200 and 500 high-powered fields should be examined (approxi-
mately 30 minutes), preferably by more than one observer.

POLYMERASE CHAIN REACTION


Methods for the rapid detection of M. tuberculosis in CSF include the Nucleic
Acid-Based Amplification Test (NAAT) that relies upon the Polymerase Chain
Reaction (PCR). It is recommended that CSF specimens be submitted for
PCR testing whenever clinical suspicion is sufficiently high to warrant em-
piric therapy and AFB stains of the initial samples are negative. However, it is
important to recognise that a negative test result neither excludes the diagnosis
nor obviates the need for continued therapy.

NEURORADIOLOGY
Computed Tomography (CT) and Magnetic Resonance Imaging (MRI) have
improved greatly characterisation and management of CNS infections. CT
can define the presence and extent of basilar arachnoiditis, cerebral oedema
and infarction, and the presence and course of hydrocephalus. The following
observations can be derived from a review of selected clinical series:
• In a patient with compatible clinical features, CT evidence of basilar
meningeal enhancement combined with any degree of hydrocephalus is
strongly suggestive of tuberculous meningitis.
• The CT scan is normal in approximately 30 percent of cases with Stage
I meningitis, and patients with a normal scan nearly always recover com-
pletely on therapy.
• Hydrocephalus combined with marked basilar enhancement is indicative
of advanced meningitic disease and carries a poor prognosis. Marked basi-
lar enhancement correlates well with vasculitis and, therefore, with a risk
of basal ganglia infarction.
The diagnosis of tuberculoma is made on the basis of clinical and radiographic
findings or by needle biopsy. The diagnosis of spinal tuberculous arachnoiditis
is based upon MRI changes of nodular arachnoiditis, combined with tissue bi-
opsy. MRI is superior to CT in defining lesions of the basal ganglia, midbrain,
and brainstem and for evaluating all forms of suspected spinal diseases.

259
Child Neurology

MANAGEMENT
Treatment of TBM is based on three different components: administration of
anti-infective drugs active against M. tuberculosis, modulation of the destruc-
tive elements of the immune response, and management of increased intrac-
ranial pressure.
Intensive phase—A —A four drug regimen that includes INH, RIF, PZA, and
either EMB or STM for two months followed by:
Continuation phase—INH —INH and RIF alone if the isolate is fully susceptible,
for an additional seven to 10 months.

CHEMOTHERAPY
Main Guidelines for the Treatment of Tuberculous Meningitis in Infants
and Children

BRITISH INFECTION SOCIETY


• Isoniazid 10-20 mg/kg/24 hours (max 500 mg) orally for 12 months
• Rifampin 10-20 mg/kg/24 hours (max 600 mg) orally for 12 months
• Pyrazinamide 30-35 mg/kg/24 hours (max 2 g) orally for 2 months
• Ethambutol 15-20 mg/kg/24 hours (max 1 g) orally for 2 months
• Prednisolone 4 mg/kg/24 hours orally for 4 weeks, followed by a reducing
course over 4 weeks

AMERICAN THORACIC SOCIETY, CDC, AND INFECTIOUS


DISEASES SOCIETY OF AMERICA
• Isoniazid 10-15 mg/kg/24 hours (max 300 mg) orally for 9-12 months
• Rifampin 10-20 mg/kg/24 hours (max 600 mg) orally for 9-12 months
• Pyrazinamide 15-30 mg/kg/24 hours (max 2 g) orally for 2 months
• Ethambutol 15-20 mg/kg/24 hours (max 1 g) orally for 2 months
• Dexamethasone 8 mg/day/24 hours orally for children weighing less than
25 kg and 12 mg/day for children weighing 25 kg or more for 3 weeks,
followed by a reducing course over 3 weeks

WORLD HEALTH ORGANISATION


• Isoniazid 10-15 mg/kg/24 hours (max 300 mg) orally for 6 months
• Rifampin 10-20 mg/kg/24 hours (max 600 mg) orally for 6 months
• Pyrazinamide 15-30 mg/kg/24 h (max 2 g) orally for 2 months
• Streptomycin 20-40 mg/kg (max 1 g) I/M or I/V for 2 months
• Prednisone 2 mg/kg/24 hours orally for 4 weeks, followed by a reducing
course over 1-2 weeks

260
Case 44

Recommended daily dosages of second-line anti-tuberculous drugs


for treatment of tuberculous meningitis in infants and children.

DRUG DOSAGE
• Ethionamide 20 mg/kg/24 hours (max 1 g/day) orally as a single daily dose
• Cycloserine 10-15 mg/kg/24 hours (max 1 g/day) orally as a single daily
dose
• Steptomycin 20-40 mg/kg/24 hours (max 1 g/day) I/M or I/V as a single
daily dose
• Para-amino-salicylic acid 200-300 mg/kg/24 hours orally in 2-4 doses
• Capreomycin 15-30 mg/kg/24 hours (max 1000 mg) orally as a single daily
dose
• Amikacin and Kanamycin 15-30 mg/kg/24 hours (max 1 g/day) I/M or I/V
as a single daily dose
• Ofloxacin 15-20 mg/kg/24 hours (max 800 mg) orally as a single daily dose
• Levofloxacin 7.5-10 mg/kg/24 hours (max 500 mg) orally as a single daily
dose
• Moxifloxacin 7.5-10 mg/kg/24 hours (max 500 mg) orally as a single daily
dose
• Ciprofloxacin 20-30 mg/kg/24 hours (max 1.5 g) orally as a single daily
dose
Glucocorticoids. There is considerable experimental evidence and a growing
base of clinical data that adjunctive glucocorticoid therapy is beneficial in both
adults and children with tuberculous meningitis.
Drug-resistant M. tuberculosis. CNS infection caused by strains resistant
to one or more first line drugs is becoming increasingly prevalent. Those most
at risk for drug-resistant disease include individuals from areas of the world
where TB is endemic, those with a history of previous anti-tuberculous treat-
ment, homeless persons, and those with exposure to source patients harbour-
ing drug-resistant organisms. The World Health Organisation estimates that
approximately 10 percent of clinical isolates worldwide of M. tuberculosis are
resistant to one or more first-line anti-tuberculosis drugs. The impact of re-
sistance on treatment outcome varies according to which drug is ineffective.
Duration of therapy. It is recommended that therapy be administered for
nine to 12 months in drug-sensitive infections. If PZA is omitted or cannot be
tolerated, treatment should be extended to 18 months.
Surgery. Patients with hydrocephalus may require surgical decompression
of the ventricular system in order to effectively manage the complications of
raised intracranial pressure. In such patients with clinical stage II disease, the
combination of serial lumbar puncture and steroid therapy may suffice while
judging the early response to chemotherapy. However, surgical intervention
should not be delayed in patients with stupor and coma or when the clinical
course on therapy is marked by progressive neurologic impairment.

261
Child Neurology

Unlike other CNS mass lesions, medical management is preferred for clinical
tuberculomas, as surgical removal has often been complicated by severe fatal
meningitis, unless the lesion produces obstructive hydrocephalus or compres-
sion of the brainstem. In the past, surgical resection was often complicated by
severe fatal meningitis.

REFERENCES
1. Bahemuka M, Murungi JH. Tuberculosis of the nervous system. A clinical, radio-
logical and pathological study of 39 consecutive cases in Riyadh, Saudi Arabia. J
Neurol Sci 1989; 90:67.
2. Marais S, Thwaites G, Schoeman JF, Török ME, Misra UK, Prasad K, et al. Tu-
berculous meningitis: a uniform case definition for use in clinical research. Lancet
Infect Dis 2010; 10:803-12.
3. Kennedy DH, Fallon RJ. Tuberculous meningitis. JAMA 1979; 241:264.
4. Farinha NJ, Razali KA, Holzel H, et al.. Tuberculosis of the central nervous system
in children: a 20-year survey. J Infect 2000; 41:61.
5. Kumar R, Singh SN, Kohli N. A diagnostic rule for tuberculous meningitis. Arch
Dis Child 1999; 81:221-4
6. Kent SJ, Crowe SM, Yung A, et al.. Tuberculous meningitis: a 30-year review. Clin
Infect Dis 1993; 17:987.
7. Thwaites GE, Chau TT, Farrar JJ. Improving the bacteriological diagnosis of tu-
berculous meningitis. J Clin Microbiol 2004; 42:378.
8. Starke JR. Tuberculosis of the central nervous system in children. Semin Pediatr
Neurol 1999; 6:318-31
9. Blumberg HM, Burman WJ, Chaisson RE, et al.. American Thoracic Society/Cen-
tres for Disease Control and Prevention/Infectious Diseases Society of America:
treatment of tuberculosis. Am J Respir Crit Care Med 2003; 167:603.
10. World Health Organization (WHO). Global tuberculosis control: surveillance,
planning, financing. Geneva: WHO, 2004.
11. Schoeman JF, Van Zyl LE, Laubscher JA, Donald PR. Effect of corticosteroids on
intracranial pressure, computed tomographic findings, and clinical outcome in
young children with tuberculous meningitis. Pediatrics 1997; 99:226.
12. Drobniewski F, Nikolayevskyy V, Maxeiner H, et al. Rapid diagnostics of tuber-
culosis and drug resistance in the industrialized world: clinical and public health
benefits and barriers to implementation. BMC Med 2013; 11:190.
13. World Health Organization. Fluorescent Light-Emitting Diode (LED) Micros-
copy for Diagnosis of Tuberculo-sis. Policy Statement WHO/HTM/TB/2011.8
Available at http://www.who.int/tb/publications/2011/led_microscopy_diagnosis_
(28 January 2015, date last accessed).
14. World Health Organization. Treatment of tuberculosis; guidance for national
tuberculosis programs on the management of tuberculosis in children. G eneva,
Switzerland: World Health Organization; 2006. Available: whqlibdoc.who.int/
hq/2006/WHO_HTM_TB_2006.371_eng.pdf. Accessed 2014 Feb7.
15. Thwaites G, Fisher M, Hemingway C, Scott G, Solomon T, Innes J. British Infec-
tion Society guidelines for the diagnosis and treatment of tuberculosis of the cen-
tral nervous system in adults and children. J Infection. 2009; 59: 167-87.

262
Case 45
A 10-year-old female child presented with difficulty in standing up from
a sitting posture, initiating walking after sitting for some time, and dif-
ficulty in climbing stairs, for past 2 years. Her mother stated that this painless
“stiffness of muscles” resolved as she walked about 15-20 steps, after which
her gait used to get normal. There was no history of progression or fluctuation
in severity or exacerbation in cold weather. She was born to a second degree
consanguineously wedded couple and had a normal neonatal course. There
was no feeding difficulty or difficulty in opening eyes after crying, during ear-
ly childhood. There was no muscle pain or cramps. She was otherwise healthy
and her motor and cognitive development has been normal. Her father has
similar symptoms but they did not interfere with his daily activities and living
and he has never discussed it with his doctor.
On physical examination,, getting up from a sitting posture was difficult and
initiation of movement was slow, which improved as she continued to walk.
Before getting up to walk, she complained of muscle stiffness. She had a normal
flat, heel, toe, and tandem gait. There were no dysmorphic facial features. She
was seen as alert and cooperative. Her speech was fluent without dysarthria.
Her pupils were equal, round, and reactive to light. Her extraocular muscles
were intact and her face was symmetric with normal strength. It took several
seconds for her to open her eyes after closing them. The tongue is midline
without fasciculation. She had normal tone with 5/5 strength throughout.
There was no dysmetria or tremor. Light touch, temperature, and vibration
were intact throughout. Reflexes were 2+ throughout with bilateral plantar
flexor responses. She had marked hypertrophy of calf muscles, thighs, gluteal
muscles and biceps. There was no evidence of handgrip or eyelid myotonia.
Rest of the clinical examination was normal. Clinical examination of both the
parents and her sibling were normal.

Answer the following questions:


• Give the most likely diagnosis.
• Briefly summarise this case.
• Localise the examination findings.
• Provide a differential diagnosis.
• Discuss an appropriate diagnostic work-up.
• Discuss the management of the patient.

263
Child Neurology

DIAGNOSIS
Myotonia Congenita

SUMMARY
The patient was healthy, developmentally normal, who presented with com-
plaints of painless muscle stiffness specially while initiating movement, symp-
toms that were also reported by her father. Her examination was remarkable
for eyelid myotonia and marked hypertrophy of calf, thighs, gluteal, and biceps
muscles. A diagnosis of myotonia congenita was entertained based on non-
progressive painless myotonia, with “Herculean appearance”, percussion myo-
tonia of the tongue and Electromyography (EMG) revealed typical repetitive
discharges of varying amplitude and frequency along with the classical “dive
bomber’s sound” indicating myotonia.

Localisation
This child’s complaints of stiffness and eyelid myotonia localise to the muscle.
Calf hypertrophy also supports localisation to the muscle.

DIFFERENTIAL DIAGNOSIS
Thomsen’s disease (autosomal-dominant myotonia congenita) and Becker’s
disease (autosomal-recessive myotonia congenita) represent two forms of
Myotonia Congenita which are chloride channelopathies. Generalised myoto-
nia is the major clinical symptom in dominant and recessive forms of chloride
channel myotonia congenita. These chloride channelopathies produce myo-
tonia as their primary symptom, and resemble the sodium channel disorders
without periodic paralysis, and/or may also resemble mild forms of DM1 and
DM2 (Myotonic Dystrophy type 1 and 2).
Other causes of muscle stiffness or poor coordination, including central nerv-
ous system diseases affecting the frontal lobes, brainstem, and cerebellum re-
quire initial consideration but are rarely confused with these channelopathies
based on a clinical picture of cognitive involvement and disturbance of higher
CNS functions.
Symptoms develop in the first or second decade of life. Myotonic stiffness oc-
curs with sudden physical exertion after a period of rest. Repeated muscle con-
tractions ameliorate the stiffness. This response is the “warm-up phenomenon,”
which helps distinguishing chloride channel myotonia congenita from certain
forms of sodium channel myotonia that display increasing muscle stiffness with
repeated contractions, a paramyotonic response. Here, transient weakness ap-
pears for a few seconds during the initial attempt at a specific movement after
a period of inactivity. Muscle strength improves to normal after several strong
contractions. Patients have prominent muscle hypertrophy, especially in the
legs, in both the autosomal-dominant and autosomal recessive forms of myo-
tonia congenita. Tendon reflexes, cerebellar function, sensation, and strength

264
Case 45

stays normal. Myotonia can be noted on hand grip, and on eye closure, and with
lid lag. If patients lie supine for 5-10 minutes and arise suddenly, generalised
myotonic stiffness in the proximal and paraspinal muscles becomes apparent.
• Myotonia congenita must be differentiated from other conditions that
cause myotonia. This may prove challenging in many cases, as there is sig-
nificant clinical overlap with other chloride and sodium channelopathies.
Several of the myotonic disorders are associated with periodic paralysis;
however, distinguishing clinical features may be determined by factors
that precipitate or alleviate myotonia, presence of other organ system in-
volvement, and findings on electrodiagnostic studies.
• Myotonia congenita should first be differentiated from the more com-
mon myotonic dystrophy. Myotonic dystrophy is an autosomal dominant
disorder caused by an expansion of the CTG trinucleotide repeat on chro-
mosome 19 that codes for a serine/threonine protein kinase. The age of
onset of myotonic dystrophy varies widely. It may present as a severe con-
genital form with respiratory distress, poor feeding, and hypotonia. Onset
in childhood is usually associated with non-muscular symptoms such as
cognitive delay, clumsiness, cardiac arrhythmias, and gastrointestinal dys-
motility. Patients may have some characteristic facies with myopathy of
the face and eyes that also affects the arms, hands, and legs, but calf hyper-
trophy does not occur. Myotonia worsens with rest and cold. A warm-up
phenomenon may be described by some patients.
• Proximal Myotonic Myopathy (PROMM) is another autosomal domi-
nant myotonic condition, but its genetic defect is unknown. It is only
rarely seen in childhood, typically presenting in adolescence or adulthood.
Affected children usually complain of myotonia and pain in the hands
and legs. Calf hypertrophy may occasionally be seen. Later in life, affected
individuals demonstrate weakness of the proximal thighs, hips, and neck
flexors in addition to muscle pain, cataracts, and arrhythmias.
• Three autosomal dominant myotonic conditions are associated with a
mutation in the skeletal muscle sodium channel gene on chromosome 17:
myotonia fluctuans, myotonia permanens, and acetazolamide-responsive
sodium channel myotonia. They do not cause periodic paralysis. All three
disorders present during the first decade of life and have clinical features
similar to myotonia congenita. Only myotonia fluctuans, however, is asso-
ciated with calf hypertrophy. Symptoms fluctuate in severity, particularly
after exercise, which is not described by the patient in myotonia congenita.
Myotonia permanens causes severe myotonia of the face, extremities, and
respiratory muscles, and can result in hypoxia. Acetazolamide-responsive
sodium channel myotonia affects the eyelids, face, extremities, and par-
aspinal muscles. Symptoms are precipitated by fasting, cold, potassium,
and infections.
• Paramyotonia congenita is an autosomal dominant condition also caused
by a mutation in the skeletal muscle sodium channel gene on chromo-
some 17. Like myotonia congenita, it can present with generalised stiffness

265
Child Neurology

and muscle cramps in early to mid-childhood. Myopathy and muscle


hypertrophy, however, are not seen. Also, in contrast to myotonia con-
genita, children with paramyotonia congenita are extremely sensitive to
cold. Stiffness, myotonia, and weakness occur on cold exposure. Although
children with myotonia congenita experience an improvement in symp-
toms with exercise, children affected with paramyotonia have worsening
of their stiffness with repeated muscle contraction.
• Calf hypertrophy is also a classic physical examination finding in
Duchenne muscular dystrophy. However, affected children present at a
much younger age with progressive proximal weakness and motor delays.
Myotonia is not seen in muscular dystrophies.

DIAGNOSTIC WORK-UP
Making the diagnosis of a myotonic disorder can be challenging. A thorough
history of alleviating measures and provocative stimuli must be taken.
• It must also be determined whether the patient has associated symp-
toms of periodic paralysis as there is significant overlap between these
channelopathies.
• A family history is essential as many of these disorders are hereditary.
Other organ involvement may also help to suggest a specific disease.
• The evaluation should start by obtaining a CK level. The CK level in my-
otonia congenita may be normal or slightly elevated to three to four times
the upper limit of normal.
• EMG usually confirms the presence of myotonia.
• Specific DNA testing can be performed for myotonia congenita. Sequence
analysis detects a causative mutation in more than 95% of affected indi-
viduals with both autosomal dominant and recessive forms of the disease,
and currently is the most efficient, least-invasive means of arriving at a
definitive diagnosis.
• A muscle biopsy can be very helpful in determining the etiology of a my-
opathy. However, in myotonia congenita the muscle biopsy is typically
normal and a biopsy is usually not indicated because genetic testing is
available.

MANAGEMENT
• A variety of pharmacologic treatments has been used to control myoto-
nia. Anti-myotonia treatment with mexiletine is often helpful. The anti-
arrhythmic drug mexiletine is probably most effective. Before treatment
with mexiletine is initiated, a baseline EKG is necessary to identify pa-
tients with unsuspected cardiac conduction abnormalities. Side effects of
mexiletine, which are usually mild and dose-related, include dysgeusia,
light headedness, and diarrhoea.

266
Case 45

• Children with moderately severe myotonia develop heel-cord shortening


and contractures at their elbows. If these contractures do not respond to
stretching and other physical therapy exercises, it is appropriate to initiate
anti-myotonia treatment, even when patients do not complain of stiffness.
• Tocainide, procainamide, quinine, and phenytoin also have been found to
be beneficial in some patients; tocainide, a lidocaine derivative similar to
mexiletine, was another useful alternative, (but it is no longer available), as
have been carbamazepine, dantrolene, and acetazolamide.
• Depolarising muscle relaxants, beta-adrenergic agonists, beta-antagonists
(propranolol), and colchicines may worsen myotonia.
• In addition, many individuals with myotonia congenita need to adapt their
lifestyle and exercise to avoid factors that precipitate symptoms. Stretching
exercises should be encouraged to avoid the development of heel cord and
elbow contractures.
• Finally, any child found to have a myotonic disorder should be referred to
a geneticist. Other family members should undergo an appropriate work-
up and genetic counseling should be provided.

REFERENCES
1. Shivakumar S, Cooper EC. Myotonia Congenita. In: NORD Guide to Rare Dis-
orders. Lippincott Williams & Wilkins. Philadelphia, PA. 2003:632-3.
2. Zhang J, et al.. Functional consequences of chloride channel gene (CLCN1) muta-
tions causing myotonia congenita. Neurology. 2000; 54:937-42.
3. Zhang J, George AL Jr, Griggs RC, Fouad GT, Roberts J, Kwieciński
Kwieci H, Connolly
AM, Ptácek LJ. Mutations in the human skeletal muscle chloride channel gene
(CLCN1) associated with dominant and recessive myotonia congenita. Neurology.
1996 Oct;47(4):993-8.
4. Sun C, Tranebjaerg L, Torbergsen T, Holmgren G, Van Ghelue M. Spectrum of
CLCN1 mutations in patients with myotonia congenita in Northern Scandinavia.
Eur J Hum Genet. 2001 Dec;9(12):903-9. Erratum in: Eur J Hum Genet. 2010
Feb;18(2):264.
5. Moxley RT, Tawil R, Thornton CA. Channelopathies: myotonic disorders and pe-
riodic paralysis. In: Swaiman KS, Ashwal S, eds. Pediatric Neurology: Principles
and Practice. 5th ed. 1673-1680.

267
46 Case
A 3-year-old female presented with sudden onset of ataxia and left-sided
dystonic posturing of the limbs of 2-day duration which was preceded
by a history of runny nose and cough for several days but no history of fever,
vomiting, abnormal movement, or skin rash. She was the third child of the
family. She was born at full term to a 32-year-old G3P2 mother by an un-
eventful vaginal delivery. Her development was normal in all areas. There was
no family history of neurologic disease.
On physical examination: She was fully alert but with hypertonia and hy-
perreflexia of all her limbs, more so on the left side. She was able to walk with
support with an ataxic gait. Her pupils were equal, round, and reactive to light.
Her extraocular muscles were intact. The tongue was midline. She had normal
bulk with increased tone and significant stiffness and posturing of the left arm
and leg. She responded to tactile stimulation throughout. Plantar responses
were extensor bilaterally. Other systemic examination was unremarkable.
Brain MRI showed symmetric and bilateral increased T2 signal intensity in
the central part of the caudate head and putamen, as well as involvement of
the globi pallidi, thalami, infra-and supratentorial brain cortex, brain stem,
and cerebellum.

Answer the following questions:


• Give the most likely diagnosis.
• Briefly summarise this case.
• Localise the examination findings.
• Provide the differential diagnosis.
• Discuss an appropriate diagnostic work-up.
• Discuss the management of the patient.

268
Case 46

DIAGNOSIS
Biotin-Thiamine-Responsive Basal Ganglia Disease (BTBGD)
These basal ganglia changes were consistent with BTRBGD. She was treated
with high-dose biotin (10 mg/kg/day) and additional thiamine (100 mg/day).
This medication improved her clinical status dramatically. The diagnosis of
BTBGD was confirmed by identification of biallelic SLC19A3 pathogenic
variants.

SUMMARY
A female child presented with sudden onset of ataxia and left-sided dystonic
posturing associated with hypertonia and hyperreflexia of all her limbs espe-
cially on the left side. She was able to walk with support with ataxic gait with
bilateral plantar extensor responses. Brain MRI showed symmetric and bilat-
eral increased T2 signal intensity in the central part of the caudate head and
part or all of the putamen, as well as involvement of the basal ganglia, thalami,
infra-and supratentorial brain cortex, brain stem, and cerebellum. These clini-
cal and radiological findings were suggestive of BTBGD.

Localisation of Lesion
This child’s findings of spasticity, hyperreflexia, and bilateral plantar extensor
responses localise to the corticospinal tracts. Progressive dystonia is caused by
basal ganglia involvement.

BIOTIN-THIAMINE-RESPONSIVE BASAL GANGLIA


DISEASE (BTBGD)
Biotin-Thiamine-Responsive Basal Ganglia Disease (BTBGD) is characterised
by recurrent subacute encephalopathy and manifests as confusion, seizures,
ataxia, dystonia, supranuclear facial palsy, external ophthalmoplegia, and/or
dysphagia; which if left untreated can eventually lead to coma and even death.
Dystonia and cogwheel rigidity are nearly always present; hyperreflexia, ankle
clonus, and Babinski responses are common. Hemiparesis or quadriparesis may
be seen. Episodes are often triggered by febrile illness or mild trauma or sur-
gery. Less frequently, BTBGD presents as chronic or slowly progressive dysto-
nia, seizures, and/or psychomotor delay. Although onset is usually in childhood
(ages three to ten years), it is extremely variable, ranging from the newborn
period to adulthood. Prompt administration of biotin and thiamine early in the
disease course results in partial or complete improvement within days.

DIFFERENTIAL DIAGNOSIS
The differential diagnosis of BTBGD primarily includes numerous neurodegen-
erative disorders, as well as inborn errors of metabolism causing dystonia and other
movement disorders. Although acquired conditions, such as basal ganglia infection
(meningitis, encephalitis), ischaemia, vasculitis, structural abnormalities (tumour,
arteriovenous malformation), and toxins can cause a progressive dystonia.

269
Child Neurology

Mitochondrial disorders, primary dystonias (idiopathic torsion dystonia, do-


pa-responsive dystonia), and Wilson’s disease deserve consideration. As PKAN
is associated with acanthocytosis, the neuroacanthocytosis syndromes are also
in the differential diagnosis. Finally, MRI T2 hyperintensity in the globus pal-
lidus can be seen in other disorders such as Leigh disease, alpha-fucosidosis,
organic acidurias, post-infectious dystonia, as well as post-infarction dystonia,
and can occasionally be confused with PKAN.
BBGD is an important treatable differential diagnosis of Leigh syndrome. The
clinical picture may completely mimic a mitochondrial disorder, including lac-
tic acidaemia and detection of a lactate peak on magnetic resonance spectrosco-
py. Importantly, atypical brain imaging findings (i.e. cortical signal alterations;
indications of vasogenic oedema) should alert clinicians to the differential di-
agnosis BBGD. Generally, biotin and thiamine treatment is suggested in any
patient with symmetrical basal ganglia lesions and neurological symptoms un-
til BBGD is excluded or a mitochondrial dysfunction has been clearly identi-
fied. Additional disorders in the differential diagnosis:
• Mitochondrial disorders (including Leigh syndrome)
• Wernicke encephalopathy
• Organic acid disorders
• Toxic encephalopathy
• Infectious encephalopathy
• Inflammatory disease (including CNS vasculitis)
• Wilson’s disease
• Juvenile Huntington disease
• Torsion dystonia (DYT1)
• Dopa-responsive dystonia
• Acute Disseminated Encephalomyelitis (ADEM)
Dopa-responsive dystonia and Wilson’s disease are important to consider be-
cause they are treatable. In its chronic stage BTBGD shares clinical features
with several conditions including Wilson’s disease, juvenile Huntington dis-
ease, and Torsion Dystonia (DYT1); however, BTBGD can be differentiated
by its acute to subacute presentation.
• Leigh Disease: Leigh disease (subacute necrotising encephalomyelopa-
thy) is a mitochondrial disease that results from a disorder in the respira-
tory chain production of adenosine triphosphate. Clinical manifestations
can be highly variable, affecting children and (rarely) young adults and
typically causing central hypotonia, developmental regression or arrest,
ophthalmoplegia, respiratory and bulbar dysfunction, and ataxia. MR im-
aging findings include symmetric areas of T2 prolongation in the basal
ganglia, periaqueductal region, and cerebral peduncles, with putaminal
involvement being a consistent feature. When Leigh disease is suspected,
MR spectroscopy (best performed with long echo times) may reveal the

270
Case 46

presence of abnormally high lactate levels in the basal ganglia, which to-
gether with elevated serum and CSF lactate levels supports the diagnosis.
• Toxic Poisoning: Carbon monoxide, methanol, and cyanide are cellular
respiratory toxins that affect the mitochondria. The diagnosis is usually es-
tablished with appropriate toxicological and laboratory tests, with imaging
being used to assess brain damage. Because of their high metabolic activity,
respiratory toxins tend to cause bilateral MR imaging abnormalities in the
basal ganglia, sometimes with diffuse involvement of other brain struc-
tures. Carbon monoxide in particular has a propensity to affect the globus
pallidus. T2 prolongation in the basal ganglia is typical in acute poisoning,
often with restricted diffusion on diffusion-weighted MR images. In car-
bon monoxide poisoning, delayed leukoencephalopathy and T1 shortening
in the globus pallidus may be encountered. Haemorrhagic necrosis of the
putamen may be observed in cyanide and methanol poisoning, and white
matter oedema may be an additional feature of methanol poisoning.
• Wilson’s Disease: Wilson’s disease is caused by the accumulation of cop-
per resulting from a deficiency of ceruloplasmin, its serum transport pro-
tein. This disease, also known as hepatolenticular degeneration, affects the
liver, brain, and other tissues. The symptoms vary and include dysarthria,
dystonia, tremors, ataxia, parkinsonian symptoms, and psychiatric prob-
lems. Kayser-Fleisher rings in the cornea are characteristically associated
with Wilson’s disease. MR imaging findings include areas of T2 prolonga-
tion in the putamen (a common finding), globus pallidus, caudate nuclei,
and thalamus. Thalamic involvement is typically confined to the ventro-
lateral aspect. The cortical and subcortical regions, mesencephalon, pons,
vermis, and dentate nuclei may also be involved. Diffusion restriction is
often seen in the early stages of the disease.
• Neurodegeneration with Brain Iron Accumulation: NBIA is a het-
erogeneous group of disorders characterised by brain degeneration and
excessive iron deposition in the basal ganglia. Two clinical categories are
recognised: (a) classic early-onset, rapidly progressive disease; and (b)
atypical late-onset, slowly progressive disease. In classic NBIA and in one-
third of cases of atypical disease, the PANK2 gene is mutated. This gene
encodes a pantothenate kinase, and affected patients (with pantothenate
kinase–associated neurodegeneration [previously known as Hallervorden-
Spatz disease]) typically present with pyramidal or extrapyramidal signs,
dystonia, and dysarthria. In NBIA, the diagnostic MR imaging feature
is bilateral hypointensity in the globus pallidus at T2-weighted imaging,
which correlates with iron accumulation observed at pathologic exami-
nation. Patients with pantothenate kinase–associated neurodegenera-
tion with the PANK2 mutation demonstrate the “eye-of-the-tiger sign,”
with a high-signal-intensity centre surrounded by the more typical hy-
pointensity in the globus pallidus. The eye-of-the-tiger sign is not seen
in PANK2 mutation–negative patients. PKAN should be distinguished
from other disorders of NBIA. This group of diseases is characterised by

271
Child Neurology

progressive extrapyramidal symptoms and iron deposition in the basal


ganglia. Aceruloplasminemia and neuroferritinopathy do not produce the
classic “eye-of-the-tiger,” MRI findings and have different genetic defects.
They also tend to present in later life.
HARP (Hypobetalipoproteinaemia, Acanthocytosis, Retinitis Pigmentosa,
Pallidal degeneration) syndrome is now considered to be part of the
PKAN spectrum of disorders. Mutations in the PKAN2 gene have been
identified in affected families.

DIAGNOSTIC WORK-UP
Biotin-Thiamine-Responsive Basal Ganglia Disease (BTBGD) is suspected in
individuals with the following:
• Acute/subacute encephalopathy with seizures, extra pyramidal manifesta-
tions (dystonia, cogwheel rigidity, dysarthria, dysphagia), and pyramidal
tract signs (quadriparesis, hyperreflexia) typically in a child, age 3-10 years,
and usually preceded by febrile illness or some other stress. Cerebellar
signs, supranuclear facial nerve palsy, external ophthalmoplegia, and ataxia
are variably present.
• Brain MRI showing the following:
° Bilateral and symmetric increased T2 signal intensity in the caudate
nucleus, putamen, thalamus, infra-and supratentorial brain cortex,
and brain stem
° Vasogenic oedema during acute crises as demonstrated by Diffusion-
Weighted Imaging (DWI)/Apparent Diffusion Coefficient (ADC)
MRI
° Chronic changes including diffuse cerebral cortical and (to a lesser
extent) cerebellar atrophy, and gliosis in the affected regions
° Spinal cord involvement (seen in one affected individual)
• Normal laboratory investigations, including tandem mass spectrometry of
blood; urine Gas Chromatography-Mass Spectrometry (GC-MS); serum
concentrations of lactic acid, ammonia, biotin, and thiamine; serum bio-
tinidase enzyme activity; urine amino acids; plasma amino acids; liver en-
zymes; coagulation profile; lipid profile; and Cerebrospinal Fluid (CSF)
cell count, protein, glucose, and cultures. Serum concentration of lactic
acid may be high in the acute stage
• Family history consistent with autosomal recessive inheritance. Note: (1)
Presumably affected (but undiagnosed) sibs may have had unexplained
coma or encephalopathy. (2) To date, consanguinity has been reported in
a large portion of families.
Biotin-Thiamine-Responsive Basal Ganglia Disease (BTBGD) is con-
firmed in individuals with biallelic SLC19A3 pathogenic variants. Sequencing
of SLC19A3 is likely to identify pathogenic variants on both alleles and there-
fore provide molecular confirmation of BTBGD in most individuals with

272
Case 46

typical clinical and radiologic findings. Although large exon or whole-gene


deletions have not been reported, apparent heterozygosity for a single patho-
genic variant in SLC19A3 identified by sequence analysis in a typically affected
individual may indicate compound heterozygosity for a second unidentified
pathogenic variant and warrants deletion analysis.

MANAGEMENT
Appropriate treatment includes the following:
• ICU care during acute encephalopathic episode, including treatment of
seizures and increased intracranial pressure.
• Biotin and thiamine therapy. Biotin is given as 5-10 mg/kg and thiamine
as 300-900 mg orally.
° Symptoms typically resolve within days
° Biotin and thiamine need to be given lifelong
° During acute decompensation thiamine may be increased to double
the regular dose and given intravenously
• In the acute stage empiric treatment with anti-microbial/anti-viral agents.
Recommended until infectious causes of the acute/subacute encephalopa-
thy are ruled out.
• Anti-epileptic drugs for effective control of seizures
• Symptomatic treatment of dystonia, including administration of L-dopa
• Rehabilitation, physiotherapy, occupational therapy, and speech therapy
• Educational programs adapted to individual needs
• Education of the family regarding the importance of lifelong compliance
with medical therapy
• Genetic counseling is the process of providing individuals and families
with information on the nature, inheritance, and implications of genetic
disorders to help them make informed medical and personal decisions.

REFERENCES
1. Ozand PT, Gascon GG, Al Essa M, et al. Biotin-responsive basal ganglia disease: a
novel entity. Brain 1998; 121:1267-1279.
2. Alfadhel M, Almuntashri M, Jadah RH, et al. Biotin-responsive basal ganglia dis-
ease should be renamed biotin-thiamine-responsive basal ganglia disease: a retro-
spective review of the clinical, radiological and molecular findings of 18 new cases.
Orphanet J Rare Dis 2013; 8:83.
3. Perez-Duenas B, Serrano M, Rebollo M, et al. Reversible lactic acidosis in a new-
born with thiamine transporter-2 deficiency. Pediatrics 2013; 131:e1670-e1675.
4. Yamada K, Miura K, Hara K, et al. A wide spectrum of clinical and brain MRI find-
ings in patients with SLC19A3 mutations. BMC Med Genet 2010; 11:171.
5. Kono S, Miyajima H, Yoshida K, et al. Mutations in a thiamine-transporter gene
and Wernicke’s-like encephalopathy. N Engl J Med 2009; 360:1792-1794.

273
Child Neurology

6. Debs R. Biotin-responsive basal ganglia disease in ethnic Europeans with novel


SLC19A3 mutations BBGD in ethnic Europeans with SCL19A3 mutations. Arch
Neurol 2010; 67:126-130.
7. Kevelam SH, Bugiani M, Salomons GS, et al. Exome sequencing reveals mutated
2013
SLC19A3 in patients with an early-infantile, lethal encephalopathy. Brain 2013;
136:1534-1543.
8. Tabarki B, Al-Shafi S, Al-Shahwan S, et al.. Biotin-responsive basal ganglia disease
revisited: clinical, radiologic, and genetic findings. Neurology 2013; 80:261-267.
9. Zeng WQ, Al-Yamani E, Acierno JS, Jr, et al.. Biotin-responsive basal ganglia disease
maps to 2q36.3 and is due to mutations in SLC19A3. Am J Hum Genet 2005;
77:16-26.
10. Gerards M, Kamps R, van Oevelen J, et al.. Exome sequencing reveals a novel Mo-
roccan founder mutation in SLC19A3 as a new cause of early-childhood fatal Leigh
syndrome. Brain 2013; 136:882-890.
11. Serrano M, Rebollo M, Depienne C, et al.. Reversible generalized dystonia and en-
cephalopathy from thiamine transporter 2 deficiency. Mov Disord 2012; 27:1295-
1298.
12. Sremba LJ, Chang RC, Elbalalesy NM, et al.. Whole exome sequencing reveals
compound heterozygous mutations in SLC19A3 causing biotin-thiamine respon-
sive basal ganglia disease. Mol Genet Metab Reports2014; 1:368-372.

274
Case 47
A 5-year-old boy was brought to the paediatric neurology clinic with one
episode of protracted vomiting while he was sleeping at night. The
child became so weak that he was unable to walk or even stand and remained
quiet in his bed. He was unresponsive to any call although his eyes were
open. He had profuse salivation with drooling of saliva and passed urine and
stool during the episode. He was taken to a local hospital for treatment and
received fluids. The whole episode lasted for about one hour. Then the child
gradually became almost normal but with a failure to recall the events in the
preceding hour. Later, the next day he started vomiting and became very pale
while playing. However, the parents noticed this time some jerky movements
involving the left hand and twitching of the facial muscles on the same side,
which lasted for around 30 minutes. He was again taken to the hospital where
he was given injection diazepam which aborted the attack. The patient had
incontinence of urine and vomited for 5-6 times as well. In the postictal period
the patient was very weak, drowsy and tired. The whole episode lasted for
about one hour, when he was brought to our clinic. The child’s intelligence
was normal with normal milestones of development. He was the second issue
of a non-consanguineous parents with no family history of seizure disorder.
On physical examination,, he was mildly tachycardic and appeared pale,
but was not ill-appearing or diaphoretic. He answered questions with
inappropriate responses and did not cooperate fully with the examination.
His pupils were dilated but reactive. The EEG revealed spike and-wave
complexes in the occipital (O1, O2) and posterior temporal (T5, T6) region
which disappeared on eye opening with a normal background. MRI brain
revealed a normal result.

Answer the following questions:


• Give the most likely diagnosis.
• Summarise the case briefly.
• Provide a differential diagnosis.
• Discuss an appropriate diagnostic work-up.
• Discuss the management of the patient.

275
Child Neurology

DIAGNOSIS
Panayiotopoulos Syndrome

SUMMARY
A 5-year-old normal boy presented with protracted vomiting during sleep
state and remained unresponsive to any call although his eyes were open. He
had profuse salivation with drooling of saliva with loss of sphincters control.
The child again had another episode, started vomiting, and became very pale
with jerky movements involving the left hand and twitching of facial muscles,
which lasted for around 30 minutes. The whole episode lasted for about one
hour and he was brought to our clinic immediately after the event. Based on
the semiology of the seizures, normal MRI brain and a three-hour video-EEG
recording, the child was diagnosed with PS.

DIFFERENTIAL DIAGNOSIS
The ILAE recognised early-onset childhood epilepsy with occipital spikes
as Panayiotopoulos type and differentiate it from the later-onset of occipital
epilepsy, described as Gastaut epilepsy. This syndrome is thought to account
for between 3-6% of all paediatric epilepsy cases (up to 15 years of age). PS
is typically seen between 3 and 5 years of age; it affects both boys and girls.
Panayiotopoulos syndrome is characterised by autonomic and behavioural
disturbances, with vomiting, deviation of the eyes, and impairment of con-
sciousness that can progress to convulsions. Seizures are long-lasting, often
more than 3 minutes. PS is about half as frequent as benign focal epilepsy with
central-temporal spikes by some estimates, and carries an excellent progno-
sis. However, seizures can be prolonged, are marked by autonomic features,
and often involve alteration in consciousness. Seizures may march to involve
clonic movement of the head and upper extremities or proceed to secondary
convulsion. A minority of patients may develop seizures during the day. The
syndrome is benign, with remission before age 12.
The EEG reveals normal background activity, but frequent surface-negative
spike and slow-wave complexes over the occipital region, occurring singu-
larly or repetitively with a frequency of 2-4 Hz. Discharges may be apparent
bilaterally with a persistent voltage asymmetry or a shifting predominance, or
they may be unilateral. The spike-wave complexes attenuate with eye-opening
and fixation, and are induced in darkness or with eye closure. Generalised
discharges may also be present, and other focal discharges can also occur. The
occipital spike wave remains present despite treatment with anti-epileptic
medications, but tends to decrease with age. PS has indicated that diagnosis
is important for several reasons, including the fact that prophylactic treatment
may not be necessary.
• Benign epilepsy with centrotemporal spikes. Benign (childhood)
Epilepsy with Centrotemporal Spikes (BECTS or BCECTS), also known
as benign Rolandic epilepsy. Age of onset is between 3 and 13 years,

276
Case 47

with a peak incidence between 7 and 9 years. However, the hallmark


Electroencephalographic (EEG) finding is of centrotemporal spikes. The
most common seizure type is a simple partial seizure with motor symp-
toms involving the face. The characteristic ictal symptoms correspond to
the origin of seizures in the Rolandic or perisylvian sensorimotor cortex,
which represents the face and oropharynx, and includes facial numbness
or twitching, guttural vocalisations, hypersalivation, drooling, dysphasia,
and speech arrest. Motor activity in the upper but not the lower extremi-
ties is also common. Three-quarters of seizures occur at night or on awak-
ening. Although these focal seizures are the most frequent seizure type,
they can go unnoticed, especially because they occur mainly in sleep. As
a result, a common presentation of BECTS is a secondary Generalised
Tonic Clonic (GTC) seizure during sleep. Overall, about half of children
with BECTS have at least one secondary generalised seizure. Partial or
generalised status epilepticus is unusual. Electroencephalogram (EEG)
findings of BECTS are characteristic, with centrotemporal sharp waves
(70 to 100 msec) that have several distinctive features:
• The morphology is biphasic, with a negative sharp peak followed by a
positive rounded component (amplitude 50 percent of the negativity).
• The sharp waves often have a “horizontal dipole,” which typically reveals
a maximum negativity in the centrotemporal region and a maximum posi-
tivity at the vertex or in the frontal region.
• The sharp waves often occur in repetitive bursts, and can be bilateral and
independent.
• Epileptiform activity is markedly activated by non-REM sleep, and oc-
casionally occurs only in sleep.
• The background EEG activity is normal. Although this distinctive EEG
pattern is highly associated with BECTS.
• Benign occipital epilepsy of childhood. Benign occipital epilepsy
of childhood, sometimes referred to as Gastaut-type, has a mean age of
presentation of 8 to 9 years (range 3 to 16 years), which is somewhat
later than in Panayiotopoulos syndrome. Seizures often include visual
symptoms, either blindness or visual hallucinations. Hemi-clonic activity,
automatisms, migraine-like headache, and versive movements can occur,
and seizures may secondarily generalise. Compared with Panayiotopoulos
syndrome, seizures are more frequent and of shorter duration; usually less
than five minutes. Most seizures occur in the daytime. The interictal EEG
is similar to that in Panayiotopoulos syndrome, but with some differences:
epileptiform activity is more predominantly occipital, often occurs in long
bursts of spike-wave complexes, and is markedly activated by eye closure.
Background activity is normal. Because clinical and EEG features of
symptomatic and benign idiopathic occipital epilepsy overlap substantially,
magnetic resonance imaging is generally recommended. Migraine with
aura is also frequently considered in the differential diagnosis. The course
of benign occipital epilepsy of childhood is less consistently benign than

277
Child Neurology

the previously described syndromes. Anti-epileptic drug treatment is


more often prescribed since the duration of epilepsy is longer and seizures
can be frequent. Any medication used in partial epilepsies is likely to be
efficacious.

DIAGNOSTIC WORK-UP
• The cornerstone of the evaluation is a sleep-deprived EEG. The most
common finding is occipital epileptiform discharges (synchronous or
unilateral), which are seen in three-quarters of the patients. However, it
should be noted that 10-25% exhibit only extra-occipital epileptiform dis-
charges (typically frontal or temporal). The location of these discharges
can vary even in the same patient. Less commonly, the EEG can be nor-
mal. Sleep activates the discharges significantly and may be the only time
epileptiform discharges are seen. Ictal discharges originate from occipital,
frontal, and temporal regions, but seizure manifestations can be subtle (in
one case, the only ictal change was tachycardia for 10 minutes before eye
deviation and convulsive activity occurred). Autonomic seizures can be
due to symptomatic, cryptogenic, and idiopathic etiologies
• The unusual symptomatology of PS requires a high level of suspicion for
epilepsy. Because of the prominent dysautonomic features, these seizures
can be mistaken for non-epileptic syndromes including syncope, gastro-
enteritis, encephalitis, and migraine
• PS can usually be distinguished from symptomatic Temporal Lobe
Epilepsy (TLE). In TLE, autonomic symptoms are usually much less pro-
nounced and occur near the end rather than at the beginning of the seizure
• PS itself is a diagnosis that is most firmly established in hindsight. Therefore,
symptomatic etiologies would be prudent
neuroimaging to evaluate for symptomatic

MANAGEMENT
• Panayiotopoulos syndrome describes a benign partial epilepsy associated
with interictal occipital spikes that begin in early childhood with rare sei-
zures that have prominent autonomic features (e.g., vomiting, pallor) and
are often prolonged. Half of patients have only a single seizure, and pro-
phylactic treatment may not be required.
• Prophylactic treatment is reserved for patients with frequent seizures or
those with severe symptoms. Although any of the anti-epileptic drugs
used for partial epilepsy may be used, there is the most experience with
carbamazepine, which is successful as monotherapy in most patients.
Withdrawal of treatment is recommended within one to two years if sei-
zures are not ongoing. Other anti-epileptic medications like oxcarbaze-
pine (Trileptal), valproic acid, topiramate, levetiracetam (Keppra), gabap-
entin (Neurontin), zonisamide (Zonegran), lacosamide (Vimpat), can be
tried if seizures are resistant to carbamazipine. In one large series, almost
90% were seizure free after treatment, but approximately 5% continued to
have seizures despite treatment.

278
Case 47

• Rectal diazepam and instruction in its use should be provided to parents


of children who have had a prolonged seizure.

REFERENCES
1. Panayiotopoulos CP. Benign childhood focal seizures and related epileptic syn-
dromes. In: A Clinical Guide to Epileptic Syndromes and Their Treatment, 2nd ed,
Panayiotopoulos CP (Ed), Springer, London 2007. p.285.
2. Kramer U, Nevo Y, Neufeld MY, et al.. Epidemiology of epilepsy in childhood: a
cohort of 440 consecutive patients. Pediatr Neurol 1998; 18:46.
3. Cavazzuti GB. Epidemiology of different types of epilepsy in school age children
of Modena, Italy. Epilepsia 1980; 21:57.
4. Tovia E, Goldberg-Stern H, Ben Zeev B, et al.. The prevalence of atypical presenta-
tions and comorbidities of benign childhood epilepsy with centrotemporal spikes.
Epilepsia 2011; 52:1483.
5. Camfield P, Camfield C. Epileptic syndromes in childhood: clinical features, out-
comes, and treatment. Epilepsia 2002; 43 Suppl 3:27.
6. Fejerman N, Caraballo R, Tenembaum SN. Atypical evolutions of benign locali-
zation-related epilepsies in children: are they predictable? Epilepsia 2000; 41:380.
Coppola G, Franzoni E, Verrotti A, et al.. Levetiracetam or oxcarbazepine as mon-
otherapy in newly diagnosed benign epilepsy of childhood with centrotemporal
spikes (BECTS): an open-label, parallel group trial. Brain Dev 2007; 29:281.
7. Taylor I, Berkovic SF, Kivity S, Scheffer IE. Benign occipital epilepsies of child-
hood: clinical features and genetics. Brain 2008; 131:2287.
8. Covanis A. Panayiotopoulos syndrome: a benign childhood autonomic epilepsy
frequently imitating encephalitis, syncope, migraine, sleep disorder, or gastroen-
teritis. Pediatrics 2006; 118:e1237.
9. Lada C, Skiadas K, Theodorou V, et al.. A study of 43 patients with panayiotopoulos
syndrome, a common and benign childhood seizure susceptibility. Epilepsia 2003;
44:81.
10. Koutroumanidis M. Panayiotopoulos syndrome: an important electroclinical ex-
ample of benign childhood system epilepsy. Epilepsia 2007; 48:1044.
11. Caraballo R, Cersósimo R, Fejerman N. Panayiotopoulos syndrome: a prospective
study of 192 patients. Epilepsia 2007; 48:1054.
12. Panayiotopoulos CP. Benign childhood epilepsy with occipital paroxysms: a 15-
year prospective study. Ann Neurol 1989; 26:51.
13. Specchio N, Trivisano M, Di Ciommo V, et al. Panayiotopoulos syndrome: a clini-
cal, EEG, and neuropsychological study of 93 consecutive patients. Epilepsia 2010;
51:2098.
14. Kivity S, Ephraim T, Weitz R, Tamir A. Childhood epilepsy with occipital parox-
ysms: clinical variants in 134 patients. Epilepsia 2000; 41:1522.
15. Verrotti A, Parisi P, Loiacono G, et al. Levetiracetam monotherapy for childhood
occipital epilepsy of gastaut. Acta Neurol Scand 2009; 120:342.

279
48 Case
A 6-month-old boy, first child of healthy parents was born after an
uneventful pregnancy. He was well and his developmental milestones
were normal until the age of 4 months, when he started to show a bad temper
and did not smile. The spells were described as brief episodes of his head
falling backward with extensions of the extremities. They used to occur in
clusters on awakening, last for several minutes and occur several times a day.
Once he was admitted to hospital, his series of tonic extensions of extremities
increased. The spells were occurring with increasing frequency with the
passage of time and sometimes he used to cry afterwards. The parents had
also noted that during this time the infant had stopped smiling, decreased his
babbling.
On physical examination, there were no dysmorphic facial features or
birthmarks. He did not smile spontaneously or responsively. No babbling or
cooing was noted. All accessible cranial nerves were intact. He had normal
bulk and tone. He moved all four extremities equally against gravity. He
had little interest in grabbing or playing with toys but there is no dysmetria.
Deep tendon reflexes were normal throughout with bilateral plantar flexor
responses.
On investigation, complete blood count, liver function test, renal function
test, serum electrolytes were normal. EEG showed bilaterally asynchronous
high voltage polyspikes, and slow wave discharges interspersed with multifocal
spikes and slow waves. MRI scan revealed a normal brain.

Answer the following questions:


• Give the most likely diagnosis.
• Briefly summarise the case.
• Provide a differential diagnosis.
• Discuss an appropriate diagnostic work-up.
• Discuss the management of this patient.

280
Case 48

DIAGNOSIS
Infantile spasms (West syndrome)

SUMMARY
A 6-month-old child infant who presented with clusters of extensor spasms
multiple times per day and developmental regression, the description of which
is consistent with a diagnosis of infantile spasms. Based on clinical history, se-
miology of the seizures and EEG recording, the child was diagnosed as having
West syndrome.

DIFFERENTIAL DIAGNOSIS
Infantile spasms constitute an age-dependent epilepsy syndrome that presents
during infancy and is typically identified by clusters of spasms and an inter-
ictal EEG pattern known as hypsarrhythmia. As William West first described
in his own son, in a letter to the Lancet in 1841, the onset of spasms is often
accompanied by psychomotor deterioration, thus meeting the criteria of an
epileptic encephalopathy. West’s syndrome refers to the classic triad of spasms,
hypsarrhythmia, and psychomotor arrest or regression. With a worldwide in-
cidence of approximately 1 per 3000 live births, infantile spasms are the most
prevalent epilepsy syndrome of infancy, and its emotional and financial costs
to society are enormous. Despite over 150 years of interest in this disorder,
various aspects, from terminology to treatment, remain controversial, in large
part because the pathogenesis is unknown. This section focuses on the clini-
cal presentation and prognosis, diagnostic evaluation, associated etiological
factors, proposed pathophysiological mechanisms, and current treatment op-
tions. Electroclinical features, regardless of its etiology, this syndrome mani-
fests during a specific period of brain maturation; most often between 4 and 8
months of life and nearly always before 2 years. The unique clinical spasms in
the setting of hypsarrhythmia or its variants, lack of response to conventional
anti-epileptic agents, and generally poor outcome distinguish infantile spasms
from similar epilepsies of infancy.
The classic seizure type, referred to as an epileptic spasm, is characterised by
symmetric, bilateral, brief contraction of the axial muscle groups. The EMG
tracing during a spasm reveals an abrupt phasic contraction lasting less than
2 seconds, which may be followed by a less intense tonic contraction lasting
from 2 to 10 seconds. Therefore, this unique seizure type is longer than a
myoclonic jerk and yet shorter than a tonic seizure.
The differential diagnosis of infantile spasms includes nonepileptic events
such as colic, gastrooesophageal reflux, Sandifer syndrome, hyperekplexia, and
behavioural events such as infantile masturbation (self gratification syndrome)
or breath-holding spells. These diagnoses can usually be excluded by taking a
thorough history. Benign myoclonus of infancy can also be considered. Infants
with this syndrome are developmentally normal and have episodes of myo-
clonus with a normal ictal and interictal EEG.
• Infantile spasms must also be differentiated from other seizure disorders
that occur in infancy, including benign myoclonic epilepsy in infancy and
severe myoclonic epilepsy in infancy (Dravet syndrome).

281
Child Neurology

Benign myoclonic epilepsy in infancy is a distinct form of idiopathic gen-


eralised epilepsy that is characterised by myoclonic seizures in infants 6
months to 2 years of age. This epilepsy may be genetically based. The in-
terictal EEG is usually normal but myoclonic jerks may be associated with
generalised polyspike and slow-wave discharges with normal background
for age.
• Severe myoclonic epilepsy in infancy occurs in the first year of life in pre-
viously normal children. It is a progressive epileptic encephalopathy with
several stages of evolution. It has distinct EEG features.
• Early infantile clonic febrile seizures, myoclonic seizures, atypical ab-
sence, and complex focal seizures occur. The EEG may be normal initially,
but then deteriorates to show background slowing as well as generalised
polyspike and slow-wave discharges.
• Thus, the clinical symptomatology, as well as the EEG, can help to dif-
ferentiate these syndromes from infantile
antile spasms.

CAUSES OF INFANTILE SPASM


Cerebral Malformations
• Lissencephaly
• Polymicrogyria
• Schizencephaly
• Focal cortical dysplasia

Genetic Syndromes
• Aicardi syndrome
• Down syndrome
• Williams syndrome

Neurocutaneous Diseases
• Tuberous sclerosis complex
• Neurofibromatosis type 1
• Incontinentia pigmenti
• Sturge-Weber syndrome

Hypoxic-Ischaemic Encephalopathy
Infection
• Prenatal meningitis/encephalitis
• Post-natal meningitis/encephalitis

Inborn Errors of Metabolism


• Pyridoxine-dependent seizures
• Maple syrup urine disease

282
Case 48

• Phenylketonuria
• Biotinidase deficiency
• Mitochondrial disorders
• Nonketotic hyperglycinaemia

DIAGNOSTIC WORK-UP
• A thorough physical examination should initially be performed to evaluate
for evidence of obvious genetic syndromes or metabolic disease. Special
attention should be paid to the skin examination and may require a Wood
lamp examination of fair skin to rule out tuberous sclerosis complex.
• The evaluation of an infant with infantile spasms begins with an EEG.
Early in the course, the EEG may be normal but usually evolves to show
hypsarrhythmia. Hypsarrhythmia describes an interictal pattern of cha-
otic, poorly organised, high-amplitude, intermixed slow waves and multi-
focal epileptiform activity.
• The work-up of a child with infantile spasms also includes neuroimaging
with an MRI to rule out structural abnormalities. Magnetic Resonance
(MR) spectroscopy may be helpful to evaluate for metabolic disease.
• A basic metabolic work-up including plasma amino acids, urine organic
acids, lactate/pyruvate, and ammonia level should be done if a metabolic
disorder is suspected or if the cause is not determined through history,
physical examination, or neuroimaging.
• Any dysmorphic child should have a high-resolution karyotype performed
and should be referred for evaluation by a geneticist. Additional genetic or
metabolic testing also may be necessary, depending on the individual case.
An ophthalmology evaluation may help rule out metabolic, genetic, or
infectious disease.

MANAGEMENT
• The best treatment for infantile spasms is still controversial. The goal of
treatment is to prevent or ameliorate the encephalopathy by stopping the
spasms and improving the EEG background. Despite numerous published
reports on the treatment of infantile spasms over the past half-century, in-
terpretation has been limited due to methodological shortcomings.
• Therefore, at present, there is no consensus regarding optimal treatment,
and there is no conclusive evidence that current medical therapies alter
neurocognitive or epilepsy outcomes. The most commonly used medi-
cal therapies in the United States for treatment of infantile spasms are
ACTH or oral corticosteroids, vigabatrin in the UK, and pyridoxine in
Japan. With the recent Food and Drug Administration (FDA) approval of
vigabatrin in 2009, its use may increase, particularly for infants with tuber-
ous sclerosis or pre-existing cortical blindness.
• Conventional anti-epileptic agents, as well as the ketogenic diet, have been
used with incomplete success when first-line agents fail. Finally, fueled by

283
Child Neurology

advances in neuroimaging, surgical resection of focal lesions has emerged as


a promising option for patients with medically intractable infantile spasms.
• The approach may be different for symptomatic and idiopathic cases.
Adrenocorticotropic Hormone (ACTH) and vigabatrin are the most ef-
fective drugs. A wide range of ACTH doses and treatment regimens has
been used. ACTH at doses of 50 to 150 International Units (IU) per me-
ter squared per day (20 to 80 IU) is typically used for no longer than 6 to
8 weeks. Although not FDA approved in the United States, vigabatrin has
been particularly helpful for children with infantile spasms and tuberous
sclerosis complex.
• Alternative therapies include oral steroids, topiramate, valproic acid, zon-
isamide, and the benzodiazepines (nitrazepam, clonazepam).
• A challenge with 100 mg of intravenous pyridoxine should be consid-
ered in infants without an obvious etiology for their seizures to rule out
pyridoxine-dependent seizures. Epilepsy surgery may also be beneficial
for patients with resectable brain lesions.
• Children with infantile spasms are at risk for psychomotor retardation
either before or after the onset of spasms and affected patients should be
evaluated by a developmental specialist during the course of their disease.
They should be referred for early interventional services, including physi-
cal, occupational, and speech therapy when appropriate.

REFERENCES
1. Hrachovy RA, Frost JD; Severe encephalopathic epilepsy in infants: infantile
spasms. Pediatr Epileps. 2008; 1: 249 -268. 2. West WJ; peculiar form of infantile
convulsions. Lancet. 1841; 1:724-725. 3.
2. Riikonen. Long-Term Outcome of West Syndrome: A Study of Adults with a History
of Infantile Spasms. Epilepsia, 1996; 37(4) : 367-72 . 9.
3. Sharma NL , Vishwanthan V; Outcome in West Syndrome. Indian Pediatr. 2008;
45(7): 559-63. 10.
4. Kaushik JS , Patra B , Sharma S , Yadav D , Aneja S; Clinical spectrum and treat-
ment outcome of West Syndrome in children from Northern India. Seizur, 2013;
22(8): 617-21.
5. West, WJ. On a peculiar form of infantile convulsions. Lancet 1841; 1:724.
6. Pellock JM, Hrachovy R, Shinnar S, et al. Infantile spasms: a U.S. consensus report.
Epilepsia 2010; 51:2175.
7. Ramgopal S, Shah A, Zarowski M, et al. Diurnal and sleep/wake patterns of epilep-
tic spasms in different age groups. Epilepsia 2012; 53:1170.
8. Wilmshurst JM, Gaillard WD, Vinayan KP, et al. Summary of recommendations for
the management of infantile seizures: Task Force Report for the ILAE Commission
of Pediatrics. Epilepsia 2015; 56:1185.
9. Philippi H, Wohlrab G, Bettendorf U, et al. Electroencephalographic evolution of
hypsarrhythmia: toward an early treatment option. Epilepsia 2008; 49:1859.
10. Dimassi S, Labalme A, Ville D, et al. Whole-exome sequencing improves the diag-
nosis yield in sporadic infantile spasm syndrome. Clin Genet 2016; 89:198.

284
Case 49
A 4-day-old baby girl presented with lethargy, seizures, and hypotonia to
her doctor who immediately referred her for hospital admission. The
patient was born through a cesarean, with Premature Rupture of Membranes
(PROM) of more than 18 hours, gestational age of 38 weeks, 3200 g weight,
50 cm height, and head circumference of 35 cm. The mother reports a
normal, healthy pregnancy with very active foetal movements. The infant has
been mostly sleeping since birth and has been feeding poorly because of her
sleepiness. During sleep, the parents also have seen frequent myoclonic jerks.
There is no history of consanguinity.
On physical examination. Her vital signs were stable. Her anterior fonta-
nelle was flat. There were no dysmorphic facial features and no hepatosple-
nomegaly. She was sleepy and unarousable. She moved her body but did not
open her eyes to external stimuli. Her pupils were equal, round, and reactive
to light. Her facial movements were symmetric. Oculocephalic, corneal, and
gag reflexes were present. She had normal bulk with marked truncal and ap-
pendicular hypotonia. The infant was able to move her extremities symmetri-
cally against gravity with deep painful stimuli. Reflexes were 2+ throughout
with bilateral plantar extensor responses.
Laboratory examination showed that blood cell count, electrolyte, glucose,
ammonia, and lactate were all within the normal range. Cranial ultrasonogra-
phy showed no abnormal findings. Urine organic acid examination and blood
liquid chromatography-mass spectrometry were unremarkable. The infant
was transferred to the intensive care unit, she frequently showed paroxysmal
general tonic postures, associated with oxygen desaturations and increased
heart rate. Seizure was aggravated by touching. Sleep EEG revealed paroxys-
mal sharp waves over both central areas. Video on long-term EEG monitor
proved seizure associated with epileptic form discharges, particularly in the
frontal region. Ictal recording showed general tonic seizure with tachycar-
dia, associated with epileptic form spikes. Brain Magnetic Resonance Imaging
(MRI) showed no remarkable findings.

Answer the following questions:


• Give the most likely diagnosis.
• Briefly summarise this case.
• Provide a differential diagnosis.
• Discuss an appropriate diagnostic work-up.
• Discuss the management of the patient.

285
Child Neurology

DIAGNOSIS
Nonketotic Hyperglycinaemia (NKH)

SUMMARY
A new born was admitted with lethargy, seizures, and hypotonia. The baby has
been mostly sleeping since birth, associated with poor feeding. During sleep,
frequent myoclonic jerks were observed by the parents. Cerebrospinal fluid
examination showed amino acid analyses (MRM mode) of glycine as CSF 51.9
µM; plasma 183.0 µM; and CSF/plasma ratio 0.28 (the normal ratio is <0.02).
Clinical and laboratory results were consistent with NKH.

DIFFERENTIAL DIAGNOSIS
Non-ketotic hyperglycaemia is a lethal hereditary disorder with autosomal re-
cessive inheritance. Patients with the disorder shortly after birth present symp-
toms such as encephalopathy, reduction of consciousness, loss of reflexes,
finally seizures and seizures are completely resistant to therapy. The effects oc-
cur due to glycine aggregation in two areas of the brain with glycine receptors
in the brain stem and spinal cord. The brain stem and spinal cord have inhibi-
tory effects and therefore cause glycine aggregation and hypotonia. However,
the receptors of the cerebrum and cerebellum region have stimulating effects,
cause seizures, myoclonus and other neurological diseases and encephalopa-
thy. In order to diagnose this disorder, primarily glycine levels in the plasma
and urine are determined and in case of high levels in the absence of ketone-
mia, the CSF glycine and the ratio of CSF glycine to plasma are measured. In
the case of aggregation of large amounts of glycine in CSF compared to plasma
(CSF glycine/plasma ratio more than 0.08) the diagnosis is confirmed. Gold
standard for diagnosis of NKH is to measure converting glycine enzyme (gly-
cine cleavage enzyme) seen in the liver biopsy.
• The patient’s young age at presentation in addition to the symptoms of
lethargy, hypotonia, and seizures are highly suggestive of an inborn er-
ror of metabolism. However, acquired disorders are much more common
than metabolic diseases and should be ruled out first. These include infec-
tion (sepsis, meningitis, encephalitis), hypoxic-ischaemic encephalopathy
with seizures, intracranial haemorrhage, neonatal stroke, head trauma,
and medication toxicity.
• Once acquired etiologies are excluded, the broad differential diagnosis of
inborn errors of metabolism causing neonatal encephalopathy, hypotonia,
and seizures must be considered. Typically, these symptoms are produced
by the accumulation of small-molecule metabolites, insufficient amounts
of a necessary enzymatic pathway product, or abnormal transport of
certain molecules. The pregnancy is often unremarkable and affected
infants are typically normal at birth. Lethargy, seizures, vomiting, poor
feeding, and irritability begin hours to days after birth. In general terms,

286
Case 49

aminoacidopathies, organic acidurias, urea cycle disorders, peroxisomal


disorders, fatty acid oxidation defects, and mitochondrial disorders should
be considered.
• When seizures are also a prominent feature, NKH, pyridoxine-dependent
seizures, folinic acid-responsive seizures, sulfite oxidase deficiency, mo-
lybdenum cofactor deficiency, and Glucose Transporter 1 (GLUT1) de-
fects enter the differential diagnosis. Seizures may even begin in utero
in these conditions, and are suggested in the case presented above by the
history of very active foetal movements.
• A thorough physical examination, family history, screening metabolic
work-up, specialised metabolic/genetic testing, and neuroimaging with
brain MRI and MR spectroscopy may be required to narrow the differ-
ential diagnosis.
• The presence of abnormal smells that accompany many inborn errors of
metabolism might also be significant. Laboratory findings suggestive of an
underlying metabolic disease include persistent metabolic acidosis with
an anion gap (especially higher than 20 mEq/L), elevated lactate, hyper-
ammonemia, and hypoglycaemia. Persistent metabolic acidosis may be
caused by organic acidurias, mitochondrial respiratory chain abnormali-
ties, or disorders of pyruvate metabolism.
• Organic acidurias such as methylmalonic aciduria, propionic aciduria, and
isovaleric aciduria can cause an elevated serum glycine level but can be
differentiated from NKH by the presence of ketosis and by measurement
of urine organic acids (which are both normal in NKH). Most amino
acid disorders do not cause an acute neonatal encephalopathy. The neo-
natal presentation of maple syrup urine disease, however, is an exception.
In these cases, the urine may smell sweet because of the presence of ke-
toacids. A variety of mitochondrial respiratory chain disorders may pre-
sent with profound encephalopathy in addition to serum and CSF lactic
acidosis. Disorders of pyruvate metabolism (pyruvate dehydrogenase and
pyruvate carboxylase deficiency) must also be considered with this clinical
and laboratory picture.
• Hyperammonemia as seen with urea cycle defects can also result in en-
cephalopathy and seizures secondary to cerebral oedema and the neurotox-
ic effects of ammonia. The most common urea cycle disorder, Ornithine
Transcarbamylase (OTC) deficiency (X-linked), presents shortly after
birth and is usually fatal in child. Heterozygous females present later with
intermittent episodes of lethargy, vomiting, and ataxia. Seizures are less
prominent. A secondary hyperammonemia may also be seen in organic
acidurias and fatty acid oxidation defects.
• Hypoglycaemia is commonly seen in sepsis, prematurity, and mater-
nal diabetes mellitus but when presenting with metabolic acidosis, it is
suggestive of an organic aciduria or disorder of gluconeogenesis (glyco-
gen storage disease type 1). Fatty acid oxidation defects like short-chain

287
Child Neurology

acyl-coenzyme A (acyl-CoA) dehydrogenase deficiency can also pre-


sent with hypoglycaemia in infancy. Disorders of peroxisome biogenesis
(Zellweger syndrome, neonatal adrenoleukodystrophy, infantile Refsum
syndrome) can be considered but are a less likely cause of neonatal en-
cephalopathy. Laboratory findings include abnormal very-long-chain fatty
acids. Urine organic acids, as well as the acylcarnitine profile, are expected
to be normal in these cases.
• Several inborn errors of metabolism may present with acute encephalopa-
thy in addition to intractable seizures and deserve special mention. The
diagnosis of pyridoxine-dependent seizures and folinic acid-responsive
seizures can be excluded as they respond to intravenous pyridoxine or
folinic acid, respectively. Sulfite oxidase and molybdenum cofactor defi-
ciency are autosomal recessive disorders that demonstrate poor feeding,
intractable seizures, axial hypotonia, and extremity hypertonicity in the
first few days to weeks of life. Routine metabolic tests do not detect these
disorders and special testing for elevated urine S-sulfocysteine and thio-
sulfate must be performed. GLUT1-deficiency syndrome may present
with a similar clinical picture. A low CSF glucose, low CSF lactate, and
normal serum glucose are characteristic of this disease.
• Finally, other disorders should be considered when a patient is found to
have elevated glycine levels. Valproic acid and D-glyceric acidaemia can
also present with secondary hyperglycinaemia as a result of a decrease in
liver GCS activity.

Metabolic disorders with prominent seizure activity


• Nonketotic hyperglycinaemia
• Pyridoxine-dependent seizures
• Folinic acid-responsive seizures
• Sulfite oxidase deficiency
• Molybdenum cofactor deficiency
• Glucose transporter deficiency disorders

DIAGNOSTIC WORK-UP
• The patient’s evaluation should begin with a thorough history of the
mother’s pregnancy, the child’s birth, feeding history, and family history.
It is important to know if foetal movements were normal, whether the
mother had any infections during the pregnancy, whether the mother
used any drugs during the pregnancy, and whether there was vaginal
bleeding or any additional complications. Increased foetal movements, as
in this case, may be indicative of in utero seizure activity. It is relevant
to know whether the child had any difficulties with feeding or breathing
after birth.

288
Case 49

• Many metabolic diseases present after the introduction of protein or glu-


cose feeds, so a detailed history of the onset of symptoms and the infant’s
diet must be obtained. Noting any family history of consanguinity and/or
unexplained infant death is also very important. Abnormal smells might
be relevant.
• At the time of presentation, the patient should have a screening work-
up performed to evaluate for the most common acquired disorders. This
screening work-up should include basic chemistries (to rule out acido-
sis and anion gap), blood gas, CBC, glucose, liver function tests, creatine
kinase, urinalysis, blood culture, and urine culture. A noncontrast head
CT also needs to be done, followed by a lumbar puncture for cell count,
glucose, protein, bacterial culture, and HSV PCR.
• Given the presence of seizures, an EEG is warranted early in evaluation.
Finally, testing for inborn errors of metabolism in an infant with encepha-
lopathy should be initiated by obtaining plasma amino acids, urine organic
acids, a lactate/pyruvate battery, and an ammonia level. Additional tests
can include urine analysis for ketones and reducing substances in addition
to serum carnitine, acylcarnitine profile, and very-long-chain fatty acids.
• CSF lactate and glycine can also be measured as part of the initial work-up
when there are strong suspicions of an inborn error of metabolism.
• A brain MRI and MR spectroscopy are further recommended and may
help point to a type of metabolic disease.
• The diagnosis of pyridoxine-dependent seizures is evaluated by giving
100 mg of pyridoxine intravenously during an EEG. Epileptiform activ-
ity should resolve while, or shortly, after the dose is given. NKH should
be suspected in individuals with elevated urine, serum, and CSF glycine.
However, the urine and serum glycine levels may often not be elevated
and therefore, simultaneous CSF and plasma glycine levels are needed.
• An abnormal CSF-to-plasma glycine ratio of >0.08 is consistent with
NKH (the normal ratio is <0.02). Once NKH is strongly suspected, con-
firmatory biochemical, quantitative amino acid analysis, and molecular
genetic testing should be done. Biochemical enzyme assays of GCS ac-
tivity obtained on liver biopsy are available on a research basis only. The
specific protein deficiency (P, T, or H) also can be identified on liver tissue
but, again, is not clinically available.
• Molecular genetic testing by sequence analysis of the GLDC, GCSH, and
ATM genes is clinically available and confirms the diagnosis in a majority
of patients.

MANAGEMENT
• No effective treatment exists for severe glycine encephalopathy; prelimi-
nary evidence suggests that children with pathogenic variants associated
with residual GCS enzyme activity treated aggressively in the first two

289
Child Neurology

years of life with sodium benzoate to reduce plasma concentration of gly-


cine and N-methyl D-Aspartate an (NMDA) receptor site antagonists
have improved outcome compared to late-treated or untreated controls.
Anti-epileptic drugs and/or ketogenic diet for seizure control; gastrostomy
tube for feeding problems; therapy for gastrooesophageal reflux; physical
therapy.
• In general, infants with suspected inborn errors of metabolism should be
taken off of enteral feeds; instead, intravenous fluids of 10% dextrose with
appropriate electrolytes should be provided. If there are concerns for a
fatty acid oxidation defect, intravenous intralipids should not be given.
Other complications, such as hypoglycaemia and metabolic acidosis,
should be aggressively medically managed.
• ment strategies may then be tailored as the work-up points toward
Management
a specific diagnosis.
• Currently, there is no effective treatment available for NKH. Sodium ben-
zoate has been used to normalise serum glycine but does not normalise
CSF glycine levels; high doses, however, can lower CSF glycine levels. It
might be useful for episodes of lethargy and seizures in the milder forms
of NKH. Sodium benzoate has not been shown to change outcome in the
classic severe neonatal form.
• Traditional anti-convulsants such as phenobarbital and phenytoin may not
be effective at seizure control in these patients. Benzodiazepines and so-
dium benzoate appear to be more effective anti-epileptic agents. Valproic
acid is contraindicated as it can cause secondary hyperglycinaemia by de-
creasing liver GCS activity. Antagonism of the NMDA glutamate recep-
tor, which is stimulated by glycine, also has been attempted without suc-
cess with dextromethorphan, ketamine, felbamate, and lamotrigine.
• Supportive care for the patient and counseling for the family plays a large
role in caring for these patients. Genetic counseling is recommended for
parents and family members. As both parents are carriers, each new child
has a 25% chance of being affected, a 50% chance of being a carrier, and
a 25% chance of being neither affected nor a carrier. Prenatal molecular
genetic testing of extracted foetal DNA or chorionic villus sampling is
available and can be offered to the parents.

REFERENCES
1. Shah DK, Tingay DG, Fink AM, et al. Magnetic resonance imaging in neonatal
nonketotic hyperglycinemia. Pediatr Neurol. 2005;33:50-52.
2. Ellaway CJ, Wilcken B, Christodoulou J. Clinical approach to inborn errors of me-
tabolism presenting in the newborn period. J Paediatr Child Health. 2002;38:511-
517.
3. Verissimo C, Garcia P, Simoes M, Robalo C, Henriques R, Diogo L, et al. Non-
ketotic hyperglycinemia: a cause of encephalopathy in children. J Child Neurol.
2013;28(2):251-4.

290
Case 49

4. Madu AE, Oliver L. Non-ketotic hyperglycinaemia: case report and review of


medical literature. J Matern Fetal Neonatal Med. 2013;26(5):537-9.
5. Hoover-Fong JE, Shah S, Van Hove JL, Applegarth D, Toone J, Hamosh A. Natu-
ral history of nonketotic hyperglycinemia in 65 patients. Neurol. 2004;63:847-53.
6. Beijer P, Lichtenbelt KD, Hofstede FC, Nikkels PG, Lemmers P, de Vries LS. A
known and a novel mutation in the glycine decarboxylase gene in a newborn with
classic nonketotic hyperglycinemia. Neuropediatrics. 2012;43(3):164-7.
7. Sel ÇG, Kiliç M, Ceylaner S, Özkan M, Aksoy A, Yüksel D, et al.. P85-2818: Non-
ketotic hyperglycinemia: A cause of severe epileptic encephalopathy and hypotonia
in children. Europ J Paediat Neurol. 2015;19:S118.
8. Hui HNT, Ho YC, Chan HB, Tam SCF, Tang NLS, Ip PLS. Hui HNT.. A Baby
with Nonketotic Hyperglycinaemia. 2004;9:243-7.
9. Roy D, Al-Asmari A, Ghazal YK, Al-Oqiel S. Nonketotic hyperglycinemia
in Suleimaniah Children’s Hospital, Riyadh, Saudi Arabia. Ann Saudi Med.
2004;24(5):378-81.
10. Dursun A C¸, Aliefendiog lu D, Aslan AT, Cos¸kun T, Dursun A C, akmak FN,
Kesimer M. Transient nonketotic hyperglycinemia: two case reports and literature
review. . Pediatr Neurol. 2003;28:151-5.
11. Neuberger JM, Schweitzer S, Rolland MO, Burghard R. Effect of sodium benzo-
ate in the treatment of atypical nonketotic hyperglycinaemia. J Inher Metab Dis.
2000;3:22-6.

291
50 Case
A 4-month-old child infant, the first child of non-consanguineous parents
was brought to the emergency room by his parents with complaints of a
hoarse voice and worsening respiratory distress. At birth he was noted to have
a heart murmur and is being seen by a cardiologist regularly for his enlarged
heart. He has had poor weight gain and has difficulty in feeding. The baby was
born at term with a birth weight of 3,500 gm. Prenatal examinations and assays
were performed in a timely manner and identified to have no abnormalities.
On physical examination, he was afebrile with tachypnoea. He was sick
looking but there were no dysmorphic facial features. His tongue was large
and protrudes from his mouth. On developmental assessment he was delayed
in motor domain mainly. A heart murmur was present and liver edge was
palpated 3 cm below the rib but no splenomegaly. He was alert with equal
pupil size, round, and reactive to light. He was able to track in all directions.
His face was symmetric. A gag was present and the tongue was midline without
fasciculation. He had normal bulk with severe axial hypotonia and little
head control. He was lying with frog-leg positioning with little spontaneous
movement. He was able to move his extremities against gravity but appears
weak. There was some tickle-induced withdrawal. Deep Tendon Reflexes: 1+
throughout with bilateral plantar flexor responses.
Echocardiography showed the presence of cardiac hypertrophy. Serum
muscle enzymes, including aspartate aminotransferase, lactate dehydrogenase
and creatine kinase were found to be elevated. Quantitative blood alpha-
glucosidase level was 20 nmol/hr/mg (normal > 60).

Answer the following questions:


• Give the most likely diagnosis.
• Summarise the case briefly.
• Provide a differential diagnosis.
• Discuss an appropriate diagnostic work-up.
• Discuss the management of the patient.

292
Case 50

DIAGNOSIS
Pompe Disease (Glycogen storage disease type II due to acid maltase deficiency)

SUMMARY
The patient was a 5-month-old infant with a history of developmental delay,
failure to thrive, and cardiomyopathy who presented for evaluation of respira-
tory distress. His examination was remarkable for macroglossia, cardiac mur-
mur, hepatomegaly, severe hypotonia, generalised weakness, and hyporeflexia.

DIFFERENTIAL DIAGNOSIS
Pompe disease or glycogen storage disease type II, is a lysosomal storage disor-
der in which deficiency of alpha-glucosidase leads to accumulation of glycogen
and finally to destruction of muscle tissue. Pompe disease is a rare, progressive,
and often fatal muscular disease. The underlying pathology is a deficiency of
the enzyme Acid Alpha-Glucosidase (GAA) that hydrolyzes lysosomal glyco-
gen. This disease is a single disease which manifests in its clinical spectrum,
that varies with respect to age at onset, rate of progression, and extent of organ
involvement. Complete deficiency of alpha-glucosidase causes a progressive
lethal cardiac and skeletal muscle disorder known as infantile Pompe disease.
Partial deficiency leads to a milder late onset phenotype. The latter condition
may present at any age and is subdivided into non-classical infantile, child-
hood, juvenile, and adult Pompe disease. Infantile Pompe disease is a rapidly
progressive disease, characterised by prominent cardiomegaly, hepatomegaly,
weakness and hypotonia, and death due to cardiorespiratory failure in the first
year. Patients with the infantile variant form (non-classic infantile Pompe dis-
ease) show slower progression and less severe Cardiomyopathy (CMP) but
present in the first year of life. Echocardiogram (ECHO) typically reveals a
hypertrophic CMP with or without Left Ventricular (LV) outflow tract ob-
struction in the early stages of the disease. In the late stages of infantile disease,
patients may have impaired cardiac function and a dilated CMP. Pompe disease
is caused by mutation of the Acid Alpha-Glucosidase gene (GAA) gene. More
than 100 different mutations in the GAA gene have been identified in families
with this disorder. Mutations of the GAA gene in Pompe disease are inherited
as an autosomal recessive trait.
The differential diagnosis for an infant presenting with hypotonia and weak-
ness is broad.
• Pompe disease can sometimes be mistaken for SMA type I (Werdnig-
Hoffmann disease), an anterior horn cell disorder. Although SMA may
result in infantile hypotonia and proximal weakness, it does not cause or-
ganomegaly. SMA may also cause tongue fasciculations, which can be a
distinguishing feature.
• Other myopathies such as congenital muscular dystrophies, congenital
myopathies, congenital myotonic dystrophy, and inflammatory myopa-
thies do not cause organomegaly and would be unlikely to explain this
patient’s constellation of symptoms.

293
Child Neurology

• Other disorders of glycogen metabolism and glycolysis should first be


considered as they most closely resemble Pompe disease. Debrancher en-
zyme deficiency (glycogen storage disease III), as well as brancher enzyme
deficiency (glycogen storage disease IV), can cause hypotonia, muscle
weakness, cardiomyopathy, and hepatomegaly in addition to an elevated
CK in infancy. Phosphorylase b kinase deficiency (glycogen storage dis-
ease VIII) is associated with four clinical phenotypes, which variably cause
exercise intolerance, myalgias, myoglobinuria, weakness, hepatic dysfunc-
tion, and cardiomyopathy. Even though liver and cardiac abnormalities
may arise in infancy, myopathic symptoms typically occur in childhood or
adolescence, in contrast to infantile-onset Pompe disease.
• Mitochondrial cytopathies are a heterogeneous group of disorders with
multisystem organ involvement and can also be considered in the differ-
ential diagnosis of this patient. Because of the non-uniform distribution of
mitochondria in various tissues and the critical role of mitochondria in oxi-
dative metabolism, a wide range of symptoms may be seen. Abnormalities
of the respiratory chain, in particular cytochrome C Oxidase (COX) defi-
ciency can frequently be demonstrated in affected patients. Mitochondrial
cytopathies variably involve muscle, heart, the CNS, the peripheral nerv-
ous system, and endocrine tissues. Muscle disease may present as hypo-
tonia, exercise intolerance, weakness, ptosis, and external ophthalmople-
gia. Hypertrophic cardiomyopathy and arrhythmias may occur. Seizures,
cognitive delay, optic atrophy, pigmentary retinopathy, ataxia, stroke, and
sensorineural hearing loss demonstrate CNS involvement.
° A peripheral axonal neuropathy is reported in many mitochondrial
diseases. Finally, endocrine manifestations such as diabetes mellitus
and short stature may be noted. A mitochondrial cytopathy would
need to be excluded if testing for Pompe disease and other glycogen
storage diseases was negative.
• Certainly, other storage diseases such as the mucopolysaccharidoses, per-
oxisomal disorders, and gangliosidosis could also be considered in the dif-
ferential diagnosis. However, cardiomyopathy and/or hepatomegaly are less
frequently the primary symptoms in these disorders. They also often pre-
sent with dysmorphisms that are not characteristic of Pompe disease.

DIAGNOSTIC WORK-UP
• Basic laboratory tests (CBC, arterial blood gas, liver function tests, basic
chemistry panel) should be done to screen for signs of infection and meta-
bolic abnormalities. A CK level can be sent for screening for neuromus-
cular involvement.
• For an undiagnosed patient, a CK level is useful as a screening test and is
usually moderately elevated (may be as high as 2,000 UI/L) in Pompe dis-
ease. However, it may be normal in adult forms. Liver function tests may
also be elevated because of hepatic involvement.

294
Case 50

• Once Pompe disease is strongly suspected, the diagnosis can be defini-


tively established by demonstrating an absence or decrease in alpha-glu-
cosidase activity in muscle tissue or skin fibroblasts obtained from a skin
biopsy. Lysosomal vacuoles are filled with glycogen.
• The clinical diagnosis is traditionally confirmed by the virtual absence
(infantile-onset) or markedly reduced (late-onset) GAA activity in tis-
sues such as cultured fibroblasts from skin biopsy, muscle biopsy, purified
lymphocytes, mononuclear cells and lymphoid cell lines. Measurement of
GAA activity in skin fibroblasts is the current gold standard. Muscle tis-
sue biopsy, although more invasive, allows muscle acid glucosidase activity
and glycogen content to be assayed directly and rapidly. It can also be used
for histological studies to assess the location and amount of glycogen ac-
cumulation in the tissue. Electron microscopy also demonstrates the pres-
ence of glycogen in lysosomes and in the cytoplasm.
• In late-onset Pompe disease, the site of muscle biopsy can impact results
due to the variability of glycogen accumulation, both between different
muscles, and between the muscle fibre type within a muscle. Muscle bi-
opsy in Pompe disease shows the presence of vacuoles that stain positively
for glycogen. In advanced stages of the disease, glycogen accumulation is
seen, both in the lysosomes, and dispersed in the cytosol. Quantitatively,
muscle glycogen content is elevated up to tenfold above normal in infan-
tile Pompe disease and to a lesser extent in late onset patients.
• New methods have been developed that assay GAA activity in Dried
Blood Spot (DBS) extracts. Isolation of GAA from DBS extracts by im-
munocapture or competitive inhibition of MGA activity using maltose or
acarbose have been used to remove the interfering MGA activity. DBS can
be convenient for rapid results.
• For patients presenting with a storage syndrome and/or dysmorphic fea-
tures in whom the diagnosis is less clear, a screening work-up includes
urine mucopolysaccharides and oligosaccharides (mucopolysaccharidos-
es), urine organic acids, plasma amino acids, lactate/pyruvate (mitochon-
drial disease), phytanic acid (Refsum disease), and plasma very-long-chain
fatty acids (peroxisomal disorders).

MANAGEMENT
• Guideline for the management of Pompe disease was developed as an ed-
ucational resource for health care providers to facilitate the prompt and
accurate diagnosis and treatment of patients. An international group of ex-
perts in various aspects of Pompe disease met to review the evidence based
management from scientific literature and expert opinions. Consensus
was developed in each area of diagnosis, treatment and management in
the development of this guideline. This management guideline specifically
addresses evaluation and diagnosis across multiple organ systems (cardiol-
ogy, pulmonary, gastrointestinal/nutrition, musculoskeletal, neurological)
involved in both infantile and late-onset Pompe disease

295
Child Neurology

• Treatment of Pompe disease is supportive. Patients need to be followed by a


multidisciplinary team of pulmonologists, cardiologists, and neurologists.
• A geneticist should also be involved early on to aid in the diagnosis, assess
carrier status of family members, and to provide counseling, particularly
if the parents wish to have other children. In patients with infantile-onset,
respiratory and cardiac supportive care can prolong life, but “do not resus-
citate” orders eventually need to be discussed with parents.
• Recombinant GAA protein has been developed independently by several
groups, including one using recombinant GAA purified from rabbit milk
and another using protein expression methods with CHO cells. Both prod-
ucts degrade lysosomal glycogen in the heart and muscle. Decreases in heart
size are usually observed 3 months after treatment has been initiated.
• The US Food and Drug Administration (FDA) approved the use of
Myozyme (alglucosidase alfa) for IOPD in 2006. Studies on IOPD have
been completed, and the results show that ERT can maintain muscular
function and improve quality of life, which has led to the approval of
Lumizyme by the FDA in 2010 for use in LOPD. Treatment should be
started early, prior to the destruction of muscle architecture, for successful
outcomes.

REFERENCES
1. Mechtler TP, Stary S, Metz TF, et al.. Neonatal screening for lysosomal storage dis-
orders: feasibility and incidence from a nationwide study in Austria. Lancet 2012;
379:335.
2. Kroos M, Hoogeveen-Westerveld M, van der Ploeg A, Reuser AJ. The genotype-
phenotype correlation in Pompe disease. Am J Med Genet C Semin Med Genet
2012; 160C:59.
3. Zhang H, Kallwass H, Young SP, et al.. Comparison of maltose and acarbose as in-
hibitors of maltase-glucoamylase activity in assaying acid alpha-glucosidase activity
in dried blood spots for the diagnosis of infantile Pompe disease. Genet Med 2006;
8:302.
4. Howell RR, Byrne B, Darras BT, et al
al.. Diagnostic challenges for Pompe disease: an
under-recognized cause of floppy baby syndrome. Genet Med 2006; 8:289.
5. Kishnani PS, Hwu WL, Mandel H, et al. A retrospective, multinational, multicen-
tre study on the natural history of infantile-onset Pompe disease. J Pediatr 2006;
148:671.
6. Bernstein DL, Bialer MG, Mehta L, Desnick RJ. Pompe disease: dramatic im-
provement in gastrointestinal function following enzyme replacement therapy. A
report of three later-onset patients. Mol Genet Metab 2010; 101:130.
7. Pardo J, García-Sobrino T, López-Ferreiro A. Gastrointestinal symptoms in late-
onset Pompe disease: Early response to enzyme replacement therapy. J Neurol Sci
2015; 353:181.
8. Goldstein JL, Young SP, Changela M, et al. Screening for Pompe disease using a
rapid dried blood spot method: experience of a clinical diagnostic laboratory. Mus-
cle Nerve 2009; 40:32.

296
Case 50

9. Chien YH, Goldstein JL, Hwu WL, et al. Baseline Urinary Glucose Tetrasaccharide
Concentrations in Patients with Infantile-and Late-Onset Pompe Disease Identi-
fied by Newborn Screening. JIMD Rep 2015; 19:67.
10. Yang CF, Yang CC, Liao HC, et al. Very Early Treatment for Infantile-Onset Pompe
Disease Contributes to Better Outcomes. J Pediatr 2016; 169:174.
11. van den Hout HM, Hop W, van Diggelen OP, et al.. The natural course of infantile
Pompe’s disease: 20 original cases compared with 133 cases from the literature.
Pediatrics 2003; 112:332.
12. Mellies U, Stehling F, Dohna-Schwake C, et al.. Respiratory failure in Pompe dis-
ease: treatment with non-invasive ventilation. Neurology 2005; 64:1465.
13. Kishnani PS, Corzo D, Leslie ND, et al.. Early treatment with alglucosidase al-
pha prolongs long-term survival of infants with Pompe disease. Pediatr Res 2009;
66:329.
14. van Gelder CM, Poelman E, Plug I, et al.. Effects of a higher dose of alglucosidase
alfa on ventilator-free survival and motor outcome in classic infantile Pompe dis-
ease: an open-label single-centre study. J Inherit Metab Dis 2016; 39:383.
15. Banugaria SG, Prater SN, Ng YK, et al.. The impact of antibodies on clinical out-
comes in diseases treated with therapeutic protein: lessons learned from infantile
Pompe disease. Genet Med 2011; 13:729.

297
COMMENTS OF WELL KNOWN NEUROLOGISTS
Paediatric Neurology can be a difficult area to many trainees.
Dr Saeed has included a varied collection of cases, some of which
doctors may come across regularly and some which most general
paediatricians may only come across once or twice in a career.
In addition to the solution to the case there is a very detailed
summary of each condition, differential diagnosis and excellent
reference section which will provoke further reading.
This is a book which will be useful for both doctors working for
post-graduate exams as well as more experienced doctors who
simply enjoy a diagnostic challenge. I thoroughly recommend it.

Dr. Gerry Mackin MBBS MRCP FRCPCH


Consultant Paediatrician
South West Acute Hospital
Enniskillen
Co Fermanagh
Northern Ireland

Paediatric Neurology is sub-speciality which deals with complex


and complicated cases. Many a time the neurological conditions
have multi-system involvement and makes cases even more
challenging. Neuro-inflammatory, Neuro-metabolic, Neuro-
infectious, Neuro-oncological and Neuro-genetic group of
diseases not only require extensive knowledge but support from
multi professional approach is mandatory for assessment and
management. The rapidly advancement made in the Genetic
front is now resulting previously ‘idiopathic’ or ‘primary’ labelled
conditions to now being found to have underlying genetic cause,
idiopathic/primary generalised epilepsies are prime example.
Various Neurology genetic panels or even Whole Exome Sequence has made it possible to
find the offending gene. This of course not only help the family concern to have a ‘label’ but
from pragmatic and practical point of view, can be of tremendous help in future pregnancies
or even family extension planning in the un-affected siblings
The author has put lot of efforts in compiling this Case-based book which is helpful learn-
ing format at all levels but specifically for trainees of all grades.
A detailed differential diagnosis and discussion will help reader in expanding knowledge
and while at the same time also help in diagnostic approach.
I recommend this book to be part of reading to all paediatricians and paediatric trainees.
Dr. Ayaz Shah
Consultant Paediatric Neurologist
Belfast Royal Hospital for Sick Children
Belfast
Northern Ireland

You might also like